You are on page 1of 296

Visit https://telegram.me/booksforcbse for more books.

Chemical
1 Acids, Bases
Reactions
Salts
Equations
and
and

6. Which of the following reactions involves the combination


Multiple Choice Questions (MCQs) of two elements?
(a) CaO + CO2 → CaCO3
DIRECTIONS : This section contains multiple choice
(b) 4Na + O2 → 2Na2O
questions. Each question has four choices (a), (b), (c) and (d)
out of which only one is correct. 1
(c) SO 2 + O 2 → SO3
2
1. Which of the following is a decomposition reaction? (d) NH3 + HCl → NH4Cl
Heat
2HgO 
(a) → 2Hg + O2 7. When hydrogen sulphide gas is passed through a blue
Heat
solution of copper sulphate, a black precipitate of copper
CaCO3 
(b) → CaO + CO 2 sulphide is obtained and the sulphuric acid so formed
Electrolysis remains in the solution. The reaction is an example of –
(c)
2H 2O → H 2 + O 2
(a) a combination reaction
(d) All of these (b) a displacement reaction
2. On the basis of following features, identify the correct (c) a decomposition reaction
option. (d) a double decomposition reaction
(i) This reaction occurs during corrosion.
8. What happens when copper rod is dipped in iron sulphate
(ii) This reaction occurs during respiration.
solution?
(a) Decomposition reaction
(a) Copper displaces iron
(b) Redox reaction
(b) Blue colour of copper sulphate solution is obtained
(c) Combination reaction
(c) No reaction takes place
(d) Endothermic reaction
(d) Reaction is exothermic
3. Which of the following is not a physical change?
9. A student added dilute HCl to a test tube containing zinc
(a) Boiling of water to give water vapour.
granules and made following observations which one is
(b) Melting of ice to give water.
correct?
(c) Dissolution of salt in water.
(a) The zinc surface became dull and black.
(d) Combustion of Liquefied Petroleum Gas (LPG).
(b) A gas evolved which burns with a pop sound.
4. Which of the following can be decomposed by the action (c) The solution remained colourless.
of light? (d) The solution becomes green in colour.
(a) NaCl (b) KCl
(c) AgCl (d) CuCl 10. A dilute solution of sodium carbonate was added to two
test tubes (A) containing dil HCl and (B) containing dilute
5. In which of the following the identity of initial substance NaOH. The correct observation was –
remains unchanged? (a) a brown coloured gas liberated in test tube A.
(a) Curdling of milk (b) a brown coloured gas liberated in test tube B.
(b) Formation of crystals by process of crystallisation (c) a colourless gas liberated in test tube A.
(c) Fermentation of grapes
(d) a colourless gas liberated in test tube B.
(d) Digestion of food
S-2 Science
11. A balanced chemical equation is in accordance with – 20. In the equation, NaOH + HNO3 → NaNO3 + H2O
(a) Avogadro’s law nitric acid is acting as –
(b) law of multiple proportion (a) an oxidising agent (b) an acid
(c) law of conservation of mass (c) a nitrating agent (d) a dehydrating agent
(d) law of gaseous volumes.
21. Fe2O3 + 2Al → Al2O3 + 2Fe
12. The equation
The above reaction is an example of a –
Cu + xHNO3 → Cu(NO3)2 + yNO2 + 2H2O
(a) combination reaction
The values of x and y are – (b) double displacement reaction
(a) 3 and 5 (b) 8 and 6 (c) decomposition reaction
(c) 4 and 2 (d) 7 and 1 (d) displacement reaction
13. Zn + H2SO4(dil) → ZnSO4 + H2↑ 22. White silver chloride in sunlight turns to –
Above reaction is – (a) grey (b) yellow
(a) decomposition reaction (c) remain white (d) red
(b) single displacement reaction 23. Black and white photography uses –
(c) combination reaction (a) decomposition of silver chloride.
(d) synthesis reaction (b) decomposition of silver bromide.
14. The reaction in which two compounds exchange their (c) both
ions to form two new compounds is – (d) none of these
(a) a displacement reaction
24. When copper powder is heated it gets coated with –
(b) a decomposition reaction
(a) black copper oxide (b) yellow copper oxide
(c) an isomerization reaction
(c) red copper oxide (d) None of these
(d) a double displacement reaction
15. When the gases sulphur dioxide and hydrogen sulphide 25. Combination of phosphorus and oxygen is an example
mix in the presence of water, the reaction is of–
(a) oxidation (b) reduction
SO2 + 2H2S → 2H2O + 2S. Here hydrogen sulphide is
(c) rancidity (d) None of these
acting as –
(a) an oxidising agent (b) a reducing agent 26. Rusting of iron is an example of –
(c) a dehydrating agent (d) a catalyst (a) reduction (b) redox
(c) oxidation (d) dissociation
16. CuO + H2 → H2O + Cu, reaction is an example of –
(a) redox reaction (b) synthesis reaction 27. Which of the following does not corrode when exposed to
(c) neutralisation (d) analysis reaction the atmosphere?
(a) Iron (b) Copper
17. A substance which oxidises itself and reduces other is
known as – (c) Gold (d) Silver
(a) oxidising agent (b) reducing agent 28. Take about 1.0g CaCO3 in a test tube. Heat it over a flame,
(c) both of these (d) none of these a colourless gas comes out. The reaction is called a
18. A redox reaction is one in which – (a) decomposition reaction
(a) both the substances are reduced. (b) displacement reaction
(b) both the substances are oxidised. (c) double decomposition reaction
(c) an acid is neutralised by the base. (d) double displacement reaction
(d) one substance is oxidised while the other is reduced. 29. Hydrogen sulphide (H2S) is a strong reducing agent.
19. In the following equations : Which of the following reactions shows its reducing
action?
Na2CO3 + x HCl → 2 NaCl + CO2 + H2O, the value of x
is– (a) Cd(NO3)2 + H2S → CdS + + 2HNO3
(a) 1 (b) 2 (b) CuSO4 + H2S → CuS + H2SO4
(c) 3 (d) 4 (c) 2FeCl3 + H2S → 2FeCl2 + 2HCl + S
(d) Pb(NO3)2 + H2S → PbS + 2CH3COOH
Chemical Reactions and Equations S-3
30. 2CuI → Cu + CuI2, the reaction is – 38. The oxidation states of P atom in POCl3, H2PO3 and
(a) redox (b) neutralisation H2P2O6, respectively are
(a) + 5, + 4, + 4 (b) + 5, + 5, + 4
(c) oxidation (d) reduction
(c) + 4, + 4, + 5 (d) + 3, + 4, + 5
31. When copper turnings are added to silver nitrate solution,
39. The process of respiration is :
a blue coloured solution is formed after some time. It is
because, copper – (a) Oxidation reaction which is endothermic
(b) Reduction reaction which is endothermic
(a) displaces silver from the solution
(c) Combination reaction which is exothermic
(b) forms a blue coloured complex with AgNO3
(d) Oxidation reaction which is exothermic
(c) is oxidised to Cu2+
40. Silver articles become black when exposed to air. It is due
(d) is reduced to Cu2+
to the formation of
32. Zn2+(aq) + 2e– → Zn(s). This is – (a) Silver oxide (b) Silver nitrate
(a) oxidation (b) reduction (c) Silver chloride (d) Silve sulphide
(c) redox reaction (d) none of these 41. A test tube along with calcium carbonate in it initially
weighed 30.08 g. A heating experiment was performed
33. A substance A reacts with another substance B to produce
on this test tube till calcium carbonate completely
the product C and a gas D. If a mixture of the gas D and
decomposed with evolution of a gas. Loss of weight
ammonia is passed through an aqueous solution of C,
during this experiment was 4.40 g. What is the weight of
baking soda is formed. The substances A and B are
the empty test tube in this experiment?
(a) HCl and NaOH (b) HCl and Na2CO3
(a) 20.08 g (b) 21.00 g
(c) Na and HCl (d) Na2CO3 and H2O (c) 24.50 g (d) 2.008 g
34. Chemically the ‘water gas’ is 42. Match chemical reactions given in the List I with the type
(a) H2O (gaseous) (b) CO2 + H2 of chemical reactions given in List II and select the correct
(c) CH4 + H2O (d) CO + H2 answer using the options given below:

35. The oxidation number of sulphur is –4 in List I List II


(Chemical reactions) (Type of Chemical
(a) H2S (b) CS2 reactions)
(c) Na2SO4 (d) Na2SO3 A. Formation of NH3 I. Decomposition
from N2 and H2
36. Identify the endothermic process from the following B. Calcination of zinc II. Double displacement
(a) Addition of conc. HCl to water carbonate.
(b) CH4(g) +2O2(g) → CO2(g) + 2H2O(1) C. Reaction of aqueous III. Combination
BaCl2 solution with
(c) H2O(1) → H2O(g) dilute H2SO4
(d) CaO(s) + H2O(1) → Ca(OH)2(aq) D. Rancidity of oils IV. Redox
V. Displacement
37. The schematic diagram is given below
(a) A-I, B-V, C-III, D-IV (b) A-III, B-IV, C-V, D-I
heat
A →
→ B + HCl (c) A-IV, B-III, C-V, D-I (d) A-III, B-I, C-II, D-IV
(solid) cool (vapour) (vapour)
43. A B C D
→

heat NaOH(aq)

HO
C →
conc. HCl
D →
2
shake well
E(aq)
(Gas) (acidic solution)

Which of the following is a correct statement ?


(a) A and E are chemically same. Cu Al Fe Zn

(b) A and D are chemically same. If we added FeSO4 to above four test tubes, in which test
tube we observe black residue?
(c) D and E are chemically same.
(a) “A” and “B” (b) “B” and “C”
(d) C and E are chemically same.
(c) “A” and “C” (d) “B” and “D”
S-4 Science
Case/Passage - 2
Chemistry in Automobiles:
DIRECTIONS : Study the given case/passage and answer the For an internal combustion engine to move a vehicle down
following questions. the road, it must convert the energy stored in the fuel into
mechanical energy to drive the wheels. In your car, the
Case/Passage - 1 distributor and battery provide this starting energy by creating
The reaction between MnO2 with HCl is depicted in the an electrical “spark”, which helps in combustion of fuels like
following diagram. It was observed that a gas with bleaching gasoline. Below is the reaction depicting complete combustion
abilities was released . [From CBSE Question Bank-2021] of gasoline in full supply of air:
[From CBSE Question Bank-2021]
HCl (aq)
2C8H18(I) + 25O2(g) → 16 ‘X’ + Y
49. Which of the following are the products obtained from the
reaction mentioned in the above case?
Product ‘X’ Product ‘Y’
(a) CO2 H2O2
(b) H2O CO
(c) CH3OH H2O
MnO2(s)
(d) CO2 H2O
Reactants Products
50. Identify the types of chemical reaction occurring during
44. The chemical reaction between MnO2 and HCl is an the combustion of fuel:
example of: (a) Oxidation & Endothermic reaction
(a) displacement reaction (b) Decomposition & Exothermic reaction
(b) combination reaction (c) Oxidation & Exothermic reaction
(c) redox reaction (d) Combination & Endothermic reaction
(d) decomposition reaction. 51. On the basis of evolution/absorption of energy, which of
the following processes are similar to combustion of fuel?
45. Chlorine gas reacts with _____ to form bleaching powder.
(i) Photosynthesis in plants
(a) dry Ca(OH)2
(ii) Respiration in the human body
(b) dil. solution of Ca(OH)2 (iii) Decomposition of vegetable matter
(c) conc. solution of Ca(OH)2 (iv) Decomposition of ferrous sulphate.
(d) dry CaO (a) (ii) & (iii) (b) (i) & (ii)
46. Identify the correct statement from the following: (c) (iii) & (iv) (d) (ii) & (i)
(a) MnO2 is getting reduced whereas HCl is getting 52. ‘A student while walking on the road observed that a
oxidized cloud of black smoke belched out from the exhaust stack
(b) MnO2 is getting oxidized whereas HCl is getting of moving trucks on the road.’ Choose the correct reason
reduced. for the production of black smoke:
(a) Limited supply of air leads to incomplete combustion
(c) MnO2 and HCl both are getting reduced.
of fuel.
(d) MnO2 and HCl both are getting oxidized.
(b) Rich supply of air leads to complete combustion of
47. In the above discussed reaction, what is the nature of fuel.
MnO2? (c) Rich supply of air leads to a combination reaction.
(a) Acidic oxide (b) Basic oxide (d) Limited supply of air leads to complete combustion
(c) Neutral oxide (d) Amphoteric oxide of fuel.
53. ‘Although nitrogen is the most abundant gas in the
48. What will happen if we take dry HCl gas instead of
atmosphere, it does not take part in combustion’. Identify
aqueous solution of HCl?
the correct reason for this statement.
(a) Reaction will occur faster. (a) Nitrogen is a reactive gas
(b) Reaction will not occur. (b) Nitrogen is an inert gas
(c) Reaction rate will be slow. (c) Nitrogen is an explosive gas
(d) Reaction rate will remain the same. (d) Only hydrocarbons can take part in combustion
Chemical Reactions and Equations S-5
Reason : A substance which helps in oxidation is known
Assertion & Reason as reducing agent.

DIRECTIONS : Each of these questions contains an assertion 63. Assertion : The balancing of chemical equations is based
followed by reason. Read them carefully and answer the question on law of conservation of mass.
on the basis of following options. You have to select the one that Reason : Total mass of reactants is equal to total mass of
best describes the two statements. products.
(a) If both Assertion and Reason are correct and Reason is
the correct explanation of Assertion. Match the Following
(b) If both Assertion and Reason are correct, but Reason is
not the correct explanation of Assertion. DIRECTIONS : Each question contains statements given in
(c) If Assertion is correct but Reason is incorrect. two columns which have to be matched. Statements (A, B, C, D)
in column I have to be matched with statements (p, q, r, s) in column II.
(d) If Assertion is incorrect but Reason is correct.
54. Assertion : Chlorine gas react with potassium iodide 64. Column II gives type of reaction mention in column I,
solution to form potassium chloride and iodine.
match them correctly.
Reason : Chlorine is more reactive than iodine therefore Column I Column II
displaces iodine from potassium iodide.
(A) C + O2 → CO2 (p) Displacement
55. Assertion : When copper strip is placed in ferrous
sulphate solution, colour of the solution changes. light
(B) AgBr  → Ag + Br (q) Combination
Reason : Iron is more reactive than copper.
(C) Zn + CuSO4 → ZnSO4 + Cu (r) Decomposition
56. Assertion : Decomposition of vegetable matter into
compost is an endothermic reaction. Cu
(D) CH3CH2OH → (s) Oxidation
Reason : Heat is required in an endothermic reaction.
CH3CHO + H2
57. Assertion : Reaction of sodium sulphate with barium
chloride is a precipitation reaction. 65. Column I Column II

Reason : Precipitation reaction produces insoluble salt. (A) KClO3 → (p) O2
58. Assertion: When a mixture of hydrogen and chlorine is ∆
placed in sunlight, hydrogen chloride is formed. (B) ZnCO3 → (q) H2O

Reason : It is an example of combination reaction. (C) H2CO3 → (r) CO2
59. Assertion : Stannous chloride gives grey precipitate with ∆
mercuric chloride, but stannic chloride does not do so. (D) C2H6 → (s) ZnO

Reason : Stannous chloride is a powerful oxidising agent A B C D


which oxidises mercuric chloride to mercury. (a) p s, r q, r q, r
60. Assertion : Corrosion of iron is commonly known as (b) p q, r s, r r, p
rusting. (c) q, r s, p p, s r
Reason : Corrosion of iron occurs in presence of water (d) r q s p
and air.
61. Assertion : In a reaction Fill in the Blanks
Zn(s) + CuSO4 (aq) → ZnSO4 (aq) + Cu(s),
Zn is a reductant but itself get oxidized. DIRECTIONS : Complete the following statements with an
appropriate word / term to be filled in the blank space(s).
Reason : In a redox reaction, oxidant is reduced by
accepting electrons and reductant is oxidized by losing 66. In a .................... reaction two or more substances combine
electrons. to form a new single substance.
62. Assertion : A reducing agent is a substance which can 67. Reactions in which heat is given out along with the
either accept electron. products are called ....................... reactions.
S-6 Science
68. Reactions in which energy is absorbed are known as
........... reactions. True / False
69. When an element displaces another element from its DIRECTIONS : Read the following statements and write your
compound, a ..................... reaction occurs. answer as true or false.
70. Two different atoms or groups of atoms (ions) are
77. The number of atoms of each element is conserved in any
exchanged in .................... reactions.
chemical reaction.
71. Precipitation reactions produce .................... salts. 78. Oxidation is the loss of electrons from a substance.
72. Reduction is the ................. of oxygen or gain of hydrogen. 79. Reduction is the gain of electrons by a substance.

73. The digestion of food in the body is an example of ......... 80. A complete chemical equation represents the reactants,
reaction. products and their physical states symbolically.
81. A magnesium ribbon burns with a dazzling flame in air
74. The addition of oxygen to a substance is called ...........
(oxygen) and changes into a white substance, magnesium
75. When calcium carbonate is heated, it decomposes to give oxide.
.................. and ................. . 82. Rusting is a double decomposition reaction.
76. The new substances produced in a reaction are called as 83. The reaction between nitrogen and hydrogen to give
.................... ammonia is an example of a combination reaction.
84. Action of heat on ferrous sulphate is an example of
decomposition reaction.
85. The formation of Cu and H2O in the reaction of copper
oxide with hydrogen is an example of a redox reaction.
Chemical Reactions and Equations S-7

ANSWER KEY & SOLUTIONS


1. (d) A decomposition reaction is a type of chemical 21. (d)
reaction in which a single compound breaks down
22. (a) White silver chloride in sunlight turns to grey.
into two or more elements or new compounds.
23. (b)
2. (b) Both are redox reactions. Redox reactions are

characterised by the transfer of electrons between 24. (a) 2Cu + O 2 → 2CuO
Black
chemical species. One species undergoes oxidation
(Oxidation)
while another species undergoes reduction. 25. (a) 4P + 3O 2 → 2P2O3
3. (d) Combustion of liquefied petroleum gas is a chemical 4P + 5O 
(Oxidation)
2 → 2P2O5
change. As it is an irreversible reaction and new
products (carbon dioxide and water vapours) are 26. (b) 2Fe(s) + O2(g) + 4H+(aq) → 2Fe2+(aq) + 2H2O(l)
formed during the change. Also, a lot of heat is
released during this reaction. 27. (c) Gold is least reactive hence does not corrode at all.

4. (c) 28. (a) CaCO3 → CaO + CO 2
5. (b) Formation of crystals by process of crystallization. Reduction

6. (b) Except (b) all other reactions involve compounds. 29. (c) FeCl3 + H2S FeCl2 + HCl + S

CuSO 4 + H 2S 
→ CuS + H 2SO 4 Oxidation
(blue) (black)
(double decomposition reaction) In the given reaction H2S undergoes oxidation,
hence behave as a reducing agent.
7. (d) reduction
30. (a)
8. (c) Iron is more reactive than copper, hence Cu will not +1 0 +2
displace iron from iron sulphate, hence no reaction 2C uI → C u + C uI 2 . Oxidation and reduction both
will take place. oxidation
9. (b) Zn + 2HCl → ZnCl2 + H2 occur so the reaction is redo
Hydrogen gas burns with a pop sound. 31. (a) Cu is more reactive than Ag.
10. (c) Na2CO3 + 2HCl → 2NaCl + H2O + CO2 0 +1
Cu + 2AgNO3 
→ CuNO3 + 2Ag
Na2CO3 + NaOH → no reaction
11. (c) +2 0

12. (c) Cu + 4HNO3 → Cu(NO3)2 + 2NO2 + 2H2O 32. (b) Zn(aq) + 2e 
→ Z n(s) ; reduction

13. (b) 14. (d) 33. (b) HCl + Na 2CO3 


→ NaCl(aq)+ CO 2 + H 2O
(A) (B) (C) (D)
15. (b) Here H2S is behaves as a reducing agent and oxidises
to H2O.
CO 2 + NH3 + NaCl(aq) 
→ NaHCO3 + NH 4Cl
Oxidation Baking soda

16. (a) CuO + H2 → Cu + H2O Hence A & B are HCl and Na2CO3
Reduction 34. (d) Water gas → CO + H2
17. (b) 18. (d) 35. (*) Let the oxidation state of S be x
19. (b) Na2CO3 + 2HCl → 2NaCl + CO2 + H2O (i) H2S
20. (b) The reaction represents a neutralisation reaction in \2+x=0
which base (NaOH) reacts with an acid (HNO3) to x = –2
form salt (NaNO3) and water (H2O).
S-8 Science
(ii) CS2 44 g CO2 is formed from 100 g CaCO3
4 + 2x = 0 ⇒ x = –2 100
(iii) Na2SO4 4.40 g CO is formed from × 44 = 10 g CaCO3
44
2(+1) + x + 4(–2) = 0 If mass of CaCO3 is 10 g, then weight of empty test
2+x–8=0 tube = 30.08 – 10.0 = 20.08 g
x=+6 42. (d) A-III, B-I, C-II, D-IV
(iv) Na2SO3 43. (d) Zn and Al are more reactive than iron, therefore they
2(+1) + x + 3(– 2) = 0 will displace iron from its salt solution giving black
2+x–6=0 residue, while Cu being less reactive than iron will
x=+4 not able to displace iron from its salt solution.
None of these, option is correct.
FeSO4 + 2Al → Al2(SO4)3 + 3Fe
36. (c) Conversion of liquid to gas is endothermic process.
FeSO4 + Zn → ZnSO4 + Fe
37. (b) (vapour)
(solid) A
heat
B + HCl FeSO4 + Cu → No reaction
cool (vapour)
NH4Cl NH3 FeSO4 + Fe → No reaction
44. (c) redox reaction
heat NaOH (aq.)
45. (a) dry Ca(OH)2
HCl (con)
C → D NH4Cl 46. (a) MnO2 is getting reduced whereas HCl is getting
NH3(g) oxidized
(shake
H2O
well) 47. (b) Basic oxide
E HCl 48. (b) Reaction will not occur
(acidic soln.)
49. (d)
A = NH4Cl; D = NH4Cl
Hence correct statement is: A and D are chemically 50. (c)
same. 51. (a)
38. (a) Let the oxidation state of P-atom in POCl3, H2PO3 52. (a)
and H4P2O6 be x.
53. (b)
(i) POCl3 54. (a) Chlorine displaces iodine from potassium iodide
x + 1(–2) + 3(–1) = 0 solution.
x – 2 – 3 = 0 55. (d) When copper strip is placed in FeSO4 solution,
x=+5 colour of the solution does not change.
56. (d) Decomposition of vegetable matter into compost is
(ii) H2PO3
an exothermic reaction.
2(1) + x + 3(–2) = 0 57. (a) Na2SO4(aq) + BaCl2(aq) →
2 + x – 6 = 0
BaSO 4 (s) + 2NaCl(aq)
x=+4 Precipitate
(iii) H4P2O6
4(1) + 2x + 6(–2) = 0 58. (a) A combination reaction is a reaction where two or
more elements or compounds combine to form a
4 + 2x – 12 = 0
single compound. Hydrogen and chlorine combine
2x = 8 to give hydrogen chloride.
x = + 4.
Reduction
39. (d) Respiration is oxidation and exothermic process.
+2 +2 +4 +1
40. (d) Layer of silver sulphide deposited on the silver 59. (c) SnCl2 + 2HgCl2 → SnCl4 + Hg2Cl2
articles when exposed to air.
Oxidation (Reducing agent)
41. (a) On thermal decomposition of calcium carbonate
Hg2Cl2 + SnCl2 → 2Hg + SnCl4

CaCO3 → CaO + CO 2
100 g 44 g
60. (b) Corrosion occurs due to oxidation of iron.
56 g
Chemical Reactions and Equations S-9
61. (a) 70. double displacement 71. insoluble
62. (d) A reducing agent is a substance which oxidizes itself 72. loss 73. Decomposition reaction
but reduces others i.e., looses electrons.
74. oxidation 75. CaO (s) and CO2 (g)
63. (a)
76. products
64. A → (q) B → (r) C → (p) D → (s)
77. True 78. True 79. True 80. True
65. (a)
81. True 82. False 83.
True 84. True
66. combination 67. exothermic
85. True
68. endothermic 69. displacement
Acids, Bases
Bases and
2 Acids,
Salts
and Salts
6. The product of complete neutralization of H3PO3 with
Multiple Choice Questions (MCQs) NaOH is :
(a) NaH2PO3 (b) Na2HPO3
DIRECTIONS : This section contains multiple choice
questions. Each question has four choices (a), (b), (c) and (d) (c) Na3PO3 (d) Na3(HPO3)2
out of which only one is correct.
7. Chemical A is used for water softening to remove
temporary hardness. ‘A’ reacts with sodium carbonate to
1. During the preparation of hydrogen chloride gas on a
generate caustic soda. What is ‘A’?
humid day, the gas is usually passed through the guard
tube containing calcium chloride. The role of calcium (a) Gypsum (b) Slaked lime
chloride taken in the guard tube is to: (c) Quick lime (d) Lime stone
(a) absorb the evolved gas. 8. An aqueous solution turns red litmus solution blue.
(b) moisten the gas. Excess addition of which of the following solution would
reverse the change?
(c) absorb moisture from the gas.
(a) Baking powder
(d) absorb Cl– ions from the evolved gas.
(b) Lime
2. To protect tooth decay we are advised to brush our teeth (c) Ammonium hydroxide solution
regularly. The nature of the tooth paste commonly used is: (d) Hydrochloric acid
(a) Acidic (b) Neutral
9. A blue litmus paper was first dipped in dil. HCl and then
(c) Basic (d) Corrosive in dil. NaOH solution. It was observed that the colour of
the litmus paper –
3. Which of the following is not a mineral acid?
(a) changed to red.
(a) Hydrochloric acid (b) Citric acid
(b) changed first to red and then to blue.
(c) Sulphuric acid (d) Nitric acid
(c) changed blue to colourless.
4. Which of the following acid is present in sour milk ? (d) remains blue in both the solutions.
(a) glycolic acid (b) lactic acid 10. The acid used in making vinegar is –
(c) citric acid (d) tartaric acid (a) formic acid (b) acetic acid
5. An aqueous solution ‘A’ turns phenolphthalein solution (c) sulphuric acid (d) nitric acid
pink. On addition of an aqueous solution ‘B’ to ‘A’, the
11. CuO + (X) → CuSO4 + H2O. Here (X) is –
pink colour disappears. The following statement is true
for solution ‘A’ and ‘B’. (a) CuSO4 (b) HCl

(a) A is strongly basic and B is a weak base. (c) H2SO4 (d) HNO3

(b) A is strongly acidic and B is a weak acid. 12. Reaction of an acid with a base is known as –
(c) A has pH greater than 7 and B has pH less than 7. (a) decomposition (b) combination
(d) A has pH less than 7 and B has pH greater than 7. (c) redox reaction (d) neutralization
Acids, Bases and Salts S-11
13. When CO2 is passed through lime water, it turns milky. 24. When an oxide of a non–metal reacts with water which of
The milkiness in due to formation of – the following is formed?
(a) CaCO3 (b) Ca(OH)2 (a) Acid (b) Base
(c) H2O (d) CO2 (c) Salt (d) None of these

14. Antacids contain – 25. ‘Alum’ is an example of –


(a) single salt (b) double salt
(a) weak base (b) weak acid
(c) acids (d) none of these
(c) strong base (d) strong acid
26. Which of the following statements is correct about an
15. 2NaOH + MgSO4 ––––→ ? aqueous solution of an acid and of a base?
(a) MgO + Na2SO4 (b) Mg(OH)2 + Na2SO4 (i) Higher the pH, stronger the acid
(c) Mg(OH)2 + Na2O (d) MgO + Na2O (ii) Higher the pH, weaker the acid
16. Bleaching powder gives smell of chlorine because it – (iii) Lower the pH, stronger the base
(a) is unstable. (iv) Lower the pH, weaker the base
(b) gives chlorine on exposure to atmosphere. (a) (i) and (iii) (b) (ii) and (iii)
(c) is a mixture of chlorine and slaked lime. (c) (i) and (iv) (d) (ii) and (iv)
(d) contains excess of chlorine.
27. A sample of soil is mixed with water and allowed to
17. Plaster of paris is made from – settle. The clear supernatant solution turns the pH paper
(a) lime stone (b) slaked lime yellowish-orange. Which of the following would change
the colour of this pH paper to greenish-blue?
(c) quick lime (d) gypsum
(a) Lemon juice (b) Vinegar
18. Chemical formula of baking soda is –
(c) Common salt (d) An antacid
(a) MgSO4 (b) Na2CO3
(c) NaHCO3 (d) MgCO3 28. Plaster of paris is obtained –

19. Washing soda has the formula – (a) by adding water to calcium sulphate.
(a) Na2CO3.7H2O (b) Na2CO3.10H2O (b) by adding sulphuric acid to calcium hydroxide.
(c) Na2CO3.H2O (d) Na2CO3 (c) by heating gypsum to a very high temperature.
20. Plaster of Paris hardens by – (d) by heating gypsum to 373 K.
(a) giving of CO2 29. What is the term for the positive and negative ions of a
(b) changing into CaCO3 compound breaking apart in solution –
(c) combining with water (a) Conglomeration (b) Oxidation
(d) giving out water (c) Dissociation (d) None of the Above
21. Which of the following is acidic in nature? 30. Of the aqueous solutions listed below, which would be
(a) apple juice (b) soap solution the best conductor of an electric current?
(a) HCl (b) H3PO4
(c) slaked lime (d) lime
(c) HOCl (d) CH3COOH
22. The reaction of metal with acid results in the formation
of– 31. Common salt besides being used in kitchen can also be
used as the raw material for making
(a) only hydrogen gas
(i) washing soda (ii) bleaching powder
(b) only salt
(iii) baking soda (iv) slaked lime
(c) both salt and hydrogen gas
(a) (i) and (ii) (b) (i), (ii) and (iv)
(d) none of these (c) (i) and (iii) (d) (i), (iii) and (iv)
23. Which of the following acid does not react with metals? 32. Which salt can be classified as an acid salt?
(a) sulphuric acid (b) phosphoric acid (a) Na2SO4 (b) BiOCl
(c) carbonic acid (d) nitric acid (c) Pb(OH)Cl (d) Na2HPO4
S-12 Science
33. An element X reacts with dilute H2SO4 as well as with 38. The chemical formula of ‘Plaster of Paris’ is
NaOH to produce salt and H2(g). Hence, it may be 1
concluded that: (a) CaSO 4 . H 2 O (b) CaSO4.2H2O
2
I. X is an electropositive element.
3
II. oxide of X is basic in nature. (c) CaSO4.H2O (d)
CaSO 4 . H 2 O
2
III. oxide of X is acidic in nature.
IV. X is an electronegative element. 39. A solution turns red litmus blue. Its pH is likely to be –
(a) I, II, III (b) IV, I, II (a) 2 (b) 4
(c) III, IV, I (d) II, III, IV (c) 5 (d) 10
34. The turmeric solution will turn red by an aqueous solution 40. A solution reacts with crushed egg-shells to give a gas
of - that turns lime water milky. The solution contains –
(a) potassium acetate (b) copper sulphate (a) NaCl (b) HCl
(c) sodium sulphate (d) ferric chloride (c) LiCl (d) KCl
35. The correct order of increasing pH values of the aqueous 41. 10 mL of a solution of NaOH is found to be completely
solutions of baking soda, rock salt, washing soda and neutralised by 8 mL of a given solution of HCl. If we take
slaked lime is 20 mL of the same solution of NaOH, the amount of
(a) Baking Soda < Rock Salt < Washing Soda < Slaked HCl solution (the same solution as before) required to
lime neutralise will be –
(b) Rock Salt < Baking Soda < Washing Soda <Slaked (a) 4 mL (b) 8 mL
lime (c) 12 mL (d) 16 mL
(c) Slaked lime < Washing Soda < Rock Salt < Baking
42. Which of the following type of medicines is used for
Soda treating indigestion ?
(d) Washing Soda < Baking Soda < Rock Salt < Slaked (a) Antibiotic (b) Analgesic
lime (c) Antacid (d) Antiseptic
36. You are provided with aqueous solutions of three salts — 43. Which of the following reaction does not results in the
A, B and C, 2-3 drops of blue litmus solution, red litmus evolution of H2 gas?
solution and phenolphthalein were added to each of these
(a) dilute sulphuric acid reacts with zinc granules.
solution in separate experiments. The change in colours
of different indicators were recorded in the following (b) dilute hydrochloric acid reacts with magnesium ribbon.
table: (c) dilute sulphuric acid reacts with aluminium powder.
(d) dilute hydrochloric acid with diute sodium hydroxide
Sample With blue With red With solution.
litmus litmus phenolphtha-
solution solution lein
solution
A No change No change No change DIRECTIONS : Study the given case/passage and answer the
B Turns red No change No change following questions.
C No change Turns blue Turns pink
Case/Passage - 1
On the basis of above observations, identify A, B, and C Marble’s popularity began in ancient Rome and Greece, where
from the following options: white and off-white marble were used to construct a variety
of structures, from hand-held sculptures to massive pillars and
(a) A = NH4 Cl, B = NaCl, C = CH3COONa
buildings.
(b) A = NH4 Cl, B = CH3 COONa, C = NaCl
(c) A = NaCl, B = NH4 Cl, C = CH3 COONa
(d) A = CH3 COONa, B = NH4 Cl, C = NaCl
37. Aqua regia is the mixture of conc. HCl and conc. HNO3
in the ratio:
(a) 1 : 3 (b) 2 : 3
(c) 3 : 1 (d) 3 : 2 [From CBSE Question Bank-2021]
Acids, Bases and Salts S-13
44. The substance not likely to contain CaCO3 is Case/Passage - 2
(a) Dolomite (b) A marble statue Frothing in Yamuna:
(c) Calcined gypsum (d) Sea shells.
The primary reason behind the formation of the toxic foam is
45. A student added 10g of calcium carbonate in a rigid high phosphate content in the wastewater because of detergents
container, secured it tightly and started to heat it. After used in dyeing industries, dhobi ghats and households.
some time, an increase in pressure was observed, the Yamuna’s pollution level is so bad that parts of it have been
pressure reading was then noted at intervals of 5 mins labelled ‘dead’ as there is no oxygen in it for aquatic life to
and plotted against time, in a graph as shown below. survive.
During which time interval did maximum decomposition
took place?
1.25

1.00
Pressure (atm)

0.75

0.50

0.25 [From CBSE Question Bank-2021]


49. Predict the pH value of the water of river Yamuna if the
(a) 15-20 min (b) 10-15 min reason for froth is high content of detergents dissolved in
(c) 5-10 min (d) 0-5 min it.
46. Gas A, obtained above is a reactant for a very important (a) 10-11 (b) 5-7
biochemical process which occurs in the presence of (c) 2-5 (d) 7
sunlight. Identify the name of the process -
50. Which of the following statements is correct for the water
(a) Respiration (b) Photosynthesis with detergents dissolved in it?
(c) Transpiration (d) Photolysis (a) low concentration of hydroxide ion (OH– )and high
47. Marble statues are corroded or stained when they concentration of hydronium ion (H3O+)
repeatedly come into contact with polluted rain water. (b) high concentration of hydroxide ion (OH–)and low
Identify the main reason. concentration of hydronium ion (H3O+)
(c) high concentration of hydroxide ion (OH–) as well
as hydronium ion (H3O+)
(d) equal concentration of both hydroxide ion (OH–)
and hydronium ion (H3O+).
The table provides the pH value of four solutions P, Q, R
and S

Solution pH value
P 2
(a) decomposition of calcium carbonate to calcium
Q 9
oxide
R 5
(b) polluted water is basic in nature hence it reacts with
calcium carbonate S 11
(c) polluted water is acidic in nature hence it reacts with 51. Which of the following correctly represents the solutions
calciumcarbonate in increasing order of their hydronium ion concentration?
(d) calcium carbonate dissolves in water to give calcium (a) P > Q > R > S (b) P > S > Q > R
hydroxide. (c) S < Q < R < P (d) S < P < Q < R
48. Calcium oxide can be reduced to calcium, by heating 52. High content of phosphate ion in river Yamuna may lead to:
with sodium metal. Which compound would act as an (a) decreased level of dissolved oxygen and increased
oxidizing agent in the above process? growth of algae
(a) sodium (b) sodium oxide (b) decreased level of dissolved oxygen and no effect of
(c) calcium (d) calcium oxide growth of algae
S-14 Science
(c) increased level of dissolved oxygen and increased Reason : Hydronium ions are responsible for corrosive
growth of algae action.
(d) decreased level of dissolved oxygen and decreased
growth of algae Match the Following
53. If a sample of water containing detergents is provided to
you, which of the following methods will you adopt to DIRECTIONS : Each question contains statements given in
neutralize it? two columns which have to be matched. Statements (A, B, C, D)
(a) Treating the water with baking soda in column I have to be matched with statements (p, q, r, s) in column II.
(b) Treating the water with vinegar
(c) Treating the water with caustic soda 62. Column II gives nature of acids and bases mention in
column I, match them correctly.
(d) Treating the water with washing soda
Column I Column II
(A) HCl (p) Strong acid
Assertion & Reason (B) HCN (q) Weak acid
(C) NaOH (r) Weak base
DIRECTIONS : Each of these questions contains an assertion
followed by reason. Read them carefully and answer the question (D) NH4OH (s) Strong base
on the basis of following options. You have to select the one that 63. Match the salts given in column I with the corresponding
best describes the two statements. acid and base given in column II.
(a) If both Assertion and Reason are correct and Reason is Column I Column II
the correct explanation of Assertion. (A) KNO3 (p) Nitric acid, Silver
(b) If both Assertion and Reason are correct, but Reason is hydroxide
not the correct explanation of Assertion. (B) AgNO3 (q) Hydrochloric
(c) If Assertion is correct but Reason is incorrect. acid, Magnesium
(d) If Assertion is incorrect but Reason is correct. hydroxide
(C) MgCl2 (r) Carbonic
54. Assertion : Aqueous solution of ammonium nitrate turns
blue litmus red. acid, Ammonium
hydroxide
Reason : Ammonium nitrate is salt of strong acid and (D) (NH4)2CO3 (s) Nitric acid,
strong base.
Potassium hydroxide
55. Assertion : All alkalis are bases but all bases are not
64. Column I Column II
alkali.
(A) NaHCO3 (p) Baking soda
Reason : Water soluble bases are alkali.
(B) NaOH (q) Alkaline
56. Assertion : Magnesium hydroxide is used as antacid. (C) KHSO4 (r) Acidic salt
Reason : Magnesium hydroxide is a strong base. (D) Ca(OH)2 (s) Bitter taste
57. Assertion : Dry HCl gas does not change the colour of
blue litmus paper to red. Fill in the Blanks
Reason : Dry HCl gas is strongly basic.
DIRECTIONS : Complete the following statements with an
58. Assertion : Sodium hydrogen carbonate is used in fire appropriate word / term to be filled in the blank space(s).
extinguisher.
65. Oxy acids contains ........ atoms in addition to hydrogen
Reason : Sodium hydrogen carbonate is a mild base.
atom.
59. Assertion : H2CO3 is a strong acid.
66. An acid that contains more than one acidic hydrogen
Reason : A strong acid dissociates completely or almost atom is called a .............. .
completely in water.
67. When an acid reacts with a metal, ............. gas is evolved
60. Assertion : Salts are the products of an acid-base reaction. and a corresponding ............. is formed.
Reason : Salt may be acidic or basic. 68. When an acid reacts with a metal carbonate or metal
61. Assertion : On adding H2SO4 to water the resulting hydrogen carbonate, it gives the corresponding salt,
aqueous solution get corrosive. .......... gas and ............... .
Acids, Bases and Salts S-15
69. .......................... is the fixed number of water molecules 76. Mixing concentrated acids or bases with water is a highly
chemically attached to each formula unit of a salt in its endothermic process.
crystalline form.
77. Acids and bases neutralise each other to form
70. ENO contains .......... and is ........... in nature. corresponding salts and water.
78. The colour of caustic soda turns pink when phenolphthalein
71. Anhydrous sodium carbonate is commonly known as
is added.
............... .
79. Hydrogen chloride gas turns the blue litmus red.
72. Soda–acid fire extinguisher contains a solution of sodium
hydrogen carbonate and ............... . 80. Sodium hydrogen carbonate is used in fire extinguisher.
73. An alkali reacts with ammonium salts to produce 81. Washing soda on strong heating gives sodium oxide and
corresponding salt, water and evolve .................. . carbon dioxide.
74. Zn(OH)2 is a ................. base. 82. Plaster of paris is obtained by heating gypsum at 373 K in
a kiln.
True / False 83. Bleaching powder is used for disinfecting drinking water.
84. Solution of sodium hydrogen carbonate is alkaline in
DIRECTIONS : Read the following statements and write your nature.
answer as true or false.

75. Acidic nature of a substance is due to the formation of


H+(aq) ions in solution.
S-16 Science

ANSWER KEY & SOLUTIONS


1. (c) Calcium chloride is good dehydrating agent so it is 25. (b) 26. (d) 27. (d) 28. (d)
used to absorb moisture from the hydrogen chloride
29. (c) Acids and bases go through a process of dissociation
gas.
when they are put into solution. They break apart
2. (c) The tooth paste commonly used is basic which help into positively and negatively charged particles.
in neutralisation of the extra acid formed during
30. (a) HCl is a strong acid.
tooth decay.
31. (c)
3. (b) Citric acid is an example of organic acid or edible
acid while HCl, H2SO4 and HNO3 are mineral acids. 32. (d) Because it can furnish H+ ions in solution.
4. (b) Lactic acid is present in sour milk.
33. (a) Element X can react with both acid and base. It
5. (c) Aqueous solution of A is basic while that of B is shows that element X is amphoteric in nature and is
acidic. Therefore A has pH greater than 7 and B has an electropositive element.
pH less than 7.
6. (b) H3PO3 is a dibasic acid. 34. (a) CH3COOK + H 2O 
→ KOH + CH3COOH
Strong base

 + – The solution will be basic in nature so it turns


H3PO3  Η + H 2 PO3 turmeric to red.
 H + + HPO32–
H 2 PO3  35. (b) Rock Salt (NaCl) < Baking Soda (NaHCO3) <
Washing Soda (Na2CO3) < Slaked lime (CaCO3)
7. (b) Chemical ‘A’ is calcium hydroxide (slaked lime).
36. (c) A neutral salt brings no change with blue
Ca(OH)2 + Na2CO3 → 2NaOH + CaCO3 ↓
litmus solution, red litmus solution and with
8. (d) The given solution is basic in nature when excess of phenolphthalein solution. An acidic salt turns blue
HCl is added, it becomes acidic. litmus to red and brings no change in red litmus
solution as well as in phenolphthalein solution.
9. (b) In acid, blue litmus changes to red and in basic
solution red litmus changes to blue. Hence blue Basic salt turns red litmus to blue and also turns
litmus first changes its color to red and then to blue. phenolphthalein solution pink.
10. (b) 6 - 12% acetic acid is known as vinegar.
Sample Solution With With With
11. (c) CuO + H2SO4 → CuSO4 + H2O blue red phenol-
12. (d) litmus litmus phthalein
solution solution solution
13. (a) Ca(OH)2 + CO2 → CaCO3 + H2O A Neutral salt No change No No change
14. (a) Antacids are weak bases which are given when a (NaCl) change
patient is suffering from acidity. These antacids B Acidic salt Turns red No No change
neutralise the acid and give relief to patient.
(NH4Cl) change
15. (b) 2NaOH + MgSO4 → Mg(OH)2 + Na2SO4 C Basic salt No change Turns Turns pink
16. (b) 17. (d) 18. (c) 19. (b) (CH3COONa) blue

20. (c) 21. (a) 22. (c)


37. (c) Aqua-regia is 3 part conc. HCl and 1 part conc.
23. (c) Carbonic acid is a weak and so it does not react with HNO3.
metal.
38. (a) Plaster of paris is calcium sulphate hemihydrate.
24. (a) A non metal oxide forms acid on treatment with
water. e.g., 39. (d) The red litmus solution turns blue in basic solution.
CO 2 + H 2O 
→ H 2CO3 The pH of basic solution is more than 7.
(Carbonic acid)

Acids, Bases and Salts S-17
40. (b) Since the gas produced turns lime water milky so 51. (c)
the gas is CO2. The egg-shell is made of CaCO3 52. (a)
which reacts with an acid (dil. HCl) to produce CO2.
53. (b)
CaCO3 + 2HCl → CaCl2 + H2O + CO2↑
54. (c) Ammonium nitrate is salt of strong acid and weak
41. (d) Since 10 mL of NaOH requires HCl = 8mL base.
8 55. (a) Bases generate hydroxide ions in water hence water
20 mL of NaOH will require HCl = × 20 mL soluble bases are called alkalis.
10
= 16 mL 56. (c) Magnesium hydroxide is a mild base and neutralise
42. (c) We use antacids for treating indigestion. the excess acid in the stomach.
57. (c) Dry HCl gas does not show acidic character in
43. (d) Zinc + Sulphuric acid →
absence of water. Therefore do not change the
Zinc sulphate + Hydrogen colour of blue litmus in dry condition.
Zn(s) + H2SO4(aq) → ZnSO4(aq) + H2(g) 58. (b) Sodium hydrogen carbonate react with acid present
(b) Magnesium + Hydrochloric acid → in fire extinguisher to produce carbon dioxide gas.

Magnesium chloride + Hydrogen 59. (d) H2CO3 (carbonic acid) is a weak acid.
Mg(s) + 2HCl(aq) → MgCl2(aq) + H2(g) 60. (b)
(c) Aluminium + Sulphuric acid → 61. (a) Because H2SO4 is a strong acid, it readily forms
hydronium ions when dissolved in water which are
Aluminium sulphate + Hydrogen
responsible for its corrosive action.
2Al(s) + 3H2SO4(aq) → Al2(SO4)3(aq) + 3H2(g)
62. A → (p); B → (q); C → (s); D → (r)
(d) dilute Hydrochloric acid + dilute sodium hydroxide
63. A → (s); B → (p); C → (q); D → (r)
→ sodiumchloride + water
64. A → (p, q, r); B → (q, s); C → (q, r); D → (q, s)
2HCl + 2NaOH → 2NaCHl + 2H2O
65. Oxygen 66. Polyprotic acid
44. (c) Calcined gypsum is CaSO4 ⋅ H2O
67. Hydrogen, salt 68.
Carbon dioxide, water
45. (d) 0-5 min
69. Water of crystallisation
46. (b) Gas is CO2 which is a important reactant in
70. Sodium hydrogen carbonate, basic
photosynthesis process.
71. Soda ash 72. Sulphuric acid
47. (c) polluted water is acidic in nature hence it reacts with
calcium carbonate 73. Ammonia 74. Diacidic
48. (d) calcium oxide 75. True 76. False 77. True 78. True
49. (a) 79. False 80.
True 81. False 82. True
50. (b) 83. True 84. True
MetalsBases
and and
3 Acids,
Salts
Non-Metals
8. Which of the following is not a characteristics of metal ?
Multiple Choice Questions (MCQs) (a) Malleable (b) Electropositive nature
(c) Ductile (d) None of these
DIRECTIONS : This section contains multiple choice
questions. Each question has four choices (a), (b), (c) and (d) 9. Zn + H2O (Steam) –––→ A + B, In the equation A and B
out of which only one is correct. are –
(a) Zn, H only (b) ZnH2 and O2
1. Which of the following metal is liquid at ordinary
temperature? (c) ZnO2 & O2 (d) ZnO & H2
(a) Aluminium (b) Mercury 10. Removal of impurities from ore is known as –
(c) Magnesium (d) Potassium (a) crushing and grinding (b) concentration of ore
2. The atomic number of an element Y is 16. The number of (c) calcination (d) roasting
electrons in Y2– ion will be:
11. Froth floatation method is used for the concentration of–
(a) 16 (b) 17
(a) oxide ores (b) sulphide ores
(c) 18 (d) 20
(c) sulphate ores (d) halide ores
3. Which of the following elements will form basic oxides?
(a) Barium (b) Aluminium 12. Heating of concentrated ore in absence of air for
(c) Carbon (d) Phosphorus conversion into oxide ore is known as –
(a) roasting (b) calcination
4. Which one of the following is not correct regarding the
electrolytic refining of copper? (c) reduction (d) none of these
(a) Basic Cu(OH)2 solution is used as cathode. 13. Pure gold is –
(b) Acidified CuSO4 solution is used as electrolyte. (a) 24 carats (b) 22 carats
(c) Impure Cu is taken as cathode (c) 20 carats (d) 18 carats
(d) Cu2+ ion gets collected at anode. 14. What is anode mud ?
5. Which of the following compound is covalent in nature? (a) Fan of anode
(a) Carbon tetrachloride (b) Ammonium chloride (b) Metal of anode
(c) Lithium chloride (d) Calcium chloride (c) Impurities collected at anode in electrolysis during
6.
A student by mistake used a wet gas jar to collect sulphur purification of metals
dioxide. Which one of the following tests of the gas is (d) All of these
likely to fail ?
(a) Odour 15. Which of the following pairs will give displacement
(b) Effect on acidified K2Cr2O7 solution reactions?
(c) Solubility test (a) ZnSO4 solution and aluminium metal
(d) None of these (b) MgCl2 solution and aluminium metal
7. Silicon is used in : (c) FeSO4 solution and silver metal
(a) solar energy devices (b) semiconductors (d) AgNO3 solution and copper metal.
(c) transistors (d) all of these
Metals and Non-Metals S-19
16. Which of the following is a chemical method for 26. The process to heat the ore in the presence of excess
preventing an iron frying pan from rusting? supply of air below its melting point is called –
(a) applying grease (a) roasting (b) calcination
(b) applying paint (c) smelting (d) liquation
(c) applying a coating of zinc 27. Graphite is a/an –
(d) all of the above (a) alloy (b) metal
17. An element reacts with oxygen to give a compound with (c) metalloid (d) non-metal
a high melting point. This compound is also soluble in 28. One of the constituents of amalgam is –
water. The element is likely to be – (a) aluminium (b) copper
(a) calcium (b) carbon (c) iron (d) mercury
(c) silicon (d) iron
29. Which of the following metal reacts with water/steam to
18. Aluminium does not oxidise readily in air because – produce oxide instead of hydroxide ?
(a) it is high in the electrochemical series. (a) Sodium (b) Potassium
(b) it is low in the electrochemical series. (c) Calcium (d) Magnesium
(c) the metal does not combine with oxygen. 30. The white phosphorus is stored
(d) the metal is covered with a layer of oxide which (a) in air (b) under water
does not rub off. (c) under kerosene (d) under CS2
19. The correct order of increasing chemical reactivity is – 31. Sodium is obtained by the electrolysis of –
(a) Zn < Fe < Mg < K (a) an aqueous solution of sodium chloride
(b) Fe < Mg < Zn < K (b) an aqueous solution of sodium hydroxide
(c) Fe < Mg < K < Zn (c) fused sodium chloride
(d) Fe < Zn < Mg < K (d) fused sodium sulphate

20. The least malleable is – 32. In the combined state, zinc is mainly found as –
(a) aluminium (b) silver (a) chloride (b) bromide
(c) gold (d) carbon (c) oxide (d) sulphide

21. The metal that reacts with cold water is – 33. Which of the following is incorrect?
(a) mercury (b) sodium (a) Chalcocite – Copper
(b) Magnetite – Iron
(c) zinc (d) tungsten
(c) Calamine – Aluminium
22. The only metal that is liquid at room temperature is – (d) Galena – Lead
(a) mercury (b) sodium
34. Among Mg, Cu, Fe, Zn the metal that does not produce
(c) zinc (d) tungsten hydrogen gas in reaction with hydrochloric acid is
23. The process of extraction of metal from its ores, is known (a) Cu (b) Zn
as – (c) Mg (d) Fe
(a) concentration (b) calcination 35. The major products of the following reaction,
(c) purification (d) metallurgy Heat
ZnS(s) + O2(g)  → ..........are
24. The compound from which metal is extracted economically (a) ZnO and SO2 (b) ZnSO4 and SO3
is – (c) ZnSO4 and SO2 (d) Zn and SO2
(a) slag (b) gangue
36. Choose the incorrect pair
(c) ore (d) mineral
(a) NO - Neutral oxide (b) Cl2O7 - Acidic oxide
25. The process to remove unwanted impurities from the ore (c) MgO - Basic Oxide (d) P4O10 - Basic oxide
is called –
37. Metal present in chloroplast is
(a) purification (b) calcination
(a) Iron (b) Copper
(c) bassemerisation (d) concentration
(c) Magnesium (d) Cobalt
S-20 Science
38. Magnesium ribbon is rubbed with sand paper before Choose the correct decreasing order of reactivity of these
making it to burn. The reason of rubbing the ribbon is to: metals amongst the following:
(a) remove moisture condensed over the surface of (a) M > L > H > K (b) K > M > H > L
ribbon. (c) M > K > L > H (d) L > H > K > M
(b) generate heat due to exothermic reaction. 47. ______ gas is evolved when Mn react with very dilute
(c) remove magnesium oxide formed over the surface HNO3
of magnesium.
(a) NO2 (b) H2
(d) mix silicon from sand paper (silicon dioxide) with
magnesium for lowering ignition temperature of the (c) N2O (d) NO
ribbon. 48. _____________ alloy is used for welding electrical wires
39. The element that cannot be used as a reducing agent is (a) Solder (b) Germen silver
(a) carbon (b) aluminium (c) Stainless steel (d) Gun metal
(c) sulphur (d) sodium 49. Aqueous solution of CsO2 is :
(a) Basic (b) Neutral
40. Al2O3 reacts with
(c) Acidic (d) Amphoteric
(a) only water (b) only acids
(c) only alkalis (d) both acids and alkalis 50. Which of the following will give displacement reactions ?
(a) NaCl solution and copper metal
41. Which of the following is an example of neutral oxide?
(a) Fe2O3 (b) Al2O3 (b) MgCl2 solution and aluminium metal
(c) C O (d) NO2 (c) FeSO4 solution and silver metal
42. A metal ‘M’ of moderate reactivity is present as its sulphide (d) AgNO3 solution and copper metal
‘X’. On heating in air, ‘X’ converts into its oxide ‘Y’ and
51. Which of the following methods is suitable for preventing
a gas evolves. On heating ‘Y’ and ‘X’ together, the metal
an iron frying pan from rusting ?
‘M’ is produced. ‘X’ and ‘Y’ respectively are
(a) Applying grease
(a) ‘X’ cuprous sulphide, ‘Y’ cuprous oxide (b) Applying paint
(b) ‘X’ cupric sulphide, ‘Y’ cupric oxide (c) Applying a coat of zinc
(c) ‘X’ sodium sulphide, ‘Y’ sodium oxide (d) All the above
(d) ‘X’ calcium sulphide, ‘Y’ calcium oxide 52. An element can react with oxygen to give a compound
43. Which of the following metals react with conc. sulphuric with high melting point. This compound is also water
soluble. The element is likely to be
acid but does not react with a solution of ferrous sulphate?
(a) Cu (b) Zn (a) Calcium (b) Carbon
(c) Fe (d) Mg (c) Silicon (d) Iron
53. Food cans are coated with tin and not with zinc because
44. The formula of phosphate salt of a metal is MPO4. The
formula of its nitrate salt will be (a) zinc is costlier than tin.
(a) MNO3 (b) M(NO3)2 (b) zinc has higher melting point than tin.
(c) zinc is more reactive than tin.
(c) M2(NO3)3 (d) M(NO3)3
(d) zinc is less reactive than tin.
45. Solder is an alloy of
(a) Pb and Sn (b) Zn and Pb
(c) Pb and Zn (d) Zn and Sn
DIRECTIONS : Study the given case/passage and answer the
46. The following observations are given for four metals:
following questions.
I. Metal H does not react with dilute HCl.
II. Metal K reacts with warm water. Case/Passage - 1
III. Metal L does not react with water but displaces Metals are electropositive elements. They can easily lose
metal H from its aqueous salt solution. electrons to form ions. Metals show distinguished physical
as well as chemical properties. Generally most of the metals
IV. Metal M reacts with cold water.
are ductile and malleable with exception such as mercury.
Metals and Non-Metals S-21
These properties make them valuable for commercial as well 61. The metal that reacts with cold water is -
as domestic uses. Reaction of a metal with water is one of (a) mercury (b) sodium
important chemical property. Metals like sodium and potassium (c) zinc (d) tungsten
reacts with cold water while magnesium reacts with hot water.
Metals like aluminium, zinc do not react with hot/cold water 62. Metal present in chloroplast is
but they easily react with steam. When a metal react with hot/ (a) Iron (b) Copper
cold water the products are metal hydroxide and hydrogen, (c) Magnesium (d) Cobalt
and when it react with steam, the product are metal oxide and 63. Which of the following metal(s) catch fire on reaction
hydrogen. Some metals like sodium, potassium react violently with water?
with water. (a) Sodium (b) Potassium
54. When zinc reacts with steam it produces: (c) Magnesium (d) both (a) and (b)
(a) Zn(OH)2 (b) ZnO
Case/Passage - 3
(c) O2 (d) ZnO2
The huge annual loss due to corrosion is a national waste and
55. Most ductile metal among the following is: should be minimized.
(a) Au (b) Ag Following are some methods which are helpful to prevent
(c) Cu (d) Al corrosion
56. During the reaction of calcium with water, pieces of
(i) Coating the iron surface with paint or oil or grease
metal start floating due to the formation of:
prevents moist oxygen from coming in contact with
(a) Ca(OH)2 (b) CO2
the metal and thus effectively prevents rusting of
(c) H2 (d) none of these iron.
57. Consider the reactions:
(ii) Galvanisation : Iron is blasted with fine sand to make the
Na(s) + H2O (l) → NaOH (aq) + H2 (g) ..........(i)
surface rough dipped in molten zinc and then cooled. A
Ca(s) + H2O (l) → Ca(OH)2(aq) + H2(g).........(ii) thin layer of zinc forms on the iron surface. Since zinc is
(a) Reaction (i) is endothermic reaction. more reactive than iron, it acts as a sacrificial metal and is
(b) Reaction (ii) is endothermic reaction. preferentially oxidised thus preventing oxidation of iron.
(c) Reaction (ii) is more exothermic than reaction (i).
(iii) Electroplating with tin, nickel or chromium also prevents
(d) Reaction (i) is more exothermic than reaction (ii).
rusting.
58. Metals can be converted into thin sheet by hammering.
(iv) Alloying (mixing iron in its molten state with other
This property is known as:
metals) prevents rusting. Stainless steel is an alloy of iron
(a) Ductility (b) Sonorous
with Cr or Ni.
(c) Malleability (d) Both (a) and (c)
64. The most durable metal plating on iron to protect against
corrosion is :
Case/Passage - 2
(a) nickel plating (b) copper plating
Elements can be classified as metals or non-metals on the
(c) tin plating (d) zinc plating
basis of their properties. The easiest way to start grouping
substances is by comparing their physical properties. Metals, 65. The most convenient method to protect the bottom of
in their pure state, have a shining surface. This property is ship made of iron is :
called metallic luster. metals are generally hard. The hardness
(a) coating it with red lead oxide.
varies from metal to metal. some metals are used for making
cooking vessels. (b) white tin plating.
(c) connecting it with Mg block.
59. Metals generally are
(a) reducing agents (d) connecting it with Pb block.
(b) oxidising agent 66. The best way to prevent rusting of iron is :
(c) both oxidising and reducing agents (a) making it cathode
(d) None of these (b) putting in saline water
60. The most abundant metal in the earth’s crust is - (c) both of these
(a) iron (b) copper (d) none of these
(c) aluminium (d) mercury
S-22 Science
Case/Passage - 4 73. Assertion: Sodium, potassium and magnesium are never
found as free elements in nature.
Some metals are chemically very reactive, whereas others
are less reactive or unreactive. On the basis of vigourness of Reason: Sodium, potassium and magnesium are reactive
reactions of various metals with oxygen, water and acids, as elements.
well as displacement reactions, the metals have been arranged
74. Assertion: Carbonate ores are changed into oxides by
in a group or series according to their chemical reactivity.
roasting process.
The arrangement of metals in a vertical column in the order
of decreasing reactivities is called reactivity series of metals Reason: It is easier to obtain a metal from its oxides.
(or activity series of metals). In reactivity series, the most
75. Assertion : Iron is found in the free state in nature.
reactive metal is placed at the top whereas the least reactive
metal is placed at the bottom. As we come down in the series, Reason : Iron is highly reactive element.
the chemical reactivity of metals decreases. Since the metals 76. Assertion : Different metals have different reactivities
placed at the bottom of the reactivity series (like silver and with water and dilute acids.
gold) are less reactive, so they are usually found in free state
(native state) in nature. Reason : Reactivity of a metal depends on its position in
the reactivity series.
67. When metal Z is added to dilute HCl solution, there is no
evolution of gas. Metal is : 77. Assertion : Zinc becomes dull in moist air.
(a) K (b) Na Reason : Zinc is coated by a thin film of its basic
(c) Ag (d) Zn carbonate in moist air.
68. Copper sulphate solution can be safely kept in a container 78. Assertion : Zinc is used in the galvanisation of iron.
made of : Reason : Its coating on iron articles increases their life by
(a) aluminium (b) lead protecting them from rusting.
(c) silver (d) zinc
79. Assertion : Nitrate ores are rarely available.
69. Metal always found in free state is :
Reason : Bond dissociation energy of nitrogen is very high.
(a) gold (b) silver
(c) copper (d) sodium Match the Following

Assertion & Reason DIRECTIONS : Each question contains statements given in


two columns which have to be matched. Statements (A, B, C, D)
in column I have to be matched with statements (p, q, r, s) in column II.
DIRECTIONS : Each of these questions contains an assertion
followed by reason. Read them carefully and answer the question 80. Column-I Column-II
on the basis of following options. You have to select the one that (A) CaO (p) Amphoteric oxide
best describes the two statements.
(B) Al2O3 (q)
Neutral oxide
(a) If both Assertion and Reason are correct and Reason is
(C) SO2 (r) Basic oxide
the correct explanation of Assertion.
(D) H2O (s) Acidic oxide
(b) If both Assertion and Reason are correct, but Reason is
not the correct explanation of Assertion. 81. Column-I Column-II
(c) If Assertion is correct but Reason is incorrect. (A) Iodine (p) Liquid metal
(d) If Assertion is incorrect but Reason is correct. (B) Diamond (q) Liquid non-metal
(C) Mercury (r) Lustrous
70. Assertion: Metals are electropositive elements.
(D) Bromine (s) Hardest substance
Reason: Metals form positive ions by losing electrons.
82. Column-I Column-II
71. Assertion: Electric wires are made up of copper.
(A) Good conductor of (p) Hydrogen
Reason: Non metals are bad conductor of electricity. electricity
72. Assertion: Potassium oxide is a basic oxide. (B) Food preservative (q) Copper
Reason: Solution of potassium oxide in water turns red (C) Allotrope of carbon (r) Nitrogen
litmus blue. (D) Manufacture of (s) Graphite
ammonia
Metals and Non-Metals S-23
83. Column-I Column - II 90. Stainless steel contains ..............., ............. and ..............
(Position of the Metal (Related Reduction 91. Froth floatation process is used for the concentration of
in the Activity Series) Process) .............. ores.
A. The bottom of the series (p) Electrolysis
92. The method of removing volatile matter from carbonate
B. The top of the series (q) Reduction by heat
ores is known as ...................... .
alone
C. The lower regions of (r) Found in native 93. Bronze is an alloy of copper and ................... .
the series state 94. The main ore of copper is ...................... .
D. The middle of the (s) Reduction using
95. In electrolytic refining, impure metal is used as ................. .
series carbon or some
other reducing agent
True / False
Fill in the Blanks DIRECTIONS : Read the following statements and write your
answer as true or false.
DIRECTIONS : Complete the following statements with an
appropriate word / term to be filled in the blank space(s). 96. Metals can form positive ions by losing electrons to non-
metals.
84. Metals combine with oxygen to form ......... oxides. 97. Different metals have same reactivities with water and
dilute acids.
85. Metals above hydrogen in the activity series can displace
............. from dilute acids. 98. A more reactive metal displaces a less reactive metal
from its salt solution.
86. The surface of some metals, such as iron, is corroded 99. Metals occur in nature only as free elements.
when they are exposed to moist air for a long period of
time. This phenomenon is known as ................... . 100. Aluminium is the most abundant metal in the earth’s
crust.
87. The best conductors of electricity are copper and ............ 101. Mercury and zinc are purified by liquation method.
88. Manganese and ............... react with very dilute nitric 102. The presence of carbon in pig iron makes it very soft and
acid to evolve hydrogen gas. malleable.

89. An alloy of any metal with mercury is called ...................... 103. Roasting is done for sulphide ores.
the electrical conductivity of an alloy is ........... than that 104. Reaction that takes place in aluminothermic process is
of pure metals.
also known as thermite reaction.
S-24 Science

ANSWER KEY & SOLUTIONS


1. (b) Mercury is liquid metal at ordinary temperature. 38. (c) When magnesium is exposed to air, a layer of oxide
is formed on its surface and it gets corroded. So, as
2. (c) The number of electrons is Y2– ion = 16 + 2 = 18
to remove the layer, magnesium ribbon is rubbed.
3. (a) Barium will form basic oxide. Aluminium will form
39. (c) Sulphur has a tendency to gain electrons. It is a non-
amphoteric oxide while carbon and phosphorus will
metal and cannot be used as reducing agent.
form acidic oxides.
40. (d) Al2O3 is an amphoteric oxide, so it can react with
4. (a) In the process of electrolytic refining, impure metal
both acids and alkalis, e.g.
is made anode and is connected to the positive
terminal of battery. Pure metal is made cathode and 41. (c) Fe2O3 is basic
is connected to the negative terminal of the battery.
Al2O3 is amphoteric
Cu2+ ions from the solution are deposited on the
cathode while Cu from impure anode dissolves into CO is neutral
the solution and the impurities settle down below
NO2 is acidic
anode as anode mud.
42. (a) Cu2S + 3 O2 → 2 Cu2O + 2SO2
5. (a) Carbon tetrachloride is a covalent compound.
2 Cu2O + Cu2S → 6 Cu + SO2
6. (d) 7. (d)
X = Cuprous sulphide, Y = Cuprous oxide
8. (d) All are characteristics of metal.
43. (a) Cu reacts with conc. H2SO4 on heating but does not
9. (d) 10. (b) 11. (b)
react with FeSO4 as it is less reactive than Fe.
12. (b) Calcination involves heating of the ore below its
44. (d) As M(NO3)3.
fusion temperature in absence of air.
45. (a) Solder is an alloy of Pb and Sn.
13. (a) 14. (c)
46. (c) Metals below hydrogen in a reactivity series does
15. (d) Copper is more reactive than silver hence displaces
not react with dilute HCl. Medium reactive metals
silver from silver nitrate solution.
reacts with warm water and highly reactive metals
16. (c) react with cold water.
17. (a) 2Ca + O2 → CaO (ionic compound) As per the given information H, K, L and M can be
identified as Cu, Mg, Pb and K/Na respectively. So
CaO + H2O → Ca(OH)2
their reactivity order will be M > K > L >H i.e.
18. (d) 19. (d) 20. (d) 21. (b) K > Mg > Pb > Cu.
22. (a) 23. (d) 24. (c) 25. (d) 47. (b) Mn + 2HNO3 → Mn (NO3)2 + H2.
26. (a) 27. (d) 28. (d) 29. (d) Hydrogen gas is evolved when Mn reacts with very
30. (b) 31. (c) 32. (d) 33. (c) dilute HNO3.

34. (a) Cu does not produce hydrogen gas on reaction with 48. (a) Solder alloy is used for welding electrical wires.
hydrochloric acid. Cu is present below hydrogen in The constituents of solder alloy are lead and tin.
reactivity series, i.e. it is less reactive than hydrogen. 49. (a) 2CsO2 + 2H2O → 2CsOH + H2O2 + O2

35. (a) 2ZnS(s) + 3O2(g)  → 2ZnO + 2SO2 CsO2 is the oxide of alkali metal. It is a basic oxide.
The sulphide ore is heated in presence of air to Due to formation of CsOH its aqueous solution is
produce its oxide form at a temperature below the basic.
melting point of the metal. The process is known as 50. (d) Copper will displace silver from silver nitrate
roasting. solution because copper lies above silver in
36. (d) Non-metals oxides are acidic in nature. reactivity series of metals.

37. (c) In chloroplast Mg is present.


Metals and Non-Metals S-25
51. (d) All the above methods are helpful in preventing iron 67. (c) Ag does not displace hydrogen from acids since it is
pan from rusting. below hydrogen in activity series.
52. (a) Calcium (Ca) combines with oxygen to form 68. (c) Since silver is less reactive than copper it does not
calcium oxide (CaO) which has a high melting point react with copper sulphate solution.
and dissolves in water to form Ca(OH)2.
69. (a) Gold is a noble metal.
53. (c) Zinc is more reactive than tin (zinc is above tin in
reactivity series) so it will react with organic acids 70. (a) Metals lose electrons to form positive ions therefore
(present in food) to form poisonous compounds. To they are known as electropositive elements.
avoid this food cans are coated with tin and not with
71. (b) Electric wires are made up of copper metal because
zinc.
metal are good conductor of electricity.
54. (b) Zn(s) + H2O(g) → ZnO(s) + H2(g)
72. (b) Potassium is a metal and hence, it forms basic oxide.

Steam It turns red litmus blue.
55. (a) Gold is most ductile metal. 73. (a) Sodium, potassium and magnesium are reactive
56. (c) Hydrogen gas is formed during the reaction which elements and found at the top of the reactivity series.
get stick to the surface of the metal and make them They do not occur in free state.
float.
74. (d) Carbonate ores are changed into oxides by
57. (d) Reaction (i) is more exothermic than reaction (ii). calcination process.
58. (c) Malleability. 75. (d)
59. (a) As metals are electropositive in nature and have 76. (a) The metals placed at the top of the series are most
tendency to lose electrons.
reactive.
60. (c)
77. (a) 78. (a)
61. (b) It is because of extremely high reactivity of sodium.
79. (a) The bond dissociation energy of N2 is very high due
62. (c) In chloroplast Mg is present. to presence of triple bond between two nitrogen
63. (d) Sodium and potassium both are extremely reactive atoms. Therefore, nitrate ores are rarely available.
and react with water so vigorously. The reaction
80. A → (r); B → (p); C → (s); D → (q)
is highly exothermic so the hydrogen evolved will
catch fire. 81. A → (r); B → (s); C → (p); D → (q)
64. (d) This is because zinc has higher oxidation potential 82. A → (q, s); B → (r); C → (s); D → (r, p)
than Ni, Cu and Sn. The process of coating of
iron surface with zinc is known as galvanization. 83. A → (r); B → (p); C → (q); D → (s)
Galvanized iron sheets maintain their lustrue due 84. basic 85. hydrogen
to the formation of protective layer of basic zinc
carbonate. 86. corrosion 87. silver 88. magnesium

65. (c) To protect the bottom of the ship it is connected with 89. amalgam, less 90. iron, chromium, carbon
more reactive metal than iron like magnesium. This 91. sulphide 92. calcination 93. tin
technique is called cathodic protection.
94. copper pyrite 95. anode
66. (a) Cathodic protection is best method to prevent iron
from rusting. In this method iron is made cathode by 96. True 97. False 98. True
application of external current.
99. False 100. True 101. False
Saline water is highly conducting and hence
accelerates the formation of rust. 102. False 103. True 104. True
4 Acids, Bases and
Life Processes
Salts
6. In the following sketch of stomatal apparatus, parts I, II,
Multiple Choice Questions (MCQs) III and IV were labelled differently by four students. The
correct labelling is:
DIRECTIONS : This section contains multiple choice
questions. Each question has four choices (a), (b), (c) and (d)
out of which only one is correct. I
II
1. A pacemaker is meant for III
IV
(a) transporting liver.
(b) transplanting heart.
(c) initiation of heart beats.
(a) I-guard cell, II-stoma, III-starch granule, IV-nucleus
(d) regulation of blood flow.
(b) I-cytoplasm II-nucleus, III-stoma, IV-chloroplast
2. Root cap has no role in water absorption because: (c) I-guard cell, II-starch, III-nucleus, IV-stoma
(a) It has no direct connection with the vascular system (d) I-cytoplasm, II-chloroplast, III-stoma, IV-nucleus
(b) It has no cells containing chloroplasts
7. The correct order of air reaching from atmosphere to the
(c) It has no root hairs lungs is through
(d) It has loosely arranged cells. (a) external nares, larynx, trachea and air sac.
3. Veins can be differentiated from arteries because the (b) larynx, trachea, air sac and external nares.
veins (c) trachea, air sac, external nares and larynx.
(a) have valves (d) air sac, trachea, larynx and external nares.
(b) have hard walls.
8. The rate at which oxygen moves from the alveoli of our
(c) have pure blood in them. lungs into our blood
(d) have thick walls. (a) depends on the difference in oxygen concentration
4. The function of the glomerulus and Bowman’s capsule of between the alveoli and the blood.
the nephron is to (b) depends on the color of the alveoli.
(a) reabsorb water into the blood. (c) depends on the availability of energy to transport
(b) eliminate ammonia from the body. gases across the membrane.
(c) reabsorb salts and amino acids. (d) none of the above
(d) filter the blood and collect the filtrate. 9. Major function of contractile vacuole is
5. An advantage of excreting nitrogenous wastes in the (a) Excretion (b) Circulation
form of uric acid is that – (c) Osmoregulation (d) All of these
(a) It is less toxic and reduces water loss and the 10. Heart beat can be initiated by
subsequent need for water.
(a) Sino-auricular node
(b) The formation of uric acid requires a great deal of
(b) Atrio-ventricular node
energy.
(c) Sodium ion
(c) Uric acid is the first metabolic breakdown products
(d) Purkinje’s fibres
of acids.
(d) Uric acid may be excreted through the lungs.
Life Processes S-27
11. Digestion of food in human starts from 21. In respiration, air passes through
(a) Duodenum (b) Small intestine (a) Pharynx → nasal cavity → larynx → trachea →
(c) Mouth (d) Large intestine bronchi → bronchioles
12. Large intestine in man mainly carries out (b) Nasal cavity → pharynx → larynx → trachea →
(a) absorption bronchi → bronchioles
(b) assimilation (c) Larynx → nasal cavity → pharynx → trachea
(c) digestion of fats (d) Larynx → pharynx → trachea → lungs
(d) digestion of carbohydrates 22. During deficiency of oxygen in tissues of human beings,
13. In Amoeba the digestion is intracellular because pyruvic acid is converted into lactic acid in the
(a) Amoeba is unicellular (a) cytoplasm (b) chloroplast
(b) Amoeba is multicellular (c) mitochondria (d) golgi body
(c) Amoeba is found in a pond
(d) Amoeba is a microscopic animal 23. Choose the function of the pancreatic juice from the
following?
14. The process of transpiration in plants helps in:
(a) Trypsin digests proteins and lipase digests
(a) Opening of stomata
carbohydrates.
(b) Absorption of CO2 from atmosphere
(c) Upward conduction of water and minerals (b) Trypsin digests emulsified fats and lipase proteins.
(d) Absorption of O2 from atmosphere. (c) Trypsin and lipase digest fats.
(d) Trypsin digests proteins and lipase emulsify fats.
15. Erythropoesis may be stimulated by the deficiency of
(a) Iron (b) Oxygen 24. Choose the correct statement that describes arteries.
(c) Protein (d) None of these (a) They have thick elastic walls, blood flows under
high pressure, collect blood from different organs
16. In the cardiac cycle, diastole is –
and bring it back to the heart.
(a) The number of heart beats per minute
(b) They have thin walls with valves inside, blood flows
(b) The relaxation period after contraction of the heart
under low pressure and carry blood away from the
(c) The forceful pumping action of the heart
heart to various organs of the body.
(d) The contraction period after relaxation of the heart.
(c) They have thick elastic walls, blood flows under
17. The breakdown of pyruvate to give carbon dioxide, water low pressure, carry blood from the heart to various
and energy takes place in organs of the body.
(a) cytoplasm (b) mitochondria (d) They have thick elastic walls without valves inside,
(c) chloroplast (d) nucleus blood flows under high pressure and carry blood
18. What is the term used when vessels open and let more away from the heart to different parts of the body.
blood through? Pseudopodia
(a) Vasoconstriction
(b) Vasodilatation 25. Food particle

(c) Increased permeability Food vacuole


(d) None of these
Which activity is illustrated in the diagram of an Amoeba
19. The chief function of lymph nodes in mammalian body is to shown above?
(a) produce RBCs (a) Ingestion (b) Digestion
(b) collect and destroy pathogens (c) Egestion (d) Assimilation
(c) produce a hormone 26. From the given picture of the
(d) destroy the old and worn out red blood cells
digestive system, identify the part
20. Select the correct statement? labelled as gastric gland.
A

(a) Heterotrophs do not synthesise their own food.


B
(a) A C
(b) Heterotrophs utilise solar energy for photosynthesis. (b) B D
(c) Heterotrophs synthesise their own food. (c) C
(d) Heterotrophs are capable of converting carbon (d) D
dioxide and water into carbohydrates.
S-28 Science
27. The diagram below represents a group of organs in the (d) Brunner’s glands are present in the submucosa of
human body. Urine leaves the urinary bladder by passing stomach and secrete pepsinogen.
through this structure labelled 34. Human urine is usually acidic because
A (a) excreted plasma proteins are acidic.
(b) potassium and sodium exchange generates acidity.
B (c) hydrogen ions are actively secreted into the filtrate.
(d) the sodium transporter exchanges one hydrogen ion
C for each sodium ion in peritubular capillaries.
D 35. Which one of the following animals has two separate
(a) A (b) B circulatory pathways?
(c) C (d) D (a) Lizard (b) Whale
28. Given alongside is a sketch of a leaf partially covered with (c) Shark (d) Frog
black paper and which is to be 36. Cow has a special stomach as compared to that of a lion
used in the experiment to show in order to
that light is compulsory for the (a) absorb food in better manner.
process of photosynthesis. At the
(b) digest cellulose present in the food.
end of the experiment, which one III
I (c) assimilate food in a better way.
of the leaf parts labelled I, II and
III will become black when (d) absorb large amount of water.
II
dipped in iodine solution? 37. Which of the following is not an enzyme?
(a) I only (b) II only (a) Lipase (b) Amylase
(c) I and III (d) II and III (c) Trypsin (d) Bilirubin
29. The phenomenon of normal breathing in a human being 38. Pancreatic juice contains more than one enzyme. Which
comprises. among the following combination is correct?
(a) an active inspiratory and a passive expiratory phase. (a) Pepsin and Lipase
(b) a passive inspiratory and an active expiratory phase. (b) Amylase and Pepsin
(c) both active inspiratory and expiratory phases. (c) Pepsin and Trypsin
(d) both passive inspiratory and expiratory phases. (d) Trypsin and Lipase
30. Filteration unit of kidney is
39. Observe the experimental sets [A] & [B].
(a) ureter (b) urethra
(c) neuron (d) nephron Observe the test tube A & B. From the list given below,
choose the combination of responses of shoot and root
31. A column of water within xylem vessels of tall trees does that are observed in B.
not break under its weight because of:
(a) Tensile strength of water (A) (B)
(b) Lignification of xylem vessels Test tube
(c) Positive root pressure Water
(d) Dissolved sugars in water
32. Roots play insignificant role in absorption of water in: Cotton
Plug
(a) Pistia (b) Pea
Plant
(c) Wheat (d) Sunflower
33. Which of the following statements is not correct? Day 5
Day 1
(a) Goblet cells are present in the mucosa of intestine
and secrete mucus.
(b) Oxyntic cells are present in the mucosa of stomach (a) Positive phototropism and positive geotropism
and secrete HCl. (b) Negative phototropism and positive geotropism
(c) Acini are present in the pancreas and secrete (c) Positive phototropism and negative geotropism
carboxypeptidase. (d) Only negative phototropism
Life Processes S-29
40. Which one of the following organisms respires through B. Adrenaline was secreted in the blood by the adrenal
the skin? glands.
(a) Blue whale (b) Salamander C. Heart beat becomes faster and pumps more blood so
(c) Platypus (d) Peacock that muscles get more oxygen.
41. The first enzyme that the food encounters in human D. Adrenocorticotropic hormone is secreted in the
digestive system is : blood and blood flows more towards the vital organs.
(a) Pepsin (b) Trypsin Select the correct combination of options given below:
(c) Chymotrypsin (d) Amylase (a) A and B (b) A and C
(c) B and C (d) C and D
42. Red blood corpuscles are formed in
(a) Liver (b) Kidneys 49. Removal of the pancreas impairs the breakdown of
(c) Small intestine (d) Bone marrow (a) lipids and carbohydrates only
(b) lipids and proteins only
43. The Excretory units of Annelids are:
(c) lipids, proteins and carbohydrates
(a) Uniferous tubule (b) Flame cells
(d) proteins and carbohydrates only
(c) Nephridia (d) Malpighian tubule
44. Open circulatory system is found in: 50. Microscopic examination of a blood smear reveals an
(a) Prawn (b) Snakes abnormal increase in the number of granular cells with
multiple nuclear lobes. Which one of the following cell
(c) Fish (d) Man
Types has increased in number?
45. Haemoglobin is dissolved in Plasma of blood in: (a) Lymphocytes (b) Monocytes
(a) Earthworm (b) Roundworm (c) Neutrophils (d) Thrombocytes
(c) Tapeworm (d) Insects
51. Which one of the following metabolic conversions
46. Which one of the following organisms has a cellular requires oxygen?
respiratory pigment dissolved in plasma and is also a
(a) Glucose to pyruvate
predaceous carnivore and shows matriphagy?
(b) Glucose to CO2 and ethanol
(a) Scorpion (b) Cockroach
(c) Glucose to lactate
(c) Earthworm (d) Sea cucumber
(d) Glucose to CO2 and H2O
47. Glucose is the prime source of energy in our body.
52. Which one of the following organs is NOT a site for the
However, it is stored in the form of glycogen in the
production of white blood cells?
muscle and liver of animals and in the form of starch in
(a) Bone marrow (b) Kidney
plants. As a result, everytime a cell requires glucose, it
must hydrolyze glycogen which is an energy consuming (c) Liver (d) Spleen
process. Why does the cell store glycogen instead of 53. Which of the following process occur only in animals?
glucose in free form? (a) Respiration (b) Nutrition
(a) Glycogen is more compact and more hydrophilic. (c) Nervous control (d) Hormonal control
(b) Storage of glucose in free form will consume more ATP.
54. Tricuspid valve is present in
(c) Glucose in the free form creates more osmotic pressure.
(a) Right atria and right ventricle
(d) Glucose is highly reactive molecule hence storing in
(b) Left atria and left ventricle
the free form can result in unwanted reactions in the
cells. (c) Wall of atrium
(d) Wall of vetricle
48. A squirrel was eating a fruit on the ground. Suddenly, it
was attacked by a dog. The squirrel rushed to the tree 55. Root pressure is effective way transporting water in
immediately and saved itself from the dangerous attack. xylem. This pressure is generated
What immediate changes are most likely to have taken (a) In bright sunlight
place in the body of the squirrel? (b) During night
A. Blood flows to the stomach for rapid digestion. (c) At very low temperature
(d) In high trees
S-30 Science
Lactic acid production has occurred in the athlete while
running in the 400 m race. Which of the following
processes explains this event?
DIRECTIONS : Study the given case/passage and answer the
following questions. Which of the following processes explains this event?
(a) Aerobic respiration (b) Anaerobic respiration
Case/Passage - 1
(c) Fermentation (d) Breathing
There is a pair of bean-shaped organs P in the human body
towards the back, just above the waist. A waste product Q 62. Study the graph below that represents the amount of
formed by the decomposition of unused proteins in the liver energy supplied with respect to the time while an athlete
is brought into organ P through blood by an artery R. The is running at full speed.
numerous tiny filters S present in organ P clean the dirty blood Choose the correct combination of plots and justification
by removing the waste product Q. The clean blood goes into provided in the following table.
circulation through a vein T. The waste substance Q, other
waste salts, and excess water form a yellowish liquid U which Plot A Plot B Justification
goes from organ P into a bag-like structure V through two (a) Aerobic Anaerobic Amount of energy is
tubes W. This liquid is then thrown out of the body through a low and inconsistent
tube X. in aerobic and high
in anaerobic
56. What is (i) organ P, and (ii) waste substance Q?
(b) Aerobic Anaerobic Amount of energy is
57. Name (i) artery R, and (ii) vein T.
high and consistent
58. What are tiny filters S known as? in aerobic and low
59. Name (i) liquid U (ii) structure V (iii) tubes W, and (iv) in anaerobic
tube X. (c) Anaerobic Aerobic Amount of energy is
Case/Passage - 2 high and consistent
in aerobic and low
All living cells require energy for various activities. This
in anaerobic
energy is available by the breakdown of simple carbohydrates
either using oxygen or without using oxygen. (d) Anaerobic Aerobic Amount of energy is
high and inconsistent
60. Energy in the case of higher plants and animals is obtained in anaerobic and low
by in aerobic
(a) Breathing (b) Tissue respiration
63. 
The characteristic processes observed in anaerobic
(c) Organ respiration (d) Digestion of food
respiration are:
61. The graph below represents the blood lactic acid (i) presence of oxygen
concentration of an athlete during a race of 400 m and (ii) release of carbon dioxide
shows a peak at point D.
(iii) release of energy
Lactic acid production has occurred in the athlete while (iv) release of lactic acid
running in the 400 m race.
(a) (i), (ii) only (b) (i), (ii), (iii) only
Respiration in athletics (c) (ii), iii), iv) only (d) (iv) only
The blood of an athlete was tested before, during and
64. Study the table below and select the row that has the
after a 400m race:
incorrect information.
Blood lactic acid concentration

12 D
10
Aerobic Anaerobic
8 Location
(a) Cytoplasm Mitochondria
(mmol/litre)

E
6
B
C
End Porduct CO2 and H2O
(b) Ethanol and
4
CO2
2
A
0 Amount of ATP High
(c) Low
0 30 60
Time in seconds Oxygen Needed
(d) Not needed
Life Processes S-31
72.
Column I Column II
Assertion & Reason
(A) Regulation metabolic (p) The removal of waste
DIRECTIONS : Each of these questions contains an assertion from an organism.
followed by reason. Read them carefully and answer the question (B) Reproduction (q) The chemical process
on the basis of following options. You have to select the one that of oxidizing organic
best describes the two statements. molecules to release
energy.
(a) If both Assertion and Reason are correct and Reason is
the correct explanation of Assertion. (C) Respiration (r) The replication of an
organism.
(b) If both Assertion and Reason are correct, but Reason is
(D) Excretion (s) The control and
not the correct explanation of Assertion.
coordination of
(c) If Assertion is correct but Reason is incorrect.
chemical processes
(d) If Assertion is incorrect but Reason is correct. within the organism.
65. Assertion: Blood of insects is colourless. 73. Column I Column II
Reason: The blood of insect does not play any role in (A) Stomach (p) The structure is the
transport of oxygen. site where the
chemical breakdown
66. Assertion: Blood pressure is arterial blood pressure.
of proteins first occurs.
Reason: It is measured by sphygmomanometer. (B) Large intestine (q) This organ absorbs
67. Assertion: Chloroplast help in photosynthesis. most of the water from
the undigested food.
Reason: Mitochondria have enzymes for dark reaction.
(C) Small intestine (r) This organ is the
68. Assertion: During physiology of excretion, deamination section of the
does not take place in liver. alimentary canal
Reason: Deamination is a process to make use of excess of where most of the food
amino acids which can not be incorporated into protoplasm. is absorbed into the
blood.
69. Assertion: Photorespiration decreases net photosynthesis. (D) Liver (s) This organ secretes
Reason: Rate of respiration in dark and light is almost the chemical bile,
same in all plants. which is used to
emulsify fats.
Match the Following
Fill in the Blanks
DIRECTIONS : Each question contains statements given in
two columns which have to be matched. Statements (A, B, C, D) DIRECTIONS : Complete the following statements with an
in column I have to be matched with statements (p, q, r, s) in column II. appropriate word / term to be filled in the blank space(s).

70. Column I Column II 74. Ninety percent of the water lost by the plants during
(A) Autotrophic (p) Leech nutrition transpiration is through the ................ of the leaf.
(B) Heterotrophic nutrition (q) Paramaecium 75. The semi-liquid mixture of partially digested food found
(C) Parasitic nutrition (r) Deer in the stomach is called ....................... .
(D) Digestion in food (s) Green plant. 76. The ......................... prevents the entry of food into the
vaculoes respiratory tract.
71. Column I Column II 77. Second heart sound heard as ............. is due to closure of
(Animal) (Respiratory Organ) .............. valves at the beginning of ventricular diastole.
(A) Fish (p) Trachea 78. Kidney eliminate the excretory waste materials as their
(B) Birds (q) Gills aqueous solution, called .................. .
(C) Aquatic (r) Lungs 79. Energy rich compound generated during photosynthesis
(D) Earthworm (s) Moist cuticle is ........................ .
S-32 Science
80. Pressure in the arteries during ventricular relaxation is 86. A complete digestive tract consists of an oral and an anal
called ........................ pressure. opening.
81. Diffusion is insufficient to meet .................... requirement 87. Only the multicellular organisms require transporting
of multicellular organisms like humans. mechanisms.
88. Humans have an open circulatory system.
True / False
89. Living organisms must maintain a constant internal
environment.
DIRECTIONS : Read the following statements and write your
answer as true or false. 90. In humans, the alveoli are the functioning units of
external respiration.
82. Translocation is the transportation of the products of
91. Circulatory system also performs the function of homeostasis.
photosynthesis.
92. Essential amino acids cannot be synthesized in human body.
83. In a general, digestion is simply hydrolysis of complex
polymers to monomers. 93. Generally gravitational water is utilized by the plants.
84. The exchange of nutrients and waste products between 94. In photosynthesis, carbon dioxide is given out by
the blood and cells occurs within the arteries. diffusion process.
85. Trypsin digests proteins into amino acids. 95. Bowman’s capsule is found in heart.
Life Processes S-33

ANSWER KEY & SOLUTIONS


1. (c) 22. (a) In the absence of oxygen, pyruvic acid is converted
into lactic acid in the cytoplasm of the muscle cell.
2. (c) Root cap is devoid of root hairs which are
instrumental in water absorption by increasing the 23. (d) 24. (d) 25. (a) 26. (b)
surface area to speed up osmosis and thus root cap is
27. (d) 28. (c)
not involved in the water absorption.
29. (a) During inspiration, muscles of ribs and diaphragm
3. (a) 4. (d) 5. (a) 6. (b)
contracts. Hence, it is an active process. During
7. (a) The correct pathway of air: nasal cavities (or oral expiration, muscles of ribs and diaphragm relax.
cavity) > pharynx > trachea > primary bronchi (right Hence, it is a passive process.
& left) > secondary bronchi > tertiary bronchi >
30. (d) Nephron filter blood in kidney.
bronchioles > alveoli (site of gas exchange).
31. (a) Due to tensile strength of water, a column of water
8. (a) During inhalation, there is a greater concentration
within xylem vessels of tall trees does not break
of oxygen in the alveoli than in the blood of the
under its weight.
pulmonary capillaries, so oxygen diffuses from the
alveoli into the blood across the capillaries. 32. (a) Pistia is a hydrophyte plant where absorption of
water by root is not important.
9. (c) 10. (a)
33. (d) Duodenum contains Brunner’s glands which secrete
11. (c) Digestion begins in the mouth with the secretion of
mucus and digestive juices.
saliva and its digestive enzymes.
34. (c) Urine has acidic nature because hydrogen ions (H+)
12. (a) The large intestine, or large bowel, is the last part
are components of an acid which are secreted into
of the digestive system in vertebrate animals. Its
the filtrate.
function is to absorb water from the remaining
indigestible food matter, and then to pass the useless 35. (b) Whale is a mammal and in mammals, two
waste material from the body. separate circulatory pathways are found - systemic
circulation and pulmonary circulation. Oxygenated
13. (a)
and deoxygenated bloods received by the left and
14. (c) Transpiration is an essential phenomenon. It’s right atria respectively pass on to the left and right
pulling action helps in absorption and transportation ventricles. Thus, oxygenated and deoxygenated
of water in the plant. It also supplies water for bloods are not mixed. This is referred to as double
photosynthesis. circulation.
15. (b) 36. (b)
16. (b) Cardiac diastole is the period of the cardiac cycle 37. (d) Bilirubin is yellow compound that occurs in the
when, after contraction, the heart relaxes and catabolic pathway which breaks down hence in
expands while refilling with blood returning from vertebrates it is not an enzyme. Other options i.e.,
the circulatory system. lipase, amylase, and trypsin are lipid digesting, starch
digesting and endopeptidase enzymes respectively.
17. (b)
38. (d) Pancreatic juice contains Pancreatic proteases (such
18. (b) Vasodilation is the widening of your blood vessels.
as trypsin and chymotrypsin), Pancreatic amylase
It happens when smooth muscles found in the
and Pancreatic lipase.
walls of arteries or large veins relax, allowing the
blood vessels to become more open. this leads to an 39. (a)
increase in blood flow through your blood vessels as
40. (b) Salamanders are a group of amphibians typically
well as a decrease in blood pressure.
characterised by a lizard-like appearance with
19. (b) 20. (a) 21. (b)
S-34 Science
slender bodies, blunt snouts, short limbs and a tail. 50. (c) Granulocytes and agranulocytes are the two types
Salamanders breath through their skin and the thin of white blood cells found in blood. Eosinophils,
membranes in the mouth and throat. neutrophils and basophils are the granulocytes.
Monocytes and lymphocytes are the agranulocytes. If
41. (d) The first enzyme that the food encounters in the
the microscopic examination of a blood smear reveals
digestive system is amylase. Digestion begins in the
an abnomal increase in the number of granular cells
mouth with the secretion of saliva and its digestive
with multiple nuclear lobes, so out of the given blood
enzymes. Saliva contains the digestive enzyme
cells, neutrophils has increased in number.
amylase, which works on carbohydrate, starch
present in breads, potatoes or pasta to help break 51. (d) Aerobic respiration uses oxygen to break down
them down into simple sugars. glucose, amino acids and fatty acids and is the main
way the body generates adenosine triphosphate (ATP),
42. (d) Red blood cells are formed in the red bone marrow
which supplies energy to the muscles.The products of
of bones. Stem cells in the red bone marrow called
this process are carbon dioxide and water.
hemocytoblasts give rise to all of the formed
elements in blood. 52. (b) In the human adult, the bone marrow produces all of
the red blood cells, 60-70 percent of the white cells
43. (c) The annelid excretory system is made up of long
(i.e., the granulocytes), and all of the platelets. The
tubular organs called nephridia. Many species have
reticuloendothelial tissues of the spleen, liver, lymph
a pair of nephridia in each segment.
nodes, and other organs produce the monocytes (4-8
44. (a) Open circulatory system is primarily found in percent of the white cells).
invertebrates, in which the blood flows freely
53. (c) Nervous control occurs only in animals.
through cavities and there are no vessels to conduct
the blood. This type of system is found in animals 54. (a) Tricuspid valve is present between right atria and
such as insects and some mollusks (snails, clams). right ventricle.
45. (a) Haemoglobin is found dissolved in the plasma 55. (b) During night, root pressure is effectively involved in
of earthworm because they don’t have proper transport of water through xylem. During day, due to
respiratory system. It is dissolved in plasma for the the opening of stomata, transpiration pull becomes the
diffusion of gases and other materials. major factor for transporting water in xylem.
46. (a) In scorpion, the respiratory pigment is dissolved in 56. (a) (i) Kidneys (ii) Urea
plasma and it is also a predaceous carnivore that
57. (b) (i) Renal artery (ii) Renal vein
shows matriphagy, (a process in which an organism
feed on its own mother). 58. (c) Nephrons

47. (c) Glycogen is insoluble thus, storing it as glycogen 59. (d) (i) Urine (ii) Bladder
will not upset the osmotic pressure rather than (iii) Ureters (iv) Urethra
glucose which is soluble in water. And if cell store
it as glucose, it will disturb the osmotic pressure 60. (b) Tissue respiration
(hypertonic) that will cause the cell to lyse. 61. (b) Anaerobic respiration
48. (3) In this condition, adrenaline is secreated by adrenal 62. (b)
gland into the blood stream which increases the
Aerobic Anaerobic Amount of energy is high
heart rate, redistributing blood to the muscles and
and consistent in aerobic
altering the blood metabolism, so as to maximise
and low in anaerobic
blood glucose levels primarily for the brain.
49. (c) The pancreas is a glandular organ. It is the part of 63. (c) (ii), (iii), (iv) only
the digestive syste, located in the abdomen and 64. (a) Location Aerobic-Cyloplasm and Anaerobic-
produces insulin and other important enzymes and Mitochondria
hormones that help break down foods. The enzymes
65. (b) The blood of an insect functions differently than the
include trypsin and chymotrypsin to digest proteins,
blood of a human. Insect blood, however, does not
amylase to break down carbohydrates and lipase, to
carry gases and has no haemoglobin which gives red
break down fats into fatty acids and cholesterol.
colour to the blood.
Life Processes S-35
66. (b) Blood pressure, sometimes called arterial blood 72. (A) → (s), (B) → (r), (C) → (q), (D) → (p)
pressure, is the pressure exerted by circulating blood
73. (A) → (p), (B) → (q), (C) → (r), (D) → (s)
upon the walls of blood vessels. Blood pressure is
measured by sphygmomanometer. 74. stomata 75. chyme

67. (c) Dark reaction occurs in the stroma region of the 76. epiglottis 77.
Dup/Dubb, semilunar
chloroplast and mitochondria is involved in the 78. urine 79.
ATP
synthesis of ATP.
80. diastolic 81. oxygen
68. (d) Deamination takes place in liver during excretion
to make excess of amino acids which can not be 82. True 83. True 84. False 85. True
incorporated into protoplasm. 86. True 87. False 88. False 89. True
69. (c) 90. True 91. True 92. True 93. False
70. (A) → (s), (B) → (r), (C) → (p), (D) → (q) 94. False 95. False
71. (A) → (q), (B) → (r), (C) → (p), (D) → (s)
Light-Reflection
5 Acids,
Salts
Bases and
and Refraction
6. The image formed by a convex mirror
Multiple Choice Questions (MCQs) (a) is always real
(b) is always virtual
DIRECTIONS : This section contains multiple choice (c) cannot say
questions. Each question has four choices (a), (b), (c) and (d) (d) None of these
out of which only one is correct.
7. In case of erect object having inverted image, linear
1. An object is at a distance of 0.5 m in front of a plane magnification is :
mirror. Distance between the object and image is (a) positive (b) negative
(a) 0.5 m (b) 1 m (c) zero (d) no definite sign.

(c) 0.25 m (d) 1.5 m 8. If object lies symmetrically and number of images
formed are 9, therefore two plane mirrors are kept at an
2. Number of images formed when two plane mirrors are angle of :
inclined at an angle 90° is (a) 72° (b) 40°
(a) 3 (b) 2 (c) 36° (d) 50°
(c) 4 (d) 5 9. Reciprocal of focal length of a lens gives the
3. Which one of the following statements is not correct? (a) power (b) radius
(a) A convex mirror is often used as driving rear-view mirror. (c) magnification (d) none of these
(b) A convex mirror is often used as a shaving mirror. 10. Magnification of a lens is given by
(c) A concave mirror is often used in a search light or a torch. image height 1
(a) object height (b)
(d) A concave mirror is often used as the reflector Radius
behind lamp in a projector
1 1
4. The relation, R = 2f holds true for : (c) (d)
focal length image distance
(a) concave mirrors only
11. A man having height 2.5 m. He oberves image of 1m
(b) convex mirrors only height erect, then mirror used is
(c) all spherical mirrors (a) concave (b) convex
(d) lens as well as for all spherical mirrors. (c) plane (d) None of these
5. A magnification greater than unity indicates : 12. Where should an object be placed in front of a convex lens
(a) real image to get a real image of the size of the object?
(b) size of the image is smaller than that of object (a) At the principal focus of the lens
(c) size of the object is smaller than that of image (b) At twice the focal length
(c) At infinity
(d) size of object is equal to that of image
(d) Between the optical centre of the lens and its
principal focus.
Light-Reflection and Refraction S-37
13. Find the angle of incidence and angle of reflection from 20. An object is placed 60 cm in front of a convex mirror.
the diagram. The virtual image formed by the mirror is located 30 cm
behind the mirror. What is the object’s magnification
(a) + 2 (b) –2
(c) + 0.5 (d) – 0.5
21. Light rays A and B fall on optical component X and come
Mirror out as C and D.
surface 35° C
A
X
(a) 45°, 40° (b) 55°, 55° B
(c) 60°, 60° (d) 30°, 30° D
14. A spherical mirror and a thin spherical lens have each a The optical component is a
focal length of –15 cm. The mirror and the lens are likely to (a) concave lens (b) convex lens
be (c) convex mirror (d) prism
(a) both concave.
22. An object is placed 20.0 cm in front of a concave mirror
(b) both convex. whose focal length is 25.0 cm. What is the magnification
(c) the mirror is concave and the lens is convex. of the object?
(d) the mirror is convex, but the lens is concave. (a) + 5.0 (b) – 5.0
(c) + 0.20 (d) – 0.20
15. Which of the following lenses would you prefer to use
while reading small letters found in a dictionary? 23. An object is placed at the radius of curvature of a concave
(a) A convex lens of focal length 50 cm. spherical mirror. The image formed by the mirror is
(b) A concave lens of focal length 50 cm. (a) located at the focal point of the mirror.
(c) A convex lens of focal length 5 cm. (b) located between the focal point and the radius of
(d) A concave lens of focal length 5 cm. curvature of the mirror.
(c) located at the center of curvature of the mirror.
16. An object is situated at a distance of f/2 from a convex
(d) located out beyond the center of curvature of the
lens of focal length f. Distance of image will be – mirror.
(a) + (f/2) (b) + (f/3)
24. If the refractive indices for water and diamond relative to
(c) + (f/4) (d) – f
air are 1.33 and 2.4 respectively, then the refractive index
17. An object is placed 60 cm in front of a concave mirror. of diamond relative to water is –
The real image formed by the mirror is located 30 cm in (a) 5.5 (b) 1.80
front of the mirror. What is the object’s magnification? (c) 3.19 (d) None of these
(a) + 2 (b) –2
25. There is an equiconvex lens of focal length of 20 cm. If
(c) + 0.5 (d) –0.5
the lens is cut into two equal parts perpendicular to the
18. Two plane mirrors are set at right angle and a flower is principle axis, the focal lengths of each part will be
placed in between the mirrors. The number of images of (a) 20 cm (b) 10 cm
the flower which will be seen is (c) 40 cm (d) 15 cm
(a) One (b) Two 26. An object is placed 20.0 cm in front of a concave mirror
(c) Three (d) Four whose focal length is 25.0 cm. Where is the image
19. A man is 6.0 ft tall. What is the smallest size plane mirror located?
he can use to see his entire image (a) 1.0 × 102 cm in front of the mirror
(a) 3.0 ft (b) 6.0 ft (b) 1.0 × 102 cm behind the mirror
(c) 12 ft (d) 24 ft (c) 5.0 × 101 cm in front of the mirror
(d) 5.0 × 101 cm behind the mirror
S-38 Science
27. Which statement best describes the property of light 33. A number of images of a candle flame can be seen in a
waves illustrated in the diagram below? thick plane mirror. The brightest image is
(a) Fourth (b) Second
(c) Last (d) First
34. A ray from air enters water, then through a thick layer of
glass placed below water. After passing through glass, it
again comes out in air medium. Then final emergent ray
will
(a) Bend towards the normal
(b) Suffer lateral displacement
(a) Some materials absorb light waves.
(c) Have the same path as if it had not passed through
(b) Some materials reflect light waves.
glass and water.
(c) Light waves are refracted by some materials.
(d) None of these
(d) Light waves are emitted by some materials.
35. A concave spherical mirror has a radius of curvature of
28. Light waves
100 cm. What is its focal length
(a) require air or another gas to travel through
(a) 50 cm (b) 100 cm
(b) require an electric field to travel through
(c) 200 cm (d) 300 cm
(c) require a magnetic field to travel through
(d) can travel through perfect vacuum 36. Light is incident on an air-water interface at an angle of
25° to the normal. What angle does the refracted ray make
29. What are the factors that determine the angle of deviation
with the normal
in a prism?
(a) 19° (b) 34°
(a) angle of incidence (b) wave length
(c) 25° (d) 90°
(c) angle of the prism (d) All the above
30. Morning sun is not so hot as the mid day sun because 37. If the speed of light in medium –1 and medium –2 are
2.5 × 108 ms–1 and 2 × 108 ms–1, respectively, then the
(a) Sun is cooler in the morning
refractive index of medium – 1 with respect to medium –
(b) Heat rays travel slowly is the morning
2 is _________.
(c) It is God gift
(d) The sun’s rays travel a longer distance through 3 2
(a) (b)
atmosphere in the morning 2.5 2.5
31. The layered lens shown below is made of two different 2.5 2.5
(c) (d)
transparent materials. 3 2
38. Under what conditions does a diverging lens form a
virtual image of a real object
(a) Only if u > f.
(b) Only if u < f.
(c) Only if u = f
(d) A diverging lens always forms a virtual image of a
real object.
A point object is placed on its axis. The object will form
(a) one image (b) infinite images 39. A lens produces a enlarged, virtual image. What kind of
(c) no image (d) two images lens is it?
(a) converging
32. An object is placed in front of a concave mirror of focal
(b) diverging
length 50.0 cm and a real image is formed 75 cm in front
of the mirror. How far is the object from the mirror (c) It could be either diverging or converging.
(a) 25 cm (b) 30 cm (d) None
(c) 150 cm (d) –150 cm
Light-Reflection and Refraction S-39
40. In an experiment to determine the focal length of a 42. On the basis of experiment ‘to trace the path of a ray
concave lens, a student obtained the image of a distant of light passing through a rectangular glass slab’ four
window on the screen. To determine the focal length of students arrived at the following interpretations :
the lens, she/he should measure the distance between the I. Angle of incidence is greater than the angle of
(a) lens and the screen only emergence.
II. Angle of emergence is less than the angle of
(b) lens and the window only
refraction.
(c) screen and the window only
III. Emergent ray is parallel to the incident ray.
(d) screen and the lens and also between the screen and IV. Emergent ray is parallel to the refracted ray.
the window
The correct interpretation is that of the student.
41. Ashima looks into the mirror and sees the reflection of the (a) I (b) II
picture behind her. (c) III (d) IV
Picture 43. Light waves
Mirror
(a) are mechanical waves
(b) are electromagnetic waves
(c) travel with the same velocity in all media
(d) requires a material medium for their propagation
Image seen by Ashima
in the mirror 44. Virtual images of object of the same size are formed by
(a) a concave mirror (b) a convex mirror
(c) a plane mirror (d) all the above
45. Two plane inclined mirrors form 5 images by multiple
reflection. The angle of inclination is
Which of the following is the picture that is behind (a) 90° (b) 60°
Ashima ? (c) 45° (d) 30°
46. A bright × (cross) mark is made on a sheet of white paper.
Over the white paper a rectangular glass-slab of thickness
3 cm is placed. On looking through, the image of the mark
(a)
appears above the mark. It is below the upper surface of
the slab by (µglass = 1.5)
(a) 2.5 cm (b) 1.5 cm
(c) 2 cm (d) 1.75 cm
47. Images formed by an object placed between two plane
mirrors whose reflecting surfaces make an angle of 90°
(b)
with one another lie on a
(a) Straight line (b) Zig-zag curve
(c) Circle (d) Ellipse
48. A diver in a swimming pool wants to send a signal to a
person lying on the edge of the pool by flashing his water-
proof torch
(c) (a) He must direct the beam of light vertically upwards
(b) He must direct the beam horizontally
(c) He must direct the beam at an angle to the vertical
which is slightly lesser than the critical angle
(d) He must direct the beam at an angle to the vertical
which is slightly greater than the critical angle
49. Two plane mirrors are inclined at an angle θ. A ray of light
(d)
is incident on one mirror and is then reflected from the
other mirror. Then the angle between the first ray and the
final ray will be
S-40 Science
(a) θ (b) 2θ (c) plane or concave mirror
(c) between θ and 2θ (d) > 2θ (d) plane or concave or convex mirror
50. A glass slab is placed in the path of a beam of convergent 57. Refraction of light from air to glass and from air to water
light, then the point of convergence of light are shown in figure (i) and (ii) below. The value of the
(a) moves towards the glass slab angle in the case of refraction as shown in figure (iii) will
(b) moves away from the glass slab be :
(c) remains at the same point 35°
(d) undergoes a lateral shift
glass
51. A real image is formed by a convex mirror when the (i)
object is placed at
air
(a) infinite 60°
(b) between center of curvature and focus
(c) between focus and pole air

(d) None of the above 60°

52. A virtual image is formed by a concave mirror when the (ii)


object is placed between
water
(a) infinity and center of curvature 41°

(b) center of curvature and focus


(c) focus and the pole
(d) All of the above 
53. Which of the following are used in a Kaleidoscope glass
(iii)
(a) plane mirrors (b) concave
water
(c) convex mirrors (d) All of the above 41°
54. When a convex lens made up of glass is immersed in
(a) 30° (b) 35°
water, its focal length
(c) 60° (d) none of the above
(a) decreases (b) does not change
(c) increases (d) None of the above 58. The focal length of a plane mirror is
(a) positive (b) negative
55. Find out the correct option from the following. (c) zero (d) infinity
(A) The magnification is positive for all virtual images
59. Rays from the sun converge at a point 15 cm in front of a
and is negative for all real images.
concave mirror. Where should an object be placed so that
(B) The magnification of concave lens and convex size of its image is equal to the size of the object?
mirror is always positive where as the magnification (a) 15 cm in front of the mirror
of convex lens and concave mirror can be positive (b) 30 cm in front of the mirror
or negative depending on the position of the object (c) between 15 cm and 30 cm in front of the mirror
before the lens. (d) more than 30 cm in front of the mirror.
(a) Only A is true
60. A convex mirror is used
(b) Only B is true.
(a) by a dentist
(c) Both A and B are true (b) for shaving
(d) Both A and B are false (c) as a rear view mirror in vehicles
56. A person standing at some distance from a mirror finds his (d) as a light reflector for obtaining a parallel beam of
image erect, virtual and of the same size. Then the mirror light.
is possibly 61. In case of a concave mirror, when the object is situated at the
(a) plane mirror principal focus, the image formed is
(b) concave mirror (a) real and inverted (b) of infinite size
(c) lies at infinity (d) All of these
Light-Reflection and Refraction S-41
62. For an object at infinity, a concave mirror produces an 70. The linear magnification for a mirror is the ratio of the size
image at its focus which is of the image to the size of the object, and is denoted by m.
(a) enlarged (b) virtual Then, m is equal to (symbols have their usual meanings):
(c) erect (d) real and point sized
f f −u
63. An inverted image can be seen in a convex mirror, (a) (b)
f −u f
(a) under no circumstances
(b) when the object is very far from the mirror f f +v
(c) (d)
(c) when the object is at a distance equal to the radius of f +v f
curvature of the mirror 71. In case of a real and inverted image, the magnification of
(d) when the distance of the object from the mirror is a mirror is
equal to the focal length of the mirror (a) positive (b) negative
64. In order to get a diminished virtual image, the object can (c) zero (d) infinity
be placed anywhere in front of a
72. Magnification produced by a rear view mirror fitted in
(a) concave mirror (b) plane mirror
vehicles
(c) convex mirror (d) none of these
(a) is less than one
65. A full length image of a distant tall building can definitely (b) is more than one
be seen by using (c) is equal to one
(a) a concave mirror (d) can be more than or less than one depending upon
(b) a convex mirror the position of the object in front of it.
(c) a plane mirror
73. The ratio of the sine of angle of incidence to the sine of
(d) both concave as well as plane mirror
angle of refraction is called
66. The concave mirrors are used in (a) refractive index
(a) reflecting telescopes (b) magic- lanterns (b) optical density
(c) cinema projectors (d) All of these (c) relative density
67. Which of the following statements is true? (d) none of these
(a) A convex lens has 4 dioptre power having a focal 74. When an objects is placed between two mirrors placed
length 0.25 m inclined to each at an angle 45° Number of images formed
(b) A convex lens has –4 dioptre power having a focal are
length 0.25 m (a) 3 (b) 5
(c) A concave lens has 4 dipotre power having a focal (c) 7 (d) None of these
length 0.25 m
75. Foam of soap always appears white as
(d) A concave lens has – 4 dioptre power having a focal
length 0.25 m (a) it contains large hydrocarbon chains.
(b) it absorbs red portion of the visible light
68. A virtual, erect and magnified image of an object is to be
(c) it reflects light of all wavelengths.
produced with a concave mirror of focal length 12 cm.
(d) it has one hydrophobic end, which is insoluble in
Which of the following object distance should be chosen
water.
for this purpose?
(a) 10 cm (b) 14 cm 76. Two lenses of focal length f1 and f2 are kept in contact
(c) 18 cm (d) 24 cm coaxially. The power of the combination will be

69. A 10 mm long awlpin is placed vertically in front of a f1 f 2 f1 + f 2


(a) (b) f1 f 2
concave mirror. A 5 mm long image of the awl pin is f1 + f 2
formed at 30 cm in front of the mirror. The focal length of
this mirror is f1 f 2
(c) f1 + f2
f1 − f 2 (d)
(a) – 30 cm (b) – 20 cm
(c) – 40 cm (d) – 60 cm
S-42 Science
77. A mirror is placed at an angle of 30° with respect to What is the distance of the final image formed by this lens
Y-axis (see figure). A light ray travelling in the negative system?
y-direction strikes the mirror. The direction of the reflected (a) 120 cm to right of lens A
ray is given by the vector
(b) 90 cm to right of lens A
y (c) 22.5 cm to right of lens B
(d) 45 cm to right of lens B

x
80. Two plane mirrors are kept on a horizontal table making
O
an angle θ with each other as shown schematically in the
30° figure. The angle θ is such that any ray of light reflected
after striking both the mirrors returns parallel to its
incident path. For this to happen, the value of θ should be

(a) (b) iˆ − 3 ˆj

iˆ − 2 ˆj
(c) 3iˆ − ˆj (d)
78. A ray of light originates from inside a glass slab and is 
incident on its inner surface at an angle θ as shown below.

Glass slab
2 (a) 30° (b) 45°
x
 0 (c) 60° (d) 90°

–2 81. An object is placed at a distance of 40 cm from a concave


mirror of focal length 15 cm. If the object is displaced
through a distance of 20 cm towards the mirror, the
displacement of the image will be
(a) 30 cm away from the mirror
Screen (b) 36 cm away from the mirror
In this experiment, the location x of the spot where the ray (c) 36 cm towards the mirror
hits the screen is recorded. Which of the following correctly (d) 30 cm towards the mirror
shows the plot of variation of x with the angle θ?
82. A pin AB of length 2 cm is kept on the axis of a convex lens
between 18 cm and 20 cm as shown in figure. Focal length
0 0 0 0 of convex lens is 10 cm. Find magnification produced for
x x x x the image of the pin.

        A B

A B C D
20 cm

(a) A (b) B (c) C (d) D


(a) 0.83 (b) 1.00
79. Two convex lenses A and B each of focal length 30 cm are
separated by 30 cm, as shown in the figure. An object O is (c) 1.25 (d) 6.78
placed at a distance of 40 cm to the left of lens A. 83. A concave mirror for face viewing has focal length of
A B 0.4 m. The distance at which you hold the mirror from
your face in order to see your image upright with a
magnification of 5 is:
O (a) 0.24 m (b) 1.60 m
(c) 0.32 m (d) 0.16 m
40 cm 30 cm
Light-Reflection and Refraction S-43
84. A convex lens of focal length 20 cm is cut into two 91. A diverging lens with magnitude of focal length 25 cm is
halves. Each of which is placed 0.5 mm and a point placed at a distance of 15 cm from a converging lens of
object placed at a distance of 30 cm from the lens as magnitude of focal length 20 cm. A beam of parallel light
shown. falls on the diverging lens. The final image formed is :
Then the image is at (a) real and at a distance of 40 cm from the divergent
(a) 60 cm lens
(b) real and at a distance of 6 cm from the convergent
(b) 30 cm lens
(c) 70 cm (c) real and at a distance of 40 cm from convergent
lens
(d) 50 cm
(d) virtual and at a distance of 40 cm from convergent
85. Focal length of a lens is 25 cm. In dioptre, power of lens lens.
will be 92. A beam of light from a source L is incident normally
(a) 0.04 (b) 0.4 on a plane mirror fixed at a certain distance x from
(c) 4 (d) 2.5 the source. The beam is reflected back as a spot on a
86. When viewed vertically a fish appears to be 4 meter below scale placed just above the source L. When the mirror
the surface of the lake. If the index of refraction of water is rotated through a small angle θ, the spot of the light
is found to move through a distance y on the scale. The
is 1.33, then the true depth of the fish is
angle θ is given by
(a) 5.32 metres (b) 3.32 metres
y x
(c) 4.32 metres (d) 6.32 metres (a) (b)
x 2y
87. Two thin lenses of focal lengths f1 and f2 are placed
x y
in contact with each other such that the combination (c) (d)
behaves as a glass slab. Then how are f1 and f2 related y 2x
to each other? 93. A glass beaker is filled with water up to 5 cm. It is
1 kept on top of a 2 cm thick glass slab. When a coin at
(a) f1 = f2 = –f1 the bottom of the glass slab is viewed at the normal
f 2 (b)
incidence from above the beaker, its apparent depth
(c) f1 = f2 (d)
f1 = f 2 from the water surface is d cm. Value of d is close to
(the refractive indices of water and glass are 1.33 and
88. A convex lens of focal length 25 cm receives light from 1.5, respectively)
the sun. A diverging lens of focal length – 12 cm is placed (a) 2.5 cm (b) 5.1 cm
37 cm to the right of the converging lens. Where is the (c) 3.7 cm (d) 6.0 cm
final image located relative to the diverging lens? 94. A convex lens is put 10 cm from a light source and it
(a) 6 cm to the left (b) 25 cm to the left makes a sharp image on a screen, kept 10 cm from the
lens. Now a glass block (refractive index 1.5) of 1.5 cm
(c) At infinity (d) 12 cm to the right
thickness is placed in contact with the light source. To get
89. A camera lens focuses light from a 12.0 m tall building the sharp image again, the screen is shifted by a distance
located 35.0 m away on film 50.0 mm behind the lens. d. Then d is:
How tall is the image of the building on the film? (a) 1.1 cm away from the lens
(b) 0
(a) 17.1 mm (b) 7.00 mm (c) 0.55 cm towards the lens
(c) 2.50 cm (d) 1.25 mm (d) 0.55 cm away from the lens
90. A hollow lens is made of thin glass and in the shape of
a double concave lens. It can be filled with air, water of
refractive index 1.33 or CS2 of refractive index 1.6. It will
act as a diverging lens, if it is DIRECTIONS : Study the given case/passage and answer the
(a) filled with air and immersed in water following questions.
(b) filled with water and immersed in CS2 Case/Passage - 1
(c) filled with air and immersed in CS2 A 5.0 cm tall object is placed perpendicular to the principal
(d) filled with CS2 and immersed in water axis of a convex lens of focal length 20 cm. The distance of the
object from the lens is 30 cm.
S-44 Science
95. What is the distance of image from the pole of lens?
(a) v = 60 cm (b) v = – 60 cm
(c) v = 30 cm (d) v = –30 cm
96. What is the power of the used lens? A B C D

(a) + 5 D (b) – 5 D
(a) A (b)
B
(c) + 0.5 D (d) – 0.5 D
(c) C (d)
D
Case/Passage - 2
Light travels through a vacuum at a speed c = 3 × 108 m/s. It 100. You are given water, mustard oil, glycerine and kerosene.
can also travel through many materials, such as air, water and In which of these media, a ray of light incident obliquely
glass. Atoms in the material absorb, reemit and scatter the at same angle would bend the most?
light, however. Therefore, light travels through the material (a) Kerosene (b) Water
at a speed that is less than c, the actual speed depending on (c) Mustard oil (d) Glycerine
the nature of the material. To describe the extent to which
101. A ray of light is incident in medium 1 on a surface that
the speed of light in a material medium differs from that in
separates medium 1 from medium 2. Let v1 and v2
a vacuum, we use a parameter called the index of refraction
represent the velocity of light in medium 1 and medium 2
(or refractive index).
respectively. Also let n12 and n21 represent the refractive
97. Figure shows a ray of light as it travels from medium A to index of medium 1 with respect to medium 2 and
refractive index of medium 2 with respect to medium 1,
respectively. If i and r denote the angle of incidence and
angle of refraction, then-
45°
45° Medium B sin i v1 sin i v2
30° Medium A (a) sin r = n21 v (b) = n21 v
2 sin r 1
60°

sin i v1 sin i v2
(c) = n12 v (d) = n12 v
sin r 2 sin r 1
medium B. Retractive index of the medium B relative to
medium A is Case/Passage - 3

3 2 Inside a substance such as glass or water, light travels more


(a) (b) slowly than it does in a vacuum. If c denotes the speed of
2 3
light in a vacuum and v denotes its speed through some
1 other substance, then v = c/n where n is a constant called
(c) (d) 2
2 the index of refraction.

98. A light ray enters from medium A to medium B as shown To good approximation, a substance’s index of refraction does
not depend on the wavelength of light. For instance, when red
and blue light waves enter water, they both slow down by about
Medium B the same amount. More precise measurements, however, reveal
that n varies with wavelength. Table presents some indices of
refraction of Custon glass, for different wavelengths of visible
Medium A light. A nanometer (nm) is 10–9 meters. In a vacuum, light
travels as c = 3.0 × 108 m/s
in the figure. The refractive index of medium B relative to Table : Indices of refraction of Custon glass
A will be
Approximately Wavelength in "Indices
(a) greater than unity (b) less than unity
colour vacuum (nm) n"
(c) equal to unity (d) zero
yellow 580 1.5
99. The path of a ray of light coming from air passing yellow orange 600 1.498
through a rectangular glass slab traced by four students
orange 620 1.496
shown as A, B, C and D in the figure. Which one of
them is correct? orange red 640 1.494
Light-Reflection and Refraction S-45
102. Inside Custon glass 108. Assertion : When a concave mirror is held under water,
(a) Orange light travels faster than yellow light its focal length will increase.
(b) Yellow light travels faster than orange light Reason : The focal length of a concave mirror is
(c) Orange and Yellow light travels equally fast independent of the medium in which it is placed.
(d) We cannot determine which color of light travels faster 109. Assertion : A convex mirror is used as a driver’s
mirror.
103. For blue-green of wavelength 520 nm, the index of
refraction of Custon glass is probably closest to Reason : Because convex mirror’s field of view is large
and images formed are virtual, erect and diminshed.
(a) 1.49 (b) 1.50 
(c) 1.51 (d) 1.52 110. Assertion : When the object moves with a velocity v , its

image in the plane mirror moves with a velocity of −2v
104. Which of the following phenomena happens because n
with respect to the object.
varies with wavelength
Reason : The minimum height of the mirror to be required
(a) A lens focuses light
h
(b) A prism breaks sunlight into different colors to see the full image of man of height h is .
2
(c) Total internal reflections ensures that light travels
down a fiber optic cable 111. Assertion : As the temperature of a medium increases the
refractive index decreases.
(d) Light rays entering a pond change direction at the
pond’s surface Reason : When a ray travels from vacuum to a medium,
then µ is known as absolute refractive index of the
medium. (µvacuum = 1).
Assertion & Reason
112. Assertion : If a spherical mirror is dipped in water, its
focal length remains unchanged.
DIRECTIONS : Each of these questions contains an assertion
followed by reason. Read them carefully and answer the question Reason : A laser light is focused by a converging lens.
on the basis of following options. You have to select the one that There will be a significant chromatic aberration.
best describes the two statements.
113. Assertion : A virtual image cannot be projected one
(a) If both Assertion and Reason are correct and Reason is
screen.
the correct explanation of Assertion.
(b) If both Assertion and Reason are correct, but Reason is Reason : Virtual images are formed by actual meeting
not the correct explanation of Assertion. of rays of light after reflection or refraction.
(c) If Assertion is correct but Reason is incorrect. 114. Assertion : Red light travels faster in glass than green
(d) If Assertion is incorrect but Reason is correct. light.

105. Assertion : The diameter of convex lens required to form Reason : The refractive index of glass is less for red
full image of an object is half the height of the object. light than for green light.

Reason : The smaller diameter lens will give full image 115. Assertion : As light travels from one medium to
of lower intensity. another, the frequency of light does not change.

106. Assertion : The image of a point object situated at the Reason : Because frequency is the characteristic of
centre of hemispherical lens is also at the centre. source.
Reason : For hemisphere Snell’s law is not valid. 116. Assertion : Light rays retrace their path when their
107. Assertion : A point object is placed at a distance of 26 cm direction is reversed (Law of reversibility of light rays)
from a convex mirror of focal length 26 cm. The image Reason : For the refraction of light, water is denser than
will not form at infinity. air, but for the refraction of sound, water is rarer than air.
1 1 1 117. Assertion : The mirrors used in search lights are parabolic
Reason : For above given system the equation + =
u v f and not concave spherical.
gives v = ∞.
Reason : Silvered plano convex lens is used in search
light.
S-46 Science
121. Light seems to travel in ...............
Match the Following
122. A light ray travelling obliquely from a denser medium to a
rarer medium bends ............. the normal. A light ray bends
DIRECTIONS : Each question contains statements given in
two columns which have to be matched. Statements (A, B, C, .............. the normal when it travels obliquely from a rarer
D) in column I have to be matched with statements (p, q, r, s) in to a denser medium.
column II. 123. In case of a rectangular glass slab, the refraction takes
place at both .................. interface and ............... interface.
118. Match the following :
The emergent ray is ........... to the direction of incident
Column I Column II
ray.
(A) Power of convex mirror (p) Positive power
(B) Power of concave mirror (q) Negative power 124. Power of a lens is the reciprocal of its .................
(C) Power of plane mirror (r) Zero power 125. The SI unit of power of a lens is ................
(D) Power of convex lens (s) Infinite power
126. The angle of incidence is ......... to the angle of reflection.
119. The graphs given apply to convex lens of focal length f,
producing a real image at a distance v from the optical 127. The reflecting surface of a spherical mirror may be curved
centre when self luminous object is at distance u from
.............. or ..............
the optical centre. The magnitude of magnification is m.
Identify the following graphs with the first named quantity 128. The inner surface of the spoon can be approximated to a
being plotted along y-axis.
........... mirror.
Column I Column II
129. The centre of the reflecting surface of a spherical mirror is
a point called the ................
v against u
(A) (p) 130. The centre of curvature of a concave mirror lies in ..........
of it.

131. Line passing through the pole and the centre of curvature
of a spherical mirror is called the ................
1 1 132. A ray parallel to the principal axis, after reflection, will
(B) against (q)
v u pass through the ....................
133. The dentists use ............... mirrors to see large images of
y the teeth of patients.
134. A transparent material bound by two surfaces, of which
m against v
(C) (r) one or both surfaces are spherical, forms a ...........
135. The degree of ............. of light rays achieved by a lens is
x
expressed in terms of its power.
136. An object is placed in front of a spherical mirror. The
v
(D) (m + 1) against (s) image is found to be virtual for all positions of the object.
f
The spherical mirror is .........
137. Two immiscible transparent liquids A and B have 1.2 and
1.5 as their refractive indices (with respect to air). The
Fill in the Blanks refractive index of B with respect to A is ........

DIRECTIONS : Complete the following statements with an True / False


appropriate word / term to be filled in the blank space(s).
DIRECTIONS : Read the following statements and write your
120. The power of a convex lens is ................. and that of a answer as true or false.
concave lens is ..............
Light-Reflection and Refraction S-47
138. The reflecting surfaces, of all types, obey the laws of 147. A ray of light passing through the optical centre of a lens
reflection. will emerge without any deviation.
139. Light travels in vacuum with an enormous speed of 148. The image in a plane mirror lies as far behind the mirror.
3 × 108 ms–1.
149. An object is placed in front of a mirror and an image of
140. The speed of light is different in different media. it is formed at the object itself. The mirror mentioned in
question is a convex mirror.
141. The refractive index of a transparent medium is the ratio
of the speed of light in vacuum to that in the medium. 150. A concave mirror can produce both real and virtual images.
142. The incident ray, the normal to the mirror at the point of 151. Light travels faster in glass than in air.
incidence and the reflected ray, all lie in the same plane. 152. The laws of reflection are valid for plane mirrors and not
143. Image formed by a plane mirror is always virtual and erect. for spherical mirrors.
144. The principal focus of a spherical mirror lies midway 153. The mirror formula is valid only if the aperture of the
between the pole and centre of curvature. mirror is small.
145. Convex mirrors enable the driver to view much larger 154. When a ray of light travels from air to water, its speeds up.
area than would be possible with a plane mirror.
155. A lens that is thicker at the middle than at the edges is a
146. A concave lens will always give a virtual, erect and diverging lens.
diminished image.
S-48 Science

ANSWER KEY & SOLUTIONS


1. (b) In plane mirror, object distance = image distance
a µg sin 41°
∴ aµg = =
∴ Distance between object and image a µw sin θ
= 0.5 + 0.5 = 1 m  sin 60° sin 60°  sin 41°
360° or  =
2. (a)
= n = 4  sin 35° sin 41°  sin θ
90° ∴ q = 35°
so numbe of images is (n – 1 ) ⇒ (4 – 1) = 3
58. (d) 59. (b) 60. (c) 61 (d) 62. (d)
3. (b) Concave mirror is used as a shaving mirror
63. (a) 64. (c) 65. (b) 66. (d) 67. (a)
4. (c) For all spherical mirrors f = R/2
68. (a) 69. (b) 70. (a) 71. (b) 72. (a)
Image height
5. (c) given, m = >1 73. (a)
object height
360°
74. (c) Number of images formed = − 1 = 7.
⇒ Image height > Object height θ
75. (c)
6. (b) Convex mirror always form virtual and erect image.
1 1 f1 + f 2
7. (b) 8. (c) 76. (b) P = P1 + P2 ⇒ P
= +  ⇒ P= f f
f1 f 2 1 2
1
9. (a) Power = 77. (c) According to laws of reflection,
focal length
angle of incidence = angle of reflection
Image height
10. (a) Magnification, m = A
Object height Incident
ray
11. (b) Diminished, erect image is formed by convex Normal
mirror.
12.
(b)
13.
(b)
14. (a)
15. (c)
30°

1 1 1 60°
16. (d) For a spherical lens − =
v u f x
O 60°
For convex lens. u = – f /2 and f is + ve
1 1 1 1 1 2 
∴ = + + =+ − r
∴v=–f
v f u f f f
17. (d) 18. (c) 19. (a) 20. (c) 21. (a) B
Reflected
22. (a) 23. (c) 24. (b) 25. (c) 26. (b) ray
27. (c) 28. (d) 29. (d) 30. (d) 31. (d) 
∴ if a vector r is along the reflected ray, then
32. (d) 33. (b) 34. (b) 35. (a) 36. (a) 
= r cos30°iˆ − sin 30 ˆj

37. (b) 38. (d) 39. (a) 40. (a) 41. (b)
 3ˆ 1 ˆ
42. (c) 43. (b) 44. (c) 45. (b) 46. (c) =r i− j
2 2
47. (c) 48. (c) 49. (b) 50. (b) 51. (d) 
=r 3 iˆ − ˆj

52. (c) 53. (a) 54. (c) 55. (c) 56. (a)
Hence, the direction of the reflected ray vector is .
sin 60° sin 60°
57. (b) aµg = and aµw = ˆ ˆ
sin 35° sin 41° 3i − j
Light-Reflection and Refraction S-49
78. (a) Angle of incidence, i = 90° – θ, decreases with 1 1 1 1 1 1
increase in θ upto angle of incidence i = critical = –   ⇒  = –
–15 v2 20 v2 20 15
angle reflection takes place so x is positive and
beyond the critical angle refraction takes place so x ∴ v2 = –60 cm
is negative. Therefore image shifts away from mirror by
Hence graph ‘A’ correctly depicts variation of x with = 60 – 24 = 36 cm
the angle θ. 82. (c) For the end B, image distance of end B will be,
1 1 1 f = 10 cm
79. (c) For lens A, = –
f v u
A B
uB = –18 cm
1 1  1 
⇒ = ––  vB = image distance of end B
30 v  40 
O As we know,
1 1 1
⇒ = +
30 v 40 40 cm 30 cm 1 1 1
= v −u
4–3 1 f B B
= = or, v = 120 cm.
120 120 1 1 1
For lens B, u = 90 cm [u = 120 – 30] v = f + u
B B
1 1 1 1 1 1
= – ⇒ = – 1 1 1 8
f v u 30 v 90 v = − =
1 1 1 3 +1 B 10 18 180
= + =
v 30 90 90 180
vB = ⇒ 22.5 cm
or, v = 22.5 cm 8
Which is positive so that it is 22.5 cm from lens B. Similarly, for the end A, image distance of end A
will be,
80. (d) Two mirrors are inclined at an angle, θ = ?
f = 10 cm
According to question, emergent ray is parallel to
incident ray uA = –20 cm
∴ deviation angle δ = 180° vA = image distance of end A
But δ = 360° – 2θ 1 1 1
= v −u
or, 360° – 2θ = 180° f A A

or, 2θ = 180°  1 1 1
= f + u
∴ θ = 90° vA A

81. (b) 1 1 1 1
f = 15 cm = − =
vA 10 20 20
O 40 cm
vA = 20 cm
So, length of image A’B’ = (vB – vA)
= 22.5 – 20 = 2.5 cm
1 1 1
Using mirror formula, = + A′B′ 2.5
f v1 u So magnification, m = ⇒ = 1.25
AB 2
1 1 1 1 1 1
– 15= v + u ⇒ =
v –15
+
40 v
1 1 83. (c) +5 =− ⇒ v =−5u
∴ v1 = –24 cm u
When object is displaced by 20 cm towards mirror 1 1 1 1 1 −1
Now, u2 = –20 Using ⇒ + = ⇒ + =
v u f −5 u u 0.4
1 1 1 ∴ u = – 0.32 m.
So, = +
f v2 u2
S-50 Science
84. (a) Given, 1 1 1
− =
Object distance, u = 30cm V −40 20
when a lens is cut along the principle axis into two or, V = 40 cm from converging lens real and
equal parts focal length remains same for each part. inverted.
\ Focal length, f = 20cm 92. (d) When mirror is rotated by angle θ reflected ray will
using lens formula be rotated by 2θ.

1 1 1 light
= - spot 
f v u
y
1 1 1 1
Þ = - = source
v 20 30 60 2
Mirror
(L) x
⇒ v = 60 cm
85. (c) Focal length of a lens, F = 25 cm y y
= 2θ ⇒ θ =
f = 0.25 m x 2x
1 1 93. (b) Given:  d1 = 5 cm, µ1 = 1.33
P = = = 4D
f 0.25    d2 = 2 cm, µ2 = 1.5
86. (a) 87. (b) 88. (a) 89. (a) d1 and d2 are the thickness of slabs of medium with
refractive index µ1 and µ2, respectively.
90. (d) Here µcs2 > µ water > µair
(1.6) (1.33) (1.0) d1 d 2
i.e., CS2 is denser than water and water is denser than air. using formula, d = µ + µ + .....
1 2

When medium outside a lens is denser than medium 5 2


Apparent depth, d = +
of lens, then a concave lens will acts like a convex 1.33 1.5
lens and vice-versa. = 5.088 cm = 5.1 cm
Hence, lens here acts as a diverging lens when filled
with CS2 and immersed in water.
94. (d)
91. (c) As parallel beam incident on diverging lens will
form image at focus. O O'
µ
∴ v = –25 cm
10 cm
15 cm

As the object and image distance is same, object is


placed at 2 f. Therefore 2 f = 10
or f = 5 cm.
 1
d t 1 − 
Shift due to slab, =
 µ
in the direction of incident ray

 2
f = –25 cm f = 20 cm ⇒ d= 1.5 1 − =
 0.5 cm
 3
The image formed by diverging lens is used as an
object for converging lens, Now, u = – 9.5 cm
So for converging lens u = –25 – 15 = –40 cm,  f = 1 1 1
Again using lens formulas − =
20 cm v −9.5 5
⇒ v = 10.55 cm
∴ Final image formed by converging lens
Thus, screen is shifted by a distance d = 10.55 – 10
= 0.55 cm away from the lens.
Light-Reflection and Refraction S-51
95. (a) Object size h0 = 5.0 cm, f = 20 cm, 100. (d) Among the given material kerosene refractive index,
Object distance u = – 30 cm µ = 1.44, water µ = 1.33, mustard oil µ = 1.46 and
glycerine µ = 1.74. Glycerine is most optically denser.
1 1 1 Therefore, ray of light bend most in glycerine.
Since, – =
v u f
sin i v
1 1 1 101. (a) = n21 = 1
= + sin r v2
v f u
102. (a) 103. (c) 104. (b)
1 1 1 1
Then = + = 105. (d) Any size of lens, can form full image, only intensity
v 20 –30 60 of image decreases with decrease in size.
∴ v = + 60 cm
106. (c) The rays from centre of hemisphere cut at the centre
Positive sign of v shows that image is formed at a after refraction - Snell’s law is valid in each case of
distance of 60 cm from the pole to the right of the refraction.
lens.
107. (d) 108. (d) 109. (a) 110. (b)
Therefore image is real and inverted. 111. (b) 112. (c)
1 1 113. (c) Virtual image is formed when the rays of light after
96. (a) Power = = = +5D
f 0.2m reflection or refraction appear to meet at a point.
97. (a) From figure, angle of incidence, i = 60° and angle of 114. (a) 115. (a) 116. (c) 117. (c)
refraction, r = 45°
118. (A) → p; (B) → p; (C) → r; (D) → p
Refractive index of the medium B relative to
119. (A) → r; (B) → p; (C) → q; (D) → s
medium A, (from Snell’s law)
120. positive, negative. 121. straight lines.
 3
  122. away from, towards 123. air-glass, glass-air, parallel
sin i sin 60°  2  3
µ BA= = = =
sin r sin 45°  1  2 124. focal length 125. dioptre
 
 2 126. equal 127. inwards, outwards.
98. (a) Since light rays in the medium B goes towards 128. concave 129. pole
normal (figure), so it has greater refractive index 130. front 131.
principal axis
i.e., denser w.r.t. medium A. Hence, refractive index
of medium B relative to medium A is greater than 132. principal focus 133.
concave
unity. 134. lens. 135.
convergence or divergence
99. (b) In a rectangular glass slab, the emergent rays are 136. convex 137. 5/4
parallel to the direction of the incident ray, as the 138. True 139. True 140. True 141. True
extent of bending of the ray of light at the opposite
parallel faces air-glass and glass-air interface of 142. True 143. True 144. True 145. True
the rectangular glass slab is equal and opposite. 146. True 147. True 148. True 149. False
This is why the ray emerges are parallel to the 150. True 151. False 152. False 153. True
incident ray. 154. False 155. False
The Human
6 Acids,
Eye and
Salts
Bases
the and
Colourful World
book lying on his desk clearly. Which of the following
Multiple Choice Questions (MCQs) statement is correct about the student?
(a) The near point of his eyes has receded away.
DIRECTIONS : This section contains multiple choice
(b) The near point of his eyes has come closer to him.
questions. Each question has four choices (a), (b), (c) and (d)
out of which only one is correct. (c) The far point of his eyes has receded away.
(d) The far point of his eyes has come closer to him.
1. The human eye possesses the power of accommodation.
6. A man driving a car can read a distant road sign clearly but
This is the power to :
finds difficulty in reading the odometer on the dashboard
(a) alter the diameter of the pupil as the intensity of of the car. Which of the following statement is correct
light changes about this man?
(b) distinguish between lights of different colours (a) The near point of his eyes has receded away.
(c) focus objects at different distances (b) The near point of his eyes has come closer to him.
(d) decide which of the two objects is closer. (c) The far point of his eyes has receded away.
2. How does the eye change in order to focus on near or (d) The far point of his eyes has come closer to him.
distant objects?
7. Which of the following is not caused by the atmospheric
(a) The lens moves in or out refraction of light?
(b) The retina moves in or out (a) Twinkling of stars at night
(c) The lens becomes thicker or thinner
(b) Sun appearing higher in the sky than it actually is
(d) The pupil gets larger or smaller
(c) Sun becoming visible two minutes before actual
3. Which of the following changes occur when you walk out sunrise
of bright sunshine into a poorly lit room? (d) Sun appearing red at sunset
(a) The pupil becomes larger 8. The sky appears dark to passengers flying at very high
(b) The lens becomes thicker altitudes mainly because : [CBSE 2020]
(c) The ciliary muscle relaxes (a) Scatterings of light is not enough at such heights.
(d) The pupil becomes smaller (b) There is no atmosphere at great heights.
4. A person got his eyes tested. The optician’s prescription (c) The size of molecules is smaller than the wavelength
for the spectacles reads: of visible light.
Left eye : – 3.00 D Right eye : – 3.50 D (d) The light gets scattered towards the earth.
The person is having a defect of vision called : 9. A near sighted person cannot see distinctly beyond 50 cm
(a) presbyopia (b) myopia from his eye. The power in diopter of spectacle lenses
(c) astigmatism (d) hypermetropia which will enable him to see distant objects clearly is
(a) +50 (b) –50
5. A student sitting on the last bench in the class cannot read
(c) +2 (d) –2
the writing on the blackboard clearly but he can read the
The Human Eye and the Colourful World S-53
10. The following one is not a primary colour (a) Astigmatism (b) Myopia
(a) Yellow (b) Red (c) Hypermetropia (d) Presbyopia
(c) Green (d) Blue 17. On entering a glass prism, sun rays are
(a) Deviated but not dispersed
11. When a mirror is rotated an angle the reflected ray moves
through double that angle, the instrument based on the (b) Deviated and dispersed
above principle is (c) Dispersed but not deviated
(a) Periscope (b) Odometer (d) Neither deviated nor dispersed.
(c) Refractometer (d) Sextant 18. A piece of cloth looks red in sun light. It is held in the
blue portion of a solar spectrum, it will appear
12. In the visible spectrum the colour having the shortest (a) red (b) black
wavelength is
(c) blue (d) white
(a) Green (b) Red 19. To get line spectrum, the substances are excited in their
(c) Violet (d) Blue (a) solid state (b) molecular state
13. The splitting of white light into several colours on passing (c) gaseous state (d) atomic state
through a glass prism is due to 20. A student can distinctly see the object upto a distance 15
cm. He wants to see the black board at a distance of 3 m.
(a) refraction (b) reflection
Focal length and power of lens used respectively will be
(c) interference (d) diffraction (a) –4.8 cm, – 3.3 D (b) –5.8 cm, –4.3 D
(c) –7.5 cm, –6.3 D (d) –15.8 cm, –6.3 D
14. 1. 21. The pupil of the eye changes in size to adjust for
(a) objects at different distances
(b) objects of different sizes
(c) different colors
2. (d) different amounts of light
22. What power lens is needed to correct for nearsightedness
where the uncorrected far point is 250 cm?
3. (a) +2.5 diopters (b) –2.5 diopters
(c) + 0.4 diopters (d) –0.4 diopters
23. What power lens is needed to correct for farsightedness
where the uncorrected near point is 50 cm?
4. (a) + 2 diopters (b) – 3 diopters
(c) + 4 diopters (d) – 2 diopters
24. In a room, artificial rain is produced at one end and a
Identify the wrong description of the above figures strong source of white light is switched on at the other
(a) 1 represents far-sightedness end. To observe the rainbow an observer must
(b) 2 correction for short sightedness (a) Look anywhere in the room
(c) 3 represents far sightedness (b) Look towards the source
(d) 4 correction for far-sightedness (c) Look towards the raindrops
15. At sun rise or at sun set the sun appears to be reddish (d) Look in a direction equally inclined to the source of
while at mid-day it looks white. This is because raindrops
(a) Scattering due to dust particles and air molecules 25. Astigmatism can be corrected by
causes this phenomenon (a) Bifocal lenses (b) Cylindrical lenses
(b) The sun is cooler at sun rise or at sunset (c) Concave lenses (d) Planoconvex lenses
(c) Refraction causes this phenomenon 26. The least distance of vision of a longsighted person is 60
(d) Diffraction sends red rays to the earth at these times. cm. By using a spectacle lens, this distance is reduced to
16. A person 20 years old cannot see objects clearly which 12 cm. The power of the lens is
are nearer than 75 cms from his eyes, the disease he is (a) + 5.0 D (b) + (20/3) D
suffering from is (c) – (10/3) D (d) + 2.0 D
S-54 Science
27. A man can see upto 100 cm of the distant object. The (a) – 2.5 D (b) + 2.5 D
power of the lens required to see far objects will be (c) – 6.25 D (d) + 1.5 D
(a) + 0.5 D (b) + 1.0 D
35. Twinkling of stars is on account of
(c) + 1 D (d) – 5.0 D
(a) Large distance of stars and storms in air
28. Dispersion is the term used to describe (b) Small size of stars
(a) the propagation of light in straight lines (c) Large size of stars
(b) The splitting of a beam of light into component colours (d) Large distance of stars and fluctuations in the density
(c) The bending of a beam of light when it strikes a mirror of air.
(d) The change that takes place in white light after 36. White light is incident at an angle to the surface of a
passage through red glass. triangular piece of glass. Which color of light deviates most
29. A given ray of light suffers minimum deviation in from its original path after leaving the glass?
an equilateral prism P. Additional prisms Q and R of (a) red (b) orange
identical shape and material are now added to P as shown (c) green (d) blue
in the figure. The ray will suffer
37. The middle vascular coat that darkens the eye chamber
(a) greater deviatio
and prevents refraction by absorbing the light rays is
(b) same deviation
(a) choroid (b) sclera
(c) no deviation
(c) retina (d) cornea
(d) total internal reflection
30. In a glass prism 38. When light rays enter the eye, most of the refraction
(a) Blue light is dispersed more than red light occurs at the
(b) Red light is dispersed more than blue light (a) crystalline lens
(c) Both red light and blue light are equally dispersed (b) outer surface of the cornea
(d) None of these (c) iris
(d) pupil
31. An optician while testing the eyes finds the vision of a
patient to be 6/12. By this he means that 39. When the light is bright,
(a) The person can read the letters of 6 inches from a (a) the iris makes the pupil expand
distance of 12 m (b) the iris and the pupil contract
(b) The person can read the letters of 12 inches from 6 m (c) the iris and the pupil remain as they are
(c) The person can read the letters of 6 m which the (d) none of the above
normal eye can read from 12 m 40. The eyelens .......... light rays to form real, inverted and
(d) The focal length of eye lens had become half that of highly diminished image on the .......
the normal eye (a) converges, retina (b) diverges, retina
32. A person cannot see objects clearly beyond 50 cm. The (c) converges, pupil (d) diverges, pupil
power of the lens to correct the vision is
41. The surface of retina has about 125 million light sensitive
(a) +5 D (b) –0.5 D
(a) rods only (b) cones only
(c) –2 D (d) +2 D
(c) rods and cones (d) neither rods nor cones
33. A long sighted person has a minimum distance of distinct
42. The ‘far point’ of a normal human eye is
vision of 50 cm. He wants to reduce it to 25 cm. He
(a) 25 cm (b) 25 m
should use a
(c) 100 m (d) at infinity
(a) Concave lens of focal length 50 cm
(b) Convex lens of focal length 25 cm 43. The property related to the sense of continuity of vision is
(c) Convex lens of focal length 50 cm called
(d) Concave lens of focal length 25 cm (a) persistence of vision
(b) colour blindness
34. A long-sighted person cannot see objects clearly at a
(c) optical illusion
distance less than 40 cm. from his eye. The power of the
lens needed to read an object at 25 cm. is (d) none of these
The Human Eye and the Colourful World S-55
44. When the ciliary muscles are relaxed, the eyelens is (c) the intensity of light
............. and distant objects can be seen clearly. (d) none of these
(a) thin (b) thick
51. Which of the following statements is correct regarding
(c) inclined (d) none of these the propagation of light of different colours of white light
45. While looking at nearby objects, the ciliary muscles in air?
........... the eyelens so as to .......... its focal length. (a) Red light moves fastest
(a) contract, increase (b) Blue light moves faster than green light
(b) contract, decrease (c) All the colours of the white light move with the
(c) expand, increase same speed
(d) expand, decrease (d) Yellow light moves with the mean speed as that of
the red and the violet light
46. The change in focal length of an eyelens to focus the
image of object at varying distances is done by the action 52. When a ray passes through a prism,
of the (a) it goes undeviated
(a) pupil (b) ciliary muscles (b) it remains parallel to the base
(c) retina (d) blind spot (c) it bends towards the base
(d) none of the above
47. Which of the following statement is correct?
(a) A person with myopia can see distant objects clearly 53. The clear sky appears blue because
(b) A person with hypermetropia can see nearby objects (a) blue light gets absorbed in the atmosphere
clearly (b) ultraviolet radiations are absorbed in the atmosphere
(c) A person with myopia can see nearby objects clearly (c) violet and blue lights get scattered more than lights
(d) A person with hypermetropia cannot see distant of all other colours by the atmosphere
objects clearly (d) light of all other colours is scattered more than violet
and blue colour lights by the atmosphere
48. A person cannot see distinctly objects kept beyond 2 m.
This defect can be corrected by using a lens of power 54. At noon the sun appears white as
(a) + 0.5 D (b) – 0. 5 D (a) light is least scattered
(c) + 0. 2 D (d) – 0. 2 D (b) all the colours of the white light are scattered away
(c) blue colour is scattered the most
49. A beam of light consisting of red, green and blue colours
is incident on a right-angled prism as shown. The (d) red colour is scattered the most
refractive index of the material of the prism for the above 55. Which of the following phenomena contributes
red, green and blue wavelengths are 1.39, 1.44 and 1.47 significantly to the reddish appearance of the sun at
respectively. The prism will sunrise or sunset?
B (a) Dispersion of light
(b) Scattering of light
90°
(c) Total internal reflection of light
(d) Reflection of light from the earth
45° 45°
A C 56. The bluish colour of water in deep sea is due to
(a) separate part of the red colour from the green and (a) the presence of algae and other plants found in water
blue colours. (b) reflection of sky in water
(b) separate part of the blue colour from the red and (c) scattering of light
green colours.
(d) absorption of light by the sea
(c) separate all the three colours from one another.
(d) not separate even partially any colour from the other 57. A student sitting on the last bench can read the letters
written on the blackboard but is not able to read the
two colours.
letters written in his text book. Which of the following
50. The rod cells correspond to statements is correct?
(a) the colour of light (a) The near point of his eyes has receded away
(b) the source of light (b) The near point of his eyes has come closer to him
S-56 Science
(c) The far point of his eyes has come closer to him (a) A concave lens of 0.5 D
(d) The far point of his eyes has receded away (b) A concave lens of 1.0 D
58. The danger signals installed at the top of tall buildings are (c) A concave lens of 2.0 D
red in colour. These can be easily seen from a distance (d) A convex lens of 2.0 D
because among all other colours, the red light 65. Select the correct statement about rainbow.
(a) is scattered the most by smoke or fog (a) We can see a rainbow in the western sky in the late
(b) is scattered the least by smoke or fog afternoon
(c) is absorbed the most by smoke or fog (b) The double rainbow has red on the inside and violet
(d) moves fastest in air in the outside
59. A person is suffering from both near sightedness and (c) A rainbow has an arc shape, since the earth is round
far sightedness. His spectacles would be made of (d) A rainbow on the moon is violet on the inside and
(a) two convex lenses with the upper lens having a red on the outside
larger focal length than the lower lens. 66. Various optical processes are involved in the formation
(b) two concave lenses with the upper lens having a of a rainbow. Which of the following provides the correct
smaller focal length than the lower lens. order in time in which these processes occur ?
(c) a concave lens as the upper lens and a convex lens as (a) Refraction, total internal reflection, refraction
the lower lens (b) Total internal reflection, refraction total internal
(d) a convex lens as the upper lens and a concave lens as reflection
the lower lens
(c) Total internal reflection, refraction, refraction
60. The stars twinkle in the night, becauses : (d) Refraction, total internal reflection, total internal
(a) Their emit light intermittently reflection.
(b) Their star’s atmosphere absorbs light intermittently 67. Pick the wrong answer in the context with rainbow.
(c) The earth’s atmosphere absorbs light intermittently (a) When the light rays undergo two internal reflections
(d) The refractive index of air in atmosphere fluctuates in a water drop, a secondary rainbow is formed.
61. A Red object when seen through a thick blue glass (b) The order of colours is reversed in the secondary
appears: rainbow.
(a) Green (b) Violet (c) An observer can see a rainbow when his front is
(c) Black (d) Red towards the sun.
(d) Rainbow is a combined effect of dispersion,
62. If a person can see on object clearly when it is placed at
refraction and reflection of sunlight.
25 cm away from him, he is suffering from :
(a) myopia 68. The reason for using red light in traffic signals to stop
(b) hyper metropia vehicles.
(c) asitgmatism (a) Red light has shorter wavelength
(d) none of these (b) Red light has longer wavelength
(c) Red light is very bright and attractive
63. A person is suffering from some sight problem. From the
given diagram say which defect he suffers from? (d) Red light has highest angle of refraction
(a) Myopia Rays from a 69. The figures represent three cases of a ray passing through
(b) Hypermetropia
distant object a prism of angle A. The case corresponding to minimum
deviation is
(c) Cataract
(d) Astigmatism
64. To read a poster on a wall, a person with defective vision
needs to stand at a distance of 0.4m from the poster. A
person with normal vision can read the poster from a (a) 1 (b) 2
distance of 2.0 m. Which one of the following lens may (c) 3 (d) None of these
be used to correct the defective vision?
The Human Eye and the Colourful World S-57
70. If for a given prism the angle of incidence is changed But in reality it is not so. Image is always formed on the retina.
from 0° to 90°, the angle of deviation This is possible because the curvature of the crystalline lens is
(a) Increases altered by ciliary muscles. When the eye is focused on infinity
(b) Decreases the muscles are relaxed and the eye lens remains thin. If the
(c) First decreases and then increases object is brought near by, the curvature increases so that the
(d) First increases and then decreases image can be formed on the retina. This property of the eye
lens is called accommodation.
71. The change in focal length of an eye lens to focus the
DIRECTIONS : Study the given case/passage and answer the image of objects at varying distances is done by the
following questions. action of _______
(a) pupil (b) ciliary muscles
Case/Passage - 1
(c) retina (d) blind spot
Human eye is spherical in shape and has diameter of about
2.5 cm. Sclerotic is a tough, opaque and white substance 72. The fluid between the retina and the lens is called ______
forming the outermost coating of the eyeball. The front portion (a) aqueous humour
is sharply curved and covered by a transparent protective (b) vitreous humour
membrane called the ‘cornea’. Inner to the sclerotic there is a (c) aqua
layer of black tissue called as choroids consisting of a mass of (d) humus
blood vessels, which nourishes the eye. The black colour does
73. The part of the eye where optic nerves enter the eye
not reflect the light and hence rules out the blurring of image by
(a) pupil (b) ciliary muscles
reflection within the eyeball.
(c) retina (d) blind spot
Behind the cornea, the space is filled with a liquid called the
aqueous humour and behind that a crystalline lens. ‘Iris’ is a 74. The inner back surface of the eyeball is called
muscular diaphragm lying between the aqueous humour and (a) pupil (b) ciliary muscles
the crystalline lens. Iris has an adjustable opening in the middle (c) retina (d) blind spot
called the pupil of the eye. The pupil appears black because all Case/Passage - 2
the light entering is absorbed by the ‘retina’, which covers the
The phenomenon of decomposition of the white light into its
inside of the rear part of the ball. Iris controls the amount of light
seven component colours when passing through a prism or
entering because the retina absorbs nearly all the light, which
through a transparent object delimited by non parallel surfaces
falls upon it. This is done by varying the aperture of the pupil
is called dispersion of light. A beam of light containing all the
with the help of the iris. In dim light the iris dilates the pupil so
visible spectrum of the light is white, because the sum of all
that more light can enter in. When the light is bright the pupil
the colors generates the white color. The light is decomposed
contracts.
in all the component colours, Violet, Indigo, Blue, Green,
The crystalline lens divides the eyeball into two chambers. The Yellow, Orange and Red, called as VIBGYOR. The band of the
chamber between the cornea and the lens is called the anterior coloured components of a light beam is called its spectrum. The
chamber filled with a fluid called aqueous humour while the phenomenon can be explained by thinking that light of different
chamber between the lens and the retina is called the posterior colours (different wavelengths) has different velocities while
chamber which is filled with a transparent gelatinous substance travelling in a medium vm = f λm.
called vitreous humour. Hence, the change in velocity of light observed when the light
The refractive indices of the cornea, pupil lens and fluid portion passes from the air to the glass, depends on the wavelength.
of the eye are quite similar. So, when a ray of light enters the 75. A prism ABC (with BC as base) is placed in different
eye, it is refracted at the cornea. This refraction produces a real orientations. A narrow beam of white light is incident on
inverted and diminished image of distant objects on the retina. the prism as shown in figure. In which of the following
When the object is kept at different distances then, we may cases, after dispersion, the third colour from the top
expect the image to be formed at different distances from the corresponds to the colour of the sky?
lens. It means, it may not form on the retina always.
S-58 Science
A C B focal length. The ciliary muscles are most strained in this
position. For an average grown-up person minimum distance
(a) (b) of object should be around 25 cm.

B C A A person suffering for eye defects uses spectacles (Eye glass).


(ii)
(i) The function of lens of spectacles is to form the image of the
C C objects within the range in which person can see clearly. The
A image of the spectacle-lens becomes object for eye-lens and
(c) (d) whose image is formed on retina.
A
B B The number of spectacle-lens used for the remedy of eye defect
(iii) (iv) is decided by the power of the lens required and the number
76. Which of the following statements is correct regarding of spectacle-lens is equal to the numerical value of the power
the propagation of light of different colours of white light of lens with sign. For example power of lens required is +3D
in air? (converging lens of focal length 100/3 cm) then number of lens
(a) Red light moves fastest will be +3.
(b) Blue light moves faster than green light
For all the calculations required you can use the lens formula
(c) All the colours of the white light move with the and lens maker’s formula. Assume that the eye lens is
same speed
equiconvex lens. Neglect the distance between eye lens and the
(d) Yellow light moves with the mean speed as that of
spectacle lens.
the red and the violet light
80. Minimum focal length of eye lens of a normal person is
77. When white light is allowed to pass through a glass prism,
(a) 25 cm (b) 2.5 cm
which colour deviates the least?
(c) 25/9 cm (d) 25/11 cm
(a) Violet (b) Red
(c) Green (d) Orange 81. Maximum focal length of eye lens of normal person is
(a) 25 cm (b) 2.5 cm
78. When white light is allowed to pass through a glass prism,
(c) 25/9 cm (d) 25/11 cm
which colour deviates the most?
(a) Indigo (b) Green 82. A nearsighted man can clearly see object only upto a
(c) Red (d) Violet distance of 100 cm and not beyond this. The number of
the spectacles lens necessary for the remedy of this defect
79. For a prism material, refractive index is highest for will be
(a) Red (a) +1 D (b) ­– 1 D
(b) Yellow (c) + 3 D (d) – 3 D
(c) Orange
(d) Violet Passage Based Questions Assertion & Reason
Case/Passage - 3
DIRECTIONS : Each of these questions contains an assertion
The ciliary muscles of eye control the curvature of the lens followed by reason. Read them carefully and answer the question
in the eye and hence can alter the effective focal length of on the basis of following options. You have to select the one that
the system. When the muscles are fully relaxed, the focal best describes the two statements.
length is maximum. When the muscles are strained the (a) If both Assertion and Reason are correct and Reason is
curvature of lens increases (that means radius of curvature the correct explanation of Assertion.
decreases) and focal length decreases. For a clear vision the (b) If both Assertion and Reason are correct, but Reason is
image must be on retina. The image distance is therefore not the correct explanation of Assertion.
fixed for clear vision and it equals the distance of retina from
(c) If Assertion is correct but Reason is incorrect.
eye-lens. It is about 2.5 cm for a grown-up person.
(d) If Assertion is incorrect but Reason is correct.
A person can theoretically have clear vision of objects situated
at any large distance from the eye. The smallest distance at 83. Assertion : When ray of light falls on the particles of a
colloidal solution, the path of the beam is visible.
which a person can clearly see is related to minimum possible
The Human Eye and the Colourful World S-59
Reason : Path of light is visible due to the scattering of
light by the colloidal particles. Fill in the Blanks
84. Assertion : Sun looks white at noon.
DIRECTIONS : Complete the following statements with an
Reason : At noon, the light has to travel longer distance appropriate word / term to be filled in the blank space(s).
through the atmosphere before reaching the eye of an observer.
91. The coloured diaphragm between the cornea and the lens
85. Assertion : When a ray of light passes through a prism, it
is ................
bends towards the thicker part of the prism.
92. The middle point of the iris has a hole, which is called
Reason : An incident ray strikes a prism, undergoes
.............
refraction and comes out as an emergent ray.
93. The screen on which the image is formed by the lens
86. Assertion: Myopia is due to the increased converging
system of the human eye is called ...............
power of the eye lens.
Reason: Myopia can be corrected by using spectacles 94. For young adult with normal vision, least distance of
made from concave lenses. distinct vision = .............

87. Assertion: The twinkling of stars is due to the fact that 95. The closest distance at which the eye can focus clearly is
refractive index of the earth’s atmosphere fluctuates. called the .................

Reason: In cold countries, the phenomenon of looming 96. For a normal eye, the range of vision is from .................
(i.e., ship appears in the sky) takes place, because 97. A person is short-sighted if his eyeball is too ....................
refractive index of air decreases with height.
98. The eye which cannot simultaneously see with the
88. Assertion: When we see an object, the image formed on same distinctness all objects or lines making different
the retina is real and inverted. inclinations is said to suffer from ..................
Reason: If the magnification of a system is less than one, 99. The defect of the eye due to which a person is unable to
then the image formed is inverted. distinguish between certain colours, known as ..............
89. Assertion: Rainbow is an example of the dispersion of 100. The ability of the eye to focus both near and distant
sunlight by the water droplets. objects, by adjusting its focal length, is called the ..........
Reason: Light of shorter wavelength is scattered much 101. The smallest distance, at which the eye can see objects
more than light of larger wavelength. clearly without strain, is called the ............. of the eye.

Match the Following 102. The splitting of white light into its component colours is
called .....................

DIRECTIONS : Each question contains statements given in 103. .................. causes the blue colour of sky and the
two columns which have to be matched. Statements (A, B, C, reddening of the Sun at sunrise and sunset.
D) in column I have to be matched with statements (p, q, r, s) in 104. Sunlight comprises ............... colours.
column II.

90. Column II gives lens that can be use to correct the defect
True / False
of vision given in column I, match them correctly.
DIRECTIONS : Read the following statements and write your

Column I Column II answer as true or false.
(A) Myopia (p) Convex lens
105. Lens which is used for correcting the presbyopia defect
(B) Hypermetropia (q) Concave lens of the eye is concave.
106. The colour that deviates maximum while passing through
(C) Astigmatism (r) Cylindrical lens
a glass prism is violet.
(D) Presbyopia (s) Bi-focal lens Fill in the
107. Water droplets act as tiny prism in the formation of
Blanks
rainbow.
S-60 Science
108. The transparent spherical membrane covering the front of 113. Hypermetropia is corrected by using a convex lens of
the eye is known as cornea. suitable power.
109. The eye which can see near object clearly is said to suffer 114. A person suffering from myopia cannot see distant objects
from hypermetropia. clearly.
110. The eye which cannot see distant objects clearly is said to 115. The sun looks red at sunset because most of the blue light
suffer from myopia. in sunrays is scattered leaving behind red and yellow
111. Colour blindness is a genetic disorder which occurs by lights.
inheritance. 116. Clouds look white because water droplets of clouds
112. In Myopia the image of distant objects is focused before scatter all colours of light equally.
the retina. 117. The sun is visible two minutes before the actual sunrise
due to atmospheric refraction.
The Human Eye and the Colourful World S-61

ANSWER KEY & SOLUTIONS


1. (c) The ability of the eye lens to change its shape to v = (–100)
focus near and distant objects clearly is called power
1 1 1 1
of accomodation. = − =

f (−100) (−∞) 100
2. (c) Curvature of eye lens is adjusted with the help of
ciliary muscles. f = – 100 cm = – 1 m

3. (a) In poorly lit room or dim light the iris expands the power = – 1 D
pupil to allow more light to enter the eye. 28. (b) 29. (a) 30. (a) 31. (c)
4. (b) 5. (d) 6. (a)
32. (c) 33. (c) 34. (d) 35. (d)
7. (d) Near the horizon at sunrise and sunset, most of the
36. (d) 37. (a) 38. (b) 39. (b)
blue light and shorter wavelengths are scattered
away an hence sun appears red. 40. (a) 41. (c) 42. (d) 43. (a)
8. (a) Scattering of light is not enough at such heights. 44. (a) 45. (b) 46. (b) 47. (c)
9. (d) 10. (a) 11.
(d) 12. (c)
48. (a)
13. (a) Dispersion arises because of basic phenomenon
49. (a) Difference in refractive indices of blue and green
refraction.
colour are less so they are seen together and red is
14. (a) 15. (a) 16. (c) 17. (b) seen separate because deviation depends on refractive
index.
18. (b) 19. (d)
20. (d) v = – 15 cm, u = – 300 cm 50. (c) 51. (c) 52. (c) 53. (c)

1 1 1 54. (a) 55. (b) 56. (c) 57. (a)


As = −
f v u 58. (b)
1 1 1 −19 59. (c) Bifocal lens– Convex lens (lower part) is used
⇒= − =
f ( −15) ( −300) 300 to read books and concave lens (upper part) for
viewing distant object.
f = – 15.8 cm = – 0.158m
60. (d) As refractive index of air in atmosphere fluctuates,
−100 × 19
Power P = = – 6.33 D starts twinkle in the night.
300
21. (d) 22. (d) 23. (a) 24. (c) 25. (b) 61. (c) Red object that reflects only red and absorbs any other
colour incident upon it.
26. (b) v = – 60 cm, u = – 12 cm
62. (a) Myopia is the defect of eye where person is not able
1 1 1 to see f ar off objects and see near by objects clearly.
∴ − =
(−60) (−12) f
63. (a) In mypoia defect, image formation will take place
⇒ 1 = 1 ⇒ f = 15 cm = 15 m before the retina.
f 15 100 64. (c) u = 2 meter, v = 0.4 meter, f = ?
100 20 1 1 1 –5 + 1 –4
Power
= = D = – = =
15 3 f −0.4 (−2) 2 2
1 1 1 1 –4 1
27. (c) = − = = – 2 ; P= = 2 D (concave lens)
f v u f 2 f
Here u = (–∞)
S-62 Science
65. (b) Rainbow is circular because locus of reflected In the above figure, from top the third colour is
rays reaching eye of observer is a circle not due to yellow. But we can see that from bottom the third
roundness of earth. colour is blue (colour of sky). So, we can obtain the
correct situation by inverting the prism. Thus the
There is no rainbow on moon as there is no atmosphere.
required orientations can be found in case (ii).
In case of a primary rainbow, violet colour is on C B Violet
inside and red colour is on outside of arc. Indigo
Blue (Sky Colour)

In case of a secondary rainbow, red colour is on M


N
inside and violet colour is on outside of arc.
In late afternoon rainbow is visible in east side when
light of sun in west side is reflected and refracted by A

a layer of water droplets. So, option (b) is correct.


66. (a) In primary rainbow, two refraction and one TIR 76. (c) Speed of light is same for all colours of white light in
(1) Refraction of incident ray air but different colours have different wavelengths
and frequencies.
(2) TIR
77. (b) Red
(3) Again refraction when rays come out of liquid
drops 78. (d) Violet

(1) 79. (d) Violet


80. (d) 81. (b)
(2)
1 1 1
82. (b) = −
f v u
Here v = 2.5 (Distance of retina as position of image
(3) is fixed)

u = – x
In secondary rainbow, two refraction and two TIR.
1 1 1
67. (c) Rainbow will be observed only when the sun is at = +
the back side of observer. f 2.5 x
1 1 1
68. (b) The primary reason why the colour red is used for For fmin : x is minimum f = 2.5 + 25
min
traffic signals is that red light is scattered the least
1 1 1
by air molecules. So, the red light is able to travel For fmax : x is maximum = +
f max 2.5 ∞
the longest distance.
For near sighted man lens should make the image of
69. (c) In case of minimum deviation, the light ray inside
the object within 100 cm range
prism becomes parallel to base of the prism.
For lens u = –∞, v = –100
70. (c) 71. (b) 72. (b) 73. (d)
1 1 1
74. (c) = − ⇒P=–1D
f lens −100 −∞
75. (b) Generally, in case of a prism (i), the formation of 83. (a) It is due to phenomenon called Tyndall effect.
spectrum is shown below
84. (c) Sun look white at noon, as light has to travel shorter
A distance through the atmosphere before reaching the
MN is the Incident white light eye of an observer.

N
85. (b) When a light ray passes through denser medium
M
Orange
Red from a rarer it undergoes refraction.
Yellow
Green
Indigo Blue (Sky Colour) 86. (b) In myopic eye due to the increased converging
power of eye lens, the image of a far off object is
Violet
B C
formed in front of the retina.
The Human Eye and the Colourful World S-63
87. (b) 96. 25 cm to infinity 97.
long
88. (c) The image formed on retina is real and inverted. If 98. astigmatism. 99. colour blindness
magnification is less than 1, then diminished image
100. accommodation of the eye.
is formed not inverted.
101. near point 102. dispersion.
89. (b)
103. Scattering of light 104.
7
90. (A) → q; (B) → p; (C) → r; (D) → p
105. False 106.
True 107.
True 108. True
91. iris 92. pupil 93. retina
109. False 110.
True 111.
True 112. True
94. 25 cm. 95. near point
113. True 114. True 115. True 116. True
117. True
Carbon and Its
7 Acids,
Salts
Bases
Compounds
and

4. What does isomerism explain?


Multiple Choice Questions (MCQs) (a) A difference in molecular formulae.
DIRECTIONS : This section contains multiple choice (b) A difference in molecular weights.
questions. Each question has four choices (a), (b), (c) and (d) (c) A difference in chemical properties and structural
out of which only one is correct. formulae.
(d) A difference in molecular composition.
1. The correct name of the given compound is:
5. Buckminister fullerene is an allotropic form of
CH3
CH3 (a) phosphorus (b) sulphur
(c) carbon (d) tin
CH3
6. The number of 4° carbon atoms in 2,2,4,4-tetramethyl
CH3 pentane is –
(a) 2, 3-diethyl heptane (b) 5-ethyl-6-methyl octane (a) 1 (b) 2
(c) 4-ethyl-3-methyl octane (d) 3-methyl-4-ethyl octane (c) 3 (d) 4
2. Which of the following options is false about a soap? 7. Which is a general formula of alkenes?
(a) The soap solution in water is neutral and can be used (a) CnH2n+2 (b) CnH2n
to wash all kinds of fabrics. (c) CnH2n–2 (d) None of these
(b) Soap forms lather only in soft water. 8. The functional group represent alcohol is –
(c) Soap is a metallic salt of higher fatty acids. (a) – OH (b) – CHO
(d) Soap cannot be used in slightly acidic medium. (c) – COOH (d) > C = O
3. Structural formula of benzene is: 9. Which of the following is the purest form of carbon?
H H (a) charcoal (b) coal
C
H—C C—H C (c) diamond (d) graphite
H H
H C C
(a) C H
C — H (b) H 10. Organic compounds will always contain –
H H
H C C C (a) carbon (b) hydrogen
H H
C (c) nitrogen (d) sulphur
H H H
11. Methane, ethane and propane are said to form a
H H H homologous series because all are –
C C (a) hydrocarbons
H H
H
C C H—C
H (d)
C—H (b) saturated compounds
(c)
H
C
H H—C C—H (c) aliphatic compounds
C
H H C (d) differ from each other by a CH2 group
C
H H H
Carbon and Its Compounds S-65
12. When methane is burnt in an excess of air, the products of 21. Chlorine reacts with saturated hydrocarbons at room
combustion are – temperature in the
(a) C and H2O (b) CO and H2O (a) absence of sunlight
(c) CO2 and H2 (d) CO2 and H2O (b) presence of sunlight
13. Which of the following gases is called ‘marsh gas’? (c) presence of water
(a) H2 (b) CH4 (d) presence of hydrochloric acid
(c) C2H4 (d) C2H2 22. Pentane has the molecular formula C5H12. It has

14. The final product of chlorination of methane in the sun (a) 5 covalent bonds (b) 12 covalent bonds
light is – (c) 16 covalent bonds (d) 17 covalent bonds
(a) CH3Cl (b) CH2Cl2 23. Carbon forms four covalent bonds by sharing its four
(c) CHCl3 (d) CCl4 valence electrons with four univalent atoms, e.g.
hydrogen. After the formation of four bonds, carbon
15. The number of oxygen molecules used in the combustion attains the electronic configuration of:
of 1 molecule of ethanol is –
(a) helium (b) neon
(a) 1 (b) 2
(c) argon (d) krypton
(c) 3 (d) 4
24. Which of the following does not belong to the same
16. General formula of alkyne is –
homologous series?
(a) CnH2n+2 (b) CnH2n
(a) CH4 (b) C2H6
(c) CnH2n–2 (d) CnHn (c) C3H8 (d) C4H8
17. When vanaspati oil reacts with hydrogen then it is 25. The enzyme involved in the oxidation of ethanol to form
converted into vanaspati ghee. In this process catalyst vinegar is –
used is :
(a) zymase (b) oxidase
(a) Fe (b) Mo
(c) acetobacter (d) invertase
(c) V (d) Ni
26. Glacial acetic acid is –
18. Observe the following pairs of organic compounds :
(a) 100% acetic acid free of water
(I) C4H9OH and C5H11OH
(b) solidified acetic acid
(II) C7H15OH and C5H11OH
(c) gaseous acetic acid
(III) C6H13OH and C3H7OH
(d) frozen acetic acid
Which of these pair is a homologous series according to 27. When ethanoic acid is heated with NaHCO3 the gas
increasing order of carbon atom? evolved is –
(a) (III) only (b) (II) only (a) H2 (b) CO2
(c) (I) only (d) All of these (c) CH4 (d) C O
19. Carbon exists in the atmosphere in the form of : 28. During decarboxylation of ethanoic acid with sodalime
(a) carbon monoxide only. (NaOH + CaO), CO2 is removed as –
(b) carbon monoxide in traces, and carbon dioxide. (a) CO2 (b) CO
(c) carbon dioxide only. (c) Na2CO3 (d) CaCO3
(d) coal 29. When ethanoic acid reacts with ethanol, a sweet
smelling product is formed. The functional group in
20. Oils on treating with hydrogen in the presence of
the product is
palladium or nickel catalyst form fats. This is an example
of : (a) aldehyde (b) ketone
(a) addition reaction (c) alcohol (d) ester
(b) substitution reaction 30. Detergents can lather well in –
(c) displacement reaction (a) soft water (b) hard water
(d) oxidation reaction (c) river water (d) any one of the above
S-66 Science
31. ‘Drinking alcohol’ is very harmful and it ruins the health. 39. The total number of electrons and the number of electrons
‘Drinking alcohol’ stands for – involved in the formation of various bonds present in one
(a) drinking methyl alcohol molecule of propanal (C2H5CHO) are respectively.
(b) drinking ethyl alcohol (a) 32 and 20 (b) 24 and 20
(c) drinking propyl alcohol (c) 24 and 18 (d) 32 and 18
(d) drinking isopropyl alcohol 40. The number of structural isomers of the compound
32. The treatment of acetic acid with lithium aluminium having molecular formula C4H9Br is
hydride produces – (a) 3 (b) 5
(a) methanol (b) ethanol (c) 4 (d) 2
(c) ethanal (d) methanal
41. A sweet smelling compound formed by reacting acetic
33. The fermentation reactions are carried out in temperature acid with ethanol in the presence of hydrochloric acid is
range of – (a) CH3COOC2H5 (b) C2H5COOH
(a) 20-30°C (b) 30-40°C (c) C2H5COOCH3 (d) CH3OH
(c) 40-50°C (d) 50-60°C
42. Percentage of nitrogen in urea (NH2CONH2) is
34. Soaps are sodium salts of fatty acids. Which of the (a) 23.3% (b) 46.7%
following fatty acids does not form soap?
(c) 69.9% (d) 11.66%
(a) butyric acid (b) oleic acid
43. The molecular formula of carboxylic acid that differs
(c) palmitic acid (d) stearic acid
from the rest is
35. The OH group of an alcohol or the —COOH group of a
(a) C13H26O2 (b) C2H4O2
carboxylic acid can be replaced by —Cl using :–
(a) phosphorus pentachloride (c) C9H18O2 (d) C7H12O2

(b) hypochlorous acid 44. During laboratory preparation CH4 gas is collected by
(c) chlorine downward displacement of water because
(d) hydrochloric acid (a) CH4 is lighter than Air
36. Which compound represents the vinegar? (b) CH4 is poisonous gas
(a) HCOOH (b) CH3CHO (c) It does not dissolve in water
(c) HCHO (d) CH3COOH (d) All the above statements are correct

37. A & B both compounds give H2 gas with sodium. If A & 45. Which one of the following statement is incorrect about
B react in presence of acid catalyst then they form ethyl graphite and diamond ?
acetate. Thus, A & B would be - (a) Graphite is smooth and slippery.
(a) CH3COOH, CH3OH (b) Diamond is good conductor of heat.
(b) HCOOH, CH3COOH (c) Graphite is a good conductor of electricity.
(c) CH3COOH, C2H5OH (d) Physical and chemical properties of graphite and
diamond are same.
(d) C3H7COOH, C3H7OH
46. A compound ‘X’ reacts with a compound ‘Y’ , to produce
38. During the cleansing action of soap dirt is surrounded by a colourless and odourless gas. The gas turns lime water
soap molecules. Soap molecule is like a tadpole which milky. When ‘X’ reacts with methanol in the presence
has a head and tail. These head and tail respectively are: of concentrated H2SO4, a sweet smelling substance is
(a) hydrophobic and hydrophilic produced. The molecular formula of the compound ‘X’
(b) hydrophobic and hydrophobic is –
(c) hydrophilic and hydrophilic (a) C2H4O (b) C2H4O2
(d) hydrophilic and hydrophobic
(c) C2H6O (d) C2H6O2
Carbon and Its Compounds S-67
47. The functional groups present in the following compound (c) A = Sodium acetate, B = Ethanoic acid, C = Methyl
are – ethanoate, D = Methanol, E = Ethanol
O (d) A = Ethanol, B = Ethanoic acid, C = Methyl
ethanoate, D = Sodium acetate, E = Methanol
C — OH
52. In shaving creams________ is added to prevent rapid
drying.
O — C — CH3 (a) Methanol (b) Glycerol
(c) Ethanol (d) Glycol
O
53. An organic compound A on heating with concentrated
(a) alcohol, ketone and ester
H2SO4 gave product B and on warming with alkaline
(b) ester and carboxylic acid KMnO4 gave compound C. Compound A on heating
(c) carboxylic acid and ketone with compound C in presence of concentrated H2SO4
(d) ester and alcohol formed compound D, which has fruity smell. Identify the
compounds A, B, C and D:
48. A compound of carbon, hydrogen and nitrogen contains (a) A = Alcohol, B = Carboxylic acid,
these elements in the ratio of their atomic mass 9 : 1 : C = Alkene, D = Ester
3.5, if its molecular mass is 108 u. What is its molecular
(b) A = Carboxylic acid, B = Ester,
formula?
C = Alkene, D = Alcohol
(a) C2H2N (b) C3H4N (c) A = Alcohol, B = Alkene,
(c) C2HN2 (d) C6H8N2 C = Carboxylic acid, D = Ester
(d) A = Alkene, B = Alcohol, C = Ester,
49. How many grams of oxygen gas will be needed for
D = Carboxylic acid
complete combustion of 2 moles of 3rd member of alkyne
series ? 54. Two organic compounds ‘A’ and ‘B’ react with sodium
metal and both produce the same gas ‘X’, but with
(a) 186 g (b) 256 g sodium hydrogen carbonate, only compound B reacts to
(c) 352 g (d) 372 g give a gas ‘Y’. Identify ‘A’, ‘B’, ‘X’ and ‘Y’:
(a) A = Ethylene, B = Ethyl alcohol,
50. A hydrocarbon ‘A’ (C3H8) on treatment with chlorine X = Carbon dioxide, Y = Hydrogen
in presence of sunlight yielded compound ‘B’ as major (b) A = Ethyl alcohol, B = Acetic acid,
product Reaction of ‘B’ with aqueous KOH gave ‘C’ X = Hydrogen, Y = Carbon dioxide
which on treatment with concentrated H2SO4 yielded (c) A = Methyl alcohol, B = Ethyl alcohol,
‘D’. Hydrogenation of ‘D’ gave back ‘A’. The sequence X = Hydrogen, Y = Carbon dioxide
of reactions involved in above conversion is: (d) A = Acetic acid, B = Formic acid,
(a) substitution, substitution, addition, dehydration X = Carbon dioxide, Y = Hydrogen
(b) substitution, substitution , dehydration, addition
55. Fermentation of sugarcane juice produces
(c) substitution, dehydration, addition, addition
(a) Ethanol (b) Ethanal
(d) addition, substitution, dehydration, substitution.
(c) Acetic acid (d) Gluconic acid
51. An organic liquid ‘A’ with acidified potassium dichromate
gave product ‘B’. The compound ‘B’ on heating with 56. Antiknocking compound in gasoline is :
methanol in presence of concentrated sulphuric acid (a) Triethyl lead (b) Trimethyl lead
formed compound ‘C’ which on subsequent treatment (c) Tetramethyl lead (d) Tetraethyl lead
with sodium hydroxide formed two product ‘D’ and ‘E’.
The product ‘D’ is known to affect the optic nerve causing 57. Identify the correct order of boiling points of the following
blindness. Intake of ‘D’ in very small quantities can cause compounds-
death. What are compound ‘A’, ‘B’, ‘C’, ‘D’ and ‘E’? (A) CH3CH2CH2CH2OH
(a) A = Ethanol, B = Ethanoic acid, C = Methanol (B) CH3CH2CH2CHO
D = Sodium acetate, E = Methyl ethanoate (C) CH3CH2CH2COOH
(b) A = Ethanol, B = Ethanoic acid, C = Methyl (a) (A) > (B) > (C) (b) (C) > (A) > (B)
ethanoate D = Methanol, E = Sodium acetate
(c) (A) > (C) > (B) (d) (C) > (B) > (A)
S-68 Science
58. Ethane with the molecular formula C2H6 has : Reactions in which the compounds react with oxygen and form
(a) 6 covalent bonds (b) 7 covalent bonds carbon dioxide and water is known as combustion reaction. This
(c) 8 covalent bonds (d) 9 covalent bonds process occurs with release of great amount of heat.
64. The reaction
59. Butanone is four-carbon compound with the functional
CH4 + Cl2 → CH3Cl + HCl is :
group :
(a) substitution reaction
(a) carboxylic acid (b) aldehyde
(b) addition reaction
(c) ketone (d) alcohol
(c) rearrangement reaction
60. While cooking, if the bottom of the vessel is getting (d) elimination reaction
blackened on the outside, it means that :
65. The reaction CH2 = CH2 + H2 → CH3 – CH3 is :
(a) the food is not cooked completely. (a) substitution reaction
(b) the fuel is not burning completely. (b) addition reaction
(c) the fuel is wet. (c) rearrangement reaction
(d) the fuel is burning completely. (d) elimination reaction
66. The reaction C2H6 + O2 → 2CO2 + 3H2O is :
(a) substitution reaction
(b) rearrangement reaction
DIRECTIONS : Study the given case/passage and answer the
(c) addition reaction
following questions.
(d) combustion reaction
Case/Passage - 1
Case/Passage - 3
A carbon atom attached to one, two, three and four other carbon
The given diagram represent an experiment in which a test tube
atoms is called primary, secondary, tertiary and quaternary
contains 1 mL of ethanol (absolute alcohol) and 1 mL glacial
carbon respectively. Now consider following compound and
acetic acid along with a few drops of concentrated H2SO4.
answer the following questions.
Observe the diagram and answer the following questions.
CH3
F E D C B A
CH3 CH2 CH2 C CH CH3 Test tube
containing
CH3 CH3 reaction
mixture
61. In above compound how many carbon atom are primary? Beaker
Water
(a) 7 (b) 5 Wire gauze
(c) 6 (d) 4 Tripod stand
62. In above compound how many carbon atoms are Burner
secondary?
(a) 2 (b) 1
(c) 3 (d) 0
67. Name the type of reaction taking place in this experiment.
63. In above compound which carbon atom is quaternary?
68. Write the chemical equation.
B
(a) D
(b)
F
(c) C
(d) 69. Why reverse of this reaction is known as saponification
reaction?
Case/Passage - 2
Reactions in which an atom or a group of atoms is replaced by 70. Give two uses of the resulting product.
some other atom or another group of atoms without causing any Case/Passage - 4
change in the structure of the remaining part of the molecule, Food, clothes, medicines, books, or many of the things are
are called substitution reactions. all based on this versatile element carbon. In addition, all
All organic compounds containing double or triple bonds living structures are carbon based. The earth’s crust has only
give addition reactions, i.e., alkenes, alkynes and aromatic 0.02% carbon in the form of minerals. The element carbon
hydrocarbons give addition reactions. occurs in different forms in nature with widely varying
Carbon and Its Compounds S-69
physical properties. Both diamond and graphite are formed 77. Assertion : The correct IUPAC name for the compound
by carbon atoms, the difference lies in the manner in which CH3 CH3
the carbon atoms are bonded to one another. Carbon has the | |
unique ability to form bonds with other atoms of carbon, H3C — CH — CH2 — CH — CH2 — CH3
giving rise to large molecules. This property is called
catenation. is 2, 4 dimethyl hexane not 3, 5 dimethyl hexane
71 From the given alternatives, whose chemical and physical Reason: When the parent chain has two or more
properties are not same? substitutents, numbering must be done in such a way that
(a) Graphite and Diamond the sum of the locants on the parent chain is the lowest
(b) Phosphorous and Sulphur possible.
(c) Carbon and Hydrogen 78. Assertion: Vegetable oil is converted into vegetable ghee
(d) Methyl alcohol and Acetic acid by hydrogenation process in presence of nickel catalyst.
72. Which of the following statements is not correct?
Reason: Unsaturated hydrocarbons add hydrogen to give
(a) Graphite is much less dense than diamond saturated hydrocarbon in presence of a catalyst.
(b) Graphite is black and soft
79. Assertion: Unsaturated hydrocarbon burns with sooty
(c) Graphite has low melting point
flame in excess supply of air.
(d) Graphite feels smooth and slippery
73. Which of the following are isomers? Reason: Saturated hydrocarbon has more carbon content.
(a) Butane and isobutene 80. Assertion : Following are the members of a homologous
(b) Ethane and ethene series :
(c) Propane and propyne CH3OH, CH3 CH2OH, CH3CH2CH2OH
(d) Butane and isobutane Reason : A series of compounds with same functional
group but differing by – CH2 – unit is called a homologous
74. Which one of the following is not an allotrope of carbon? series.
(a) Soot (b) Graphite
81. Assertion : Diamond and graphite are allotropes of
(c) Diamond (d) Carborundum
carbon.
75. Pentane has the molecular formula C5H12. It has
(a) 5 covalent bonds (b) 12 covalent bonds Reason : Some elements can have several different
structural forms in the same physical state. These
(c) 16 covalent bonds (d) 17 covalent bonds
differing forms are called allotropes.
82. Assertion : Carbon monoxide is extremely poisonous in
Assertion & Reason nature.

DIRECTIONS : Each of these questions contains an assertion Reason : Carbon monoxide is formed by complete
followed by reason. Read them carefully and answer the question combustion of carbon.
on the basis of following options. You have to select the one that 83. Assertion : Carbon has ability to form long carbon
best describes the two statements. chains.
(a) If both Assertion and Reason are correct and Reason is
the correct explanation of Assertion. Reason : Carbon has a unique property to form long
straight and branched chains called catenation.
(b) If both Assertion and Reason are correct, but Reason is
not the correct explanation of Assertion. 84. Assertion : All alcohols have similar chemical properties.

(c) If Assertion is correct but Reason is incorrect. Reason : All alcohols contains similar hydroxy (–OH)
functional group.
(d) If Assertion is incorrect but Reason is correct.
85. Assertion : Hydrogenation converts an oil into a fat,
76. Assertion: Ethanoic acid is called as glacial acetic acid. called vegetable ghee.
Reason: On cooling it freezes to form ice-like flakes. Reason : Hydrogenation is carried out in presence of a
They appear like a glaciers. catalyst, usually finely divided nickel.
S-70 Science
91. The soft crystalline form of carbon is ................ .
Match the Following
92. Next homologue of ethane is ............... .
DIRECTIONS : Each question contains option given in two 93. Valency of carbon in ethylene is ............... .
columns. options (A, B, C, D) in column I have to be matched 94. Ethylene burns in air to form CO2 and ............... .
with options (p, q, r, s) in column II.
95. The molecular mass of any two adjacent homologues
86. Column I Column II differ by ................. amu.
UV light
(A) Combustion reaction (p) C3H8 + Cl2 → 96. The purest form of carbon is ..................... .
C3H7Cl + HCl
97. The general formula of alcohols is ................ .
(B) Oxidation reaction (q) CH2= CH2+H2
Ni/Pd
98. The functional group present in carboxylic acids is
→ CH3– CH3 .....................
(C) Addition reaction (r) 2CH4+ O2(g)
99. Detergents cause ................. pollution.
300 °C
→
Molybdenum oxide
HCHO + 2H2O True / False
(D) Substitution reaction (s) C2H5OH + 3O2 →
DIRECTIONS : Read the following statements and write your
2CO2 + 3H2O answer as true or false.
87. Column I Column II
100. Carbon is a versatile element.
(A) — CHO (p) Azo compounds
(B) — CONH2 (q) Aldehydes 101. Carbon forms covalent bonds with itself and other
elements such as hydrogen, oxygen, sulphur, nitrogen
(C) — NH2 (r) Acid amides
and chlorine.
(D) — N = N — (s) Amines
88. Column I Column II 102. Carbon and its compounds are some of our major sources
of fuels.
(A) CH2 = CH2 (p) Saturated
103. Graphite is a good conductor of electricity.
(B) CH2 (q) Unsaturated
104. The simplest saturated hydrocarbon is methane.
CH2 CH2 105. Ethanol is the first member of the alcohol homologous
(C) CH3 – CH2 – CH3 (r) Acyclic series.
CH 106. Diamond is a good conductor of electricity.
(D)
CH CH (s) Cyclic
107. Graphite is used in pencils.
CH CH
CH 108. When hydrocarbons burn in air, carbon dioxide and
hydrogen are produced with heat energy.
Fill in the Blanks 109. If a hydrocarbon has double or triple covalent bond, it is
saturated.
DIRECTIONS : Complete the following statements with an
appropriate word / term to be filled in the blank space(s). 110. Unsaturated hydrocarbons give addition reactions.
111. By hydrogenation, vegetable oils are converted into
89. The ability of carbon to form chains gives rise to a vanaspati ghee.
............... series of compounds.
112. Invertase and amylase are two enzymes involved in
90. Newly discovered allotrope of carbon is ............. . fermentation of ethanol from sugar.
Carbon and Its Compounds S-71

ANSWER KEY & SOLUTIONS


1. (c) The name of the compound is 4-ethyl-3-methyl H H H H H
octane. | | | | |
22. (c) H − C − C − C − C − C − H
2. (a) The soap solution in water is not neutral and cannot be | | | | |
used to wash all kinds of fabrics. H H H H H
3. (c) Benzene molecule contains alternate single and Pentane has 16 covalent bonds
double bonds. Its formula is C6H6.
(12C – H and 4C – C bonds)
4. (c) Isomers have same molecular formula, molecular
weight and molecular composition but different 23. (b) 24. (d) 25. (c) 26. (a)
chemical properties and structural formulae 27. (b) CH3COOH + NaHCO3 

because the properties are based on the position of Ethanoic acid
atoms. CH3COONa + H 2O + CO2
5. (c) 28. (c)
CH3 CH3 O

| | 4° ||

acid
29. (d) CH3COOH + CH3CH 2OH 
6. (b) CH3 – C – CH2 – C – CH3 → CH3COCH 2CH3
| | ester
CH 3 CH 3
30. (d) 31. (b)
7. (b) LiAlH
32. (b) CH3COOH  4 → CH CH OH
3 2
8. (a) – OH ⇒ alcohol Acetic acid Ethanol

– CHO ⇒ aldehyde 33. (a) 34. (a) 35. (a) 36. (d)
– COOH ⇒ Carboxylic acid 37. (c) 38. (d)
C = O ⇒ Ketone H H H
| | |
9. (c) Diamond is the purest form of carbon. 39. (a) H − C− C− C =O
10. (a) | |
H H
11. (d) Methane (CH4), ethane (C2H6) and propane Propanal
(CH3CH2CH3) differ from each other by CH2 group.
Total no. of electrons of C atoms = 3 × 6 = 18
Hence these are said to form a homologous series.
Total no. of electrons of O atoms = 1 × 8 = 8
12. (d) 13. (b) 14. (d)
Total no. of electorns of H atoms = 6 × 1 = 6
15. (c) C2 H5OH + 3O 2 
→ 2CO 2 + 3H 2O
Total no. of electrons in one molecule
16. (c)
= 18 + 8 + 6 = 32 electrons
17. (d) Catalysts like Pd, Pt or Ni are used in hydrogenation
process. Total no. of bonds = 10
18. (c) C4H9OH and C5H11OH represent homologous series Each bond contains 2e–. Therefore no. of electrons
in increasing order of C atoms, other two also represent involved in bonding.
homologous series, but in decreasing order because
= 2 × 10 = 20 electrons.
they differ from each other by a CH2 group.
40. (c) (a) CH3CH2 CH2CH2 – Br
CH3CH2CH2CH2OH, CH3CH2CH2CH2CH2OH
1 – Bromobutane
19. (b) 20. (a) 21. (b)
S-72 Science
(b) CH3 – CH – CH2 – CH3 48. (d) C6H8N2 i.e. 12 × 6 + 8 × 1 + 14 × 2 = 108 u.
|
Br 49. (c) 2C4 H 6 + 11O 2 
→ 8CO 2 + 6H 2O
2-Bromobutane For 2 moles of hydrocarbon, 11 moles of O2 is required
CH3
| ∴ 11 moles = 11 × 32 = 352 g
(c)
3CH – CH – CH2Br
50. (b) C3H8(A) is propane.
1-Bromo-2-methyl propane Sunlight
(i) CH3CH2CH3 + Cl2 →
CH3 (A)
|
(d) CH3 – C – CH3 CH3CH2CH2Cl + HCl (Substitution)
| (B)
Br
2-Bromo-2-methyl propane (ii) CH3CH2CH2Cl + aq. KOH →
(B)
41. (a) Acetic acid reacts with ethanol in the presence
CH3CH2CH2OH + KCl (Substitution)
of hydrochloric acid to produce ethyl acetate (ester)
 (C)
which is a sweet smelling compound. conc. H SO
2 4→
(iii) CH3CH2CH2OH 
CH3COOH + C2 H5OH → CH3COOC2 H5 + H2O
Acetic acid Ethanol Ethyl acetate (C)
CH3CH= CH2 + H2O (Dehydration)
42. (b) NH2CONH2 → Molecular mass = 60   (D)
% of N = (28/60) × 100 = 46.7% Ni/Pt/Pd
(iv) CH3CH = CH2 + H2 →
43. (d) C13H26O2, C2H4O2, C9H18O2 → These acids (D)
contains single bond between carbon atoms (C–C). CH3CH2CH3 (Addition)
Its general formula is (CnH2nO2). (A)
C7H12O2 → It contains double bond (C = C). Its Acidified
general formula is (CnH2n–2O2). K2Cr2O7
51. (b) CH3CH2OH → CH3COOH
44. (c) CH4 is non-polar compound, so it cannot be (A) (B)
conc.
dissolved in water. H2SO4
CH3COOH + CH3OH → CH3COOCH3 + H2O
45. (d) Graphite and diamond show different physical (B) (D) Methyl ethanoate
and chemical properties. Diamond is colourless (C)

transparent substance. It does not conduct electricity. NaOH


Graphite is greyish black. It is good conductor of
electricity. CH3COONa + CH3OH
(E) (D)
46. (b) X + Y → gas (Colourless, odourless)
52. (b) Glycerol is added to shaving cream to prevent rapid
CH3COOH + NaHCO3 → CH3COONa + CO2 ↑ + H2O drying.
(Acetic acid)
53. (c) When ethanol (A) i.e. organic compound reacts with
CO 2 + Ca ( OH )2 → CaCO3 + H 2O conc. H2SO4, it forms ethene (B) along with water.
(lime water) (Milky water) When ethanol (A) reacts with alk. KMnO4, it will

H 2SO 4 get oxidised to ethanoic acid (C). Ethanol reacts
CH3COOH + CH3OH  → CH COOCH + H O
3 3 2 with ethanoic acid to form an ester called ethyl
(X)
ethanoate (D), which gives a fruity smell.
Here compound (X) is acetic acid.
Concentrated H2SO4
CH3CH2OH → CH2 = CH2 + H2O
O → Carboxylic (A) 443 K/∆
(B)
|| group
Alkaline
C–OH KMnO4 + Heat

47. (b)
CH3COOH
(C)
O  C  CH3 Concentrated
H2SO4
|| CH3CH2OH + CH3COOH ← → CH COOC H + H O
O → Ester group (A) (C)
3
(D)
2 5 2
Carbon and Its Compounds S-73
54. (b) When alcohol i.e. ethanol (A) and acid i.e. acetic 67. Esterification reaction
acid (B) reacts with a sodium metal, then it liberates
68. CH3COOH + CH3–CH2OH →
Acid
hydrogen gas (X).
Ethanoic acid Ethanol
2CH3CH2OH + 2Na → 2CH3CH2 O Na + H2↑ CH3– C – O – CH2 – CH3
(A) (X)

O
Ester
2CH3COOH +2Na → 2CH3 COONa + H2↑
(B) (X) 69. Reverse reaction is known as saponification reaction
because it is used in the prepration of soap.
When an acid reacts with carbonates or bicarbonates,
it librates CO2 gas. So, when acitic acid(B) reacts 70. Esters are used in making perfumes and as a flavouring
with NaHCO3, it librates CO2(Y) gas. agent.

CH3COOH + NaHCO3 → 71. (d) Due to presence of different functional groups


(B) methyl alcohol and acetic acid. Possess different
CH3 COO Na + H2O + CO2↑ physical and chemical properties.
  
(X) 72. (c)
55. (a) Fermentation of sugarcane produces ethanol. 73. (d) Butane and isobutane have same chemical formula
56. (d) Antiknocking agent in gasoline is tetraethyl lead but different arrangement of atoms and have
(TEL). It raises the octane value of gasoline. different structure.
H H H H H H H
57. (b) Carboxylic acid molecule has higher extent of
H-bonding than alcohol, therefore order of b.p. is H–C–C–C–C–H , H–C–C–C–H
H H H H H H
CH3CH2CH2COOH > CH3CH2CH2CH2OH H–C–H
(Butane)
 > CH3CH2CH2CH O H (Isobutane)

H H 74. (d) Carborundum is SiC (silicon carbide).


| |
58. (b) H − C − C − H H H H H H
| | | | | | |
H H
75. (c) H — C — C — C — C — C —H
i.e., it has seven covalent bonds. | | | | |
H H H H H
(6 C – H bonds and one C – C bond) pentane
59. (c) The structure of butanone is 76. (a) Ethanoic acid is also known as glacial acetic acid.
O
|| 77. (a) The correct IUPAC name for the compound is 2,
CH − CH − C − CH3 4-dimethyl hexane not 3, 5 dimethyl hexane.
3 2
H H
it has C = O (ketonic group) as its functional group. R R | |
Ni Catalyst
78. (a) C = C R – C – C –R
60. (b) The fuel is not burning completely, hence produce R R
H2
| |
carbon particles which get deposited on the bottom R R
of vessel. 79. (c) Unsaturated hydrocarbon has more carbon content
as compare to saturated hydrocarbon of comparable
61. (b) 62. (a) 63. (d)
molecular mass.

CH3 80. (a) CH3OH, CH3CH2OH, CH3CH2CH2OH belongs
|
1° 2°
CH3 CH2 CH2
2° 4° 3°
C CH CH3

to same homologous series with – OH functional
| | group and each member is differ by –CH2 – unit.
CH3 CH3
1° 1° 81. (a) 82. (c) 83. (a) 84. (a)
64. (a) 65. (b) 66. (d) 85. (b) Hydrogenation or hardening of oil converted various
unsaturated fatty glycerides to saturated glycerides
S-74 Science
by the addition of hydrogen in the presence of a 95. 14 96. diamond
catalyst, usually finely divided nickel.
97. CnH2n+1OH 98. –COOH
86. A – (s), B – (r), C – (q), D – (p)
99. water
87. A – (q), B – (r), C – (s), D – (p)
100. True 101. True 102. True
88. A – (q, r), B – (p, s), C – (p, r), D – (q, s)
103. True 104. True 105. False
89. homologous 90. fullerene
106. False 107. True 108. False
91. graphite 92. propane
109. False 110. True 111. True
93. 4 94. water
112. False
Periodic
8 Acids, Bases and
Classification
Salts
Elements
of

6. If Cl, Br and I, are Dobereiner’s triad and the atomic


Multiple Choice Questions (MCQs) masses of Cl and I are 35.5 and 127 respectively the
atomic mass of Br is –
DIRECTIONS : This section contains multiple choice (a) 162.5 (b) 91.5
questions. Each question has four choices (a), (b), (c) and (d)
out of which only one is correct. (c) 81.25 (d) 45.625
7. Newland could classify elements only upto –
1. The three elements calcium, strontium and barium form a
(a) copper (b) chlorine
triad.What is the basis of this grouping?
(i) Elements are in the increasing order of their atomic (c) calcium (d) chromium
weights. 8. Mendeleev classified elements in –
(ii) The atomic weight of the middle element is equal to (a) increasing order of atomic groups
the average of the atomic weight of extreme elements. (b) eight periods and eight groups
(iii) Elements in a triad have similar chemical properties. (c) seven periods and nine groups
(a) Only (i) and (ii) (b) Only (ii) and (iii) (d) eight periods and seven groups
(c) Only (i) and (iii) (d) (i), (ii) and (iii)
9. Noble gases were included in Mendeleev’s periodic table
2. Which one of the following elements will form an acidic in the –
oxide? (a) 1st group (b) 7th group
(a) An element with atomic number 7
(c) 8th group (d) none of these
(b) An element with atomic number 3
10. The long form of periodic table consists of –
(c) An element with atomic number 12 (a) seven periods and eight groups
(d) None of these
(b) seven periods and eighteen groups
3. On the basis of following features identify correct option. (c) eight periods and eighteen groups
(i) These elements majorly forms acidic oxides.
(d) eighteen periods and eight groups
(ii) These elements are majorly non-metals.
11. In the modern periodic table which of the following does
s-block elements
(a) (b) p-block elements not have appropriate position?
(c) d-block elements (d) f-block elements (a) Transition elements
4. Hydrogen has three isotopes 1H, 2H and 3H. On what (b)
Inert gases
basis these elements were placed in modern periodic
table ? (c)
Inner transition elements
(a) Atomic mass (b) Atomic number (d)
Halogens
(c) Both (a) and (b) (d) None of these 12. An element M has atomic number 9 and atomic mass 17.
5. An element ‘X’ is forming an acidic oxide. Its position in Its ion will be represented by –
modern periodic table will be (a) M (b) M2+
(a) Group 1 and Period 3 (b) Group 2 and Period 3 (c) M– (d) M2–
(c) Group 13 and Period 3 (d) Group 16 and Period 3
S-76 Science
13. The correct order of first IE of C, N, O, F is – (a) halogens (b) noble gases
(a) F > O > N > C (b) C > N > O > F (c) noble metals (d) light metals
(c) O > N > F > C (d) F > N > O > C 24. How many periods are there in the long form of the
14. Elements belonging to the same group have similar periodic table?
properties because – (a) 6 (b) 7
(a) they have similar electronic configuration of the (c) 8 (d) 9
outermost shell.
25. The elements with atomic numbers 3, 11, 19, 37 and 55
(b) their atomic numbers go on increasing as we move belong to
down the group. (a) alkali metals (b) alkaline earth metals
(c) all of them are metallic elements. (c) halogens (d) noble gases
(d) none of the above 26. The elements with atomic numbers 9, 17, 35, 53 and 85
15. The atoms of elements belonging to the same group of belong to
periodic table have the same – (a) alkali metals (b) alkaline earth metals
(a) number of protons (c) halogens (d) noble gases
(b) number of electrons 27. Each transition series contains a total of –
(c) number of neutrons (a) 2 elements (b) 8 elements
(d) number of electrons in the outermost shell (c) 10 elements (d) 18 elements
16. Which of the following is the correct order of relative 28. The number of elements in each of the inner transition
size? series are –
(a) I– > I+ > I (b) I– > I > I+ (a) 2 (b) 8
(c) I > I+ > I– (d) I+ > I– > I (c) 10 (d) 14
17. The element with the smallest size in the group 13 is – 29. The number of elements in the third period of the periodic
(a) beryllium (b) carbon table are –
(c) aluminium (d) boron (a) 2 (b) 8
(c) 18 (d) 32
18. The element present in the 4th period is –
(a) chlorine (b) iodine 30. The total number of elements in VII A group of the
periodic table are –
(c) fluorine (d) bromine
(a) 3 (b) 5
19. The most metallic element in the fifth period is –
(c) 7 (d) 9
(a) silver (b) rubidium
31. The total number of elements in the group IB are –
(c) gold (d) rhodium
(a) 3 (b) 5
20. If the two members of a Dobereiner triad are chlorine and
(c) 7 (d) 9
iodine, the third member of this triad is –
(a) fluorine (b) bromine 32. Which of the following elements has the least nonmetallic
character?
(c) sodium (d) calcium
(a) fluorine (b) chlorine
21. If the two members of a Dobereiner triad are phosphorus
(c) bromine (d) iodine
and antimony, the third member of this triad is –
(a) arsenic (b) sulphur 33. About how many known elements are there till date?
(c) iodine (d) calcium (a) 10 (b) 50
(c) 118 (d) 200
22. According to Mendeleev periodic law, the properties of
elements are periodic function of their – 34. Elements in the modern periodic table are arranged
(a) atomic masses (b) atomic numbers according to increasing –
(c) atomic volumes (d) densities (a) atomic number (b) atomic weight
(c) number of neutrons (d) chemical reactivity
23. The elements with atomic numbers 2, 10, 18, 36, 54 and
86 are all – 35. Which of these things you will not find in the periodic
table?
Periodic Classification of Elements S-77
(a) element name and symbol 47. Elements in which 4f orbitals are progressively filled are
(b) atomic weight called as –
(a) transition elements (b) lanthanides
(c) atomic orbital radius
(c) actinides (d) inert gases
(d) atomic number
48. Which of the following elements is a lanthanide (Rare–
36. Which scientist came up with the concept of a periodic earth element)?
table that included all of the known elements?
(a) cadmium (b) californium
(a) Joseph Priestly (b) Dmitri Mendeleev
(c) cerium (d) cesium
(c) Antoine Lavoisier (d) Albert Einstein
49. If the valene shell electronic configuration for an element
37. The alkali metals are in which group of the periodic is ns2np5, this element will belong to the group of –
table? (a) alkali metals (b) inert metals
(a) Group 1 (b) Group 2 (c) noble gases (d) halogens
(c) Group 3 (d) Group 4 50. If an atom has electronic configuration 1s2 2s2 2p6 3s2
38. As you go down the group, the alkali metals become – 3p6 3d 3 4s2, it will be placed in –
(a) brighter (b) hotter (a) second group (b) third group
(c) more reactive (d) less reactive (c) fifth group (d) sixth group
51. On moving from left to right across a period in the table
39. Where are the transition metals in the periodic table?
the metallic character –
(a) In group 0 (b) In group 1 (a) increases
(c) In group 2 (d) In a central block (b) decreases
40. The noble gases are unreactive because (c) remains constant
(a) they react with sodium. (d) first increases and then decreases
(b) they have a full outer shell of electrons. 52. Which of the following is the atomic number of a
metal?
(c) they have a half outer shell of neutrons.
(a) 32 (b) 34
(d) they are too thin.
(c) 36 (d) 38
41. Which of the following element is not in the liquid state? 53. Which has the maximum atomic radius?
(a) Hg (b) Li (a) Al (b) Si
(c) Ga (d) Br (c) P (d) Mg
42. Which of the following elements does not belongs to 54. Which one of the following ions has the highest value of
alkaline earth metal group? ionic radius?
(a) Rb (b) Sr (a) O2– (b) B3+
(c) Ba (d) Ra (c) Li+ (d) F–

43. Arrange the following in increasing order of their atomic 55. Which one of the following is the smallest in size?
radius : Na, K, Mg, Rb – (a) N3– (b) O2–
(c) F– (d) Na+
(a) Mg < K < Na < Rb (b) Mg < Na < K < Rb
(c) Mg < Na < Rb < K (d) Na < K < Rb < Mg 56. The size of the following species increases in the order –
44. Which is metalloid? (a) Mg2+ < Na+ < F– < Al
(a) Pb (b) Sn (b) F– < Al < Na+ > Mg2+
(c) Si (d) Zn (c) Al < Mg2+ < F– < Na+
45. Which shows variable valency? (d) Na+ < Al < F– < Mg2+
(a) s–block elements (b) p–block elements 57. The correct order of radii is –
(c) d–block elements (d) Radioactive elements (a) N < Be < B
46. Dobereiner triads is – (b) F– < O2– < N3–
(c) Na < Li < K
(a) Li, K, Rb (b) Mg, S, As
(c) Cl, Br, I (d) P, S, As (d) Fe3+ < Fe2+ < Fe4+
S-78 Science
58. Which of the following is correct regarding ionic radii? 65. The maximum number of electrons that can be filled in
(a) Ti4+ < Mn7+ (b) 35Cl– < 37Cl– the shell with the principal quantum number n = 4 is
(c) K+ > Cl– (d) P3+ > P5+ (a) 64 (b) 26
59. Consider following as a portion of the periodic table from (c) 18 (d) 32
Group No. 13 to 17. Which of the following statements 66. The ionic radii of N3–, O2–, F–, Na+ follow the decreasing
is/are true about the elements shown in it? order
I. V., W, Y and Z are less electropositive than X. (a) N3– > O2– > F– > Na+
II. V, W, X and Y are more .electronegative than Z. (b) N3– > Na+ > O2– > F–
III. Atomic size of Y is greater than that of W. (c) Na+ > O2– > N3– > F–
IV. Atomic size of W is smaller than that of X. (d) O2– > F– > Na+ > N3–
67. Consider the elements A, B, C and D with atomic numbers
V Z
6, 7, 14 and 15, respectively. Which of the following
Y statements are correct concerning these elements?
W I. D will lose electron more easily than C.
II. C will gain electron more easily than B.
X III. The element with highest electronegativity is D.
(a) I, II and III (b) II and III IV. The element with largest atomic size is C.
(c) I and IV (d) III and IV (a) I and II (b) II and III
60. Mendeleev’s periodic law states that the properties of (c) II and IV (d) III and IV
elements are a periodic function of their 68. Which of the following statement can help a chemistry
(a) reactivity of elements (b) atomic size student to predict chemical properties of an element?
(c) atomic mass (d) electronic configuration I. Position of element in the periodic table
II. Atomic number of the element
61. Chemical symbol of metal tungusten is
III. Number of shells in the atom
(a) W (b) Xe
IV. Number of electron in the outer most shell
(c) Y (d) Zr
(a) I, II and III (b) I, II and IV
62. Which is incorrect order of size? (c) I, III and IV (d) II, III and IV
(a) Na > Na+ (b) Na+ > Mg2+
69. The element which normally exist in the liquid state are
(c) Cl– > Cl (d) F– > O2–
(a) Bromine and Iodine
63. The anion O2– is isoelectronic with (d) Mercury and chlorine
(a) F+ (b) F– (c) Iodine and mercury
(c) N2– (d) N+3 (d) Bromine and mercury
64. A part of the modern periodic table is presented below in 70. Which gas being filled in weather balloon?
which the alphabets represent the symbols of elements. (a) Helium (b) Neon
Group 1 12 14 15 16 17 (c) Hydrogen (d) Nitrogen
→ 71. Manya, Kartik, Gurnoor and Sheena had arranged the
Period ↓ ions F–, Na+, O2– and Mg2+ in decreasing orders of their
2 M Q V ionic radii.
Manya – O2– > Mg2+ > F– > Na+
3 A J R W
Kartik – Mg2+ > Na+ > O2– > F–
4 E L T
Gurnoor – O2– > F– > Na+ > Mg2+
5 G X
Sheena – F– > Na+ > O2– > Mg2+
Consult the above part of the periodic table to predict
which of the following is a covalent compound- Who had provided the correct order of their decreasing
(a) RQ2 (b) AT ionic radii?
(a) Manya (b) Kartik
(c) JQ (d) JX2
(c) Gurnoor (d) Sheena
Periodic Classification of Elements S-79
72. Consider the elements A, B, C and D with atomic
numbers 11, 12, 16 and 17, respectively. Which among
the following statements regarding these elements are
correct? DIRECTIONS : Study the given case/passage and answer the
following questions.
I. The element C will gain electron more easily than
element D. Case/Passage - 1
II. The element B tends to lose electron more readily Metallic Character The ability of an atom to donate electrons
than C. and form positive ion (cation) is known as electropositivity
III. The oxide of A will be least basic while that of D or metallic character. Down the group, metallic character
will be most basic. increases due to increase in atomic size and across the period,
from left to right electropositivity decreases due to decrease
IV. The energy required to remove an electron from
in atomic size. Non-Metallic Character The ability of an atom
outermost shell from A will be minimum while that
to accept electrons to form a negative ion (anion) is called
from D will be maximum. non-metallic character or electronegativity. The elements
(a) I and III only (b) I and IV only having high electro-negativity have a higher tendency to gain
(c) II and III only (d) II and IV only electrons and form anion. Down the group, electronegativity
decreases due to increase in atomic size and across the period,
73. Which of the following is the correct order of reactivity
from left to right electronegativity increases due to decrease in
of metals?
atomic size.
(a) Mg > Al > Zn > Fe 300
(b) Mg > Zn > Fe > Al 250 Cs(262)
(c) Al > Mg > Zn > Fe Atomic radius/pm 200 K(231) Rb(244)
(d) Mg > Zn > Al > Fe Na(186)
150
Li(152)
74. The following is the correct decreasing order of the ionic 100 I(133)
Cl(99) Br(114)
radii- F(72)
50
(a) K+ > Ca2+ > S2– > Cl–
(b) K+ > Ca2+ > Cl– >S2– Atomic number (Z)
(c) Ca2+ > K+ >> Cl– >S2– 78. Which of the following correctly represents the decreasing
order of metallic character of Alkali metals plotted in the
(d) S2– > Cl– > K+ > Ca2+
graph? [CBSE Sample Issued 2021]
75. Electro-negativity of the following elements increase in (a) Cs > Rb > Li > Na > K
the order:
(b) K > Rb > Li > Na > Cs
(a) C, N, Si, P (b) Si, P, C, N
(c) Cs > Rb > K > Na > Li
(c) P, Si, N, C (d) N, Si, C, P
(d) Cs > K > Rb > Na > Li
76. Which of the following statements is not a correct
79. Hydrogen is placed along with Alkali metals in the
statement about the trends when going from left to right
modern periodic table though it shows non-metallic
across the periods of periodic table. character
(a) The elements become less metallic in nature. (a) as Hydrogen has one electron & readily loses
(b) The number of valence electrons increases. electron to form negative ion
(c) The atoms lose their electrons more easily. (b) as Hydrogen can easily lose one electron like alkali
(d) The oxides become more acidic. metals to form positive ion
(c) as Hydrogen can gain one electron easily like
77. Element X forms a chloride with the formula XCl2,
Halogens to form negative ion
which is a solid with a high melting point? X would
most likely be in the same group of the periodic table (d) as Hydrogen shows the properties of non-metals
as : 80. Which of the following has highest electronegativity?
(a) Na (b) Mg (a) F (b) Cl
(c) Al (d) Si (c) Br (d) I
S-80 Science
81. Identify the reason for the gradual change in lose electrons easily. Thus, metallic character generally
electronegativity in halogens down the group. decreases across a period and increases down a group.
(a) Electronegativity increases down the group due to 86. The non metallic character on moving along a period –
decrease in atomic size (a) increases (b) decreases
(b) Electronegativity decreases down the group due to (c) depends on the period (d) remains the same
decrease in tendency to lose electrons 87. Group 1 and group 2 elements are considered as strong
(c) Electronegativity decreases down the group due to metals because
increase in atomic radius/ tendency to gain electron (a) they have incomplete octet.
decreases (b) they can easily gain electrons.
(d) Electronegativity increases down the group due to (c) they can easily lose electrons.
increase in forces of attractions between nucleus & (d) they form anions.
valence electrons 88. Which of the following is the correct decreasing order of
82. Which of the following reason correctly justifies that metallic character?
“Fluorine (72pm) has smaller atomic radius than Lithium (a) Ca > Sc > Ti > K (b) K > Ca > Sc > Ti
(152pm)”? (c) K > Sc > Ca > Ti (d) Ti > Sc > Ca > K
(a) F and Li are in the same group. Atomic size increases Case/Passage - 4
down the group
Question numbers 1 – 3 are based on the periodic table. Study
(b) F and Li are in the same period. Atomic size increases the part of the modern periodic table presented below in which
across the period due to increase in number of shells the alphabets represent the symbols of elements and answer the
following questions.
(c) F and Li are in the same group. Atomic size decreases
down the group Group 1 12 14 15 16 17
(d) F and Li are in the same period and across the period →
atomic size/radius decreases from left to right. Period ↓
Case/Passage - 2 2 M Q V
The table given below refers to the elements of the periodic
table with atomic number from 3 to 18. These elements are 3 A J R W
shown by letters. (not by the usual symbols of the elements). 4 E L T
3 4 5 6 7 8 9 10
5 G X
A B C D E F G H
11 12 13 14 15 16 17 18 89. Consult the above part of the periodic table to predict
I J K L M N O P which of the given combination is a covalent compound:
RQ2, AT, JQ, JX2.
83. Which of the following are noble gases?
(a) H and P (b) G and O 90. Considering the above part of the periodic table, which of
(c) D and L (d) A and I the given element is the most electropositive element?

84. Which are halogens? 91. Which of the given element is the most electronegative
element?
(a) H and L (b) C and M
(c) G and O (d) E and P 92. Study the data of the following three categories A, B and C.

85. Which of the following elements have valency 4? Category Name of the Atomic Mass
(a) F and N (b) C and K element
(c) D and L (d) H and P Li 7
A Na 23
Case/Passage - 3 K 39
Group VII A elements are strong non-metals because they can N 14
easily accept an electron to form an anion whereas group 1 A
B P 31
element are strong metals because they can very easily lose one
As 74
electron to form cation.
Metals have the tendency to lose their valence electrons and B 10.8
form positive ions, so metallic character is related to the C Al 27
ionisation potential. Elements having low ionisation potential, Ga 69.7
Periodic Classification of Elements S-81
(i) From the given three categories A, B and C, Pick the Reason : Nitrogen has smaller atomic size than that of
one which forms Dobereiner’s Triads. oxygen.
(ii) Why did Mendeleev placed elements of category A, 100. Assertion : According to Mendeleev, periodic properties
B and C in three different groups? of elements are functions of their atomic number.
(iii) Is Newland law of octaves applicable to all the three Reason : Atomic number is equal to the number of
categories? protons.
Give reason to justify your answer. 101. Assertion : Elements in the same vertical column have
similar properties.
Assertion & Reason Reason : Elements have periodic dependence upon the
atomic number.
DIRECTIONS : Each of these questions contains an assertion
followed by reason. Read them carefully and answer the question Match the Following
on the basis of following options. You have to select the one that
best describes the two statements.
DIRECTIONS : Each question contains statements given in
(a) If both Assertion and Reason are correct and Reason is two columns which have to be matched. Statements (A, B, C, D)
the correct explanation of Assertion. in column I have to be matched with statements (p, q, r, s) in
(b) If both Assertion and Reason are correct, but Reason is column II.
not the correct explanation of Assertion.
102. Column II give period to which an element in column I
(c) If Assertion is correct but Reason is incorrect.
belongs, match them correctly.
(d) If Assertion is incorrect but Reason is correct.
Column I Column II
93. Assertion: Ionic size of N3– is greater than F–.
(A) Hydrogen (p) 3
Reason: N3– and F– are isoelectronic anions.
(B) Sodium (q) 4
94. Assertion: Atomic radius of aluminium atom is larger
(C) Calcium (r) 6
than magnesium atom.
(D) Barium (s) 1
Reason: Effective nuclear charge increase from
magnesium to aluminium. 103. Match the column –
95. Assertion: Oxygen atom is divalent in most of its Column I Column II
compounds.
(A) Element with largest (p) boron
Reason: Valency is the number of valence electrons size in second period
present in the outermost shell of its atom. (B) Element with smallest (q) fluorine
96. Assertion: Silicon, germanium are the metalloids in the size in group 13
modern periodic table.
(C) Element with maximum (r) bromine
Reason: Silicon, germanium has properties of metal as non-metallic character.
well as non metals. (D) Element with smallest (s) lithium
97. Assertion: Bulbs are usually filled with chemically active size in fourth period
gases.
104. Column I Column II
Reason: Nitrogen and argon gases are filled in order to (A) s-block elements (p) Alkali metals
prolong the life of the filament.
p-block elements
(B) (q) Alkaline earth metals
98. Assertion : Group 1 elements are known as the alkali
(C) Representative
elements.
elements (r) Halogens
Reason : s-orbital can accommodate only two electrons.
(D) High ionisation (s) Noble gases
99. Assertion : Nitrogen has higher ionization energy than
that of oxygen. energy
S-82 Science

Fill in the Blanks True / False

DIRECTIONS : Complete the following statements with an DIRECTIONS : Read the following statements and write your
appropriate word / term to be filled in the blank space(s). answer as true or false.

105. The law of triads was given by ........... . 116. As nuclear charge increases, atomic orbitals become
106. According to modern periodic law, the elements are smaller and more stable.
arranged in the periodic table in the order of their 117. As number of shells increases, atomic orbitals become
increasing ................ . larger and less stable.
118. Atomic radii decrease from left to right across a row of
107. Elements with eight electrons in their outermost energy
the periodic table.
shell are called ................. .
119. Atomic radii increase from top to bottom down a column
108. If two elements have the same number of valence of the periodic table.
electrons, then they belong to the same .............. of the
periodic table. 120. Fluorine has highest electron affinity in the periodic table.
121. Noble gases are placed extremely left in the periodic
109. The elements in groups 1, 2 and 13 to 18 are known as
table.
......... elements.
122. Magnesium is more metallic in nature than sodium.
110. The valency of an atom is equal to its .................... .
123. The number of shells increases in a given period from left
111. The atomic size in a period .............. from left to right. to right in the periodic table.
112. Dobereiner grouped the elements into triads and 124. The elements silicon, germanium and arsenic are called
Newlands gave the ................ . metalloids.
113. Mendeleev arranged the elements in increasing order of 125. Elements are classified on the basis of similarities in their
their .............. and according to their .......... properties. properties.
114. Mendeleev predicted the existence of some yet to be 126. Rows in the periodic table are called periods.
discovered elements on the basis of ....... in his periodic 127. The columns of the periodic table are called groups.
table.
128. You will find metals on the extreme right side of the
115. Elements in the modern periodic table are arranged in ...... periodic table.
vertical columns called ............. and .............. horizontal
rows called .............. . 129. Although the order of elements is based on atomic number,
vertical families share similar chemical properties.
Periodic Classification of Elements S-83

ANSWER KEY & SOLUTIONS


1. (d) Dobereiner noticed that strontium had similar 30. (b) The VII A group has 5 elements.
chemical properties as that of calcium and barium F, Cl, Br, I and At
and its atomic weight fell midway between the two.
31. (a) Group I B contain Cu, Ag and Au.
Hence, elements like calcium, strontium and barium
form a triad based on the given characteristics. 32. (d) Non-metallic character decreases in a group from
top to bottom, hence iodine will be least non-
2. (a) Non-metals form acidic oxides. Element with
metallic.
atomic number 7 (electronic configuration 2, 5) is
non-metal (N) and hence will form an acidic oxide, 33. (c) There are about 118 known elements listed in the
other elements Li(3) and Mg(12) are metals and periodic table.
hence form basic oxides. 34. (a) The elements of the modern periodic table are
3. (b) p-block elements majorly forms acidic oxides and organized according to increasing atomic number.
are non-metals. The atomic number represents the number of
electrons which is equal to number of protons in a
4. (b) On the basis of atomic number, elements were
neutral atom.
placed in the modern periodic table.
35. (c) You will not get information about the atomic radius
5. (d) Elements of group 16 and period 3 are non metals.
of an atom. Periodic table will have the atomic
Non metals generally form acidic oxides. Elements
number, atomic weight, name, and symbol for each
of group 1 and 2 form basic oxides while elements
element.
of group 13 form amphoteric oxides.
36. (b) Dmitri Mendeleev is credited with designing the
6. (c) According to Dobereneir’s triad the atomic mass
modern periodic table.
of Br will be average of the atomic masses of Cl
&I 37. (a) 38. (c) 39. (d) 40. (b)
35.5 + 127 41. (b) 42. (a) 43. (b) 44. (c)
=
Atomic mass of Br = 81.25
2 45. (c) 46. (c) 47. (b) 48. (c)

7. (c) 8. (c) 9. (d) 10. (b) 49. (d) 50. (c) 51. (b)

11. (c) 52. (d) 38 is the atomic no. of stronium (Sr) which is s-block
element and all elements of s-block are metals.
12. (c) The element is halogen and has one electron less
than inert gas configuration, hence its ion can be 53. (d) Mg, as we move across the period atomic radius
represented as M– ion. decreases.

13. (d) In a period, the value of ionisation potential 54. (a) O2– has the highest value of ionic radii as this can be
increases from left to right with breaks where  Nuclear charge 
explained on the basis of Z/e  
the atoms have stable configurations hence the  No. of electrons 
correct order will be
When Z/e ratio increases, the size decreases and
F > N > O > C when Z/e ratio decreases, size increases.
14. (a) 15. (d) 16. (b) 17. (d) 55. (d) Na+ < F– < O2– < N3–
18. (d) On moving along a period atomic radii decreases. All are isoelectronic, effective nuclear charge is
19. (b) The metallic character decreases as we move from highest for Na+ so it has smallest size.
left, to right in a period. 56. (a) Mg2+ < Na+ < F– < Al
20. (b) 21. (a) 22. (a) 23. (b) F– has bigger size than Mg2+ and Na+
24. (b) 25. (a) 26. (c) 27. (c) 57. (b) Ionic radii decreases significantly from left to right
28. (d) 29. (b) in a period among representative elements.
S-84 Science
58. (d) Nuclear charge per electron is greater in P5+. Atomic Electronic
Therefore, its size is smaller. Element
Number configuration
59. (c) Electropositive nature increases from top to bottom A(11) Na 2, 8, 1
in a group and decrease along a period. Therefore X B(12) Mg 2, 8, 2
is most electropositive.
C(16) S 2, 8, 6
Atomic size decreases along a period and increases D(17) Cl 2, 8, 7
down the group. Therefore W < X.
60. (c) Mendeleev’s periodic law states that the physical From the above table it is clear that:
and chemical properties of the elements are a (a) Chlorine (D) will gain electrons more easily than
periodic function of their atomic mass. sulphur (C).
61. (a) Tungsten → W (b) The oxide of sodium (A) which is an alkali metal,
62. (d) Size of F– is less than O2– will be the most basic while that of chlorine (D)
which is a halogen, will be the most acidic.
63. (b) Anion O2– has 10 electrons.
73. (a) The correct order of reactivity is as follows.
Anion F– has 10 electrons.
Mg > Al > Zn > Fe
64. (a) R and Q are members of Group 16th which is
non metallic having O, S, Se, Te etc. These are Alkali metals and alkaline earth metals are very
characterised by showing the formation of covalent reactive elements in the periodic table and reactivity
bond. decreases on moving from left to right in the periodic
table.
65. (d) The maximum number of electrons that can be filled in
the shell with principle quantum number(n) = 2n2. 74. (d) Among the isoelectronic species greater the ratio
larger will be the ionic radii, hence the order is
Maximum number of electrons = 2(4)2 = 32
66. (a) Increase in positive charge decrease the ionic radii S2– > Cl– > K+ > Ca2+
of cation while increase in negative charge increases 75. (b) Electro-negativity increases on moving left to right
the ionic size of anion. So N3– > O2– > F– > Na+. in the periodic table, while it decreases on moving
67. (c) down the group.

68. (b) Position, atomic number and number of electrons The correct order of E.N. is
in the outermost shell of an element predicts its Si < P < C < N
chemical properties. (1.8) (2.1) (2.5) (3.0)
69. (d) Bromine and mercury exist in the liquid state. 76. (c) As we move from left to right in a period no. of
70. (a) Helium is filled in weather balloon. shells remains same whereas one more electron is
71. (c) The ions F–, Na+, O2– and Mg2+ are isoelectronic added thus the size of atom decreases and hence the
species having same electronic configuration valence electron becomes more and more near to
but their nuclear charges differ from each other the nucleus and hold of nucleus on valence electron
because of their difference in the number of increases, due to this, the tendency of an atom to
protons in the nucleus. With increase in the lose valence electron decreases.
number of protons in the nucleus, the electrons are 77. (b) X forms XCl2 suggests that X is in +2 state in XCl2.
more attracted towards nucleus thereby causing Therefore X would most like be in gp 2 of periodic
the decrease in ionic radius. Therefore, the given table.
ions are 78. (c) Cs > Rb > K > Na > Li
F– : no. of proton = 9 and no. of electron = 10 79. (b) As Hydrogen can easily lose one electrion like alkali
Na+ : no. of proton = 11 and no. of electron = 10 metals to form positive ion
O2– : no. of proton = 8 and no. of electron = 10 80. (a) F
Mg2+ : no. of proton = 12 and no. of electron = 10 81. (c) Electronegativity decreases down the group due to
72. (d) increase in atomic radius/ tendency to gain election
decreases.
Periodic Classification of Elements S-85
82. (d) F and Li are in the same period and across the period 98. (b) Group I elements are known as alkali metals as the
atomic size/radius decreases from left to right. hydroxides of these metals are soluble in water and
83. (a) H and P have complete octet. these solutions are highly alkaline in nature.

84. (c) G and O 99. (c) Nitrogen has higher ionisation energy as it has stable
half filled electronic configuration.
Both have 7 electrons in their outermost shell.
100. (d) According to Mendeleev, periodic properties of
85. (c) Both have four electrons in their outermost shell. elements is a function of their atomic masses.
86. (a) 87. (c) 101. (b)
88. (b) On moving along a period metallic character 102. A → (s); B → (p); C → (q); D → (r)
decreases.
103. A → (s); B → (p); C → (q); D → (r)
89. R and Q are members of group 16th having elements,
O, S, Se, Te etc. RQ2 is characterised by showing the 104. A → (p, q); B → (r, s); C → (p, q, r); D → (r, s)
formation of covalent bond. 105. Dobereiner 106. atomic number
90. Element ‘G’ is the most electropositive element. 107. noble gases 108. group
91. Element ‘V’ is the most electronegative element 109. main group 110. combining capacity
92. (i) Dobereiner’s Triads is A. 111. decreases 112. law of octaves
(ii) Mendeleev placed elements of category A,B and C 113. atomic masses, chemical 114. gaps
in three different groups because they have different
115. 18, groups, 7, periods
physical and chemical properties.
116. True 117. True 118. True
(iii) No, Newland’s Law of octaves is not applicable for
all three categories. 119. True 120. False

Because the law of octaves states that every eighth 121. False
element has similar properties when the elements Noble gases are placed extremely right in the periodic
are arranged in the increasing order of their atomic table.
masses. 122. False 123. False
93. (b) N3– and F– are isoelectronic anions. N3–
has only The number of shells remain same in a given period.
seven protons and F– has nine. Therefore, N3– has
124. True 125. True
larger ionic size.
126. True
94. (d) Atomic radius of aluminium atom is smaller than
magnesium atom because of increased effective Rows in the periodic table are called periods. The columns
nuclear charge. of the periodic table are called groups.
95. (c) Valency of an element is determined by the number 127. True
of valence electrons present in the outermost shell of 128. False
its atom.
Inert gases are found on the far right of the periodic table.
96. (a) Silicon and germanium are among the metalloids in Halogens are in the second group form the right. Metals
the modern periodic table. of all types are found around the left. Transition metal are
97. (d) Bulbs are usually filled with chemically inactive gases. found in the middle side of the periodic table.
Nitrogen and argon gases are inactive and are filled 129. True
in order to prolong the life of the filament.
How Do
9 Acids, Bases
Organisms
Salts
Reproduce
and

8. Gemmule formation in sponges is helpful in:


Multiple Choice Questions (MCQs) (a) Parthenogenesis

DIRECTIONS : This section contains multiple choice (b) Sexual reproduction


questions. Each question has four choices (a), (b), (c) and (d) (c) Only dissemination
out of which only one is correct.
(d) Asexual reproduction
1. The vegetative reproduction in sweet potato is done by – 9. In mammals, the testes lie in scrotal sacs due to
(a) stem (b) leaf (a) presence of urinary bladder.
(c) root (d) flower (b) presence of rectum.
2. Menstrual cycle is generally of (c) long vas-deferens.
(a) 21 days (b) 28 days (d) requirement of low temperature for
(c) 38 days (d) 40 days spermatogenesis.
3. Tunica albuginea is the covering around 10. The following figure represents :
(a) ovary (b) testes
(c) kidney (d) heart
4. Example of external fertilization is
(a) fish and frog (b) frog and monkey
(c) dog and goat (d) goat and fish
5. Plants of desired qualities are produced by:
(a) Cutting
(b) Grafting
(c) Layering (a) Budding in Hydra
(d) Any one of (a), (b),(c) (b) Budding in Planaria
6. Budding and fission are processes used by (c) Regeneration in Planaria
(a) diocious species. (d) Regeneration in Hydra
(b) hermaphroditic organisms.
11. The correct sequence of reproductive stages seen in
(c) organisms requiring new gene combinations for flowering plants is
each generation.
(a) gametes, zygote, embryo, seedling
(d) asexually reproducing species.
(b) zygote, gametes, embryo, seedling
7. Ovulation in mammals is caused by: (c) seedling, embryo, zygote, gametes
(a) FSH and TSH (b) FSH and LH
(d) gametes, embryo, zygote, seedling
(c) FSH and LTH (d) LTH and LH
How Do Organisms Reproduce S-87
12. (a) transport gametes
(b) produce mature gametes
(c) eliminate waste products through the urethra
(d) express secondary sex characteristics
15. Vegetative propagation is possible by
(a) Root (b) Stem
(c) Leaves (d) All of these
16. Ovaries produce
(a) oestrogen and progesterone
(b) oestrogen only
The figure shown above illustrates which method of (c) progesterone only
vegetative propagation? (d) testosterone
(a) Bud formation
17. Site of fertilization in mammals is
(b) Grafting (a) ovary (b) uterus
(c) Layering (c) vagina (d) fallopian tube
(d) Spore formation 18. Fertilization does not occur in which part?
13. In figure, the parts A, B and C are sequentially: (a) fallopian tube (b) ampulla
(c) oviduct (d) vagina
19. Seminiferous tubules are composed of
(a) Spermatogonia (b) Glandular epithelium
(c) Sensory epithelium (d) Germinal epithelium
(a) cotyledon, plumule and radicle
20. Cowper’s glands are found in
(b) plumule, radicle and cotyledon
(a) male mammals (b) female mammals
(c) plumule, cotyledon and radicle (c) male amphibians (d) female amphibians
(d) radicle, cotyledon and plumule
21. Progesterone is secreted by
14. The diagrams below represent the reproductive systems (a) corpus luteum (b) thyroid
in the human male and female. (c) thymus (d) testes
22. The general method of asexual reproduction, in yeast
fungus is –
(a) by spores (b) by budding
(c) binary fission (d) gemma
23. Oral-contraceptives prevent the
(a) fertilization
(b) ovulation
(c) implantation
(d) entrance of sperms in vagina
24. The process of development of organism like itself is
called
(a) budding (b) flowering
(c) reproduction (d) none of the above
25. Which of the following organisms do not depend on
reproduction to exchange genetic information?
The blockages shown at A and B would most likely (a) animals (b) plants
interfere with the ability to (c) bacteria (d) fungi
S-88 Science
26. Like animals, plants produce (a) Vitelline membrane (b) Graafian follicles
(a) many more sperm than eggs. (c) Stroma (d) Germinal epithelium
(b) a few more sperm than eggs. 36. In budding,
(c) equal numbers of sperm and eggs. (a) Outgrowth develops earlier than nuclear division
(d) fewer sperm than eggs. (b) Nucleus divides earlier than development of
27. The asexual process replaced by the sexual method is outgrowth
known as: (c) Both occur simultaneously
(a) Semigany (b) Amphimixis (d) There is no fixed sequence
(c) Apospory (d) Apomixis 37. A student upon observing a permanent slide of binary
28. The cyclic period of sexual activity in non-human female fission in Amoeba drew the following diagram. Identify
mammals is called the fault in the diagram:
(a) Menstruation (b) Luteinization
(c) Oogenesis (d) Oestrous cycle
29. Characters transmitted from parents to offspring are
present in
(a) cytoplasm (b) ribosome
(c) golgi bodies (d) genes (a) Size of daughter cell to be formed is not appropriate.
30. In sweet potato, vegetative propagation takes place by (b) Constriction has appeared before nuclear division.
(a) Root (b) Stem (c) Constriction is not equally deep on the either side.
(c) Leaves (d) Fruit (d) None of the above
31. If a starfish is cut into pieces, each piece grow into a 38. In the process of binary fission:
complete animal. The process is called (a) Cytoplasm divides first followed by division of
(a) regeneration (b) reproduction nucleus
(c) healing of wounds (d) growth (b) Nucleus divides first followed by division of
cytoplasm
32. In the figure of budding in Yeast, structures a, b, c and d
should be labelled respectively as (c) Both nucleus and cytoplasm divide simultaneously
(d) Cell protrudes followed by division of nucleus
39. Given below are stages of binary fission in Amoeba.
Which one out of the following would you select as
correct sequence of these stages?

(a) Nucleus of bud, bud, Yeast, nucleus


(b) Dividing nucleus of bud, bud, Yeast, nucleus
(c) Nucleus of bud, bud, Yeast, dividing nucleus of yeast (a)
A, B, C, D (b) A, C, A, B
(d) Dividing nucleus of Yeast, Yeast, bud, nucleus of bud. (c)
B, D, A, C (d) C, A, D, B
33. The nutritive medium for the ejaculated sperms is given by 40. (A):
Vigrous contraction of the uterus at the end of
(a) Seminal fluid (b) Vaginal fluid pregnancy causes expulsion.
(c) Uterine lining (d) Fallopian tube (B): The stimulatory reflex between the uterine contraction
and oxytocin results in weakening contractions.
34. If both ovaries are removed from a rat, then which
hormone is decreased in blood (a) Both the statements (A) and (B) are correct
(a) Oxytocin (b) Oestrogen (b) Statement (A) is correct but (B) is incorrect.
(c) Prolactin (d) Gonadotrophin (c) Both the statements (A) and (B) are incorrect

35. Which part of the ovary in mammals acts as an endocrine (d) Statement(B) is correct but (A) is incorrect.
gland after ovulation ?
How Do Organisms Reproduce S-89
41. (A):
The middle piece is called as power house of the sperm. (d) Two pollen grains sending two pollen tubes inside
(B): The numerous mitochondria coiled around axial the ovary, resulting in the formation of two seeds
filament produce energy for the movement of the tail. inside the fruit.

(a) Both the statements (A) and (B) are correct 49. During oogenesis in mammals, the second meiotic
division occurs
(b) Statement (A) is correct but (B) is incorrect.
(a) before fertilisation
(c) Both the statements (A) and (B) are incorrect
(b) after implantation
(d) Statement(B) is correct but (A) is incorrect.
(c) before ovulation
42. Which of the following cells during gametogenesis is (d) after fertilisation
normally dipoid?
(a) Spermatid (b) Spermatogonia 50. The correct route that sperm follows when it releases
from the testis of a mammal:
(c) Secondary polar body (d) Primary polar body
(a) Vas deferens → Epididymis → Urethra
43. In human females, meiosis II is not complete until? (b) Urethra → Epididymis → Vas deferens
(a) Fertilisation (b) Uterine implantation (c) Epididymis → Urethra → Vas deferens
(c) Birth (d) Puberty (d) Epididymis → Vas deferens → Urethra
44. The human embryo gets nutrition from the mother blood
51. Which of the following is not a part of the female
with the help of a special organ called
reproductive system in human beings?
(a) Zygote (b) Ovary
(a) Ovary (b) Uterus
(c) Oviduct (d) Placenta
(c) Vas deferens (d) Fallopian tube
45. The source of mammalian hormone ‘relaxin’ is
(a) ovary (b) stomach 52. The anther contains
(c) intestine (d) pancreas (a) sepals (b) ovules
46. Grafting in monocot plants is not possible because they (c) carpel (d) pollen grains
have
(a) Parallel venation
(b) Have only one cotyledon
(c) Have cambium DIRECTIONS : Study the given case/passage and answer the
following questions.
(d) Have scattered vascular bundles
Case/Passage - 1
47. In the flowering plants sexual reproduction involves
several events beginning with the bud and ending in When the branches of a plant growing in the field are pulled
a fruit. These events are arranged in four different towards the ground and a part of them is covered with moist
combinations. Select the combination that has the correct soil (leaving the tips of the branches exposed above the
sequence of events. ground), then after sometime new roots develop from the parts
(a) Embryo, zygote, gametes, fertilization.   of branches buried in the soil. On cutting these branches from
(b) Gametes, fertilization, zygote, embryo. the parent plant, new plants are produced from the cut parts of
(c) Fertilization, zygote, gametes, embryo. branches which had developed roots.
(d) Gametes, zygote, embryo, fertilization. 53. What is this method of propagation of plants known as?
48. “Double fertilization” is a complex mechanism of 54. What type of branches should a plant have to be able to
flowering plants that is also unique to angiosperms. be propagated by this method?
Choose the most appropriate statement from the options
listed below that explains this phenomenon. 55. Name any two plants which are grown for their flowers
(a) Fertilization in two flowers of the same plant and propagated by this method.
forming endosperms. 56. Name any two plants which are grown for their fruits and
(b) Two male gametes fertilize two eggs inside the ovule propagated by this method.
as a result the ovary gives rise to bigger fruits.
57. Name one plant which gets propagated by this method
(c) Two fertilizations occur in a flower-one fertilization naturally by forming runners (soft horizontal stems
results in the formation of a diploid zygote and the
running above the ground).
second fertilization results in the formation of a
triploid endosperm.
S-90 Science
Case/Passage - 2 64. The seminiferous tubules of the testes are lined by the
germinal epithelium consisting of
When an insect sits on the flower of a plant then some particles,
present on the top of little stalks in the flower get stick to its (a) sertoli cells
body hair. When this insect now sits on the flower of another (b) cells of germinal epithelium
similar plant, then particles attached to the hair of insect are (c) cells of Leydig or interstitial cells
shifted to top of a flask-shaped organ at the centre of a flower.
This particle grows a long tube B from the top of flask-shaped (d) secondary spermatocytes
organ through which C moves down and reaches the bottom 65. Another name for Bulbourethral gland is
of the flask-shaped organ. Here C fuses with the nucleus of D, (a) Meibomian gland
present in structure E.
(b) Prostate gland
The fusion of C and D forms a new cell F which grows and
(c) Perineal gland
develops into a seed of the plant.
(d) Cowper’s gland
58. What are these particles? Name the process by which
these particles are transfered from one flower to other 66. In man, Cryptorchidism is the condition when
flower of another similar plant. (a) testes do not descent into the scrotum
59. What is the name of tube B? (b) there are two testes in each scrotum
60. What is C which moves down through the tube B? (c) testis degenerates in the scrotum
61. Name D and E. (d) testis enlarges in the scrotum
62. What is F? 67. Which of these is an accessory reproductive gland in
Case/Passage - 3 male mammals
The male reproductive system consists of portions which (a) Inguinal gland
produce the germ-cells and other portions that deliver the (b) Prostate gland
germ-cells to the site of fertilisation. The formation of
(c) Mushroom-shaped gland
germ-cells or sperms takes place in the testes. These are
located outside the abdominal cavity in scrotum because (d) Gastric gland
sperm formation requires a lower temperature than the
normal body temperature. We have discussed the role of Assertion & Reason
the testes in the secretion of the hormone, testosterone,
in the previous chapter. In addition to regulating the
DIRECTIONS : Each of these questions contains an assertion
formation of sperms, testosterone brings about changes in followed by reason. Read them carefully and answer the question
appearance seen in boys at the time of puberty. The sperms on the basis of following options. You have to select the one that
formed are delivered through the vas deferens which best describes the two statements.
unites with a tube coming from the urinary bladder. The
(a) If both Assertion and Reason are correct and Reason is
urethra thus forms a common passage for both the sperms
the correct explanation of Assertion.
and urine. Along the path of the vas deferens, glands like
the prostate and the seminal vesicles add their secretions (b) If both Assertion and Reason are correct, but Reason is
so that the sperms are now in a fluid which makes their not the correct explanation of Assertion.
transport easier and this fluid also provides nutrition. (c) If Assertion is correct but Reason is incorrect.
The sperms are tiny bodies that consist of mainly genetic (d) If Assertion is incorrect but Reason is correct.
material and a long tail that helps them to move towards
68. Assertion: Asexual reproduction is also called
the female germ-cell.
blastogenesis.
63. The seminiferous tubules of the testes are lined by the
Reason: In asexual reproduction, there is no formation
germinal epithelium consisting of
and fusion of gametes.
(a) spermatids
69. Assertion: Amoeba shows multiple fission during
(b) cells of Sertoli
unfavourable conditions.
(c) spermatogonium
Reason: Chances of survival are less during unfavourable
(d) spermatocytes conditions.
How Do Organisms Reproduce S-91
70. Assertion: Urethra in human male acts as urinogenital (C) Condom (r ) Protozoan
canal. (D) Trichomoniasis (s) Corpus spongiosum
Reason: Urethra carries only urine while sperms are
carried by vasa deferentia only.
Fill in the Blanks
71. Assertion: In morula stage, cells divide without increase
in size. DIRECTIONS : Complete the following statements with an
Reason: Zona pellucida remain undivided till cleavage is appropriate word / term to be filled in the blank space(s).
complete. 74. In many invertebrate organisms, both sexes are found in
the same individual. This is called ..................
Match the Following
75. During the birth process, the pituitary hormone ...............
signals the uterus to contract.
DIRECTIONS : Each question contains statements given in
two columns which have to be matched. Statements (A, B, C, D) 76. A technique to produce genetically alike individuals from
in column I have to be matched with statements (p, q, r, s) in a single cell is known as ..............
column II. 77. Budding is a common method of asexual reproduction in
72. Column A Column B yeast and .............

(A) Animals which give (p) Hydra 78. In ..............., vegetative propagation occurs by leaves.

birth to young one 79. Surgically when fallopian tube is removed or ligated, it is
called ...............
(B) Animal which (q) Planaria
80. An egg cell of a plant is contained in an ............ present
produces bud
in an ovary.
(C) An animal which (r) Placenta
81. Transfer of pollen from one flower to stigma of another
shows regeneration flower of same species is termed ........................ .
(D) Provides nutrition to (s) Cross-pollination 82. Ovaries are also responsible for the production of
the developing hormone called ........................ .
embryo from the 83. Plants raised by vegetative propagation bear early ............
and .................... .
mother
84. Future shoot hidden in a seed is called .................. .
(E) The pollen transferred (t) Germination
85. The gametes are formed in most of the multicellular
from one flower to
organisms by a process of cell division called ............... .
another flower of
86. The two parts tied together during grafting are called
other plant ........ and ............ .
(F) The process in which (u) Viviparous 87. Simply break up into smaller pieces upon maturation is
embryo develops into found in .................. .
seedling
True / False
(G) Fertilised egg in (v) Menstruation
humans gets DIRECTIONS : Read the following statements and write your
implanted in answer as true or false.
(H) When egg in humans (w) Uterus 88. Basic event in reproduction is creation of DNA copy.
is not fertilised, 89. Copper–T is a contraceptive device used by women.
what happens? 90. At the time of birth, a baby girl has thousands of immature
73. Column A Column B eggs.
(A) Seminal vesicle (p) Latex sheath 91. Sperms mature at a temperature higher than that of human
body.
(B) Urinogenital duct (q) Semen plasma
S-92 Science
92. The only function of the testes is to produce sperm. 103. Placenta is the name of a vital connection between mother
93. Animal development is limited to the period prior to bird and embryo.
hatching. 104. The male germ-cell produced by pollen grain contains
94. Onset of menstruation is termed as menopause. half the amount of DNA as compared to the other body
cells of the plant.
95. In Spirogyra, asexual reproduction takes place by
fragmentation. 105.
Vegetative propagation produces plants that are
genetically similar to the parent plant.
96. Vegetative propagation by leaves occurs in sweet potato.
106. Regeneration is the same as reproduction.
97. Transfer of male gametes to the stigma of flower is called
pollination. 107. Before cell division copying of DNA is not essential.

98. In mammals including man, fertilization takes place 108. Sexually transmitted diseases can be prevented by using
externally. condoms.

99. The ovulation takes place 10-12 days after the start of 109. Plants produced by vegetative propagation are genetically
menstruation. similar to the parent plant.

100. In human-beings, male can produce sperms upto the age 110. Sexual reproduction does not lead to variation in a
of 45-50 years. population.

101. In fission, many bacteria and protozoa simply divide into 111. The ovary of a flower grows into a fruit.
two or more daughter cells.
102. Plants that produce asexually do not produce flower.
How Do Organisms Reproduce S-93

ANSWER KEY & SOLUTIONS


1. (c) Sweet potato is an example of vegetative propagation 26. (a) 27. (d)
by roots. Sweet potato bears adventitious buds. 28. (d) Oestrus cycle is the period in the sexual cycle of
When it is planted in the soil, new plants are female mammals, except the higher primates, during
produced from the roots. which they are in heat i.e., ready to accept a male
2. (b) and to mate.
3. (b) Tunica albuginea (testicles) is a layer of connective 29. (d) 30. (a)
tissue which covers the testicles.
31. (a) Regeneration means the regrowth of a damaged
4. (a) External fertilization is a mode of reproduction in or missing organ part from the remaining tissue.
which a male sperm fertilizes a female egg outside Starfish exhibit regeneration.
of the female body. Example: fish and frog.
32. (a) 33. (a) 34. (b) 35. (b)
5. (b) Grafting is a horticultural technique used to join
36. (a) 37. (b) 38. (b) 39. (c)
parts from two or more plants so that they appear
to grow as a single plant. It is often used to combine 40. (c) Vigorous contraction of the uterus at the end of
the desirable or advantageous characteristics of two pregnancy causes parturition. Parturition is induced
different plants. by a complex neuroendocrine mechanism. The signals
for parturition originate from the fully developed
6. (d) 7. (b) 8. (d) 9. (d)
foetus and the placenta which induce mild uterine
10. (c) 11. (a) 12. (c) 13. (c) contractions called foetal ejection reflex.
14. (a) 15. (d) 16. (a) 17. (d)
41. (a) The middle piece of human contains mitochondria
18. (d) coiled a round the axial filament called mitochondrial
19. (d) The germinal epithelium is the innermost layer spiral. They provide energy for the movement of the
of the testicle. It is also known as the wall of the sperm. So it is called as the ‘power house of the
seminiferous tubule within the testes. sperm’.

20. (a) The Cowper’s glands (or bulbourethral glands) are 42. (b) Spermatogonia are diploid male germ cells which
a pair of exocrine glands in the mammals (male undergo meiosis to form sperms.
reproductive system). They produce thick clear 43. (a) Meiosis II does not complete untill fertilisation
mucus prior to ejaculation that drains into the occurs in females (in human being). If fertilisation
spongy urethra. takes place, it results in a fertilized mature ovum and
21. (a) The corpus luteum secretes progesterone, which second polar body.
is a steroid hormone responsible for the changes 44. (d) It is a connecting link between mother and
occur in the endometrium (its development) and its developing foetus, which provides nutrients and
maintenance. removes the waste from baby’s blood.
22. (b) 45. (a) Relaxin is produced in the corpus luteum, the
23. (b) The birth control pill works by stopping sperm from placenta and the uterus in females as well as in other
joining with an egg (which is called fertilization). reproductive structures, this varies by species. It also
The hormones in the pill stop ovulation. No promotes the development of mammary glands in
ovulation means there’s no egg hanging around for pregnant mammals.
sperm to fertilize, so pregnancy can’t happen. 46. (d) Grafting in monocot plants is not possible because
24. (a) they have scattered Vascular Bundles.

25. (c) Bacteria have three mechanism for genetic 47. (b)
information transfer without undergoing reproduction 48. (c) In double fertilization, one male gamete fuses with
i.e., transformation, transduction and conjugation. the egg and results in the formation of a diploid
S-94 Science
zygote and this process is called fertilization. The 62. (e) F is fertilised egg (zygote).
other male gamete fuses with the two polar nuclei
63. (b) 64. (a) 65. (d) 66. (a)
to form a triploid endosperm. This process is called
triple fusion. After fertilisation, the fertilized ovule 67. (b)
forms the seed while the tissues of the ovary become 68. (b) Development of the offspring from reproductive units,
the fruit. such as buds or fragments, in asexual reproduction is
called blastogenesis. In asexual reproduction, only
49. (d) A primary oocyte begins the first meiotic division,
one parent is involved, so also called uniparental
but then arrests until later in life when it will finish
reproduction.
this division in a developing follicle. This results in
a secondary oocyte, which will complete meiosis 69. (a)
if it is fertilized. So, the second meiotic division 70. (c) The urethra is the tube that allows urine to pass out
occurs after fertilisation. of the body. In men, it’s a long tube that runs through
50. (d) The sperm releases from the testis, enters into the penis. It also carries semen in men.
epididymis which leads to vas deferens. Then 71. (a)
sperms are transferred into the urethra. 72. (A) → (u), (B) → (p), (C) → (q), (D) → (r), (E) → (s),
51. (c) Ovary, uterus and fallopian tube are the part of female (F) → (t), (G) → (w), (H) → (v)
reproductive system but vas deferens is a part of male
reproductive system in human beings. The sperms are 73. (A) → (q), (B) → (s), (C) → (p), (D) → (r)
delivered through the vas deferens which unites with a 74. hermaphroditism 75. oxytocin
tube coming from the urinary bladder. 76. cloning 77. Hydra
52. (d) Stamens are the male reproductive parts of flowers. A 78. Bryophyllum 79. tubectomy
stamen consists of an anther (which produces pollen)
80. ovule 81. cross-pollination
and a filament.
82. oestrogen/progesterone
53. (a) Layering
83. flowers, fruits 84. plumule
54. (b) Slender branches (Thin branches)
85. meiosis 86. stock, scion
55. (c) Jasmine and China rose 87. Spirogyra
56. (d) Lemon and Guava 88. True 89. True 90. True 91. False
57. (e) Strawberry 92. False 93 False 94. False 95. True
58. (a) These particles are known as pollen grains; cross 96. False 97. True 98. False 99. True
pollination 100. False 101. True 102. False 103 True
59. (b) Pollen-tube 104. True 105. True 106. False 107. False
60. (c) C is male gamete. 108. True 109. True 110. False 111. True
61. (d) D is female gamete (ovum or egg); E is ovule
10 Acids, Bases
Heredity
Salts
Evolution
andand

7. In fruit flies wild-type colour is red and is dominant to white


Multiple Choice Questions (MCQs) eye colour. Because this eye colour is located on the:
(a) Y chromosome only. (b) autosome only.
DIRECTIONS : This section contains multiple choice
(c) X and Y chromosomes (d) X chromosome only.
questions. Each question has four choices (a), (b), (c) and (d)
out of which only one is correct. 8. The diagram below represents a portion of a nucleic acid
molecule.
1. A pair of contrasting characters is called
(a) phenotype (b) genotype
(c) allele (d) gene
2. An allele is said to be dominant if
(a) it is expressed only in heterozygous combination.
X
(b) it is expressed only in homozygous combination.
(c) it is expressed in both homozygous and heterozygous
condition. The part indicated by arrow X could be
(d) it is expressed only in second generation. (a) adenine (b) ribose
3. Which of the following is a test cross? (c) deoxyribose (d) phosphate
(a) TT × tt (b) Tt × tt 9. Mendel’s concept of segregation implies that the two
(c) Tt × TT (d) tt × tt members of an allelic pair of genes –
4. Which is the example of homologous organs? (a) are distributed to separate gametes
(a) Forelimbs of man and wings of bird (b) may contaminate one another
(b) Wings of birds and wings of insects (c) are segregated in pairs
(c) Vermiform appendix and nictitating membrane
(d) are linked
(d) Archaeopteryx and Balanoglossus
10.
5. Which of the following features do humans lack that
other primates have ?
(a) Forward-facing eyes
(b) Short snouts
(c) Flexible shoulder and elbow joints
(d) Opposable big toes
6. A pure tall plant can be differentiated from a hybrid tall The figure above shows :
plant: (a) A bird sitting on a tree
(a) By measuring length of plant (b) Connection between Birds and Reptiles
(b) By spraying gibberellins (c) Connection between Pisces and Aves
(c) If all plants are tall after self-pollination (d) All of the above
(d) If all plants are dwarf after self-pollination
S-96 Science
11. If a homozygous red-flowered plant is crossed with a 18. Who out of the following was of the strong opinion that
homozygous white-flowered plant, the offspring would acquired characteristics are inherited:
be: (a) Lamarck (b) Lysenko
(a) Half red-flowered (b) Half white-flowered (c) Mendel (d) Huxley
(c) All red-flowered (d) Half pink-flowered 19. Guanine pairs with:
12. Which of the following would stop evolution by natural (a) Adenine (b) Cytokine
selection from occurring? (c) Thymine (d) None of the above
(a) If humans became extinct because of a disease epidemic. 20. Chemically a nucleotide has a:
(b) If a thermonuclear war killed most living organisms (a) Pentose group (b) Nitrogenous base
and changed the environment drastically. (c) Phosphate group (d) All of these
(c) If ozone depletion led to increased ultraviolet
21. Which of the following rediscovered the Mendel’s work?
radiation, which caused many new mutations.
(a) Correns (b) De Vries
(d) If all individuals in a population were genetically
identical, and there was no genetic recombination, (c) Tschermark (d) All of these
sexual reproduction, or mutation. 22. Mendel’s law of segregation is based on separation of
13. If the fossil of an organism is found in the deeper layers alleles during:
of earth, then we can predict that: (a) Gametes formation
(b) Seed formation
(a) The extinction of organism has occurred recently.
(c) Pollination
(b) The extinction of organism has occurred thousands (d) Embryonic development
of year ago.
23. Mendel formulated the law of purity of gametes on the
(c) The fossil position in the layers of earth is not related
basis of :
to its time of extinction.
(a) Dihybrid cross (b) Monohybrid cross
(d) Time of extinction cannot be determined.
(c) Back cross (d) Test cross
14. A Mendelian experiment consisted of breeding tall
24. The earliest living organisms were:
pea plants bearing violet flowers with short pea plants
bearing white flowers. The progeny all bore violet (a) Multicellular (b) Eukaryotes
flowers, but almost half of them were short. This (c) Prokaryotes (d) None of these
suggests that the genetic make-up of the tall parent can 25. Sudden inheritable change is called:
be depicted as:
(a) Recombination (b) Mutation
(a) TTWW (b) TTww (c) National selection (d) Segregation
(c) TtWW (d) TtWw
26. When one gene pair hides the effect of the other unit, this
15. Normal maize has starchy seed which remain smooth phenomenon is referred as –
when dry. A mutant form has sugary seed which go (a) Dominance (b) Mutation
crinkled when dry. When a mutant was crossed with a
(c) Epistasis (d) None of these
normal plant, an F1 was produced which had smooth
seeds. What would be the relative ratios of the different 27. The genotype of offspring formed from Tt × tt will be –
seed types, if the F1 was allowed to self (a) TT and tt (b) Tt and tt
(a) 1 smooth : 3 sugary (b) 3 smooth : 1 sugary (c) only tt (d) only TT
(c) 1 smooth : 1 sugary (d) All sugary 28. A complete set of chromosomes inherited as a unit from
16. Mendel formulated some laws which are known as one parent, is known as:
(a) Laws of germplasm (a) Karyotype (b) Gene pool
(b) Laws of origin of species (c) Genome (d) Genotype
(c) Laws of recapitulation
29. Which of the following is Heterozygous?
(d) Laws of inheritance
(a) TTRR (b) ttrr
17. Vestigial organs are: (c) TT (d) Tt
(a) Primitive organs
30. An experiment to prove that organic compounds were the
(b) Primordial organs basis of life, was performed by:
(c) Organs reduced due to disuse (a) Oparin (b) Miller
(d) Organs marked only in embryonic stage (c) Melvin (d) Fox
Heredity and Evolution S-97
31. The idea of “Survival of fittest” was given by: (b) Terminal fruit and wrinkled seed
(a) Darwin (b) Herbert Spencer (c) White testa and yellow pericarp
(c) Germplasm RNA (d) Somatic DNA (d) Green coloured pod and rounded seed
32. If two parents have the genotypes AA × aa, the probability 38. From heredity point of view, which marriage is not suitable?
of having an aa genotype in the F, generation is – (a) Man Rh (–) and Woman Rh (+)
(a) 25 percent (b) 50 percent (b) Both Rh (+)
(c) 75 percent (d) None of these (c) Both Rh (–)
33. Heredity deals with the study of (d) Man Rh (+) and Woman Rh (–)
(a) resemblances and differences between the parents 39. Which of the following are fossils?
and offsprings.
(a) Pollen grains buried in the bottom of a peat bog.
(b) resemblances between the parents and offsprings.
(b) The petrified cast of a clam’s burrow.
(c) differences between the parents and offspring.
(c) The impression of clam shell made in mud, preserved
(d) none of the above
in mudstone.
34. The given figure shows bones in the forelimbs of three (d) All of the above.
mammals.
40. Which of the following evolutionary mechanisms acts
to slow down or prevent the evolution of reproductive
isolation?
(a) Natural selection (b) Gene flow
(c) Mutation (d) Genetic drift
41. In natural selection,
(a) the genetic composition of the population changes at
random over time.
(b) new mutations are generated over time.
For these mammals, the number, position, and shape of (c) all individuals in a population are equally likely to
the bones must likely indicates that they may have contribute offspring to the next generation.
(a) developed in a common environment. (d) individuals that possess particular inherited
(b) developed from the same earlier species. characters survive and reproduce at a higher rate
than other individuals.
(c) identical genetic makeup.
(d) identical methods of obtaining food. 42. A heterozygous red-eyed female Drosophila mated with
a white-eyed male would produce
35. A male child will be born if
(a) red-eyed females and white-eyed males in the F1
(a) father is healthy
(b) white-eyed females and red-eyed males in the Fl
(b) mother is well fed during pregnancy
(c) half red and half white-eyed females and all white
(c) genetic composition of child has XY set of eyed males in the F1
chromosomes
(d) half red and half white-eyed females as well as
(d) genetic composition of child has XX set of males in the F1
chromosomes.
43. Sex-linked disorders such as color blindness and
36. According to the evolutionary theory, formation of a new haemophilia are
species is generally due to
(a) caused by genes on the X chromosome
(a) sudden creation by nature.
(b) caused by genes on the autosome
(b) accumulation of variations over several generations.
(c) caused by genes on the Y chromosome
(c) clones formed during asexual reproduction.
(d) expressed only in men
(d) movement of individuals from one habitat to another.
44. The smallest unit that can evolve is a:
37. Which of the following is dominant character according
(a) Species (b) Genotype
to the Mendel?
(c) Gene (d) Population
(a) Dwarf plant and yellow fruit
S-98 Science
45. Both prokaryotic and eukaryotic fossils are found in: 51. In order to find out different types of gametes produced
(a) Azoic (b) Coenozic by a pea plant having the genotype AaBb, it should be
crossed to a plant with the genotype:
(c) Proterozoic (d) Archaeozoic
(a) AABB (b) AaBb
46. (A): Genes pass from one generation to another. (c) aabb (d) aaBB
(B): The unit of inheritance is genes. 52. (A): A good example of multiple alleles is ABO blood
(a) Statement (A) and (B) both are correct. group system.
(b) Statement (A) is correct but (B) is incorrect. (B):When IA and IB alleles are present together in ABO blood
(c) Statement (A) is incorrect but (B) is correct group system, they both express their own types.
(d) Statement (A) and (B) both are incorrect. (a) Statement (A) and (B) both are correct.
(b) Statement (A) is correct but (B) is incorrect.
47. The given figure shows an example of:
(c) Statement (A) is incorrect but (B) is correct
(d) Statement (A) and (B) both are incorrect.
53. Suppose that in sheep, a dominant allele (B) produces
Tondril black hair and a recessive allele (b) produces white hair.
If you saw a black sheep, you would be able to identify
Thorn
(a) its phenotype for hair colour.
(b) its genotype for hair colour.
Bougainvillea Cucurbita (c) the genotypes for only one of its parents.
(a)
Homologous organs (b) Convergent evolution (d) the genotypes for both of its parents.
(c)
Divergent evolution (d) Both (a) and (c) 54. Refer the given statements and select the correct option.
48. Which of the following statements best describe the (i) Percentage of homozygous dominant individuals
theory of natural selection? obtained by selfing Aa individuals is 25%.
(a) All organisms are equally suited to their environment. (ii) Types of genetically different gametes produced by
genotype AABbcc are 2.
(b) Random selection will determine which organisms
survive. (iii) Phenotypic ratio of monohybrid F2 progeny in case
of Mirabilis jalapa is 3 : 1.
(c) Organisms better adapted to their environment have
(a) All the statements are correct.
greater reproductive success.
(b) Statements (i) and (ii) are true, but statement (iii) is
(d) Organisms that produce the most offspring are better
false.
suited to their environment.
(c) Statements (i) and (iii) are true, but statement (ii) is
49. (A): Mendel was successful in his hybridisation. false.
(B): Garden pea was proved as ideal experimental material. (d) Statements (ii) and (iii) are true, but statement (i) is
(a) Statement (A) and (B) both are correct. false.
(b) Statement (A) is correct but (B) is incorrect. 55. In a case of mammalian coat color, the principal gene
(c) Statement (A) is incorrect but (B) is correct identified is ‘C’ which codes for a tyrosinase enzyme.
(d) Statement (A) and (B) both are incorrect. In case of rabbits four different phenotypes are observed
Full Color > Chinchilla > Himalayan > Albino (in order
50. Match the following columns and select the correct of the expression of gene ‘C’ and its alleles). In a progeny
answer from the codes given below. obtained after crossing two rabbits, the percentages of
Column-I Column-II Chinchilla, Himalayan and Albino rabbits were 50, 25
and 25 respectively. What must have been the genotypes
A. Non-parental gene exchange I. Crossing over
of the parent rabbits?
B. Non-sister chromatids II. X and Y
C. Sex chromosome III. Autosome-linked (a) CchCch X Cchc (b) CchCh X Cchc
disease (c) Cchc X Chc (d) ChCh X CchCch
D. Thalassaemia IV. Recombination
56. The gene for hemophilia is present on X chromosome. If a
(a) A – IV; B – I; C – II; D – III hemophilic male marries a normal female, the probability
(b) A – II; B – I; C – IV; D – III of their son being hemophilic is
(c) A – II; B – IV; C – III; D – I (a) nil (b) 25%
(d) A – II; B – IV; C – I; D – III (c) 50% (d) 100%
Heredity and Evolution S-99
57. In the experiment conducted by Mendel, RRyy (round 64. Which of the following carry hereditary characters to the
green) and rrYY (wrinkled, yellow) seeds of pea off spring in the organism?
plant were used. In the F2 generation 240 progeny (a) Ribosome (b) Chromosome
were produced, out of which 15 progeny had specific
(c) Plasma (d) Lysosome
characteristics. What were the characteristics?
(a) round and green (b) round and yellow 65. If the genotypes determining the blood groups of a couple
are IAIO and IAIB, then the probability of their first child
(c) wrinkle and yellow (d) wrinkle and green
having type O blood is:
58. A breeder crossed a pure bred tall plant having white (a) 0 (b) 0.25
flowers to a pure bred short plant having blue flowers. (c) 0.50 (d) 0.75
He obtained 202 F1 progeny and found that they are all
tall having white flowers. Upon selfing these F1 plants, 66. A cross was carried out between two individuals
he obtained a progeny of 2160 plants. Approximately, heterozygous for two pairs of genes. Assuming
how many of these are likely to be short and having blue segregation and independent assortment, the number of
flowers? different genotypes and phenotypes obtained respectively
(a) 1215 (b) 405 would be:
(c) 540 (d) 135 (a) 4 and 9 (b) 6 and 3
(c) 9 and 4 (d) 11 and 4
59. Varieties of vegetables such as cabbage, broccoli and
cauliflower have been produced from a wild cabbage 67. A plant with red coloured flowers is crossed with a plant
species. Such process of producing new varieties of having white flowers. The red and white colour of the
living organisms is called flower is controlled by a single gene. Red is dominant over
(a) Natural selection (b) Artificial selection white. The F1 progeny is self-pollinated and the flower
colour in F2 is observed. Given the above information,
(c) Speciation (d) Genetic drift what is the expected phenotypic ratio of plants with
60. Which of the following are pairs of analogous organs? different flower colours?
(I) Forelimbs of horse – Wings of bat (a) All plants with red flowers
(II) Wings of bat –Wings of butterfly (b) Red : white in the ratio of 3 : 1
(III) Forelimbs of horse – Wings of butterfly (c) Pink : white in the ratio of 3 : 1
(IV) Wings of bird – Wings of bat (d) Red : pink : white in a ratio of 1 : 2 : 1
(a) (I) and (II) (b) (II) and (IV) 68. Which one of the following is a correct statement about
(c) (III) and (IV) (d) (II) and (III) primates evolution?
61. Of the periods listed below, which one is the earliest (a) Chimpanzees and gorillas evolved from macaques
period when ostracoderms, the jawless and finless fishes, (b) Humans and chimpanzees evolved from gorillas
appeared? (c) Humans, chimpanzees and gorillas evolved from a
(a) Devonian period (b) Cambrian period common ancestor
(c) Carboniferous period (d) Silurian period (d) Humans and gorillas evolved from chimpanzees
62. Which one of the following options lists the primary 69. The gene for the genetic disease “Haemophilia” is present
energy source(s) for all forms of life on the earth? on the ‘X’ chromosome. If a haemophilic male marries a
(a) Light, inorganic substances normal female, what would be the probability of their son
being haemophilic.
(b) Inorganic substances, organic substances
(a) 50% (b) 100%
(c) Light, organic substances
(c) Nil (d) 3 : 1
(d) N2, CO2
70. A Mendelian experiment consisted of breeding tall pea
63. Four important events given below may have led to the
plants bearing violet flowers with short pea plants bearing
origin of life on the earth.
white flowers. The progeny all bear violet flowers, but
(A) Formation of amino acids and nucleotides almost half of them were short. This suggests that the
(B) Availability of water genetic make-up of the tall parent can be depicted as
(C) Organization of Cells (a) TTWW (b) TTww
(D) Formation of complex molecules
(c) TtWW (d) TtWw
(a) A, B, C and D (b) B, A, D and C
(b) A, D, B and C (d) B, C, A and D
S-100 Science
71. An example of homologous organs is (e) Out of A-B and A-D types of seeds, which one
(a) our teeth and elepahant’s tusks will be produced in (i) minimum numbers, and (ii)
(b) our arm and a dog’s fore-leg maximum numbers, in the F2 generation?
(c) potato and runners of grass. Case/Passage - 2
Most human chromosomes have a maternal and a paternal copy, and
(d) all of the above. we have 22 such pairs. But one pair, called the sex chromosomes, is
odd in not always being a perfect pair. Women have a perfect pair
of sex chromosomes, both called X. But men have a mismatched
pair in which one is a normal-sized X while the other is a short one
DIRECTIONS : Study the given case/passage and answer the called Y. So women are XX, while men are XY.
following questions. 74. If a normal cell of human body contains 46 pairs of
chromosomes then the numbers of chromosomes in a sex
Case/Passage - 1
cell of a human being is most likely to be:
72. Question number (a) - (d) are based on the images (A) and (a) 60 (b) 23
(B) given below. Study them and answer the following
(c) 22 (d) 40
questions.
75. Which of the following determines the sex of a child?
Frog Lizard
(a) The length of the mother’s pregnancy
Bat (b) The length of time between ovulation and copulation
(c) The presence of an X chromosome in an ovum
(d) The presence of a Y chromosome in a sperm

Bird Human 76. In human males, all the chromosomes are paired perfectly
except one. These unpaired chromosomes are:
(a) Large chromosome (b) Small chromosome
(c) Y chromosome (d) X chromosome
Bird
77. The process where characteristics are transmitted from
parent to offsprings is called:
(a) Variation (b) Heredity
  (c) Gene (d) Allele
    (A)     (B) (e) None of the above
(a) What term can be used for the structure given in 78 Who have a perfect pair of sex chromosomes?
image A?
(a) Girls only
(b) What term can be used for the structure given in
image B? (b) Boys only
(c) Which image shows a common ancestry? (c) Both girls and boys
(d) Which image has a common function but different (d) It depends on many other factors
origin?
73. A person first crossed pure-breed pea plants having round- Assertion & Reason
yellow seeds with pure-breed pea plants having wrinkled-
green seeds and found that only A-B type of seeds were DIRECTIONS : Each of these questions contains an assertion
produced in the F1 generation. When F1 generation pea followed by reason. Read them carefully and answer the question
plants having A-B type of seeds were cross-breed by self- on the basis of following options. You have to select the one that
pollination, then in addition to the original round-yellow best describes the two statements.
and wrinkled-green seeds, two new varieties A-D and (a) If both Assertion and Reason are correct and Reason is
C-B type of seeds were also obtained. the correct explanation of Assertion.
(a) What are A-B type of seeds?
(b) If both Assertion and Reason are correct, but Reason is
(b) State whether A and B are dominant traits or not the correct explanation of Assertion.
recessive traits.
(c) What are A-D type of seeds? (c) If Assertion is correct but Reason is incorrect.
(d) What are C-B type of seeds? (d) If Assertion is incorrect but Reason is correct.
Heredity and Evolution S-101
79. Assertion: Chromosomes are known as hereditary vehicles.
Match the Following
Reason: The chromosomes are capable of self-
reproduction and maintaining morphological and
DIRECTIONS : Each question contains statements given in
physiological properties through successive generations.
two columns which have to be matched. Statements (A, B, C, D)
80. Assertion: Ear muscles of external ear in man are poorly in column I have to be matched with statements (p, q, r, s) in
developed. column II.
Reason: These muscles are useful which move external 89. Match the genetic cross of the parents on the left with
ear freely to detect sound efficiently. the genotypes of the offspring most likely to be produced
81. Assertion: Although living organism always arise from from that cross on the right.
other living organism, life should certainly have had a Column I Column II
beginning.
(A) BB × bb (p) 100% BB
Reason: The study of the conditions and the mechanisms
(B) Bb × Bb (q) 100% bb
involved in the creation of most primitive living structures
on earth is actually the problem of origin of life. (C) BB × BB (r) 25% BB, 50%Bb,
25%bb
82. Assertion: The establishment of reproductive isolations
in an event of biological significance. (D) bb × bb (s) 100% Bb

Reason: In the absence of reproductive isolation species 90. Match the physical evidence of evolution with the best
can merge back into single population. description of that particular type of evidence.
83. Assertion: DNA fingerprinting is a method in which Column I Column II
polymerase chain reaction followed by DNA probe is (A) Fossils (p) Comparing
used. similarities and
Reason: A DNA fingerprint is inherited and therefore, differences between
resembles that of parents. amino acid sequences
in two organisms.
84. Assertion: The birds have large, light spongy bones with (B) Embryology (q) Comparing and
air sacs. contrasting cell
Reason : These adaptations help them during flight. structures found
within an organism.
85. Assertion: We have lost all the direct evidence of origin
(C) Cytology (r) The remains of dead
of life.
organisms that are
Reason: The persons responsible for protecting evidences studied.
were not skilled. (D) DNA evidence (s) Comparisons of the
86. Assertion: Among the primates, chimpanzee is the early development
closest relative of the present day humans. stages of an organism.
91.
Reason: The banding pattern in the autosome numbers 3
Column A Column B
and 6 of man and chimpanzee is remarkably similar.
(A) Genetic changes (p) Homologous organ
87. Assertion: Human ancestors never used their tails and so
(B) Independent (q) Fossil
the tail expressing gene has disappeared in them.
assortment
Reason: Lamarck’s theory of evolution is popularly (C) Natural selection (r) Analogous organ
called theory of continuity of germplasm.
(D) Dihybrid ratio (s) XY
88. Assertion: The genetic complement of an organism is (E) Male human beings (t) 9:3:3:1
called genotype.
(F) Wing of a bat and a (u) Darwin
Reason: Genotype is the type of hereditary properties of wing of a bird
an organism. (G) Remnant of ancient (v) Mendel
animals
(H) Arm of a man and (w) DNA copying
wing of a bird
S-102 Science
111. The age of fossil is usually determined by analysing the
Fill in the Blanks ............ present in the rock from which fossil is recovered.
112. Theory of natural selection was proposed by ...............
DIRECTIONS : Complete the following statements with an
appropriate word / term to be filled in the blank space(s). 113. Wind of bat and wing of bird are the example of the
................ organs.
92. Mendel performed his experiments on ...............
114. Forelimbs of frog and lizard are the example of the
93. According to modern concept, Mendel’s factor is called a ................ organs.
....................
94. Mendelian factors or genes as well as chromosomes are True / False
present in ................
DIRECTIONS : Read the following statements and write your
95. The traits which express themselves in F1 generation are
answer as true or false.
called ................
96. The phenotypic ratio between tall and dwarf is ................ 115. Mouth parts of insects show divergent evolution.
97. The phenotypic ratio in dihybrid cross is ................. 116. Life can originate on earth from pre-existing life only.
98. There are ................ pairs of chromosomes in human. 117. The atmosphere of the primitive earth was reducing.
99. The offspring can be of two types with XX and ................ 118. Variations arising during the process of reproduction
chromosomes. cannot be inherited.
100. The traits which are acquired by an organism during its 119. Sex is determined by different factors in various species.
lifetime are called ................ . 120. Changes in the non-reproductive tissues caused by
101. Transmission of traits from one generation to the next environmental factors are inheritable.
generation is called ................ 121. Exchange of genetic material takes place in asexual
102. ................ traits are unable to express in a hybrid. reproduction.

103. Two types of nucleic acids are DNA and ................ . 122. A cross between a true tall and pure dwarf pea plant
resulted in production of all tall plants because tallness is
104. Chromosome consists of a DNA molecule and ................. the dominant trait.
105. If tall plant contains TT gene then dwarf plant contains 123. Reduction in weight of an organism due to nutrition is
................ . genetically controlled.
106. Mendel choose ................................ characters in Pea for 124. New species may be formed if DNA undergoes significant
his experiments. changes or chromosome number changes in the gametes.
107. Broccoli has been developed from ................ cabbage 125. Both the parents contribute DNA equally to the offspring.
through artificial selection. 126. Sex of the child is determined by the type of ovum
108. ................................ speciation occurs in geographically provided by the mother.
separated populations. 127. A recessive trait can also be common as blood group O.
109. The first organisms were ................ and not autotrophs. 128. Attached ear lobe is recessive trait.
100. The study of fossils, a branch of biology called ........... 129. Charles Darwin discovered the law of independent
was founded by Goerges Cuiver. assortment.
Heredity and Evolution S-103

ANSWER KEY & SOLUTIONS


1. (c) flowers, it means that the tall plant having violet
flowers has WW genotype for violet flower colour.
2. (c) An allele is dominant if it is expressed in both
Since the progeny is both tall and short, the parent
homozygous and heterozygous states. Dominant
plant was not a pure tall plant. Its genotype must be
alleles expresses itself in the homozygous as well as
Tt. Therefore, the cross involved is:
heterozygous condition. It is denoted by capital letter.
TtWw × ttww
3. (b) A test cross involves mating of an unknown genotypic
individual with a known homozygous recessive.    ↓
4. (a) 5. (d) TtWw – ttww
6. (c) Pure breeding varieties are the one that carries same Therefore, half the progeny is tall, but all of them
allele for a particular character and pass the trait have violet flowers.
without change from one generation to next upon
15. (c) Suppose the genotype of a normal plant with
selfing. For example, pure breeding tall (TT) plant
smooth seeds is SS and that of wrinkled seeds is
carries two copies of “T” allele and selfing of these
ss. For a cross between SS and ss, all the offsprings
plants produce a uniform progeny of tall plants
produced will have genotype Ss (smooth seeds). If
only. Hybrids are the one which carries contrasting
the F1 with smooth seeds Ss is allowed to self, then
allele of a particular gene.
3 smooth and 1 maize plant with wrinkled seeds
7. (a) In fruit flies, the wild-type eye color is red (XW) and will be formed.
is dominant to white eye color. Because this eye-color
Genotypes: Ss (smooth seeds) × Ss (smooth seeds)
gene is located on the X chromosome only. Males are
said to be hemizygous, because they have only one Gametes   S s
allele for any X-linked characteristic.
S   SS    Ss
8. (a) The part indicated by arrow ‘X” is adenine.
s   Ss    ss
Nitrogenous bases: As is evident from Punnette square, 3 plants with
3' 5' Adenine smooth seeds and 1 with wrinkled seeds are formed.
Thymine
16. (d) 17. (c) 18. (a) 19 (b)
Guanine
Cytosine 20. (d) A nucleotide is made up of three parts: a phosphate
group, a 5-carbon sugar, and a nitrogenous base.
The four nitrogenous bases in DNA are adenine,
cytosine, guanine and thymine.
Base pair
21. (d)
Sugar-phosphate 22. (a) According to Mendel’s monohybrid cross, during
backbone gamete formation, the alleles for each gene segregate
from each other so that each gamete carries only one
3' 5' allele for each gene. It is called Law of Segregation.
23. (b)
9. (a) The law of segregation states that the two alleles of
a single trait will separate randomly, meaning that 24. (c) The first living things on Earth, single-celled micro-
there is a 50% either allele will end up in either organisms or microbes lacking a cell nucleus or cell
gamete. membrane known as prokaryotes, seem to have first
appeared on Earth almost four billion years ago,
10. (b) 11. (c) 12. (d) 13. (b) just a few hundred million years after the formation
14. (c) The genetic make-up of the tall parent can be of the Earth itself.
depicted as TtWW Since all the progeny bore violet
25. (b)
S-104 Science
26. (c) Epistasis is the phenomenon wherein the effect of one 49. (a) Mendel choose garden pea as plant material for his
gene (locus) is dependent on the presence of one or experiments, since it had the following advantages:
more ‘modifier genes’, i.e., the genetic background. (i) Well defined characters.
27. (b) 28. (c) (ii) Bisexual flowers.
29. (d) Heterozygous means that an organism has two (iii) Predominantly self-fertilisation.
different alleles of a gene. Thus Tt is heterozygous. (iv) Easy hybridisation.
30. (b) 31. (a) 32. (d) 33. (a)
Besides these features, garden pea, being self-
34. (b) The given figures show the forelimbs of three fertilised, had pure lines due to natural self
mammals which indicate the homology among fertilisation for a number of years. Therefore, any
themselves. Homologous organs are those organs variety used was pure for the characters it carried.
which are dissimilar in shape, size and function but Mendel’s success was mainly based on the fact that
their origin, basic plan and development are similar. he considered a single character at one time.
Such differences are due to divergent evolution or
50. (a) Crossing over takes place only between non-sister
adaptation for varied conditions.
chromatids.
35. (c) 36. (b)
51. (c) In order to find out the gamete or the genotype of an
37. (d) In peas, the dominant seed shape is round and the unknown individual, scientists perform a test cross.
recessive is wrinkled (w). The dominant trait for In test cross, the individual in question is crossed
pod color is green and recessive is yellow (y). with the homozygous recessive parent.
38. (d)
52. (a) ABO system consists of four blood groups-A, B,
39. (d) A fossil is the mineralized partial or complete form AB and O. ABO blood groups are controlled by
of an organism, or of an organism’s activity, that has gene I. The gene has three alleles IA, IB and i. This
been preserved as a cast, impression or mold. Thus all phenomenon is known as multiple allelism. IA and
the given options are the examples of fossils. IB are completely dominant over i. When IA and IB
40. (b) Gene flow is the movement of genes from one are present together, they both express themselves
population to another population. It helps to prevent and produce AB blood group. This phenomenon is
the genetic evolution of reproductive isolation. known as co-dominance.
41. (d) 42. (d) 53. (a) Black is the phenotype of the sheep. Without further
43. (a) There are several disorders that are caused by information, you cannot identify the genotype of a
black sheep because it could be either BB or Bb.
abnormal sex-linked traits. A common Y-linked
The possible genotypes of the parents of a black sheep
disorder is male infertility. In addition to
could be BB × BB, BB × Bb, Bb × bb, or Bb × Bb.
hemophilia, other X-linked recessive disorders
include color blindness, muscular dystrophy etc. 54. (b)
44. (d) A population is the smallest unit of living organisms 55. (c) Here :
that can undergo evolution.
Genotype of Full Colour - CchCch
45. (d)
Genotype of Chinchilla - CchCh or Cchc
46. (c) Chromosomes carry gene that passes on the traits of
parents to the offspring during genetic recombination. Genotype of Himalayan - ChCh or Chc
47. (d) The given figure of Bougainvillea and Cucurbita Genotype of Albino - cc
shows an example of homologous organ or divergent
Parents : Cchc × Chc
evolution. Homologous organs are those organs
which are dissimilar in shape, size and function but
their origin, basic plan and development are similar. h
Gametes Cch c C c
Other examples in animals are forelimbs of frog,
reptile, birds and mammals. Such differences are
due to divergent evolution or adaptation for varied
conditions. ch h
C C C chc h
Cc cc
48. (c) The theory of natural selection states that those
Chinchilla Himalayan Albino
individuals that are better adapted to their 50% 25% 25%
environment will have greater reproductive success.
Heredity and Evolution S-105
56. (a) Pure breeding – Yellow round × Wrinkled green
X Y (Male, Haemophilic) XX (Female, Normal) traits seeds seeds
(YYRR) (yyrr)
X Y X X F1 – Yellow round seeds
(YyRr)

Gametes
XX XX XY XY
Female – Carrier yR yr
YR Yr
Male
(Normal)
Since all the male progeny will get the X chromosome YR Yr yR yr
form their mother, they will all be normal.
YYRR YYRr YyRR YyRr
57. (d)
YR yellow yellow yellow yellow
58. (d) According to dihybrid phenotypic ratio round round round round
YYRr Yyrr YyRr Yyrr
9 : 3 : 3 : 1, Yr yellow yellow yellow yellow
round wrinkled round wrinkled
TW — 9 Tw —3
YyRR YyRr yyRR yyRr
tW – 3 tw –1 yR yellow yellow green green
round round round round
The total number of short and blue flowered plants is
YyRr Yyrr yyRr yyrr
1 1080 yr yellow yellow green green
× 2130 = = 135 round wrinkled round wrinkled
16 8
59. (b) Artificial selection is characterized by intentional The genotype ratio is 1 : 2 : 1 : 2 : 4 : 2 : 1 : 2 : 1
reproduction of an organism having desirable traits. The phenotypic ratio is 9 : 3 : 3 : 1
It is also called as selective breeding. ∴ The number of different genotypes and phenotypes
60. (b) Analogous organs are such organs which show obtained would be 9 and 4, respectively.
anatomically different structures but doing similar
67. (b) Phenotypic ratio can be determined by doing a test
functions. Example : wings of a bat and wings of a
pigeon. cross and identifying the frequency of a trait or trait
combinations that will be expressed based on the
61. (b) 62. (a) 63. (b) genotypes of the offspring.
64. (b) Chromosomes carry genes, which are the hereditary Parents Plant-1 × Plant-2
characters to the offspring. Phenotype Red White
65. (a) The genotypes of offspring of parents having IAIO coloured
flowers
coloured
flowers
and IAIB blood groups are: Genotype RR × rr
A O A B
Parents I I × I I r
Gametes R
F1 generation Rr
(all red
coloured
Genotype A A A B A O B O flowers)
I I I I I I I I
Phenotype Blood Group A Blood Group AB Blood Group A Blood Group B Self fertilization
Offspring
Rr × Rr

From the above cross, it is shown that none of the F1 gametes r r


R R
offspring will be of blood group O.
∴ The probability of their first child having type O R r
blood is zero.
F2 generation R RR Rr
66. (c) In the given question, both parents are heterozygous Red
RR
Red
rr
r
for two pairs of genes. This means the cross is a Red white
dihybrid cross. Phenotype : Red : White
Let us assume a dihybrid cross, 3 : 1
S-106 Science
68. (c) According to the new genetic research-when external ear freely and these muscles are called
combined with known fossils-the lineage that led vestigial organs.
to humans, chimps, and gorillas evolved from a
common ancestor about 10 million years ago. 81. (a) 82. (a)

69. (c) 83. (a) There is an inheritable quality to fingerprints. Pattern


Parents: XhY × XX types are often genetically inherited, but the individual
Haemophilic man Normal female details that make a fingerprint unique are not.
84. (a) Birds have very light, honeycombed or hollow
Xh Y
bones which help them to minimise the pull of
X XhX XY
gravity, mostly by reducing body weight.
X XhX XY
  85. (c)
70. (c) Since, all the progeny bore violet flowers, it 86. (a) The banding pattern seen on stained chromosomes
implies that the tall plant with violet flowers has from humans and chimpanzee show striking
genotype ‘WW’ for violet flower colour. Since, similarities which indicates that they have
the progeny obtained is both tall and short, the evolutionary relationships (cytogenetic evidence).
parent plant was not a pure tall plant and bears
87. (c) According to Lamarck’s theory, continuous disuse
genes that determine short height of the plant. of organs make them weak. The theory of continuity
Therefore, the genotype of the plant with respect of germplasm was given by Weismann.
to height would be ‘Tt’. So, if a cross is carried
88. (a) Genotype of the organism include all dominant and
out between tall parent with violet flowers
recessive characters.
(TtWW) and short parent with white flowers
(ttww), the progeny obtained is TtWw (8) : ttWw 89. (A) → (s), (B) → (r), (C) → (p), (D) → (q)
(4) : ttWw(4). All the progeny bear violet flowers 90. (A) → (r), (B) → (s), (C) → (q), (D) → (p)
but half of them are tall and half are short.
91. (A) → (w), (B) → (v), (C) → (u), (D) → (t), (E) → (s),
71. (d) Homologous organ have same origin but different
(F) → (r), (G) → (q), (H) → (p)
functions like each of the organ above. Our teeth
and elephant’s tusks, our arm and a dog’s fore-leg 92. garden pea 93. gene
and potato and runners of grass are the examples of 94. pairs 95. dominant
homologous organs.
96. 3 : 1 97. 9 : 3 : 3 : 1
72. (a) Homologous organs
98. 23 99. XY
(b)
Analogous organs
100. acquired traits 101. heredity
(c)
Image A shows common ancestry
102. recessive 103. RNA
(d)
Image B i.e., analogous organs have a different
origin but common functions. 104. protein 105. tt
73. (a) Round yellow 106. Seven 107. Wild
(b)
A (round) and B (yellow) are dominant traits 108. Allopatric 109. heterotrophs
(c)
Round-green 110. paleontology 111. radioactive materials
(d)
Wrinkled-yellow 112. Darwin 113. analogous
(e)
(i) A–D (ii) A–B 114. homologous
74. (b) 75. (d) 76. (c) 77. (b) 115. True 116. True 117. True 118. False
78 (a) 79. (a) 119. True 120. False 121. False 122. True
80. (c) Ear muscles of external ear in man are poorly 123. False 124. True 125. True 126. False
developed. These muscles are useless which move
127. True 128. True 129. False
11 Acids, Bases and
Electricity
Salts
7. Device used to measure electric current is:
Multiple Choice Questions (MCQs) (a) Ammeter (b) Voltmeter
(c) Galvanometer (d) Generator
DIRECTIONS : This section contains multiple choice
questions. Each question has four choices (a), (b), (c) and (d) 8. Reciprocal of resistance is called:
out of which only one is correct. (a) Inductance (b) Conductance
(c) Resistivity (d) None of these
1. The charge of 150 coulomb flows through a wire in one 9. Find the equivalent resistance between A and B of
minute. What is the electric current flowing through it? following circuit:
(a) 2.5 A (b) 3.5 A C
(c) 4.5 A (d) 5.5 A 5  6
A B
2. Find the current I flown in the circuit
(a) 0.05 A I
6 D 5
(b) 5 A
30  6 5
(c) 50 A (a) Ω (b) Ω
1.5 V 2 2
(d) 500 A
11 1
3. A current of 10 A flows through a conductor for 2 (c) Ω (d) Ω
2 2
minutes. What is the amount of charge passed through the
conductor? 10. The maximum resistance which can be made using four
(a) 1200 C (b) 150 C 1
resistors each of resistance W is [CBSE 2020]
(c) 18 C (d) 1.8 C 2
4. A current of 10 A flows through a conductor for 2 minutes. (a) 2W (b) 1 W
Find the total number of electrons flowing through the (c) 2.5 W (d) S W
conductor. 11. A current of 1 A is drawn by a filament of an electric bulb.
(a) 75 × 10 20 (b) 70 × 1015 Number of electrons passing through a cross section of
15
(c) 60 × 10 (d) 11 × 1012 the filament in 16 seconds would be roughly
(a) 1020 (b) 1016
5. A metal wire 80 cm long and 1.00 mm2 in cross-section
has a resistance of 0.92 ohm. It’s resistivity is: (c) 1018 (d) 1023
(a) 0.000 115 ohm m (b) 0.0 115 ohm m 12. The proper representation of series combination of cells
(c) 1.15 ohm m (d) None of these (Figure) obtaining maximum potential is
6. 1 Ampere is equivalent to:
(i)
1coulomb 1volt
(a) (b)
1 sec 1 sec

1volt meter (ii)


(c) (d) None
1 sec
S-108 Science
21. Three resistances of 2Ω, 3Ω and 5Ω are connected in
(iii) parallel to a 10V battery of negligible internal resistance.
The potential difference across the 3Ω resistance will be
(a) 2 V (b) 3 V
(c) 5 V (d) 10 V
(iv)
22. Two unequal resistances are connected in parallel. Which
of the following statement is true
(a) (i) (b) (ii) (a) current in same in both
(c) (iii) (d) (iv) (b) current is larger in higher resistance
13. A cylindrical conductor of length l and uniform area of (c) voltage-drop is same across both
crosssection A has resistance R. Another conductor of (d) voltage-drop is lower in lower resistance
length 2l and resistance R of the same material has area 23. You are given n identical wires, each of resistance R.
of cross section [CBSE 2020] When these are connected in parallel, the equivalent
(a) A/2 (b) 3A/2 resistance is X. When these will be connected in series,
(c) 2A (d) 3A then the equivalent resistance will be
(a) X/n2 (b) n2X
14. If ‘i’ is the current flowing through a conductor of
resistance ‘R’ for time ‘t’. then the heat produced (Q) is (c) X/n (d) nX
given by 24. A piece of wire of resistance R is cut into five equal
parts. These parts are then connected in parallel. If the
i2R iR 2 equivalent resistance of this combination is R’, then the
(a) (b) ratio R/R’ is
t t
2 (a) 1/25 (b) 1/5
(c) i Rt (d) iRt2
(c) 5 (d) 25
15. An electric kettle consumes 1 kW of electric power when
25. 2 ampere current is flowing through a conductor from a
operated at 220 V. A fuse wire of what rating must be
10 volt emf source then resistance of conductor is
used for it?
(a) 20 Ω (b) 5 Ω
(a) 1 A (b) 2 A
(c) 12 Ω (d) 8 Ω
(c) 4 A (d) 5 A
26. Charge on an electron is 1.6 × 10–19 coulomb. Number of
16. A cylindrical rod is reformed to twice its length with no electrons passing through the wire per second on flowing
change in its volume. If the resistance of the rod was R, of 1 ampere current through the wire will be
the new resistance will be (a) 0.625 × 10–19 (b) 1.6 × 10–19
(a) R (b) 2R (c) 1.6 × 10–19 (d) 0.625 × 1019
(c) 4R (d) 8R
27. 20 coulomb charge is flowing in 0.5 second from a point
17. What is the current through a 5.0 ohm resistor if the
in an electric circuit then value of electric current in
voltage across it is 10V
amperes will be
(a) zero (b) 0.50 A (a) 10 (b) 40
(c) 2.0 A (d) 5.0 A (c) 0.005 (d) 0.05
18. The length of a wire is doubled and the radius is doubled. 28. In this circuit, the value of I2 is
By what factor does the resistance change
(a) 0.2 A I1 10 
(a) 4 times as large (b) twice as large
(c) unchanged (d) half as large (b) 0.3 A 1.2A I2 15 
19. Resistance of a metallic conductor depends on _______. (c) 0.4 A
(a) its length (b) its area of cross section I3 30 
(c) its temperature (d) All the above (d) 0.6 A

20. A 24V potential difference is applied across a parallel 29. A letter ‘A’ is constructed of a uniform wire of resistance
combination of four 6 ohm resistor. The current in each 1 ohm per cm. The sides of the letter are 20 cm and the
resistor is cross piece in the middle is 10 cm long. The resistance
(a) 1 A (b) 4 A between the ends of the legs will be
(c) 16 A (d) 36 A (a) 32.4 ohm (b) 28.7 ohm
(c) 26.7 ohm (d) 24.7 ohm
Electricity S-109
30. A wire of resistance R is cut into ten equal parts which are 37. The unit for specific resistance is
then joined in parallel. The new resistance is (a) ohm × second (b) ohm × m
(a) 0.01 R (b) 0.1 R (c) ohm (d) ohm/cm
(c) 10 R (d) 100 R
38. If the temperature of a conductor is increased, its
31. If a wire is stretched to make its length three times, its resistance will
resistance will become (a) not increase
(a) three times (b) one-third (b) increase
(c) nine times (d) one-ninth (c) decrease
32. The resistivity of a wire depends on (d) change according to the whether
(a) length (b) area of cross-section 39. The equivalent resistance between points a and b of a
(c) material (d) All the above network shown in the figure is given by
33. The effective resistance between the points A and B in the 3
(a) R
figure is 4
D R b
(a) 5 Ω 3 3 4
(b) R R
(b) 2 Ω 6 3
A C R O a
(c) 3 Ω 5 R
(c) R
(d) 4 Ω 3 3 4
B 4
34. In the circuits shown below the ammeter A reads 4 amp. (d) R
and the voltmeter V reads 20 volts. The value of the 5
resistance R is 40. Two wires of resistance R1 and R2 are joined in parallel.
R
A The equivalent resistance of the combination is
(a) R1R2/(R1 + R2) (b) (R1 + R2)
(c) R1 × R2 (d) R1/R2
V
41. In the given circuit, the potential of the point E is
(a) slightly more than 5 ohms
E 1
(b) slightly less than 5 ohms (a) Zero A + – D
(c) exactly 5 ohms
8V
(d) None of the above (b) – 8 V
35. Three resistors are connected to form the sides of a (c) – 4/3 V
triangle ABC as shown below. B C
B
60
C (d) 4/3 V 5

42. The resistance of a thin wire in comparison of a thick


40 100 wire of the same material
(a) is low
(b) is equal
(c) depends upon the metal of the wire
A
(d) is high
The resistance of side AB is 40 ohms, of side BC 60
43. If the specific resistance of a wire of length l and radius r
ohms and of side CA 100 ohms. The effective resistance
is k then resistance is
between the point A and B in ohms is
(a) 50 (b) 64 (a) kpr2/l (b) pr2/lk
(c) 32 (d) 100 kl/pr2
(c) (d) k/lr2
36. If one micro-amp. current is flowing in a wire, the number 44. If a charge of 1.6 × 10–19 coulomb flows per second
of electrons which pass from one end of the wire to the through any cross section of any conductor, the current
other end in one second is constitute will be
(a) 6.25 × 1012 (b) 6.25 × 1015 (a) 2.56 × 10–19 A (b) 6.25 × 10–19 A
(c) 6.25 × 10 18 (d) 6.25 × 1019 –19
(c) 1.6 × 10 A (d) 3.2 × 10–19 A
S-110 Science
45. The number of electrons flowing per second through any 55. The resistance of some substances become zero at very
cross section of wire, if it carries a current of one ampere, low temperature, then these substances are called
will be (a) good conductors (b) super conductors
(a) 2.5 × 1018 (b) 6.25 × 1018 (c) bad conductors (d) semi conductors
(c) 12.5 × 1018 (d) 5 × 1018
56. The resistance of wire is 20Ω. The wire is stretched to
46. The number of electron passing through a heater wire in three time its length. Then the resistance will now be
one minute, if it carries a current of 8 ampere, will be (a) 6.67 Ω (b) 60 Ω
(a) 2 × 1020 (b) 2 × 1021 (c) 120 Ω (d) 180 Ω
(c) 3 × 1020 (d) 3 × 1021
57. When the resistance of copper wire is 0.1 Ω and the
47. The heat produced in a wire of resistance ‘x’ when a radius is 1 mm, then the length of the wire is (specific
current ‘y’ flow through it in time ‘z’ is given by resistance of copper is 3.14 × 10–8 ohm × m)
(a) x2×y ×z (b) y × z2 × x (a) 10 cm (b) 10 m
(c) x×z×y 2 (d) y × z × x (c) 100 m (d) 100 cm
48. In a wire of length 4m and diameter 6mm, a current of 58. When the resistance wire is passed through a die the cross-
120 ampere is passed. The potential difference across the section area decreases by 1%, the change in resistance of
wire is found to be 18 volt. The resistance of wire will be the wire is
(a) 0.15 ohm (b) 0.25 ohm (a) 1% decrease (b) 1% increase
(c) 6.660 ohm (d) None of the these (c) 2% decrease (d) 2% increase
49. The resistance of an incandescent lamp is 59. The lowest resistance which can be obtained by
(a) greater when switched off 1
connecting 10 resistors each of ohm is
10
(b)
smaller when switched on
(c)
grater when switched on 1 1
(a) Ω (b) Ω
(d) Same whether it is switched off or switched on 250 200
50. If resistance of a wire formed by 1.cc of copper be 2.46Ω.
1 1
The diameter of wire is 0.32 mm, then the specific (c) Ω (d) Ω
resistance of wire will be 100 10
(a) 1.59 × 10–6 ohm. cm (b) 2.32 × 10–6 ohm. cm 60. The resistance 4R, 16R, 64R, ..........∞ are connected in
(c) 3.59 × 10–6 ohm. cm (d) 1.59 × 10–8 ohm. cm series, their resultant will be
51. A given piece of wire length , cross sectional area A and (a) 0 (b) ∞
resistance R is stretched uniformly to a wire of length 2. (c) 4/3 R (d) 3/4 R
The new resistance will be 61. Resistance R, 2R, 4R, 8R .............. ∞ are connected in
(a) 2 R (b) 4 R parallel. Their resultant resistance will be
(c) R/2 (d) Remains unchanged (a) R (b) R/2
(c) 0 (d) ∞
52. A given piece of wire of length , radius r and resistance
R is stretched uniformly to a wire of radius (r/2). The new 62. The equivalent resistance between points X and Y is
resistances will be X
(a) 2 R (b) 4 R R R R R
(c) 8 R (d) 16 R R R R R
Y
53. There are two wires of the same length and of the same
(a) R (b) 2R
material and radial r and 2r. The ratio of their specific
(c) R/2 (d) 4 R
resistance is
(a) 1 : 2 (b) 1 : 1 63. The equivalent resistance between points A and B is
(c) 1 : 4 (d) 4 : 1
A 7 5 B
54. Specific resistance of a wire depends on the
(a) length of the wire 2 6
(b) area of cross-section of the wire
(a) 4Ω (b) 4.5 Ω
(c) resistance of the wire
(c) 2 Ω (d) 20 Ω
(d) material of the wire
Electricity S-111
64. Three resistances 4Ω each of are connected in the form of 73. When a wire of uniform cross-section a, length  and
an equilateral triangle. The effective resistance between resistance R is bent into a complete circle, resistance
two corners is between any two of diametrically opposite points will be
(a) 8Ω (b) 12Ω (a) R/4 (b) R/8
(c) 3/8 Ω (d) 8/3 Ω (c) 4R (d) R/2
65. Two wires of same metal have the same length but their 74. A solenoid is at potential difference 60V and current
cross-sections area in the ratio 3 : 1. They are joined in flows through it is 15 ampere, then the resistance of coil
series. The resistance of the thicker wire is 10Ω. The will be
total resistance of the combination will be
(a) 4 Ω (b) 8 Ω
(a) 40 Ω (b) 40/3 Ω
(c) 0.25 Ω (d) 2 Ω
(c) 5/2 Ω (d) 100 Ω
75. A strip of copper and another of germanium are cooled
66. A certain piece of silver of given mass is to be made like
from room temperature to 80 K. The resistance of
a wire. Which of the following combination of length
(L) and the area of cross-sectional (A) will lead to the (a) Each of these increases
smallest resistance (b) Each of these decreases
(a) L and A (c) Copper strip increases and that of germanium decreases
(b) 2L and A/2 (d) Copper strip decreases and that of germanium increases
(c) L/2 and 2A 76. In the circuit shown in the figure, the current through
(d) Any of the above, because volume of silver remains same
67. A certain wire has a resistance R. The resistance of another
wire identical with the first except having twice of its
diameter is
(a) 2 R (b) 0.25 R
(c) 4 R (d) 0.5 R
68. What length of the wire of specific resistance 48 × 10–8
Ω-m is needed to make a resistance of 4.2 Ω (diameter of
wire = 0.4 mm)
(a) 4.1 m (b) 3.1 m (a) the 3Ω resistor is 0.50A
(c) 2.1 m (d) 1.1 m (b) the 3Ω resistor is 0.25A
69. The resistance of an ideal voltmeter is (c) the 4Ω resistor is 0.50A
(a) zero (b) very low (d) the 4Ω resistor is 0.25A
(c) very large (d) Infinite 77. Two electric lamps each of 100 watts 220V are connected
70. Masses of 3 wires of same metal are in the ratio 1 : 2 : 3 in series to a supply of 220 volts. The power consumed
and their lengths are in the ratio 3 : 2 : 1. The electrical would be –
resistances are in ratio (a) 100 watts (b) 200 watts
(a) 1 : 4 : 9 (b) 9 : 4 : 1 (c) 25 watts (d) 50 watts
(c) 1 : 2 : 3 (d) 27 : 6 : 1 78. If it takes 8 minutes to boil a quantity of water electrically,
71. We have two wires A and B of same mass and same how long will it take to boil the same quantity of water
material. The diameter of the wire A is half of that B. If using the same heating coil but with the current doubled
the resistance of wire A is 24 ohm then the resistance of (a) 32 minutes (b) 16 minutes
wire B will be (c) 4 minutes (d) 2 minutes
(a) 12 ohm (b) 3.0 ohm 79. An electric bulb is filled with
(c) 1.5 ohm (d) None of the above (a) hydrogen (b) oxygen and hydrogen
72. The electric resistance of a certain wire of iron is R. If its (c) ammonia (d) nitrogen and argon
length and radius are both doubled, then 80. When current is passed through an electric bulb, its
(a) The resistance will be doubled and the specific filament glows, but the wire leading current to the bulb
resistance will be halved does not glow because
(b) The resistance will be halved and the specific (a) less current flows in the leading wire as compared to
resistance will remain unchanged that in the filament
(c) The resistance will be halved and the specific (b) the leading wire has more resistance than the
resistance will be doubled filament
(d) The resistance and the specific resistance, will both (c) the leading wire has less resistance than the filament
remain unchanged (d) filament has coating of fluorescent material over it
S-112 Science
81. Which of the following terms does not represent electrical 89. A current I passes through a wire of length l, radius r and
power in a circuit? resistivity ρ. The rate of heat generated is
(a) I2R (b) IR2
I 2ρ 
I 2ρ  (b)
(c) VI (d) V2/R (a)
r π r2
82. The power dissipated in the circuit shown in the figure is
30 watts. The value of R is I 2ρ 
(c) (d) none of these
(a) 20 Ω πr
R 90. The resistance R1 and R2 are joined in parallel and a current
(b) 15 Ω
is passed so that the amount of heat liberated is H1 and H2
(c) 10 Ω 5 respectively. The ratio H1/H2 has the value
(d) 30 Ω 10  R2/R1 (b)
(a) R1/R2
83. The filament of an electric bulb is of tungsten because
R12/R22
(c) (d) R22/R12
(a) Its resistance is negligible
91. Power dissipated across the 8Ω resistor in the circuit
(b) It is cheaper shown here is 2 watt. The power dissipated in watt units
(c) Its melting point is high across the 3Ω resistor is
(d) Filament is easily made
(a) 1.0 1 3 i1
84. When the current passes through the filament, it gets
heated to incandescence and give light while the (b) 0.5 i
connecting wires are not heated because (c) 3.0
(a) The connecting wires are good conductor of heat (d) 2.0 8 i2
while the filament is bad conductor
(b) The connecting wires are of low resistance while the 92. In house electrical circuits the fuse wire for safety should
filament is of high resistance be of
(c) The density of connecting wires is less than that of (a) High resistance – high melting point
the filament (b) Low resistance – high melting point
(d) The connecting wires are bad conductor of heat (c) Low resistance – low melting point
while the filament is good conductor (d) High resistance – low melting point
85. Which one of the following heater element is used in 93. What is the equivalent resistance of the following
electric press arrangement between M and N
(a) copper wire (b) nichrome wire
(c) lead wire (d) iron wire
86. What should be the characteristic of fuse wire? M R R R N

(a) High melting point, high specific resistance
(b) Low melting point, low specific resistance (a) R/2 (b) R/3
(c) High melting point, low specific resistance (c) R/4 (d) R/6
(d) Low melting point, high specific resistance
94. If a wire of resistance 1Ω is stretched to double its length,
87. The heating element of an electric heater should be made then resistance will be
with a material, which should have
(a) high specific resistance and high melting point 1
(a) Ω (b) 2 Ω
(b) high specific resistance and low melting point 2
(c) low specific resistance and low melting point 1
(d) low specific resistance and high melting point (c) Ω (d) 4 Ω
4
88. Resistance of conductor is doubled keeping the potential 95. Across a metallic conductor of non-uniform cross section
difference across it constant. The rate of generation of a constant potential difference is applied. The quantity
heat will which remains constant along the conductor is :
(a) become one fourth (b) be halved (a) current (b) drift velocity
(c) be doubled (d) become four times (c) electric field (d) current density
Electricity S-113
96. In the circuit diagram shown below, VA and VB are the 102. In the circuit given, the ratio of work done by the battery
potentials at points A and B respectively. Then,VA – VB is to maintain the current between point A and B to the work
10  A 20 
done for the whole circuit is
1
2 A 1 B 2
1
10  B 5 I

30V 1.3V
(a) –10 V (b) –20 V
1 1
(c) 0 V (d) 10 V (a) (b)
117 13
97. The diameter of a wire is reduced to one-fifth of its
1
original value by stretching it. If its initial resistance is (c) (d) 1
R, what would be its resistance after reduction of the 12
diameter? 103. What is the current supplied by the battery in the circuit
shown below? Each resistance used in circuit is of 1 kW
R R
(a) (b) and potential difference VAB= 8V
625 25
(c) 25 R (d) 625 R (a) 64 mA A
98. A heater coil is cut into two equal parts and only one part
(b) 15 mA
is used in the heater, the heat generated now will be
(a) doubled (b) four times (c) 9.87 mA
(c) one fourth (d) halved B
99. The resistance of a wire is R. After melting it is remouled (d) 1 mA
such that its area of cross section becomes n times its
initial area of cross section. It new resistance will be 104. A wire of resistance R is bent to form a A B
R square ABCD as shown in the figure. The
(a) nR (b) effective resistance between E and C is:
n
(E is mid-point of arm CD)
R 7 D C
(c) n2R (d) (a) R (b) R
E
n2 64
100. The resistance of a wire is ‘R’ ohm. If it is melted and 3 1
stretched to ‘n’ times its original length, its new resistance (c) R (d) R
4 16
will be :
105. Which of the following acts as a circuit protection device?
R
(a) (b) n2R (a) conductor (b) inductor
n
(c) switch (d) fuse
R
(c) 2 (d) nR 106. If the ammeter in the given circuit reads 2 A, What is
n the value of resistence R (the resistance of ammeter is
negligible).
101. Three electric bulbs of rating 40 W – 200 V; 50 W – 200
3
V and 100 W – 200 V are connected in series to a 600 V
R
supply. What is likely to happen as the supply is switched
on?
6
(a) Only 50 W bulb will fuse
(b) Both 40 W and 50 W bulbs will fuse.
(c) All the three bulbs will emit light with their rated 6V
powers. A
(d) 100 W bulb will emit light of maximum intensity. (a) 1 W (b) 2 W
(c) 3 W (d) 4 W
S-114 Science
107. A circuit to verify Ohm’s law uses ammeter and voltmeter (a) 0 V 6V 2k
in series or parallel connected correctly to the resistor. In
the circuit : (b) 4.8 V
8k
(a) ammeter is always used in parallel and voltmeter is V
(c) 6.0 V
series
(b) Both ammeter and voltmeter must be connected in (d) 1.2 V A
parallel
(c) ammeter is always connected in series and voltmeter 113. A student in a town in India, where the price per unit (1
in parallel unit = 1 kW-hr) of electricity is `5.00, purchases a 1 kVA
(d) Both, ammeter and voltmeter must be connected in UPS (uninterrupted power supply) battery. A day before
series
the exam, 10 friends arrive to the student’s home with their
108. An electric bulb is rated 220V and 100W. When it is laptops and all connect their laptops to the UPS. Assume
operated on 110V, the power consumed will be that each laptop has a constant power requirement of 90 W.
(a) 100W (b) 75W Consider the following statements
(c) 50W (d) 25W I All the 10 laptops can be powered by the UPS if
connected directly.
109. From a power station, the power is transmitted at a very
II All the 10 laptops can be powered if connected using
high voltage because –
an extension box with a 3A fuse.
(a) it is generated only at high voltage
(b) it is cheaper to produce electricity at high voltage III If all the 10 friends use the laptop for 5 hours, then
the cost of the consumed electricity is about `22.50.
(c) electricity at high voltage is less dangerous
(d) there is less loss of energy in transmission at high Select the correct option with the true statements.
voltage (a) I only (b) I and II only
110. Two electric bulbs rated P1 watt V volts and P2 watt V (c) I and III only (d) II and III only
volts are connected in parallel and applied across V volts.
114. A copper wire is stretched to make it 0.5% longer. The
The total power (in watts) will be
percentage change in its electrical resistance if its volume
P1 + P2 (b)
(a) P1 P2 remains unchanged is:
(a) 2.0% (b) 2.5%
PP P1 + P2 (c) 1.0% (d) 0.5%
(c) 1 2 (d)
P1 + P2 P1 P2
115. Six similar bulbs are connected as
111. In the circuit, wire 1 is of negligible resistance. Then, shown in the figure with a DC source A B
R1 R2 of emf E, and zero internal resistance.
The ratio of power consumption by the
bulbs when (i) all are glowing and (ii)
Wire 1 in the situation when two from section
+ – + – A and one from section B are glowing,
1 2
will be: E
(a) current will flow through wire 1, if ε1 ≠ ε2 (a) 4 : 9 (b) 9 : 4
(c) 1 : 2 (d) 2 : 1
ε1 ε 2
(b) current will flow through wire 1, if ≠
R1 R2
(c) current will flow through wire 1, if

ε1 + ε 2 ε −ε DIRECTIONS : Study the given case/passage and answer the


≠ 1 2 following questions.
( R1 + R2 ) ( R1 − R2 )

(d) no current will flow through wire 1 Case/Passage - 1


Two tungston lamps with resistances R1 and R2 respectively at
112. In the circuit shown below, a student performing Ohm’s full incandescence are connected first in parallel and then in
law experiment accidently puts the voltmeter and the series, in a lighting circuit of negaligible internal resistance. It
ammeter as shown in the circuit below. The reading in is given that: R1 > R2.
the voltmeter will be close to
Electricity S-115
116. Which lamp will glow more brightly when they are 121. Which of the following terms does not represent electrical
connected in parallel? power in a circuit?
(a) Bulb having lower resistance (a) I2R (b) IR2
(b) Bulb having higher resistance (c) VI (d) V2/R
(c) Both the bulbs
122. An electric bulb is rated 220V and 100W. When it is
(d) None of the two bulbs operated on 110V, the power consumed will be–
117. If the lamp of resistance R1 now burns out, how will the (a) 100 W (b) 75 W
illumination produced change?
(c) 50 W (d) 25 W
(a) Net illumination will increase
123. Two conducting wires of the same material and of equal
(b) Net illumination will decrease
lengths and equal diameters are first connected in sereis
(c) Net illumination will remain same
and then in parallel in an electric circuit. The ratio of heat
(d) Net illumination will reduced to zero produced in series and in parallel combinations would
118. Which lamp will glow more brightly when they are be–
connected in series? (a) 1 : 2 (b) 2 : 1
(a) Bulb having lower resistance (c) 1 : 4 (d) 4 : 1
(b) Bulb having higher resistance
124. In an electrical circuit three incandescent bulbs. A, B
(c) Both the bulbs and C of rating 40 W, 60 W and 100 W, respectively are
(d) None of the two bulbs connected in parallel to an electric source. Which of the
119. If the lamp of resistance R2 now burns out and the lamp following is likely to happen regarding their brightness?
of resistance R1 alone is plugged in, will the illumination (a) Brightness of all the bulbs will be the same
increase or decrease? (b) Brightness of bulb A will be the maximum
(a) Illumination will remain same
(c) Brightness of bulb B will be more than that of A
(b) Illumination will increase
(d) Brightness of bulb C will be less than that of B
(c) Illumination will decrease
125. In an electrical circuit, two resistors of 2Ω and 4Ω
(d) None
respectively are connected in series to a 6V battery.
120. Would physically bending a supply wire cause any The heat dissipated by the 4Ω resistor in 5s will be
change in the illumination?
(a) 5 J (b) 10 J
(a) Illumination will remain same
(c) 20 J (d) 30 J
(b) Illumination will increase
(c) Illumination will decrease Case/Passage - 3
(d) It is not possible to predict from the given datas Answer the following questions based on the given circuit.
Case/Passage - 2 3
The rate at which electric energy is dissipated or consumed in 4
an electric circuit. This is termed as electric power, A 6 B
P = IV, According to Ohm’s law V = IR
We can express the power dissipated in the alternative forms
2 V2
P I=
= R
R 3V
If 100W – 220V is written on the bulb then it means that the bulb 126. The potential drop across the 3Ω resistor is
will consume 100 joule in one second if used at the potential
(a) 1 V (b) 1.5 V
difference of 220 volts. The value of electricity consumed in
houses is decided on the basis of the total electric energy used. (c) 2 V (d) 3 V
Electric power tells us about the electric energy used per second 127. The equivalent resistance between points A and B is
not the total electric energy. (a) 7Ω (b) 6Ω
The total energy used in a circuit = power of the electric circuit
× time. (c) 13Ω (d) 5Ω
S-116 Science
128. The current flowing through in the given circuit is Case/Passage - 6
(a) 0.5 A (b) 1.5 A Answer the following questions based on the given circuit.
(c) 6 A (d) 3 A 2V
Case/Passage - 4 I + – ((
Answer the following questions based on the given circuit.
0.5
6
A 12 3.0 5.0 4.0 B I1
Y
I2
I
12V 2
135. The total resistance of the circuit is
(a) 2 Ω (b) 4 Ω
129. The equivalent resistance between points A and B, is
(c) 1.5 Ω (d) 0.5 Ω
(a) 12 Ω (b) 36 Ω 136. The current flowing through 0.5Ω resistor is
(c) 32 Ω (d) 24 Ω (a) 1 A (b) 1.5 A
130. The current through each resistor is (c) 3 A (d) 2.5 A
(a) 1 A (b) 2.3 A 137. The current flowing through 6Ω resistor is
(c) 0.5 A (d) 0.75 A (a) 0.50 A (b) 0.75 A
(c) 0.80 A (d) 0.25
131. The potential drop across the 12Ω resistor is
(a) 12 V (b) 6 V
(c) 8 V (d) 0.5 V Assertion & Reason

Case/Passage - 5 DIRECTIONS : Each of these questions contains an assertion


followed by reason. Read them carefully and answer the question
Answer the following questions based on the given circuit. on the basis of following options. You have to select the one that
10 best describes the two statements.

A 3.0 4.0 B (a) If both Assertion and Reason are correct and Reason is
the correct explanation of Assertion.
5.0
(b) If both Assertion and Reason are correct, but Reason is
not the correct explanation of Assertion.
3.0 I
(c) If Assertion is correct but Reason is incorrect.

12V (d) If Assertion is incorrect but Reason is correct.

138. Assertion : Fuse wire must have high resistance and low
132. The equivalent resistance between points A and B melting point.
(a) 6.2 Ω (b) 5.1 Ω
Reason : Fuse is used for very small current flow only.
(c) 13.33 Ω (d) 1.33 Ω
139. Assertion : Alloys are commonly used in electrical
133. The current through the battery is heating devices like electric iron and heater.
(a) 2.33 A (b) 3.12 A
Reason : Resistivity of an alloy is generally higher than
(c) 4.16 A (d) 5.19 A that of its constituent metals but the alloys have low
134. The current through the 4.0 ohm resistor is melting points than their constituent metals.
(a) 5.6 A (b) 0.98 A 140. Assertion : In a simple battery circuit, the point of lowest
(c) 0.35 A (d) 0.68 A potential is negative terminal of the battery.
Reason : The current flows towards the point of higher
potential as it flows in such a circuit from the negative to
positive terminal.
Electricity S-117
141. Assertion : The equation V = Ri can be applied to those Reason : The power dissipated in the circuit is directly
conducting devices which do not obey Ohm’s law. proportional to the resistance of the circuit.
Reason : V = Ri is a statement of Ohm’s law.
142. Assertion : All electric devices shown in the circuit are Match the Following
ideal. The reading of each of ammeter (A) and voltmeter
(V) is zero. DIRECTIONS : Each question contains statements given in
E two columns which have to be matched. Statements (A, B, C,
D) in column I have to be matched with statements (p, q, r, s) in
column II.
V
152. Column I Column II

A (A) Ohm (p) ρL


R A

Reason : An ideal voltmeter draws almost no current due to 1 volt


(B) Resistance (q)
very large resistance, and hence (A) will read zero. 1 ampere
143. Assertion : If ρ1 and ρ2 be the resistivities of the materials (C) Resistivity (r) zero resistance
of two resistors of resistances R1 and R2 respectively and (D) Super conductor (s) ohm-meter
R1 > R2, then ρ1 > ρ2.
153. Column II gives name of material use for device given in
 column I
Reason : The resistance R = ρ ⇒ ρ1 > ρ2 if R1 > R2
A Column I Column II
144. Assertion : Insulators do not allow flow of current (A) Resistance of resistance box (p) tungsten
through themselves. (B) Fuse wire (q) maganin
Reason : They have no free-charge carriers. (C) Bulb (r) tin-lead alloy
145. Assertion : Positive charge inside the cell always goes (D) Insulator (s) glass
from positive terminal to the negative terminal.
Reason : Positive charge inside the cell may go from
negative terminal to the positive terminal.
Fill in the Blanks
146. Assertion : Wire A is thin in comparison to wire B of
same material and same length then resistance of wire A DIRECTIONS : Complete the following statements with an
appropriate word / term to be filled in the blank space(s).
is greater than resistance of wire B.
Reason : Resistivity of wire A is greater than resistivity
154. The rate of flow of electric charge is called .................
of wire B.
147. Assertion : Resistivity of material may change with 155. If there is no current, a voltmeter connected across a
temperature. resistor will register..................... voltage.
Reason : Resistivity is a material property & independent
156. Combined resistance is the sum of separate resistances
on temperature.
provided that the various conductors are connected in
148. Assertion : When current through a bulb decreases by
..........
0.5%, the glow of bulb decreases by 1%.
Reason : Glow (Power) which is directly proportional to 157. In a parallel circuit, each circuit element has the same
square of current. ....................
149. Assertion : Long distance power transmission is done at 158. Copper is a preferred material for making wire because of
high voltage. its low........................
Reason : At high voltage supply power losses are less.
150. Assertion : Resistance of 50W bulb is greater than that of 159. The S.I. unit of electric current is ..............
100 W. 160. .............. is a property that resists the flow of electrons in
Reason : Resistance of bulb is inversely proportional to a conductor.
rated power.
151. Assertion : A resistor of resistance R is connected to an 161. The S.I. unit of resistance is .................
ideal battery. If the value of R is decreased, the power 162. The potential difference across the ends of a resistor
dissipated in the circuit will increase. is ............... to the current through it, provided its
.................... remains the same.
S-118 Science
163. The resistance of a conductor depends directly on its 184. Rate at which electric work is done is called ............... .
............., inversely on its ................., and also directly
proportioned on the .............. of the conductor.
True / False
164. 1 volt × 1 coulomb = ..............
165. Potential difference is a .............. quantity. DIRECTIONS : Read the following statements and write your
answer as true or false.
166. The resistance of a semiconductor ........ with increase in
temp. 185. The quantity of charge flowing through a point multiplied
167. The S.I. unit of resistivity is .............. . by time is a current.

168. Physical quantity represented by coulomb per second is 186. The resistivity of all pure metals increases with the rise in
.............. . temperature.

169. Two resistances of 2 Ω each are connected in parallel. 187. Ohm’s law is a relation between the power used in a
The equivalent resistance is .............. . circuit to the current and the potential difference.
170. The resistance of a wire is .............. proportional to the 188. Direction of current is taken opposite to the direction of
square of its radius. flow of electrons.
171. Kilowatt is the unit of electrical .............. but kilowatt- 189. The equivalent resistance of several resistors in series is
hour is the unit of electrical .................. equal to the sum of their individual resistances.
172. Energy spent in kilowatt-hour 190. In parallel combination, the reciprocal of equivalent
resistance is the sum of the reciprocal of individual
volt ×..........×.......... resistance.

1000
191. The series arrangement is used for domestic circuits.
173. A fuse is a short piece of wire of high .............. and low
..................... 192. The reciprocal of resistance is called specific resistance.

174. Fuse wire has a ............... melting point and is made of an 193. Resistivity is measured in ohm-metre.
alloy of ..............and ................. If the current in a circuit 194. The resistance of a wire is directly proportional to length.
rises too high, the fuse wire .................
195. The resistance of a wire is directly proportional to area.
175. A fuse is connected in ................. to the ......................
196. The resistivity of alloys decreases with the rise in temp.
wire.
197. The filament resistance of glowing bulb is greater, to its
176. Electric energy is produced by the ............. of charges.
resistance when it is not glowing.
177. Energy converted per unit charge is measured with an
198. The commercial unit of electrical energy is kilowatt-hour
instrument called a .....................
(kWh).
178. The electrical energy dissipated in a resistor is given by
199. Pure tungsten has high resistivity and a high melting
W = ......................
point (nearly 3000°C).
179. The unit of power is ..................
200. When a metallic conductor is heated the atoms in the
180. One watt of power is consumed when 1 A of current flows metal vibrate with greater amplitude and frequency.
at a potential difference of ...............
201. One kilowatt is equal to 10 horse power.
181. 1 kW h = ...................
202. Fuse is a thin wire which melts and breaks the electric
182. The alloy which is used for making the filament of bulbs circuit due to only high voltage.
is ...............
183. Power transmission is carried out at high.........................
and low .......................... .
Electricity S-119

ANSWER KEY & SOLUTIONS

1. (a) Q = 150 C, t = 60 sec so, = Q 150


I = = 2.5 A
t 60
V 1.5V
2. (a) V = IR ⇒ I = = = 0.05 A (4 + 4) × 4 8
R 30 Ω 64. (d) R AB = = Ω 
4+4+4 3 A B
3. (a) Q = I × t ⇒ Q = 10 A × (2 × 60 sec) = 1200 C
65. (a) Same metal means same specific resistance
4. (a) Charge on one electron e = – 1.6 × 10 –19 C
R1 A2 1
= =   ⇒  R2 = 3R1 = 3 × 10 = 30 W
So, number of electrons flown R2 A1 3
Q 10 × 2 × 60 R = R1 + R2 = 40 W
n= = = 75 × 1020
e 1.6 × 10−19
66. (c) R ∝  . Hence minimum for option (c)
l A A
5. (a) R = ρ ⇒ ρ = R = 0.000115Ω m
A l 1 th
6. (a) 7. (a) 8. (b) 67. (b) R ∝ 1 ∝ 12 . Hence
A r 4
2
(5 + 6) 11 0.4
9. (c) Requivalent= = Ω ρ 4.2 × π  × 10−3 
2 2 68. (d) R = ;= RA  2  1.1m
 = =
5 + 6 = 11 A ρ 4.8 × 10 −8
11

69. (c) Ideal voltmeter should not draw any current flow

A B 2
A B source hence its resistance = ∞.
5 + 6 = 11 Practically infinite resistance is not possible,
10. (a) To get the maximum resistance, all four resistors but ideal voltmeter is possible with the help of
should be connected in series, potentiometer that you will learn in higher classes.

1 1 1 1 70. (d) = ρ d ρ2 ; 2


∴R = Ω + Ω + Ω + Ω = 2Ω R = R∝
2 2 2 2 A m m
m m m
11. (a) 12. (a) 13. (c) 14. (c) V = A, d = = ⇒ A= 
 V A d  
15. (d) 16. (c) 17. (c) 18. (d)
9 4 1 1
R1 : R2 : R3 ≡ : : ≡ 9 : 2 : = 27 : 6 :1
19. (d) 20. (b) 21. (d) 22. (c) 1 2 3 3
23. (b) 24. (d) 25. (b) 26. (d) 71. (c) Same material → same density, specific resistance
27. (b) 28. (c) 29. (c) 30. (b) as they are material property.

31. (c) 32. (c) 33. (b) 34. (a) ρ  ρV ρm


R
= = =
35. (c) 36. (b) 37. (b) 38. (b) A A2 dA2
1 1
39. (b) 40. (a) 41. (c) 42. (d) R ∝ 2 ∝ 4
A r
43. (c) 44. (c) 45. (b) 46. (d)
R A rB4 4 24
47. (a) 48. (a) 49. (c) 50. (a) = = 2= 16 ⇒ R=
B = 1.5 Ω
RB rA4 16
51. (b) 52. (d) 53. (b) 54. (d)
72. (b) = ρ ρ
55. (b) 56. (d) 57. (b) 58. (d) R =
A πr 2
59. (b) 60. (b) 61. (b) 62. (b)
ρ2 R
=
R′ = 2
63. (b) π(2r) 4
S-120 Science
Specific resistance will remain same as it is a material 92. (d) In house electrical circuits the fuse wire for safety
property but remember it depends on temperature. should be of high resistance and low melting point.
73. (a) Two resistance (R/2) will be in parallel, hence 93. (b) An the three resistors are connected in parallel
Req = R/4
1 1 1 1 1 3
= + +   ⇒ = R =R
74. (a) V = i × R; R = 60/15 = 4 Ω Re q R R R Req R eq 3

75. (d) Copper is a conductor while germanium is a semi-


conductor. Resistance of temperature decreases with 94. (d) Resistance (R) = ρL
A
temperature while that of semi-conductor increases
Length is stretched to double
hence resistance of copper strip decreases and that
of germanium increases. L′ = 2L
A
76. (d) 2Ω, 4Ω, 2Ω on right side are in series resultant Area A′ = ∴ R′ = ρ × 2L
parallel to 8Ω then in series with 2Ω, 2Ω then in 2 A
parallel with 8Ω, then in series with 3Ω, 2Ω. Thus, 2
Req = 9 ohm.
R′ = 4 ρL ⇒ R′ = 4R
i = 9/9 = 1 amp flow from battery. A
Passing through 3Ω it will divide into equal parts R = 1Ω  ∴ New Resistance, R′ = 4Ω
(1/2 amp) in 8Ω (near to cell) and remaining section
then again divide into equal parts (1/4 amp) in 8Ω 95. (a) Here, metallic conductor can be considered as the
(middle one) and remaining section hence 1/4 amp. combination of various conductors connected in
passes through 4Ω. series. And in series combination current remains
same.
77. (d) 78. (d) 79. (d)
80. (c) 81. (b) 82. (c) 83. (c)
84. (b) 85. (a) i

86. (d) Fuse wire should be such that it melts immediatley


when strong current flows through the circuit. The V
same is possible if its melting point is low and
resistivity is high. 96. (d) Q 10Ω and 20Ω are in series = (10 + 20)Ω = 30Ω
and 10Ω and 5Ω are in series = (10 + 5)Ω = 15Ω
87. (a) A heating wire should be such that it produces more
heat when current is passed through it and also does 30 × 15 450
Reff= = = 10Ω
not melt. It will be so if it has high specific resistance 15 + 30 45
and high melting point.
88. (b) The rate of generation of heat, for a given potential So the total current I= V= 30= 3 Ampere
R 10
difference is, P = V2/ R
In branch CA current = 1A
89. (b) The rate of heat generation
In branch CB current = 2A
= I2R = I2(rl/rr2).
∴ VC – VA = 10 Volt ....(i)
90. (a) Heat produced, H = V2 t /R i.e., H ∝1/R
so H1/H2 = R2 / R1. & VC – VB = 20 Volt ....(ii)
91. (c) Power = V ⋅ I = I2R Subtracting (i) from (ii), VA – VB = 10 volt.
= Power 2 1 1A
i2 = = = d
R 8 4 2 97. (d) Let the Diameter of wire =
5
Potential over 8Ω = Ri2 = 8 × 1 = 4V Radius will be = 5
r
2
This is the potential over parallel branch. So, Changed Area will be = A = πr2
4 2 πr 2
i1= = 1A r πr 2
4 = π   =  ⇒  A =   ⇒  25A = πr2
 5 25 25
Power of 3Ω = i12R = 1 × 1 × 3 = 3W
Electricity S-121
Hence stretched length will be = 25 l 102. (b) After simplifying the given circuit, we get,
ρ  ρ(25) 1
Change resistance (R) = = = 625 R 2 3 2
A A / 25
A B
98. (a) Resistance of the heater be R.
New resistance of heater is R/2
1.3 V
V2 V2 V2
Initial power = Final power
= = 2
R R/2 R R AB = 1 Ω
3
∴  Heat generated is doubled.
Then equivalent resistance across the battery,

99. (d) R = ρ ; New area = nA ∴ New length =
1
Req = 2 + + 2 =
13

A n 3 3
ρ R V 1.3
R′
⇒  = = So current in circuit, I = R ⇒ 13 × 3 amp
n A n2
2 eq

3
ρ I = amp
100. (b) We know that, R = 10
A Power dissipated across arm AB,
2
ρ 2 PAB = I 2 × RAB =  3  × 1
or R = ⇒ R ∝ 2
Volume  10  3
3
According to question 2 = n1 PAB = = 0.03 Watt
100
R2 n 2l 2 Total power dissipated in circuit,
R = 21
1 l1 2
Pckt = I 2 × Req =  3  × 13
R2  10  3
or, = n2
R1 39
Pckt = = 0.39 watt
⇒  R2 = n2R1 100
Ratio of power across A and B to total power = Ratio
101. (b) Resistance of 40 W – 200 V, 50 W – 200 V, of work done across A and B to total circuit
100 W – 200 V are respectively. Q W = P × t
V2 200 × 200
R40 = = = 1000 Ω So, P=
AB W=AB 0.03
=
1
P40 40 Pckt Wckt 0.39 13
R50 = 800 Ω and R100= 400 Ω 103. (b) After simplifying the given circuit, we get

600 600
I = = = 0.2727 A
1000 + 800 + 400 2200

P1 40 A B C
I40 = = = 0.2 A
V 200

P2 50 5
I50 = = = = 0.25 A
V 200 20

P3 100
I100 = = = 0 .5 A
V 200 8V

Clearly, 0.2 A & 0.25 A < 0.27 A hence both 40 W Resistance between arm AB, Rnet AB= 1 =
kΩ
1000

and 50 W bulbs will fuse. 5 5
S-122 Science
1 1000 6V
Resistance between arm BC, Rnet BC = kΩ
= Ω Req = = 3 W ...(ii)
3 3 2A
So, Rnet = Rnet AB + Rnet BC where unknown resistance R, from (i) and (ii)

We get, Rnet= 1000 + 1000 R = 3 W – 2 W


5 3
8000 R = 1 W
Rnet= Ω
15 107. (c) Ammeter : In series connection, the same current
According to ohm’s law, V = IR flows through all the components. It aims at
measuring the current flowing through the circuit
8 × 15
I= = 15mA and hence, it is connected in series.
8000
104. (b) Here RDA = RAB = RBC = R/4 Voltmeter : A voltmeter measures voltage change
between two points in a circuit. So we have to place
and RDE = REC = R/8 the voltmeter in parallel with the circuit component.
Now RED, RDA, RAB, RBC are in series. 108. (d) 109. (d)

R R R R R + 2R + 2R + 2R 7R 110. (a) In parallel combination, total power P = P1 + P2


\ Rs = =+ + + =
8 4 4 4 8 8 111. (d) Current leaving the cell must be equal to current
 7R  R  going into the cell.
  
R =  8  8  7R
∴  eq = R1 R2
R 64
7 R/8
i2
E C
i1 i1 i2
E1 E2

R/8 For any value of E or R current going from first loop


to second loop must be zero.
105. (d) Fuse is an safety device that operates to provide
over current protection of an electrical circuit. A Hence, there is no current through the wire 1.
fuse is mainly a metal wire that melts when too 112. (c) The resistance of ammeter is very low and resistance
much current flows through it due to low melting of voltmeter is very high. When ammeter is put in
point and protects electric appliances. parallel to 8kΩ resistor, nearly whole of current
106. (a) I = 2 A goes through the ammeter.

3 The equivalent circuit is as follows


R + – 2k
Low
6 resistance

V
6V High resistance
A Hence, maximum potential drop occurs in the

voltmeter.
Two resistance are in parallel, So, reading of voltmeter is nearly 6 V.
1 1 1 2 +1 3 1 113. (c) Power delivered by the UPS battery is 1kVA i.e.
= + = = =
R1 3 6 6 6 2 1000 V.A = 1000W

When all the laptops connected directly to UPS then
\ R1 = 2W ...(i)
total power requirement
Voltage 90 × 10 = 900W,
Req =
Current
Electricity S-123
So battery (UPS) can provide power to all laptops. 116. (a) When the lamps are connected in parallel, then
If all laptops are used for 5 hours, then cost of potential difference V across each lamp will be
electricity consumed as the cost of electricity is same and will be equal to potential necessary for
`5.00 per unit. full brightness of each bulb. Because illumination
produced by a lamp is proportional to electric power
900 × 5 × 3600 consumed in it, and power consumed,
= × 5 = 22.5
3.6 × 106
ρ V2 V2
114. (c) Resistance, R = P1 = < = P2
A R1 R2

  ρ 2 Hence, illumination produced by 2nd bulb will be


R =ρ × = higher than produced by lst bulb, i.e., bulb having
A  V
lower resistance will shine more brightly.
[Q  Volume (V) = Al]
117. (b) When R1 burns out, then power is dissipated in R2
Since resistivity and volume remains constant
only. Because internal resistance is quite low in
therefore % change in resistance
lighting circuit, potential difference is still equal to
∆R 2∆ V, hence, power dissipated in 2nd lamp, i.e.,
=  = 2 × (0.5) =
1%
R 
V 2 V 2 V 2 
115. (b) When all bulbs are glowing < + 
R2  R1 R2 
R i/3 R i/3
i.e., net power consumed initially. In other words,
R i/3 R i/3 net illumination will now decrease.
118. (b) When two lamps are connected in series, the
R i/3 R i/3
potential difference across each lamp will be
different but current I flowing through each lamp
will be same.
E
Hence, P1 = I2R1 > I2R2 = P2
R R 2R
Req = + = i.e., illumination produced by lst lamp will be higher
3 3 3 as compared to that produced by 2nd lamp, i.e., lamp
E 2 3E 2 having higher resistance will glow more brightly.
Power (P=
i)
= …(i)
R eq 2R
119. (b) When lamp of resistance R2 burns out and only
When two from section A and one from section B lamp of resistance R1 is connected in the circuit
are glowing, then then current flowing the circuit will change. Let
new current be I´. Because potential difference still
i/2 R
remains same (due to low internal resistance), hence
R i
I´R1 = I (R1 + R2)
i/2 R I ( R1 + R2)
or I ′ =
R1
If P´ is the power consumed, then
E
(R1 + R2)(R1 + R2)
R 3R ′2R1 I 2
P′ I=
=
Req = +R= R1
2 2
When both the lamps were present then total power
2E 2
Power (Pf) = …(ii) consumed was given by:
3R
Dividing equation (i) by (ii) we get PS= P1 + P2 = I2 (R1 + R2), i.e., P´ > PS

Pi 3E 23R i.e., illumination gets increased when only one bulb


=
= 9:4 is used.
Pf 2 R 2 E 2
S-124 Science
120. (a) If a water pipe is given bend at some points, then it
definitely reduces the flow of water in the pipe but P.D. across 3 Ω = 1 × 3 = 1.5 volt
2
this is not true in case of an electric current flowing 4 2
in a conductor because electric current is established
in a conductor due to drift motion of electrons in
it along the line of the potential gradient. Hence,
illumination is not affected due to bending along the
length of supply wires.
3V
121. (b) P = VI = V2/R = I2R
127. (b) 128. (a)
V2 V 2 220 × 220
122. (d) P = ⇒R= = = 484 Ω
R P 100 129. (d) Given : R1 = 12Ω, R2 = 3.0Ω, R3 = 5.0Ω, R4 = 4.0Ω,
All four resistors are in series combination, so
V2 110 × 110
P= = = 25W Rs = R1 + R2 + R3 + R4
R 484
= 12W + 3.0W + 5.0W + 4.0W = 24W
123. (c) RS = R1 + R2 = R + R = 2R
130. (c) The current through all resistors in series is the same
1 1 1 1 1 2
R= R + R = + = V V 12V
P 1 2 R R R I= = = = 0.50 A
R Rs 24Ω
RP = R/2
131. (b) Potential drop across, 12Ω resistor
H1 V 2 R P RP R 1
V = IR = 12Ω (0.5A)
= 2 = R = 2 × 2R= 4= 1: 4 .
H2 RS V S
or V = 6V
124. (c) The bulb with the highest wattage glows with
132. (b) Here we have a variety of series-parallel
maximum brightness. Brightness of bulb B (100 W)
combinations.
is maximum.
We follow the general procedures outlined in the text.
Correct order of brightness will be,
The 10 Ω and the 5.0 Ω are in parallel
Bulb of 100 W > Bulb of 60 W > Bulb of 40 W.
(10Ω) (5.0Ω)
125. (c) Given, resistors, R1 = 2Ω and R2 = 4Ω R p1
= = 3.33 Ω
10Ω + 5.0Ω
Voltgage, V = 6 V A 3.0  3.3  4.0  B
2 4

I 3.0  I

6V 12 V
Equivalent Resistance,
R1 + R2 = 2 + 4 = 6Ω [Series combination]
= (i)
The circuit reduces to figure (a)
Now the 3.33 Ω and the 4.0 Ω are in series.
V 6
Current, I=
= = 1A. Rs1 = 3.33W + 4.0W = 7.33W
R 6
Heat dissipated in 4Ω Resistor The circuit reduces to figure (ii)
= I2Rt = 1 × 4 × 5 = 20 J
3.0  7.33  B
[∵ I = 1A, R = 4Ω, t = 5 sec.] A

3× 6
126. (b) Equivalent resistance of 3Ω and 6Ω = = 2Ω
3+ 6
3.0  I
as they are in parallel they have same p.d.
3 1
i= = 12 V
6 2
Electricity S-125
(ii)
The 7.33 Ω and the 3.0 Ω are in parallel. V 2
136. (a) Main current I=
= = 1A
(7.33Ω) (3.0Ω) R 2
R=
p2 = 2.13Ω
7.33Ω + 3.0Ω Current flowing through 0.5Ω resistor = 1A
A 3.0  2.13  B 137. (d) P.D. across the junctions :
V1 = IR1 = 1 × 1.5 = 1.5V

I Hence current I1, flowing through 6Ω resistor


V1 1.5
I =
1 = = 0.25 A
12 V 6 6
138. (c)

(iii)
The circuit reduces to figure (iii). 139. (c) Alloys are used in electrical heating device because
Finally, the 2.13 Ω and the 3.0 Ω are in series. they have high resistivity or resistance as compared
to pure metals and high melting point.
R= Rs2= 2.13Ω + 3.0Ω= 5.13 Ω= 5.1Ω
140. (c) It is clear that in a battery circuit, the point of lowest
A 5.13  B potential is the negative terminal of battery.
and current flows from higher potential to lower
potential.
I
141. (c) It is common error to say that V = Ri is a statement
of Ohm’s law. The essence of Ohm’s law is that the
12 V value of R is independent of the value of V. The
equation V = Ri is used for finding resistance of all
conducting devices, whether they obey Ohm’s law
(iv)
The circuit reduces to figure (iv). or not.

133. (a) From Ohm’s law, I= V= 12V = 2.33A= 2.3A 142. (d) (A) will read zero but (V) will read E
R 5.13Ω
143. (d) ρ is the characteristic of the material of resistors. It
134. (d) To find the current through the 4.0 ohm resistor, we
does not depend on the length and cross-sectional
need to expand the combinations.
area of resistors. But R depends on the length and
In figure (iii), the current through the 2.13 Ω and 3.0 the cross-sectional are of the resistor.
Ω is the same as the total current, 2.33 A. So, R1 may be greater than R2 even when r1 ≤ r2.
The voltage across the 2.13 Ω is then V2.13 = (2.13
144. (a) 145. (d)
Ω) (2.33 A) = 4.96 V.
In figure (ii), the voltage across the 7.33 Ω and 3.0 146. (c) Resistivity is a material property.
Ω is the same as that across the 2.13 Ω, 4.96V. 147. (c) r = r0(1 + aDT)
So, the current through the 7.33 Ω is 148. (b) Glow = Power (P) = I2R
4.96V
I 7.33
= = 0.677 A
7.33Ω \  dP =2  dI  =2 × 0.5 =1%
P  I 
In figure (i), the current through the 3.33 Ω and the 2
4.0 Ω is the same as the current through the 7.33 Ω 149. (a) Power loss = i²R =  P  R
Therefore, I4.0 = 0.68 A. V 
[P = Transmitted power]
135. (a) E.m.f. of the battery = 2V
Effective resistance of the parallel resistors is given V2 1
150. (b) P = ; R ∝ (same rated voltage)
by R1. R P
2
1 1 1 1+ 3 4 2
= =   ⇒  R1= 3= 1.5 Ω 151. (c) Here, P = E , so P ∝ R only when I is constant.
= + = R
R1 6 2 6 6 3 2
Here I increases as R is decreased. Hence the reason
Total resistance of the circuit, R = 1.5 + 0.5 = 2Ω
is wrong.
S-126 Science
152. (A) → (q); (B) → (p); (C) → (s); (D) → (r) 171. power, energy 172. ampere, hour

153. (A) → q; (B) → r; (C) → p; (D) → s 173. resistance, melting point 174. low, lead, tin, melts

154. electric current 155.


zero 175. series, live 176. separation

156. series 177. voltmeter 178. V × I × t

157. potential difference 158. resistivity 179. watt (W) 180. 1V

159. ampere 160. Resistance 181. 3,600,000 J 182. Tungsten

161. ohm (Ω) 183. voltage, current 184.


electric power

162. directly proportional, temperature 185. False 186. True 187. False

163. length, area of cross-section, resistivity 188. True 189. True 190. True

164. joule 165. Scalar 191. False 192. False 193. True

166. decreases 167. ohm-meter 194. True 195. False 196. False
197. True 198. True 199. True
168. electric current
200. True 201. False 202.
False
169. 1 Ω 170. inversely.
Magnetic Effects
12 Acids,
of Bases
Electric
Salts
Current
and

(a) clockwise (b) downwards


Multiple Choice Questions (MCQs) (c) anticlockwise (d) upwards

DIRECTIONS : This section contains multiple choice 6. The most suitable material for making the core of an
questions. Each question has four choices (a), (b), (c) and (d) electromagnet is:
out of which only one is correct. (a) soft iron (b) brass
(c) aluminium (d) steel
1. A magnet attracts:
(a) plastics (b) carbon 7. In an electric motor, the direction of current in the coil
changes once in each:
(c) aluminium (d) iron and steel
(a) two rotations (b) one rotation
2. A plotting compass is placed near the south pole of a bar (c) half rotation (d) one-fourth rotation
magnet. The pointer of plotting compass will:
(a) point away from the south pole 8. An electron beam enters a magnetic field at right angles
to it as shown in the Figure.
(b) point parallel to the south pole
(c) point towards the south pole
(d) point at right angles to the south pole Magnetic
field
3. Which of the following statements is incorrect regarding
magnetic field lines?
Electron beam
(a) The direction of magnetic field at a point is taken
to be the direction in which the north pole of a The direction of force acting on the electron beam will
magnetic compass needle points. be:
(b) Magnetic field lines are closed curves (a) to the left (b) to the right
(c) If magnetic field lines are parallel and equidistant, (c) into the page (d) out of the page
they represent zero field strength 9. The force experienced by a current-carrying conductor
(d) Relative strength of magnetic field is shown by the placed in a magnetic field is the largest when the angle
degree of closeness of the field lines between the conductor and the magnetic field is:
4. The magnetic field lines in the middle of the current (a) 45o (b) 60o
o
(c) 90 (d) 180o
carrying solenoid are
(a) circles 10. The force exerted on a current-carrying wire placed in a
(b) spirals magnetic field is zero when the angle between the wire
(c) parallel to the axis of the tube and the direction of magnetic field is:
(d) perpendicular to the axis of the tube (a) 45o (b) 60o
(c) 90o (d) 180o
5. The front face of a circular wire carrying current behaves
like a north pole, The direction of current in this face of 11. A circular loop placed in a plane perpendicular to the
the circular wire is: plane of paper carries a current when the key is ON.
S-128 Science
The current as seen from points A and B (in the plane of 17. The magnetic lines of force, inside a current carrying
paper and on the axis of the coil) is anti clockwise and solenoid, are
clockwise respectively. The magnetic field lines point (a) along the axis and are parallel to each other
from B to A. The N-pole of the resultant magnet is on the (b) perpendicular to the axis and equidistance from each
face close to other
(a) A (c) circular and they do not intersect each other
(b) B (d) circular at the ends but they are parallel to the axis
(c) A if the current is small, and B if the current is large inside the solenoid.
(d) B if the current is small and A if the current is 18. Which of the following determines the direction of
large magnetic field due to a current carrying conductor?
(a) Faraday’s laws of electromagnetic induction
A. B. Variable (b) Fleming’s left-hand rule
resistance
(c) Lenz’s rule
+ –
(d) Maxwell’s cork screw rule
19. Along the direction of current carrying wire, the value of
+ – magnetic field is
A (a) zero
K (b) infinity
12. A small magnet is placed perpendicular to a uniform (c) depends on the length of the wire
magnet field. The forces acting on the magnet will (d) uncertain
result in 20. The value of magnetic field due to a small element of
(a) Rotational motion current carrying conductor at a distance r and lying on the
(b) Translatory motion plane perpendicular to the element of conductor is
(c) No motion at all (a) zero
(d) Translational and rotational motion both (b) maximum
13. Which one of the following substances is the magnetic (c) inversely proportional to the current
substances? (d) none of the above
(a) Mercury (b) Iron 21. The value of intensity of magnetic field at a point due to
(c) Gold (d) Silver a current carrying conductor depends
(a) Only on the value of current
14. Magnetic lines do not intersect on one-another because
(b) Only on a small part of length of conductor
(a) they are at a distance
(c) On angle between the line joining the given point
(b) they are in the same direction
to the mid point of small length and the distance
(c) they are parallel to another between the small length and the given point
(d) at the point of intersection there will be two direction (d) On all of the above
of the magnetic force which is impossible
22. The direction of magnetic lines of forces close to a
15. By removing the inducing magnet, the induced magnetism straight conductor carrying current will be
is (a) along the length of the conductor
(a) Finished after some time (b) radially outward
(b) Finished just after (c) circular in a plane perpendicular to the conductor
(c) Not finished for a long time (d) helical
(d) Not changed
23. When an electron beam is moving in a magnetic field,
16. A current carrying wire in the neighbour hood produces then the work done is equal to the
(a) no field (a) charge of electron
(b) electric and magnetic fields (b) magnetic field
(c) electric field only (c) product of electronic charge and the magnetic field
(d) magnetic field only (d) zero
Magnetic Effects of Electric Current S-129
24. A current carrying loop lying in a magnetic field behaves 32. The current in a generator armature is AC because
like a. (a) the magnetic field reverses at intervals
(a) A magnetic dipole (b) the current in the field coils is AC
(b) magnetic pole (c) the rotation of the armature causes the field through
(c) magnetic material it to reverse
(d) non-magnetic material (d) the commutator feeds current into it in opposite
directions every half cycle
25. Two identical coaxial circular loops carry a current i each
circulating in the same direction. If the loops approach 33. The current in the armature of a motor is reversed every
each other, you will observe that half cycle due to the action of a(n)
(a) the current in each increases, (a) armature (b) field coil
(b) the current in each decreases, (c) brush (d) commutator.
(c) the current in each remains the same, 34. In an electric motor, the energy transformation is
(d) the current in one increases whereas that in the other (a) from electrical to chemical
decreases
(b) from chemical to light
26. An induced e.m.f. is produced when a magnet is (c) from mechanical to electrical
plunged into a coil. The strength of the induced e.m.f. is (d) from electrical to mechanical
independent of
35. The direction of induced current is obtained by
(a) the strength of the magnet
(a) Fleming’s left hand rule
(b) number of turns of coil
(b) Maxwell’s cork-screw rule
(c) the resistivity of the wire of the coil
(c) Ampere’s rule
(d) speed with which the magnet is moved
(d) Fleming’s right hand rule
27. The laws of electromagnetic induction have been used in
the construction of a 36. A metal sheet is placed in a variable magnetic field which
is increasing from zero to maximum. Induced current
(a) galvanometer (b) voltmeter
flows in the directions as shown in figure. The direction
(c) electric motor (d) generator
of magnetic field will be - N
28. Direction of induced e.m.f. is determined by - (a) normal to the paper, inwards
(a) Fleming’s left hand rule (b) normal to the paper,outwards W E
(b) Fleming’s right hand rule (c) from east to west
(c) Maxwell’s rule (d) from north to south
(d) Ampere’s rule of swimming S

29. The phenomenon of electromagnetic induction is – 37. A magnet NS is placed along the axis of a circular coil.
The magnet is moved away from the coil. The induced
(a) the process of charging a body.
current in the coil is:
(b) the process of generating magnetic field due to a
(a) Zero 
current passing through a coil.
(b) Clockwise N S
(c) producing induced current in a coil due to relative
motion between a magnet and the coil. (c) Anti-clockwise
(d) the process of rotating a coil of an electric motor. (d) None of these
Coil
30. The device used for producing electric current is called a 38. Four situations are given below-
(a) generator (b) galvanometer I. An infinitely long wire carrying current
(c) ammeter (d) motor II. A rectangular loop carrying current
III. A solenoid of finite length carrying current
31. In an electric motor, conversion takes place of
IV. A circular loop carrying current.
(a) Chemical energy into electrical energy
(b) Electrical energy into mechanical energy In which of the above cases will the magnetic field
produced be like that of a bar magnet?
(c) Electrical energy into light
(a) I (b) I and III
(d) Electrical energy into chemical energy
(c) Only III (d) Only IV
S-130 Science
39. The similar magnets of steel are ................. than the oriented with their planes parallel to each other. The
magnets of soft iron Y-axis passes vertically through loop A (dashed line).
(a) stronger (b) of equal strength There is a current IB in loop B as shown in the diagram.
(c) weaker (d) none of the above Possible actions which we might perform on loop A are
Y
40. A bar of soft iron is placed flat on the table. A bar magnet
is taken and its south pole is placed on one end of the bar
IB
of soft iron. The magnet is held almost vertically. The bar
is stroked from one end to the other with magnet. On the –X X
other end of the bar, magnet is lifted and again placed on
the first end and the bar is again stroked. The end of the
bar where the magnet is lifted will be
A
B
(a) south pole
(b) no pole
(c) south and north both type
(d) north pole (I) move A to the right along X-axis closer to B
41. When a bar magnet is broken into two pieces? (II) move A to the left along X-axis away from B
(a) we will have a single pole on each piece (III) as viewed from above, rotate A clockwise about
(b) each piece will have two like poles Y-axis
(c) each piece will have two unlike poles (IV) as viewed from above, rotate A anti-clockwise about
y-axis
(d) each piece will be lose magnetism
Which of the actions will induce a current in A only in
42. The permanent magnets are kept with soft iron pieces at
the direction shown?
ends as keepers
(a) Only (I) (b) Only (II)
(a) to magnetise the soft iron pieces
(c) Only (I) and (IV) (d) Only (II) and (III)
(b) to increase the strength of the magnets
(c) to avoid self demagnetisation 47. An electron move with velocity v in a uniform magnetic
(d) for physical safety of the magnets field B. The magnetic force experienced by the electron is
(a) Always zero
43. Whenever the magnetic flux linked with a coil changes,
(b) Never zero
an induced e.m.f. is produced in the circuit. The e.m.f.
lasts (c) Zero if v is perpendicular to B
(a) for a short time (d) Zero if v is parallel to B
(b) for a long time
(c) for ever
(d) so long as the change in flux takes place
DIRECTIONS : Study the given case/passage and answer the
44. A magnet is moved towards a coil (i) quickly (ii) slowly, following questions.
then the induced e.m.f. is
Case/Passage - 1
(a) larger in case (i)
(b) smaller in case (i) A solenoid is a long helical coil of wire through which a current
(c) equal in both the cases is run in order to create a magnetic field. The magnetic field of
(d) larger or smaller depending upon the radius of the the solenoid is the superposition of the fields due to the current
coil through each coil. It is nearly uniform inside the solenoid and
close to zero outside and is similar to the field of a bar magnet
45. The laws of electromagnetic induction have been used in
having a north pole at one end and a south pole at the other
the construction of a
depending upon the direction of current flow. The magnetic
(a) galvanometer (b) voltmeter
field produced in the solenoid is dependent on a few factors
(c) electric motor (d) generator
such as, the current in the coil, number of turns per unit length
46. The diagram below shows two circular loops of wire (A etc. The following graph is obtained by a researcher while
and B) centred on and perpendicular to the X-axis and doing an experiment to see the variation of the magnetic field
Magnetic Effects of Electric Current S-131
with respect to the current in the solenoid. The unit of magnetic (a) Only IV (b) I and III and IV
field as given in the graph attached is in milli-Tesla (mT) and (c) I and II (d) Only II
the current is given in Ampere.
52. From the graph deduce which of the following statements
is correct.
(a) For a current of 0.8A the magnetic field is 13 mT
(b) For larger currents, the magnetic field increases non-
linearly.
(c) For a current of 0.8A the magnetic field is 1.3 mT
(d) There is not enough information to find the magnetic
field corresponding to 0.8A current.
Case/Passage - 2
For a conductor of length L carrying a current of I in a field B
  
the force experienced by the conductor F = I L × B

48. What type of energy conversion is observed in a linear If the current-carrying conductor in the form of a loop of any

solenoid? arbitrary shape is placed in a uniform field, then, F = 0 i.e.,
(a) Mechanical to Magnetic the net magnetic force on a current loop in a uniform magnetic
(b) Electrical to Magnetic field is always zero. Here it must be kept in mind that in this
(c) Electrical to Mechanical situation different parts of the loop may experience elemental
(d) Magnetic to Mechanical force due to which the loop may be under tension or may
experience a torque.
49. What will happen if a soft iron bar is placed inside the
solenoid? Direction of force can be determined by fleming’s left hand
(a) The bar will be electrocuted resulting in short- rule, right hand palm rule or screw rule.
circuit. 53. The direction of induced current is obtained by
(b) The bar will be magnetised as long as there is current (a) Fleming’s left hand rule
in the circuit. (b) Maxwell’s cork-screw rule
(c) The bar will be magnetised permanently. (c) Ampere’s rule
(d) The bar will not be affected by any means. (d) Fleming’s right hand rule
50. The magnetic field lines produced inside the solenoid are 54. An electron moving with uniform velocity in x-direction
similar to that of … enters a region of uniform magnetic field along
(a) a bar magnet 10 y-direction. Which of the following physical quantity(ies)
(b) a straight current carrying conductor is (are) non-zero and remain constant? y
(c) a circular current carrying loop I. Velocity of the electron
II. Magnitude of the
(d) electromagnet of any shape
momentum of the electron.
51. After analysing the graph a student writes the following III. Force on the electron. x

statements. IV. The kinetic energy of electron. e
I. The magnetic field produced by the solenoid is (a) Only I and II. (b) Only III and IV.
inversely proportional to the current. (c) All four (d) Only II and IV.
II. The magnetic field produced by the solenoid is
55. Which of the following can produce a magnetic field?
directly proportional to the current.
(a) Electric charges at rest
III. The magnetic field produced by the solenoid is
directly proportional to square of the current. (b) Electric charges in motion
IV. The magnetic field produced by the solenoid is (c) Only by permanent magnets
independent of the current. (d) Electric charges whether at rest or in motion

Choose from the following which of the following would 56. A wire is lying horizontally in the north-south direction
be the correct statement(s). and there is a horizontal magnetic field pointing towards
S-132 Science
the east. Some positive charges in the wire move north
and an equal number of negative charges move south. Assertion & Reason
The direction of force on the wire will be
DIRECTIONS : Each of these questions contains an assertion
followed by reason. Read them carefully and answer the question
N on the basis of following options. You have to select the one that
best describes the two statements.

W E (a) If both Assertion and Reason are correct and Reason is


the correct explanation of Assertion.
(b) If both Assertion and Reason are correct, but Reason is
S not the correct explanation of Assertion.

(c) If Assertion is correct but Reason is incorrect.
(a) east
(b) down, into the page (d) If Assertion is incorrect but Reason is correct.
(c) up, out of the page 61. Assertion : Magnetic field interacts with a moving charge
(d) west and not with a stationary charge.
Reason : A moving charge produces a magnetic field.
57. Four situations are given below-
I. An infinitely long wire carrying current 62. Assertion : No net force acts on a rectangular coil
II. A rectangular loop carrying current carrying a steady current when suspended freely in a
uniform magnetic field.
III. A solenoid of finite length carrying current
IV. A circular loop carrying current. Reason : Force on coil in magnetic field is always non-
zero.
In which of the above cases will the magnetic field
produced be like that of a bar magnet? 63. Assertion : Force experienced by moving charge
will be maximum if direction of velocity of charge is
(a) I (b) I and III
perpendicular to applied magnetic field.
(c) Only III (d) Only IV
Reason : Force on moving charge is independent of
Case/Passage - 3
direction of applied magnetic field.
The strength of the magnetic field produced by a current-
64. Assertion : There is no change in the energy of a charged
carrying circular coil (or circular wire) depends on (i) Current
particle moving in a magnetic field although a magnetic
flowing through the coil. (ii) Radius of the circular coil. (iii)
force is acting on it.
Number of turns of wire in the circular coil.
Reason : Work done by centripetal force is always zero.
58. A long horizontal power line is carrying a current of 100
65. Assertion : In a conductor, free electrons keep on moving
A in the east-west direction. The direction of magnetic
but no magnetic force acts on a conductor in a magnetic
field at a point 1.0 m below it is
field.
(a) south to north
Reason: Force on free electrons due to magnetic field
(b) north to south
always acts perpendicular to its direction of motion.
(c) east to west
(d) west to east 66. Assertion : A proton moves horizontally towards a
vertical long conductor having an upward electric current.
59. What type of curve we get, between magnetic field and
It will deflect vertically downward.
distance along the axis of a current carrying circular coil?
Reason : Seeing the proton and the conductor from the
(a) Straight (b) Circular
side of the proton, the magnetic field at the site of the
(c) Parabolic (d) None of these   
proton will be towards right. Hence the force =
F qv × B
60. If a current carrying straight conductor is placed is east-west will deflect the proton vertically downward.
direction, then the direction of the force experienced by the
67. Assertion : A spark occurs between the poles of a switch
conductor due to earth’s magnetic field is:
when the switch is opened.
(a) downward (b) upward
Reason : Current flowing in the conductor produces
(c) east-west (d) west east
magnetic field.
Magnetic Effects of Electric Current S-133
75. The magnetic lines of force are the lines drawn in a
Match the Following magnetic field along which a ............. pole would move.

DIRECTIONS : Each question contains statements given in 76. An electric current can be used for making temporary
two columns which have to be matched. Statements (A, B, C, magnets known as .................
D) in column I have to be matched with statements (p, q, r, s) in 77. The unit of magnetic field is ......................
column II.
78. The N-pole of a compass points to the .............. pole of a
68. Column I Column II permanent magnet.
(A) An electric motor (p) to a battery 79. The force that a magnetic field exerts on a current is
works on always perpendicular to the ............. and to the ..............
(B) An electric motor is also (q) direct current
80. In a magnetic field pointing away from you, an electron
(C) A commutator is used to (r) reverse the direction
traveling to the right will experience a force in the
of flow of current. .............. direction.
(D) Commutator rings are (s) known as DC
81. Magnetic fields are produced by .................
connected MOTOR
82. You are looking into a solenoid, at its S-pole, along its
69. Equal currents i flow in two wires along x and y axis as
axis. From your view point, the direction of the current in
shown. Match the following :
the solenoid is ................
83. Crowding the wires of a solenoid more closely together
i
will .............. the strength of the field inside it.

i 84. Magnetic field lines emerge from the ............. pole of a


solenoid or a permanent magnet.
85. You are looking down the axis of a solenoid, and the
Column I Column II current from your position is clockwise. The end of the
(A) Magnetic field in (p) inwards solenoid facing you is a .............. pole.
first quadrant 86. A generator converts mechanical energy into .........
(B) Magnetic field in (q) outwards energy. It works on the basis of ......................
second quadrant
87. In our houses we receive AC electric power of ......... with
(C) Magnetic field in (r) may be inwards or a frequency of ..............
third quadrant outwards
88. The frequency for A.C. (alternating current) in USA is
(D) Magnetic field in
................
fourth quadrant
89. The armature in a motor rotates within a(n) ............. field.

Fill in the Blanks 90. To produce DC, the output of a generator must be fed
through a (n) ...............
DIRECTIONS : Complete the following statements with an 91. In any generator, the current in the armature is of the
appropriate word / term to be filled in the blank space(s). .......... type.

70. A compass needle is a .................... magnet. 92. In an AC generator, maximum number of lines of force
pass through the coil when the angle between the plane of
71. Field lines are used to represent a ................
coil and lines of force is .....................
72. Field lines are shown closer together where the magnetic
field is .................... True / False
73. A metallic wire carrying an electric current has associated
with it a ........... field. DIRECTIONS : Read the following statements and write your
answer as true or false.
74. The field lines about the wire consist of a series of
concentric circles whose direction is given by the ............ 93. A magnetic field exists in the region surrounding a
rule. magnet, in which the force of the magnet can be detected.
S-134 Science
94. The pattern of the magnetic field around a conductor due 98. Two magnetic lines of force never intersect each other.
to an electric current flowing through it depends on the
99. The field lines inside the infinite solenoid are in the form
shape of the conductor.
of parallel straight lines.
95. A current-carrying conductor when placed in a magnetic
100. An electric generator works on the principle of
field always experiences a force.
electromagnetic induction.
96. The direction of force on a current carrying conductor
101. In a DC electric motor a pair of split rings is used as
placed in a magnetic field can be reversed by reversing
commutator.
the direction of current flowing in the conductor.
102. The magnitude of induced current can be increased by
97. The direction of force on a current carrying conductor
decreasing the speed of rotation of coil.
placed in a magnetic field cannot be reversed by reversing
the direction of magnetic field.
Magnetic Effects of Electric Current S-135

ANSWER KEY & SOLUTIONS

1. (d) Magnet attracts iron and steel Case-I, If q = 0°


2. (c) \ F = 0
3. (c) If magnetic field lines are parallel and equidistant Case-II, If q = 90°
then it represents uniform magnetic field.
\ F = q v B ≠ 0
4. (c) 5. (c)
48. (c) Electrical to Mechanical
6. (a) Soft iron is the most suitable material for making the
49. (b) The bar will be magnetised as long as there is current
core of an electromagnet
in the circuit.
7. (c) The direction of current changes in each half rotation
50. (a) A bar magnet
in an electric motor.
51. (d) Only II
8. (c) 9. (c) 10. (d) 11. (a) 12. (a)
52. (a) For a current of 0.8 A the magnetic field is 13 mT
13. (b) 14. (d) 15. (b) 16. (d) 17. (a)
53. (d) Fleming’s right hand rule.
18. (d) 19. (a) 20. (b) 21. (d) 22. (c)
54. (d) Velocity and force change due to change in direction
23. (d) 24. (a) 25. (b) 26. (c) 27. (d)
but magnitude of PE and KE of electron remain
28. (b) 29. (c) 30. (a) 31. (b) 32. (c) constant speed is constant.
33. (d) 34. (d) 35. (d) 36. (b) 55. (b) Magnetic field (B) is produced by moving charge.
37. (b) The induced current in coil is in clockwise direction 56. (b) According to Fleming’s left hand rule the direction
when N pole of powerful magnet is moved to right. of force on the wire will be down into the page.
38. (c) A long coil of finite length of wire carrying current 57. (c) A long coil of finite length of wire carrying current
consisting of closely packed loops is called solenoid consisting of closely packed loops is called solenoid
whose magnetic field resembles that of a bar magnet. whose magnetic field resembles that of a bar magnet.
39. (c) 40. (d) 41. (c) 42. (c) 43. (d) 58. (b) 59. (d) 60. (a)

44. (a) 45. (d) 61. (a) A moving charge experiences a force in magnetic
field. It is because of interaction of two magnetic
46. (a) In loop shown current is anti-clockwise. fields, one which is produced due to motion charge
A and other in which charge is moving.
IA IB
B 62. (c) Force acting on each pair of the opposite sides of the
coil are equal.
63. (c) Force on moving charge will be maximum if
direction of velocity of charge is perpendicular to
So to induce a anti-clockwise current in A, flux direction of magnetic field
going into A must be increased and by bringing A
closer to B, we get a anti-clockwise current in A. 64. (a) Magnetic force is always perpendicular to the
This is in accordance with Lenz’s law. direction of motion of charged particle, i.e., work
done on the charge particle moving on a circular
47. (d) We know, Lorentz Force path in magnetic field is zero.
F = q v B sin q 65. (c) In a conductor, the average velocity of electrons is
where q is angle between the direction s of v and B. zero. Hence no current flows through the conductor.
Hence, no force acts on this conductor.
S-136 Science
66. (a) 67. (b) 82. clockwise 83. increase
68. (A) → q, (B) → s, (C) → r, (D) → p 84. North 85. south
69. (A) → r, (B) → q, (C) → r, (D) → p 86. electrical, electromagnetic induction.
70. small 71. magnetic field 87. 220 V, 50 Hz. 88.
60 Hz
72. greater. 73. magnetic 89. magnetic 90. commutator
74. right-hand 75. north magnetic 91. A.C 92. 90 degree
76. electromagnets 77. tesla 93. True 94. True 95. False 96. True
78. South 79. field, current 97. False 98. True 99. True 100. True
80. downward 81. currents 101. True 102. False
13 Acids,
Our Bases and
Environment
Salts
(c) Because respiration ceases at such low temperature.
Multiple Choice Questions (MCQs) (d) Because there is no transpiration.

DIRECTIONS : This section contains multiple choice 7. In order to maintain proper ecological balance
questions. Each question has four choices (a), (b), (c) and (d) (a) the existing forests would be cleared and new ones
out of which only one is correct. should be planted.
(b) some quick growing annuals should be planted if a
1. An example of a producer in the aquatic food web would tree must be cut for other uses.
be:
(c) tree must be cut whenever necessary because the
(a) Duckweed (b) Ducks underground part performs the useful purpose.
(c) Fish (d) Insects (d) a tree should be planted in place of one to be cut.
2. Which one is recyclable waste? 8. It is said, the Tajmahal may be destroyed due to
(a) Paper (a) Flood in Yamuna river
(b) Torn clothes (b) Decomposition of marble as a result of high
(c) Metallic and plastic discards temperature
(d) All the above (c) Air pollutants released from oil refinery of Mathura
3. Which of the following does not form part of particulate (d) All the above
matter?
9. In an ecosystem, the function of the producers is to
(a) Dust (b) Fly ash
(a) convert organic compounds into inorganic compounds.
(c) Aerosols (d) Nitric oxide
(b) trap solar energy and convert it into chemical energy.
4. Which of the following are environment-friendly practices? (c) utilize chemical energy.
(a) Carrying cloth-bags to put purchases in while shopping (d) release energy.
(b) Switching off unnecessary lights and fans
10. Free services provided to humans by ecosystems include
(c) Walking to school instead of getting your mother to
drop you on her scooter (a) control of atmospheric carbon dioxide concentration.
(d) All of the above (b) prevention of soil erosion.
5. Habitat together with functions of species constitute: (c) filtering of pollutants from water and air.
(a) Trophic level (b) Boundary (d) all of the above
(c) Topography (d) Niche 11. Carcinogenic chemicals produced during recycling of
6. Plants are killed in winter by frost: plastics and polythene is/are

(a) Because of dessication and mechanical damage to (a) formaldehyde


the tissue. (b) polycyclic aromatic compounds
(b) Because no photosynthesis take place at such a low (c) vinyl chloride
temperature. (d) dioxins and furans
S-138 Science
12. Organisms of a higher trophic level which feed on several 21. Pyramids of energy are
types of organisms belonging to a lower trophic level (a) always upright (b) always inverted
constitute the
(c) mostly upright (d) mostly inverted
(a) food web (b) ecological pyramid
22. Individuals of any species at a place form
(c) ecosystem (d) food chain
(a) biotic community (b) ecosystem
13. A decrease in the grass population will most immediately
(c) population (d) biome
decrease the available energy for the
(a) mouse (b) snake 23. City garbage can be used to produce
(c) hawk (d) frog (a) sewage sludge (b) useful articles
(c) biogas and manure (d) all of the above.
14. Why do scientists think that human-induced global
warming will be more harmful to plants and animals than 24. Acid rain is the downpour of
were past, natural climate fluctuations ? (a) carbon dioxide in rain (b) dust in rain
(a) Because temperatures will change faster (c) sulphur dioxide in rain (d) oxygen in rain
(b) Because the temperature changes will be larger
25. Select the most dangerous pollutant
(c) Because species now are less adaptable than species
in the past (a) CO (b) SO2
(d) Because ecosystems are now more complicated than (c) NO2 (d) CO2
they used to be 26. Ozone layer is essential because it absorbs most of the
15. Each step in a food chain is called a (a) infrared radiations (b) heat
(a) trophic level. (b) consumer level. (c) solar radiation (d) ultraviolet-radiation
(c) food web. (d) producer.
27. The biotic and abiotic components interacting with each
16. As a biologist, if you become very interested in the study other in a pond form
of the interaction of organisms with each other and the
(a) a community (b) a population
environment your subspeciality would be
(c) an ecosystem (d) a biome
(a) Zoology (b) Ecology
(c) Botany (d) Herpetology 28. Environment consists of

17. The last chain of food is (a) land, air, water


(a) producers (b) decomposers (b) light, temperature and rainfall
(c) parasites (d) none of these (c) plants, animals and microbes

18. Trophic levels are formed by – (d) All the above.

(a) only plants 29. Carnivores represent


(b) only animals (a) primary consumers
(c) only carnivores (b) secondary and tertiary consumers
(d) organisms linked in food chain (c) reducers
19. The part of earth comprising water is called an (d) zooplankton.
(a) atmosphere (b) hydrosphere 30. World environment day is celebrated on
(c) lithosphere (d) none of the above (a) 15th March (b) 15th April
20. The maximum energy is stored at following tropical level (c) 4th May (d) 5th June
in any ecosystem
31. Flow of energy in an ecosystem is always
(a) Producers (b) Herbivores
(a) unidirectional (b) bidirectional
(c) Carnivores (d) Top carnivores
(c) multi-directional (d) no specific direction
Our Environment S-139
32. Which of the following is a biodegradable waste? 42. Rag pickers remove
(a) Radioactive wastes (b) Aluminium cans (a) plastic, polythene, paper and metal wastes
(c) DDT (d) Cattle dung (b) rags, cardboard, glass articles

33. For corrosion of metals, there should be (c) both (a) and (b)

(a) Exposed surface of metal (d) food articles.

(b) Moisture 43. As energy is passed from one trophic level to another, the
(c) Air amount of usable energy
(d) All these (a) increases
(b) decreases
34. Sun gives radiations in the form of
(c) remains the same
(a) Infra-red radiation (b) Visible light
(d) energy is not passed from one trophic level to another
(c) Ultra-violet (d) All these
44. In the biosphere, which of the following is the ultimate
35. In an ecosystem green plants are known as source of energy?
(a) primary consumers (b) secondary consumers (a) Carbon (b) Water
(c) producers (d) tertiary consumers (c) Sunlight (d) Nitrogen
36. Carbon monoxide is a pollutant because 45. Sulphur dioxide affects
(a) It reacts with O2 (a) Haemoglobin of blood (b) Arteries
(b) It inhibits glycolysis (c) Alveoli of lungs (d) Nerves
(c) Reacts with haemoglobin
46. Pyramid of energy in a forest ecosystem is
(d) Makes nervous system inactive
(a) Always inverted
37. The presence of which of the following pollutants in the (b) Always upright
atmosphere has caused damage to Taj Mahal? (c) Both upright and inverted depending on ecosystem
(a) CO2 (d) First upright then inverted
(b) SO2
47. CO2 absorbs some of the ............. that radiates from the
(c) Pb particles
surface of earth to space
(d) Radioactive disintegrations
(a) ozone (b) heat
38. Which of the following does not affect ozone layer? (c) ultraviolet light (d) smog
(a) Cl2 (b) CH3Cl 48. Human-caused changes to the nitrogen cycle are expected
(c) NO (d) CFCl3 to result in
39. Which one is present in maximum number in an (a) an increase in acid rain.
ecosystem? (b) an increase in the loss of species from ecosystems.
(a) Herbivores (b) Carnivores (c) higher concentrations of a greenhouse gas.
(c) Producers (d) Omnivores. (d) all of the above

40. The percentage of solar radiation absorbed by all the 49. The biological process by which carbon is returned to its
green plants for the process of photosynthesis is about reservoir is
(a) 1% (b) 5% (a) photosynthesis (b) denitrification
(c) 8% (d) 10% (c) carbon fixation (d) cellular respiration

41. Decrease in number of trees may cause 50. Which of the following constitute a food-chain?
(a) increase in rainfall (a) grass, wheat and mango
(b) decrease in rainfall (b) grass, goat and human
(c) increase in temperature (c) goat, cow and elephant
(d) conservation of nutrients in soil (d) grass, fish and goat
S-140 Science
51. As a black widow spider consumes her mate, what is the 59. Which two of the following statements regarding food
lowest trophic level she could be occupying chains are correct?
(a) third (b) first (i) Removal of 80% tigers resulted in increased growth
of vegetation.
(c) second (d) fourth
(ii) Removal of most carnivores resulted in increased
52. Among the most dangerous non-biodegradable waste is population of deer.
(a) cow-dung (b) plastic articles (iii) Length of food chain is limited to 3 – 4 trophic levels
(c) garbage (d) radioactive waste due to energy loss.
(iv) Length of food chain may vary from 2 – 3 trophic
53. In an ecosystem, the 10% of energy is transferred from
levels.
one trophic level to the next in the form of
(a) (i) and (iv) (b) (i) and (ii)
(a) heat energy (b) light energy
(c) (ii) and (iii) (d) (iii) and (iv)
(c) chemical energy (d) mechanical energy
60. Match the terms given in column-I with their definition
54. Organisation involved in formulating programmes for given in column-II and choose the correct option.
protecting environment is:
Column - I Column - II
(a) WHO (b) UNDP A. Food chain I. An organism that eats meat.
(c) UNEP (d) UNICEF B. Food web II. An organism that eats plants.
55. In every food chain green plants are C. Heterotrophs III. An organism that makes its
food from light or chemical
(a) decomposers (b) producers
energy.
(c) consumers (d) None of the above.
D. Autotrophs IV. An organism that gets its energy
56. The decomposers in an ecosystem by eating other organisms.
E. Carnivore V. The sequence of organisms
(a) convert inorganic material, to simpler forms.
of who eats whom in a
(b) convert organic material to inorganic forms. biological community.
(c) convert inorganic materials into organic compounds. F. Herbivore VI. The network of all the
interrelated food chains in a
(d) do not break-down organic compounds.
biological community.
57. If a grasshopper is eaten by a frog, then the energy transfer (a) A – V; B – VI; C – IV; D – III; E – I; F – II
will be from (b) A – VI; B – IV; C – III; D – I; E – II; F – V
(c) A – III; B – I; C – II; D – V; E – VI; F – IV
(a) producer to decomposer
(d) A – II, B – V; C – VI; D – IV; E – III; F – I
(b) producer to primary consumer
61. Assertion: Pond ecosystem is upright in the pyramid of
(c) primary consumer to secondary consumer number.
(d) secondary consumer to primary consumer
Reason: Phytoplanktons are maximum and secondary
58. The diagram below shows a food pyramid. consumers are lesser in number.
(a) Statement (A) and (B) both are correct.
(b) Statement (A) is correct but (B) is incorrect.
(c) Statement (A) and (B) both are incorrect.
(d) Statement (A) is incorrect but (B) is correct.
62. Which of the following ecological pyramid is never
inverted?
Which level of the food pyramid contains consumers (a) Pyramid of number in forest ecosystem
with the least biomass? (b) Pyramid of biomass in pond ecosystem
(a) snakes (b) frogs (c) Pyramid of energy in parasitic food chain
(c) crickets (d) green plants (d) Pyramid of biomass in parasitic food chain
Our Environment S-141
63. Match column - I with column - II and select the correct 69. What is the main reason for increase in temperature in a
answer using the codes given below. glass house?
Column - I Column - II (a) Sunlight is completely absorbed by plants in the
A. Phosphorus I. Atmosphere glass house
B. Carbon II. Producers (b) Radiation fails to escape from the glass house completely
C. Goat III. Rock (c) Plant do not utilize sunlight in a glass house
D. Grasses IV. T2 (d) Plants produce heat inside the glass house
(a) A – III; B – II; C – IV; D – I 70. Read the following statements carefully.
(b) A – III; B – I; C – IV; D – II (I) Energy transfer in the biotic world always proceeds
(c) A – I; B – III; C – II; D – IV from the autotrophs.
(d) A – II; B – III; C – IV; D – I
(II) Energy flow is unidirectional.
64. Which of the following pair is incorrectly matched ? (III) Energy availability is maximum at the tertiary level.
(a) Autotrophs – Fungi (IV) There is loss of energy from one trophic level to the
(b) Primary consumers – Zooplankton other.
(c) Secondary consumers – Fishes Select the relevant statements for the forest ecosystem
(d) Decomposers – Fungi (a) I, II and IV (b) I, II and III
65. Abundance of coliform bacteria in a water body is (c) I, III and IV (d) II, III and IV
indicative of pollution from
71. In a highly pesticide polluted pond. Which of the
(a) petroleum refinery (b) metal smelter following aquatic organisms will have the maximum
(c) fertilizer factory (d) domestic sewage amount of pesticide per gram of body mass?

66. Prolonged exposure to the fumes released by incomplete (a) Lotus (b) Fishes
combustion of coal may cause death of a human because Spirogyra
(c) (d) Zooplanktons
of
72. DDT is non-biodegradable chemical when it enters food
(a) inhalation of unburnt carbon particles. chain it gets accumulated in each trophic level. This
(b) continuous exposure to high temperature. phenomenon is called as -
(c) increased level of carbon monoxide. (a) Eutrophication
(d) increased level of carbon dioxide. (b) Chemical amplification

67. Which among grass, goat, tiger and vulture in a food chain, (c) Biomagnification
will have the maximum concentration of harmful chemicals (d) Chemical magnification
in its body due to contamination of pesticides in the soil?
73. The following diagram shows a simple version of energy
(a) Grass since it grows in the contaminated soil flow through food web.
(b) Goat since it eats the grass Sunlight

(c) Tiger since it feeds on the goat which feeds on the Plants
grass
(d) Vulture since it eats the tiger, which in turn eats the Animals

goat, which eats the grass Decomposers

68. Which of the following is an result of biological ?


magnification?
(a) Top level predators may be harmed by toxic What happens to energy having the decomposers?
chemicals in environment. (a) It is used by the decomposers itself.
(b) Increase in carbon dioxide (b) It is reflected from the surface of earth.
(c) The green-house effect will be most significance at (c) It is lost as heat
the poles (d) It is used in natural biocomposting
(d) Energy is lost at each trophic level of a food chain
S-142 Science
74. Which of the following groups contain only biodegradable (a) T4 (b) T2
items ?
(c) T1 (d) T3
(a) Grass, flowers and leather
(b) Grass, wood and plastic 81. What will happen if deer is missing in the food chain
(c) Fruit-peels, cake and lime-juice given below?
(d) Cake, wood and grass Grass → Deer → Tiger
(a) The population of tiger increases.
75. Which of the following constitute a food-chain?
(b) The population of grass decreases.
(a) Grass, wheat and mango
(c) Tiger will start eating grass.
(b) Grass, goat and human
(d) The population of tiger decreases and the population
(c) Goat, cow and elephant
of grass increases.
(d) Grass, fish and goat

76. Which of the following are environment-friendly practices?


(a) Carrying cloth-bags to put purchases in while shopping.
(b) Switching off unnecessary lights and fans. DIRECTIONS : Study the given case/passage and answer the
following questions.
(c) Walking to school instead of getting your mother to
drop you on her scooter. Case/Passage - 1
(d) All of the above. Food chains are very important for the survival of most species.
When only one element is removed from the food chain it can
77. Organisms which synthesise carbohydrates from result in extinction of a species in some cases. The foundation
inorganic compounds using radiant energy are called of the food chain consists of primary producers.
(a) decomposers (b) producers
Primary producers, or autotrophs, can use either solar energy
(c) herbivores (d) carnivores
or chemical energy to create complex organic compounds,
78. Which of the statement is incorrect? whereas species at higher trophic levels cannot and so must
consume producers or other life that itself consumes producers.
(a) All green plants and blue-green algae are producers.
Because the sun’s light is necessary for photosynthesis, most
(b) Green plants get their food from organic compounds.
life could not exist if the sun disappeared. Even so, it has
(c) Producers prepare their own food from inorganic recently been discovered that there are some forms of life,
compounds. chemotrophs, that appear to gain all their metabolic energy
(d) Plants convert solar energy into chemical energy. from chemosythesis driven by hydrothermal vents, thus
showing that some life may not require solar energy to thrive.
79. Which of the following limits the number of trophic
levels in a food chain? [From CBSE Question Bank-2021]
(a) Decrease in energy at higher trophic levels.
(b) Deficient food supply. Food Webs

(c) Polluted air.


Secondary
(d) Water. carnivores
80. In the given diagram, the various trophic levels are shown
in a pyramid. At which trophic level is maximum energy
available?
Primary
Carnivores

Herbivores
Our Environment S-143
82. If 10,000 J solar energy falls on green plants in a 87. The mussel can be described as
terrestrial ecosystem, what percentage of solar energy (a) Producer (b) Primary consumer
will be converted into food energy? (c) Secondary consumer (d) decomposer
(a) 10,000 J 88. Which trophic level is incorrectly defined?
(b) 100 J (a) Carnivores – secondary or tertiary consumers
(c) 1000 J (b) Decomposers – microbial heterotrophs
(c) Herbivores – primary consumers
(d) It will depend on the type of the terrestrial plant.
(d) Omnivores – molds, yeast and mushrooms
83. If Ravi is consuming curd/yogurt for lunch, which 89. The given figure best represents:
trophic level in a food chain he should be considered as
occupying ?
(a) First trophic level (b) Second trophic level
(c) Third trophic level (d) Fourth trophic level
84. The decomposers are not included in the food chain.The
correct reason for the same is because decomposers:
(a) Act at every trophic level of the food chain
(b) Do not breakdown organic compounds
(c) Convert organic material to inorganic forms (a) Grassland food chain (b) Parasitic food chain
(d) Release enzymes outside their body to convert
(c) Forest food chain (d) Aquatic food chain
organic material to inorganic forms
85. Matter and energy are two fundamental inputs of an 90. Why do all food chains start with plants?
ecosystem. Movement of (a) Because plants are easily grown
(a) Energy is bidirectional and matter is repeatedly (b) Because plants are nutritious
circulating.
(c) Because plants can produce its own energy
(b) Energy is repeatedly circulation and matter is
unidirectional. (d) Because plants do not require energy
(c) Energy is unidirectional and matter is repeatedly 91. In the food web, what two organisms are competing for
circulating. food?
(d) Energy is multidirectional and matter is bidirectional. G E
86. Which of the following limits the number of trophic F D
levels in a food chain? C B
(a) Decrease in energy at higher trophic levels
(b) Less availability of food A
(c) Polluted air (a) A and B (c) A and C
(d) Water (b) D and F (d) B and D

Case/Passage - 2 92. Consider the following statements concerning food


The diagram below shows a food web from the sea shore chains:
dogfish (i) Removal of 80% tigers from an area resulted in
greatly increased growth of vegetation
(ii) Removal of most of the carnivores resulted in an
crab dogwhelk increased population of herbivores.
(iii) The length of the food chains is generally limited to
3 – 4 trophic levels due to energy loss
(iv) The length of the food chains may vary from 2 to 8
mussel barnacle peri
trophic levels
Which two of the above statements are correct?
microscopic animals (a) (i), (iv) (b) (i), (ii)
[From CBSE Question Bank-2021] (c) (ii), (iii) (d) (iii), (iv)
S-144 Science
93. Which of the following group of organisms are not 96. The given figure best represents:
included in ecological food chain?
(a) Carnivores (b) Saprophytes
(c) Herbivores (d) Predators
Case/Passage - 3
Biosphere is a global ecosystem composed of living organisms
and abiotic factors from which they derive energy and nutrients.
And ecosystem is defined as structural and functional unit of
the biosphere comprising of living and non-living environment (a) Grassland food chain (b) Parasitic food chain
that interact by means of food chains and chemical cycles (c) Forest food chain (d) Aquatic food chain
resulting in energy flow, biotic diversity and material cycling
to form a stable, self-supporting system
[From CBSE Question Bank-2021] Assertion & Reason

DIRECTIONS : Each of these questions contains an assertion


followed by reason. Read them carefully and answer the question
on the basis of following options. You have to select the one that
best describes the two statements.
(a) If both Assertion and Reason are correct and Reason is
the correct explanation of Assertion.
(b) If both Assertion and Reason are correct, but Reason is
not the correct explanation of Assertion.
94. Which trophic level is incorrectly defined? (c) If Assertion is correct but Reason is incorrect.
(a) Carnivores – secondary or tertiary consumers (d) If Assertion is incorrect but Reason is correct.
(b) Decomposers – microbial heterotrophs 97. Assertion: In an ecosystem, the function of producers is
(c) Herbivores – primary consumers to convert organic compounds into inorganic compounds.
(d) Omnivores – molds, yeast and mushrooms Reason: Green plants, the producers, transduce solar
energy.
The diagram below shows a food web from the sea shore 98. Assertion: Ecology is study of relationship between
dogfish living organisms and their environment.
Reason: The biotic community and non-living environment
of an area function together to form an ecosystem.
crab dogwhelk 99. Assertion: Animals adopt different strategies to survive
in hostile environment.
Reason: Praying mantis is green in colour which merges
with plant foliage.
mussel barnacle peri
100. Assertion: Abiotic component of an ecosystem involves
cycling of material and flow of energy.
Reason: This is essential to keep biotic factors alive.
microscopic animals 101. Assertion: The crown fires are most destructive as they
95. The mussel can be described as burn the tree top.
(a) Producer Reason: Due to crown fire, the temperature of that area
(b) Primary consumer may rise upto 700°C.
(c) Secondary consumer 102. Assertion: Trophic levels are formed by only plants.
(d) decomposer Reason: Food chains and webs are formed due to linked
organisms on the basis of their nutrition.
Our Environment S-145
103. Assertion: A network of food chains existing together in 110. The hierarchies within a food web are called ...........
an ecosystem is known as food web. levels.
Reason: An animal like kite cannot be a part of a food web. 111. Without the ................. in a food web many chemicals
104. Assertion: Supersonic jets cause pollution as they thin would not be recycled.
out ozone.
112. Decrease in ozone in stratosphere is linked to release of
Reason: Depletion of ozone cause green house effect.
synthetic chemicals like ..........................
105. Assertion: Tropical rain forests are disappearing fast
from developing countries such as India. 113. The ........... make the energy from sunlight available to
Reason: No value is attached to these forests because the rest of the ecosystem.
these are poor in biodiversity. 114. The use of chemicals like CFCs has endangered the
.......... layer.
Match the Following
115. The waste we generate may be ................. or ..................
DIRECTIONS : Each question contains options given in two 116. All the interacting organisms in an area together with
columns which have to be matched. options (A, B, C, D) in the non-living constituents of the environment form an
column I have to be matched with options (p, q, r, s) in column II. .................... .
106. Column I Column II 117. Gardens and crop fields are example of ....................
(A) Grass (p) Primary carnivore ecosystem.
(B) Grasshopper (q) Secondary carnivore 118.
The decomposers comprising micro-organisms like
(C) Frog (r) Producer .................... and .....................
(D) Hawk (s) Primary consumer 119. The energy flows from .................... to the heterotrophs
107. Column A Column B and decomposers.
(A) Third trophic level (p) Ozone 120. The flow of energy is always .................... in food chains.
(B) Accumulation of (q) CFCs 121. The interlocking pattern of various food chains is referred
pesticides at higher as .................... .
trophic level
122. The disposal of the waste we generate is causing serious
(C) Green plants (r) Herbivore
....................problems.
(D) Flow of energy in (s) Biomagnification
an ecosystem 123. The various populations of living organisms in an area
together form .................... community.
(E) Consists of 3 atoms (t) Decomposers
of oxygen 124. All the ecosystems taken together in a geographical area
form a bigger unit known as .................... .
(F) Main cause of (u) Producers
depletion of ozone 125. Hydrosphere, lithosphere and atmosphere along with
living organism form .................... .
layer
(G) Second trophic level (v) Unidirectional 126. The plants trap .................... energy and convert it into
.................... energy.
(H) Break-down of dead (w) Carnivores
organic compounds 127. The energy available at each successive trophic level is
....................of the previous level.
Fill in the Blanks 128. Nitrogen-fixing bacteria live in nodules on the roots of
.................... plants.
DIRECTIONS : Complete the following statements with an
129. Nitrates and nitrites present in the soil are changed into
appropriate word / term to be filled in the blank space(s).
....................by micro-organisms.
108. Climate refers to the prevailing ............... conditions. 130. The increased nitrogen in rivers and lakes boosts the
109. The total amount of ................ ................ per unit time growth of .................... and other phytoplankton at the
produced in an ecosystem is called the gross primary cost of other aquatic organisms.
productivity.
S-146 Science
138. The materials like plastics are not acted upon by physical
True / False process.

DIRECTIONS : Read the following statements and write your 139. Secondary consumers in a food chain are always carnivores.
answer as true or false. 140. Carbon dioxide causes depletion of ozone layer thereby
allowing more UV-radiations to reach the earth.
131. Non-biodegradable articles are the ones which cannot be
digested. 141. Organisms can make organic compounds from inorganic
substances by using the radiant energy of the sun in the
132. Ozone is formed in stratosphere by action of ultraviolet presence of chlorophyll.
radiations on oxygen.
142. Ecology is the scientific study of the interaction of
133. Earth is kept warm due to green house flux. organisms with each other and the environment.
134. Biodegradable wastes should be separated and kept in 143. The abiotic components of the environment are the living
blue colour bins for garbage collectors. factors.
135. Blue green algae are producers. 144. The amount of usable energy remains constant as it is
passed from one trophic level to another.
136. The reproduction and other activities of living organisms
are affected by the abiotic components of ecosystem. 145. The energy within an ecosystem is fixed and never changes.

137. Specific enzymes are needed for the break-down of a 146. Human population and technology are having a
particular substance. destructive impact on the biosphere.
Our Environment S-147

ANSWER KEY & SOLUTIONS


1. (a) 2. (d) 3. (d) 4. (d) 67. (d) The increase in concentration of harmful chemical
5. (d) 6. (a) 7. (d) 8. (c) substance like pesticides in the body of living
9. (b) 10. (d) 11. (d) 12. (a) organisms at each trophic level of a food chain
is called biological magnification. The organism
13. (a) 14. (a) 15. (a) 16. (b)
which occurs at the highest trophic level in the
17. (b) 18. (d) 19. (b) 20. (a) food chain will have the maximum concentration
21. (a) 22. (c) 23. (c) 24. (c) of harmful chemicals in its body. Since vulture
25. (a) 26. (d) 27. (c) 28. (d) occupies the top level as it eats the tiger, which in
29. (b) 30. (d) 31. (a) 32. (d) turn eats the goat, which eats the grass in the food
chain. So, it will have the maximum concentration
33. (d) 34. (d) 35. (c) 36. (c)
of harmful chemicals in its body.
37. (b) 38. (a) 39. (c) 40. (a)
68. (a) The accumulation of harmful chemicals with an
41. (c) 42. (c) 43. (b) 44. (c) increase in trophical level is known as biological
45. (c) 46. (b) 47. (b) 48. (d) magnification.
49. (d) 50. (b) 51. (d) 52. (d) 69. (b) This process can be seen in green house effect. Infra-
53. (c) 54. (c) 55. (b) 56. (b) red radiations fails to escape from glass house. As a
result temperature rises in a glass house.
57. (c) 58. (c)
70. (a) 71. (b)
59. (c) Statement (ii) and (iii), regarding food chain, are
correct. 72. (c) The phenomenon of accumulation of non-
biodegradable chemicals, e.g., DDT, in a food chain
60. (a)
at each trophic level is called biomagnifaction.
61. (a) In the graphical representation pyramid of number
73. (d) Decomposers are present at the final level in a
shows the arrangement of number of individuals
food web. They breakdown dead and decaying
(population size) of different trophic levels in a food
organic matter (plants and animals) and convert
chain in an ecosystem.
into nutrients in the soil. They naturally increase the
The pyramid of pond ecosystem is upright, because decomposition process and therefore used in natural
the base of this pyramid is occupied by the maximum biocomposting.
number of phytoplanktons (autotrophs) and
number of individuals which gradually decreases 74. (c) and (d) The term biodegradable is used to describe
towards the primary and secondary consumers side materials that decompose through the actions
respectively. of bacteria, fungi, and other living organisms.
Temperature and sunlight may also play roles in the
62. (c) 63. (b) decomposition of biodegradable plastics and other
64. (a) Fungi is not autotrophic. Autotrophs are producers substances.
which make their own food through the process of Example: human and animal excreta, plant products
photosynthesis. like rubber, paper, wood, leaves, cotton, and wool,
65. (d) Domestic sewage contains faecal matter, having dead remains of living organisms, kitchen waste,
coliform bacteria E. coli. If a water body has agricultural waste.
coliform bacteria, it indicates pollution from 75. (b) Food chain is a linear sequence of organisms
domestic sewage. which starts from producer organism and ends with
66. (c) Incomplete combustion of coal produces carbon decomposer species. In the given group, grass is
monoxide which is highly toxic and can cause death a producer, goat is a herbivore and human is top
of human. carnivore.
S-148 Science
76. (d) Eco-friendly products promote green living that 96. (a) Grassland food chain
helps to conserve energy and also prevent air, water
97. (a) 98. (a)
and noise pollution. They prove to be a boon for the
environment and also prevent human health from 99. (a) Animals blend with the surroundings or background to
deterioration. remain unnoticed for protection and aggression.

77. (b) Organisms which synthesise carbohydrates from 100. (a) 101. (a) 102.  (d)
inorganic compounds using radiant energy are called 103. (c) In the food web, different food chains are
producers e.g., all green plants. The producers make
interconnected. Each chain consists of different
the energy from sunlight available to the rest of the
trophic levels i.e., producers, consumers and
ecosystem.
detrivores. So, kite can also be a part of food web.
Organisms which consume the food produced, either
directly from producers or indirectly by feeding on 104. (a)
other consumers are the consumers. 105. (c) Tropical rain forests have disappeared mainly
Microorganisms which break-down the complex due to man’s activities. Due to over population in
organic substances into simple inorganic substances countries like India, rain forests are cut to make
used by plants are called decomposers. place available for man to live and build houses. To
Carnivores and herbivores are types of consumers. build buildings and factories man has incessantly
cut down trees. This has caused the depletion of rain
78. (b) Producers capture the solar energy and convert it
forests.
into chemical energy. All green plants and certain
blue-green algae which can produce food by 106. (A) → r (B) → s (C) → p (D) → q
photosynthesis come under this category and are
called the producer. 107. (A) → (w), (B) → (s), (C) → (u), (D) → (v), (E) → (p),
(F) → (q), (G) → (r), (H) → (t)
79. (a) There is a loss of energy as we go from one trophic
108. weather 109. organic material
level to the next, this limits the number of trophic
levels in a food-chain. 110. trophic 111. decomposers
80. (c) 81. (d) 112. Chorofluorocarbons 113. producers
82. (b) 100 J 114. ozone

83. (c) Third Trophic level 115. biodegradable, non-biodegradable.

84. (a) Act at every trophic level of the food chain 116. ecosystem 117. artificial

85. (c) Energy is unidirectional and matter is repeatedly 118. bacteria, fungi 119. autotrophs
circulating 120. unidirectional 121. food web
86. (a) Decrease in energy at higher trophic level 122. environmental 123. biotic
87. (c) Secondary consumer 124. biome 125. biosphere
88. (d) Omnivores – molds, yeast and mushrooms 126. light, chemical 127. 10%

89. (a) Grassland food chain 128. leguminious 129. ammonia

90. (c) Because plants can produce its own energy 130. algae

91. (d) B and D 131. True 132. True 133. True 134. False

92. (c) (ii), (iii) 135. True 136. True 137. True 138. False

93. (b) Saprophytes 139. True 140. False 141. True 142. True
143. False 144. False 145. False 146. True
94. (d) Omnivores – molds, yeast and mushrooms

95. (c) Secondary consumer


1 Acids, Bases and
Real Numbers
Salts
7. The sum of exponents of prime factors in the prime-
Multiple Choice Questions (MCQs) factorisation of 196 is
(a) 3 (b) 4
DIRECTIONS : This section contains multiple choice (c) 5 (d) 2
questions. Each question has 4 choices (a), (b), (c) and (d) out
of which only one is correct. 8. When 2256 is divided by 17, then remainder would be
(a) 1 (b) 16
1. What is the largest number that divides 70 and 125, leaving (c) 14 (d) None of these
remainders 5 and 8 respectively?
(a) 13 (b) 9 9. The sum of three non-zero prime numbers is 100. One of
them exceeds the other by 36. Then, the largest number is
(c) 3 (d) 585
(a) 73 (b) 91
2. What is the largest number that divides 245 and 1029, (c) 67 (d) 57
leaving remainder 5 in each case?
p
(a) 15 (b) 16 10. The rational number of the form , q ≠ 0, p and q
q
(c) 9 (d) 5
are positive integers , which represents 0.134 i.e.,
3. A class of 20 boys and 15 girls is divided into n groups so (0.1343434....) is
that each group has x boys and y girls. Values of x, y and
134 134
n respectively are (a) (b)
999 990
(a) 3, 4 and 8 (b) 4, 3 and 6
(c) 4, 3 and 7 (d) 7, 4 and 3 133 133
(c) (d)
999 990
4. If p, q are two consecutive natural numbers, then H.C.F.
(p, q) is 11. The least number which is a perfect square and is divisible
(a) p (b) q by each of 16, 20 and 24 is
(c) 1 (d) pq (a) 240 (b) 1600
(c) 2400 (d) 3600
5. Given that L.C.M. (91, 26) = 182, then H.C.F. (91, 26) is
(a) 13 (b) 26 12. If n is an even natural number, then the largest natural
number by which n (n + 1) (n + 2) is divisible is
(c) 17 (d) 9
(a) 6 (b) 8
6. Which of the following statement is true? (c) 12 (d) 24
(a) Every point on the number line represents a rational
number. 13. The least number which when divided by 15, leaves a
remainder of 5, when divided by 25, leaves a remainder
(b) Irrational numbers cannot be represented by points on
of 15 and when divided by 35, leaves a remainder of 25,
the number line.
is
22 (a) 515 (b) 525
(c) is a rational number.
7 (c) 1040 (d) 1050
(d) None of these.
M-2 Mathematics
14. The number 313 – 310 is divisible by 22. On dividing a natural number by 13, the remainder is 3 and
(a) 2 and 3 on dividing the same number by 21, the remainder is 11. If
(b) 3 and 10 the number lies between 500 and 600, then the remainder
on dividing the number by 19 is
(c) 2, 3 and 10
(a) 4 (b) 6
(d) 2, 3 and 13
(c) 9 (d) 13
15. A number lies between 300 and 400. If the number is added
to the number formed by reversing the digits, the sum is 23. Let a1, a2,..., a100 be non-zero real numbers such that
888 and if the unit’s digit and the ten’s digit change places, a1 + a2 + ...+ a100 = 0
the new number exceeds the original number by 9. Then, Then,
the number is 100 100

(a) 339 (b) 341 ∑ ai 2ai > 0and


(a) ∑ ai 2−ai < 0
i =1 i =1
(c) 378 (d) 345 100 100
16. Which of the following will have a terminating decimal ∑ ai 2ai ≥ 0and
(b) ∑ ai 2−ai ≥ 0
i =1 i =1
expansion?
100 100
77 23
(a) (b) ∑ ai 2ai ≤ 0and
(c) ∑ ai 2−ai ≤ 0
210 30 i =1 i =1
125 23 100 100
(c)
441
(d)
8 (d) The sign of ∑ ai 2ai or ∑ ai 2−ai depends on the
i =1 i =1
17. I. The L.C.M. of x and 18 is 36. choice of ai’s
II. The H.C.F. of x and 18 is 2. 24. The value of 0.235 is :
What is the number x ? 233 233
(a) (b)
(a) 1 (b) 2 900 990
(c) 3 (d) 4 235 235
(c) (d)
999 990
18. If a = 23 × 3, b = 2 × 3 × 5, c = 3n × 5 and
25. Consider the following statements: For any integer n,
L.C.M. (a, b, c) = 23 × 32 × 5, then n =
(a) 1 (b) 2 n2 + 3 is never divisible by 17.
I.
(c) 3 (d) 4 n2 + 4 is never divisible by 17.
II.
19. If p 1 and p 2 are two odd prime numbers such that Then,
p1 > p2, then p12 – p22 is
(a) both I and II are true
(a) an even number
(b) both I and II are false
(b) an odd number
(c) I is false and II is true
(c) an odd prime number
(d) I is true and II is false
(d) a prime number
1
20. When a natural number x is divided by 5, the remainder 26. Given that = 0.142857 , which is a repeating decimal
7
is 2. When a natural number y is divided by 5, the
remainder is 4. The remainder is z when x + y is divided having six different digits. If x is the sum of such first three
2z − 5 1
by 5. The value of is positive integers n such that = 0.abcdef , where a, b, c,
3 n
(a) –1 (b) 1 d, e and f are different digits, then the value of x is
(c) –2 (d) 2 (a) 20 (b) 21
(c) 41 (d) 42
21. The largest non-negative integer k such that 24k divides
13! is 27. If m = n2 – n, where n is an integer, then m2 – 2m is divisible by
(a) 2 (b) 3 (a) 20 (b) 24
(c) 4 (d) 5 (c) 30 (d) 16
Real Numbers M-3
28. The unit digit in the expression 55725 + 735810 + 22853 is (c) Every real number is rational.
(a) 0 (b) 4 (d) Every point on a number line is associated with a real
(c) 5 (d) 6 number.

29. For some integer m, every even integer is of the form 37. Which of the following statement(s) is/are not correct?
(a) m (b) m+1 (a) There are infinitely many even primes.
(c) 2m (d) 2m + 1 (b) Let ‘a’ be a positive integer and p be a prime number
such that a2 is divisible by p, then a is divisible by p.
30. For some integer q, every odd integer is of the form (c) Every positive integer different from 1 can be
(a) q (b) q+1 expressed as a product of non-negative power of 2
(c) 2q (d) 2q + 1 and an odd number.
33 p is an irrational number.
31. The decimal expansion of the rational number 2 will (d) If ‘p’ is a positive prime, then
terminate after 2 .5
(a) one decimal place
(b) two decimal places
(c) three decimal places DIRECTIONS : Study the given Case/Passage and answer the
(d) more than 3 decimal places following questions.

32. Product of two co-prime numbers is 117. Their L.C.M. Case/Passage-I


should be To enhance the reading skills of grade X students, the school
(a) 1 nominates you and two of your friends to set up a class
(b) 117 library. There are two sections-section A and section B of
(c) equal to their H.C.F. grade X. There are 32 students in section A and 36 students
in section B.
(d) Lies between 1 to 117

33. Which of the following statement(s) is/are always true?


(a) The sum of two distinct irrational numbers is
rational.
(b) The rationalising factor of a number is unique.
(c) Every irrational number is a surd.
(d) None of these

34. Which of the following statement(s) is/are not correct?


73
(a) is a non-terminating repeating decimal. [From CBSE Question Bank-2021]
54
38. What is the minimum number of books you will acquire
(b) If a = 2 + 3 and b = 2 – 3 , then a + b is for the class library, so that they can be distributed equally
irrational. among students of Section A or Section B?
(c) If 19 divides a3, then 19 divides a, where a is a positive (a) 144 (b) 128 (c) 288 (d) 272
integer.
39. If the product of two positive integers is equal to the product
(d) Product of L.C.M. and H.C.F. of 25 and 625 is 15625.
of their HCF and LCM is true then, the HCF (32, 36) is
35. The product of unit digit in (795 – 358) and (795 + 358) is (a) 2 (b) 4 (c) 6 (d) 8
(a) 8 40. 36 can be expressed as a product of its primes as
(b) lies between 3 and 7 (a) 22 × 32 (b) 21 × 33
(c) 6 3
(c) 2 × 3 1 (d) 20 × 30
(d) lies between 3 and 6
41. 7× 11 × 13 × 15 + 15 is a
36. Which of the following statement(s) is/are not correct? (a) Prime number
(a) Every integer is a rational number. (b) Composite number
(b) The sum of a rational number and an irrational number (c) Neither prime nor composite
is an irrational number. (d) None of the above
M-4 Mathematics
42. If p and q are positive integers such that p = ab2 and Observe the following factor tree and answer the following:
q = a2b, where a, b are prime numbers, then the LCM 48. What will be the value of x?
(p, q) is (a) 15005 (b) 13915
(a) ab (b) a2b2 (c) a3b2 (d) a3b3 (c) 56920 (d) 17429
Case/Passage-II
49. What will be the value of y?
A seminar is being conducted by an Educational Organisation, (a) 23 (b) 22 (c) 11 (d) 19
where the participants will be educators of different subjects.
The number of participants in Hindi, English and Mathematics 50. What will be the value of z?
are 60, 84 and 108 respectively. (a) 22 (b) 23 (c) 17 (d) 19
51. According to Fundamental Theorem of Arithmetic 13915
is a
(a) Composite number
(b) Prime number
(c) Neither prime nor composite
(d) Even number
52. The prime factorisation of 13915 is
[From CBSE Question Bank-2021] (a) 5 × 113 × 132 (b) 5 × 113 × 232
2
(c) 5 × 11 × 23 (d) 5 × 112 × 132
43. In each room the same number of participants are to be
seated and all of them being in the same subject, hence
maximum number participants that can accommodated Assertion & Reason
in each room are
(a) 14 (b) 12 (c) 16 (d) 18 DIRECTIONS : Each of these questions contains an Assertion
followed by Reason. Read them carefully and answer the
44. What is the minimum number of rooms required during
the event? question on the basis of following options. You have to select
the one that best describes the two statements.
(a) 11 (b) 31 (c) 41 (d) 21
45. The LCM of 60, 84 and 108 is (a) If both Assertion and Reason are correct and Reason is
the correct explanation of Assertion.
(a) 3780 (b) 3680 (c) 4780 (d) 4680
(b) If both Assertion and Reason are correct, but Reason is
46. The product of HCF and LCM of 60,84 and 108 is not the correct explanation of Assertion.
(a) 55360 (b) 35360 (c) 45500 (d) 45360
(c) If Assertion is correct but Reason is incorrect.
47. 108 can be expressed as a product of its primes as (d) If Assertion is incorrect but Reason is correct.
(a) 23 × 32 (b) 23 × 33
13
(c) 22 × 32 (d) 22 × 33 53. Assertion : is a terminating decimal fraction.
3125
Case/Passage-III
Reason : If q = 2n.5m where n, m are non-negative integers,
A Mathematics exhibition is being conducted in your school p
and one of your friendsis making a model of a factor tree. He then is a terminating decimal fraction.
q
has some difficulty and asks for your help in completing a quiz
for the audience. 54. Assertion : Denominator of 34.12345. When expressed
p
x in the form , q ≠ 0 , is of the form 2m × 5n, where m, n
q
5 2783 are non-negative integers.
Reason : 34.12345 is a terminating decimal fraction.
253 y
55. Assertion : The H.C.F. of two numbers is 16 and their
product is 3072. Then, their L.C.M = 162.
11 z

Reason : If a, b are two positive integers, then
[From CBSE Question Bank-2021] H.C.F × L.C.M. = a × b.
Real Numbers M-5
56. Assertion : 2 is a rational number.
Reason : The square roots of all positive integers are Fill in the Blanks
irrationals.
DIRECTIONS : Complete the following statements with an
57. Assertion : If L.C.M. {p, q} = 30 and H.C.F {p, q} = 5, appropriate word / term to be filled in the blank space(s).
then p.q = 150.
Reason : L.C.M. of (a, b) × H.C.F of (a, b) = a.b. 63. 5 is a/ an ............... number.
58. Assertion : n2 – n is divisible by 2 for every positive integer. 1
64. is a/ an ............... number.
Reason : 2 is not a rational number. 2

59. Assertion : n2 + n is divisible by 2 for every positive 65. The exponent of 2 in the prime factorisation of 144,
integer n. is ...............

Reason : If x and y are odd positive integers, from 66. 7 5 is a/ an ............... number.
x2 + y2 is divisible by 4.
67. 6 + 2 is a/ an ............... number.
Match the Following 68. An .............. is a series of well defined steps which gives
a procedure for solving a type of problem.
DIRECTIONS : Each question contains statements given in 69. An........... is a proven statement used for proving another
two columns which have to be matched. Statements (A, B, C, D) statement.
in column-I have to be matched with statements (p, q, r, s) in column-II.
70. L.C.M. of 96 and 404 is ..................
60. Column-I Column-II 71. H.C.F. of 6, 72 and 120 is ..............
(A) Irrational number is (p) rational number
72. 156 as a product of its prime factors .............
always
(B) Rational number is (q) irrational number 35
73. is a ..................... decimal expansion.
always 50
(C) 3 6 is not a (r) non-terminating,
non-repeating True / False
(D) 2 2 is an (s) terminating decimal
61. Column-I Column-II DIRECTIONS : Read the following statements and write your
answer as true or false.
(A) H.C.F of the smallest (p)  6
composite number and 74. Given positive integers a and b, there exist whole numbers
the smallest prime q and r satisfying a = bq + r, 0 ≤ r < b.
number 75. Every composite number can be expressed (factorised) as
(B) H.C.F of 336 and 54 (q)  5 a product of primes and this factorisation is unique, apart
(C) H.C.F of 475 and 495 (r)  2 from the order in which the prime factors occur.
62. Column-I Column-II 76. 2 and 3 are irrational numbers.
551
(A) (p) a prime number 77. If x = p/q be a rational number, such that the prime factorisation
2 × 56 × 79
3
of q is of the form 2n5m, where n, m are non-negative integers.
(B) Product of ( )
5 – 3 (q) is an irrational number Then x has a decimal expansion which is terminating.
78. Any positive odd integer is of the form 6q + 1 or 6q + 3 or
and ( 5 + 3 is ) 6q + 5, where q is some integer.
(C) 5 – 4 (r) is a terminating decimal 79. The quotient of two integers is always a rational number.
representation 80. 1/0 is not rational.
422 81. The number of irrational numbers between 15 and 18 is
(D) 3 4 (s) is a non-terminating but
2
× 5 repeating decimal infinite.
representation 82. Every fraction is a rational number.
M-6 Mathematics

ANSWER KEY & SOLUTIONS


1. (a) Required number = H.C.F. {(70 – 5), (125 – 8)} 134 − 1 133
10. (d) 0.134 = =
= H.C.F. (65, 117) = 13. 990 990
2. (b) Required number = H.C.F.{(245 – 5), (1029 – 5)} 11. (d) The L.C.M. of 16, 20 and 24 is 240. The least multiple
of 240 that is a perfect square is 3600 and also we
= H.C.F. (240, 1024) = 16.
can easily eliminate choices (a) and (c) since they
3. (c) H.C.F. of 20 and 15 = 5 are not perfect number. Hence, the required least
So, 5 students are in each group. number which is also a perfect square is 3600 which
is divisible by each of 16, 20 and 24.
20 + 15 35
∴ n= = =7 12. (d) Out of n and n + 2, one is divisible by 2 and the other
5 5
by 4, hence n (n + 2) is divisible by 8. Also n, n + 1,
Hence, x = 4, y = 3 and n = 7 n + 2 are three consecutive numbers, hence one of them is
4. (c) 1 divisible by 3. Hence, n (n + 1) (n + 2) must be divisible
by 24. This will be true for any even number n.
91 × 126 91 × 126 13. (a) The number divisible by 15, 25 and 35 = L.C.M.
5. (a) H.C.F. (91, 126) = = = 13
L.C.M.(91, 126) 182 (15, 25, 35) = 525
6. (d) All the given statements are false. Since, the number is short by 10 for complete division
by 15, 25 and 35.
7. (b) 196 = 22 ⋅ 72, sum of exponents = 2 + 2 = 4
Hence, the required least number = 525 – 10 = 515.
2256 (24 )64 14. (d) 313 – 310 = 310 (33 – 1) = 310 (26) = 2 × 13 × 310
8. (a) When 2256 is divided by 17 then, =
24 + 1 (24 + 1) Hence, 313 – 310 is divisible by 2, 3 and 13.
By remainder theorem when f (x) is divided by x + a 15. (d) Sum is 888 ⇒ unit’s digit should add up to 8. This is
the remainder = f (– a) possible only for option (d) as “3” + “5” = “8”.
16. (d)
Here, f (a) = (24)64 and x = 24 and a = 1
17. (d) L.C.M × H.C.F = First number × second number
∴ Remainder = f (–1) = (–1)64 = 1
36 × 2
Hence, required number = =4.
9. (c) Since, the sum of all the three prime numbrs is 100. 18
18. (b) Value of n = 2.
Then, there are two cases
19. (a) Since, p1 and p2 are odd primes and sum of two odd
Case 1: All the three numbers should be even because number is an even number.
100 is an even number. But this case is not possible So, p1 + p2 is an even number.
as there is only one even prime. Since, multiple of even number is always even.
Case 2 : One prime is even and other two primes are Therefore, (p1 + p2) (p1 – p2) is even
odd. 2 2
Hence, p1 − p2 = ( p1 + p2 )( p1 − p2 ) is an even
Since, 2 is only even prime, so it must be one of three number.
primes. 20. (a) Since, x is divided by 5, the remainder is 2
Let p and p + 36 be the other two primes. therefore x = 5m + 2
Then, according to question similarly, y = 5n + 4 consider x + y = 5(m + n) + 6
2 + p + (p + 36) = 100; 2p + 38 = 100 = 5(m + n) + 5 + 1 = 5(m + n + 1) + 1
But given that when x + y is divided by 5, the remainder
62
2p = 100 – 38 = 62; p = = 31 is z
2
\ z = 1
So, all the three primes are 2, 31 and 67.
2z – 5 2 (1) – 5
Hence, largest prime number is 67. Now, = = –1
3 3
Real Numbers M-7
21. (b) We know that 1
26. (c) = 0.142857
13! = 2 × 3 × 4 × 5 × 6 × 7 × 8 × 9 × 10 × 11 × 12 × 13 7
= 210 × 35 × 52 × 7 × 11 × 13 ⇒ 24k = (23 × 3)k The second positive integer whose reciprocal have six
different repeating decimals is
where k is largest non-negative integer
1
When 13! is an divided by 24k, we get = 0.076923
13
210 ¥ 35 ¥ 52 ¥ 7 ¥ 11 ¥ 13
And the third positive integer whose reciprocal have
23k .3k
six different repeating decimals is
= 210– 3k . 35 – k . 52 × 7 × 11 × 13 1
= 0.047619
\ 10 – 3k is integer. 21
Then, maximum value of k = 3. Therefore, the values of x are 7, 13, 21
22. (a) Given: The natural number, when divided by 13 leaves Hence, the required sum is = 7 + 13 + 21 = 41
remainder 3
27. (b) ∵ m = n2 – n = n(n – 1)
The natural number, when divided by 21 leaves
Now, m2 – 2m = m(m – 2)
remainder 11
= n(n – 1)(n2 – n – 2) = n(n – 1)(n – 2)(n + 1)
So, 13 – 3 = 21 – 11 = 10 = k
Since we know that product of any four consecutive
Now, LCM (13, 21) = 273
integers is always divisible by 24.
But the number lies between 500 and 600
\ m2 – 2m is divisible by 24.
\ 2 LCM (13, 21) – k = 546 – 10 = 536
28. (d)
For given numbers,
536 = 19 × 8 + 4 \ remainder = 4
(55)725, unit digit = 5; (73)5810, unit digit = 9
23. (a) Let a1, a2, a3, ..., a100 be non-zero real number and
(22)853, unit digit = 2
a1 + a2 + a3 + ... + a100 = 0 Unit digit in the expression
ai − ai
ai ⋅ 2 > ai and ai ⋅ 2 < ai 55725 + 735810 + 22853 is 6
100 100 100 100 29. (c) 30. (d) 31. (b)
∴ ∑ a1 ⋅ 2ai > ∑ ai and ∑ a1 ⋅ 2−ai < ∑ ai 32. (b)
Since, H.C.F. of co-prime number is 1.
i =1 i =1 i =1 i =1
100 100 ∴ Product of two co-prime numbers is equal to their
⇒ ∑ a1 ⋅ 2ai > 0 and ∑ a1 ⋅ 2−ai < 0 L.C.M. So, L.C.M. = 117
i =1 i =1
33. (d)
Hence, option (a) is correct.
34. (a)
24. (c) Let x = 0.235 ...(i)
35. (a) Unit digit in (795) = Unit digit in [(74)23 × 73]
1000 x = 235.235 ...(ii) = Unit digit in 73 (as unit digit in 74 = 1)
235
Subtract (i) from (ii), 999 x = 235 ⇒ x = = Unit digit in 343
999
Unit digit in 358 = Unit digit in (34)4 × 32
25. (d) Let us consider that n2 + 3 is divisible by 17
[as unit digit 34 = 1]
∴ n2 + 3 = 17K [K ∈ N] = Unit digit is 9
⇒ n2 = 17K – 3 ⇒ n2 = 3 (17m – 1) [∵ K = 3m] So, unit digit in (795 – 358)
= Unit digit in (343 – 9) = Unit digit in 334 = 4
3(17m – 1) is a perfect square, which is not possible.
Unit digit in (795 + 358) = Unit digit in (343 + 9)
∴ n2 + 3 is never divisible by 17. = Unit digit in 352 = 2
In, n2 + 4, put n = 9 So, the product is 4 × 2 = 8
So, (9)2 + 4 = 81 + 4 = 85 which is divisible by 17. 36. (c)
37. (a)
∴ I is true and II is false.
M-8 Mathematics
38. (c) For getting least number of books, 53. (a) Reason is correct.
taking LCM of 32, 36 Since, the factors of the denominator 3125 is of the
432, 36 form 20 × 55.
88, 9 13
\ is a terminating decimal
91, 9 3125
1, 1 Since, assertion follows from reason.
⇒ 4 × 8 × 9 = 288 54. (a) Reason is clearly true.
39. (b)
HCF of 32, 36 is
3412345 682469 682469
Again, 34.12345 = = = 5 4
4 32, 36 100000 20000 2 ×5
8, 9
Its denominator is of the form 2m × 5n, where
=4
m = 5, n = 4 are non-negative integers
40. (a) 36 is expressed as prime
\ Assertion is true. Since, reason gives assertion
36 = 2 × 2 × 3 × 3 = 22 × 32
41. (b) 7 × 11 × 13 × 15 + 15 \ (a) holds.
⇒ 15 (7 × 11 × 13 + 1) 55. (d) Here, reason is true [standard result]
so given no. is a composite number.
3072
42. (b) Given a, b are prime number. So Assertion is false. ∵ = 192 ≠ 162
16
LCM of p, q, where p = ab2, q = a2b 56. (c) Here, reason is not true.
p = a × b × b
∵ 4 = ±2, which is not an irrational number.
q = a × b × a
a × b × b × a ⇒ a2b2 \ Reason does not hold. Clearly, assertion is true.
43. (b) For maximum number of participants, taking HCF of 57. (a) 58. (b) 59. (a)
60, 84 and 108
12 60, 84, 108 60. (A) → (r) [Q 12 = 3 × 4 Q it is a composite number]
5, 7, 9 (B) → (s) [Q g.c.d. between 2 and 7 = 1]
= 12 (C) → (p) [Q 2 is a prime number]
44. (d) Minimum number of rooms required are (D) → (q) [Q 2 is not a rational number]
5 + 7 + 9 = 21
61. (A) → (r); (B) → (p); (C) → (q)
45. (a) LCM of 60, 84, 108 is
62. (A) → (s); (B) → (p); (C) → (q); (D) → (r)
12 × 5 × 7 × 9 = 3780
46. (d) Product is = 12 × 3780 = 45360 63. irrational 64. irrational
47. (d) 108 = 2 × 2 × 3 × 3 × 3 = 22 × 33 65. 4 66. irrational
48. (b) x = 5 × 2783 = 13915 67. irrational 68. algorithm
49. (c) y = 253 ) 2783 ( = 11
69. lemma 70. 9696
50. (b) z = 11) 253 ( = 23
71. 6 72. 22 × 3 × 13
51. (a) Composite number having more than 2 factors.
52. (c) Prime factorisation of 13915 = 73. terminating
5 13915 74. True 75. True
11 2783 76. True 77. True
11 253 78. True 79. False
23 23 80. True 81. True
1
82. True
⇒ 5 × 11 × 11 × 23
⇒ 5 × 112 × 23
2 Acids, Bases and
Polynomials
Salts
6. If f (x) = 2x3 – 6x + 4x – 5 and g(x) = 3x2 – 9, then the
Multiple Choice Questions (MCQs) value of f (1) + g(–2) is
(a) –3 (b) –2
DIRECTIONS : This section contains multiple choice
(c) 3 (d) 2
questions. Each question has 4 choices (a), (b), (c) and (d) out
of which only one is correct. 7. Factor of the polynomial x3 – 3x2 – 10x + 24 are:
(a) (x – 2)(x + 3)(x – 4)
1. If the zeroes of the polynomial f (x) = k2x2 – 17x + k + 2,
(k > 0) are reciprocal of each other than value of k is (b) (x + 2)(x + 3)(x + 4)

(a) 2 (b) –1 (c) (x + 2)(x – 3)(x – 4)

(c) –2 (d) 1 (d) (x – 2)(x – 3)(x – 4)

2. If one zero of the quadratic polynomial 8. The zeroes of the polynomial are

5 p(x) = x2 –10x –75


2x2 – 8x – m is , then the other zero is
2 (a) 5, – 15 (b) 5, 15
2 2 (c) 15, – 5 (d) – 5, – 15
(a) (b) –
3 3
9. If a and b are zeroes of the polynomial
3 −15
(c) (d) 2t2 – 4t + 3, then the value of a2b + ab2 is :
2 2
3. Let p(y) = y4 – 3y2 + 2y + 5, then the remainder when p(y) 3
(a) (b) 2
is divided by (y – 1). 4
(a) 2 (b) 3 (c) 3 (d) 4

(c) –5 (d) 5 10. The zeroes of the polynomial x2 – 3x – m(m + 3) are


m, m + 3
(a) (b) –m, m +3
4. If the polynomials ax3 + 4x2 + 3x – 4 and x3 – 4x + a
leave same remainder when divided by (x – 3), find the m, –(m + 3)
(c) (d) –m, –(m + 3)
value of a. 11. The value of x, for which the polynomials
(a) –1 (b) 1 x2 – 1 and x2 – 2x + 1 vanish simultaneously, is
1 1 (a) 2 (b) –2
(c) −
(d)
2 2 (c) –1 (d) 1
5. Let f(x) = x2 – 27x + 196. If f (a) = a, then what is the 12. If x = 0.7 , then 2x is
value of a.
(a) 1.4 (b) 1.5
(a) 7 (b) 14
(c) 1.54 (d) 1.45
(c) 21 (d) 6
M-10 Mathematics
13. Lowest value of x2 + 4x + 2 is 21. The polynomial, f(x) = (x – 1)2 + (x – 2)2 + (x – 3)2 +
(a) 0 (b) –2 (x – 4)2 has minimum value, when x = ...................
(c) 2 (d) 4 (a) 40 (b) 20

14. If a3 – 3a2b + 3ab2 – b3 is divided by (a – b), then the (c) 10 (d) 2.5
remainder is 22. If one zero of the quadratic polynomial x2 + 3x + k is 2,
a2 – ab
(a) + b2 then the value of k is
(b) a2 + ab + b2 (a) 10 (b) –10
(c) 1 (c) 5 (d) –5
(d) 0 23. If one of the zeroes of the quadratic polynomial
(k –1) x2 + kx + 1 is –3, then the value of k is
15. A quadratic polynomial when divided by x + 2 leaves
a remainder of 1 and when divided by x – 1, leaves a 4 −4
(a) (b)
remainder of 4. What will be the remainder if it is divided 3 3
by (x + 2) (x – 1) ? 2 −2
(c) (d)
(a) 1 (b) 4 3 3

(c) x + 3 (d) x – 3 24. The zeroes of the quadratic polynomial x2 + 99x + 127
are
16. If the polynomials ax3 + 4x2 + 3x – 4 and x3 – 4x + a leave (a) both positive
the same remainder when divided by x – 3, then the value
(b) both negative
of a is
(c) one positive and one negative
(a) 1 (b) –1
(c) 19/14 (d) –5/14 (d) both equal
25. Which of the following given options is/are correct?
17. If the value of a quadratic polynomial p(x) is 0 only at
x = –1 and p(–2) = 2, then the value of p(2) is 2
(a) + 3 is a polynomial
(a) 18 (b) 9 x

(c) 6 (d) 3 (b) x + 5 is a polynomial

18. If x2 – 4 is the factor of 2x3 + k1x2 + k2x + 12, where k1, 2


(c) is a polynomial
k2 are constant, then the value of k1 + k2 is 3x – 4
(a) 11 (b) 5 1 3
(d) 5 x 2 + x + is a polynomial
2 7
(c) –11 (d) –5
26. Which of the following given options is/are correct?
19. If x = 3 + 32/3 + 31/3, then the value of (a) Degree of a zero polynomial is ‘0’.
x3 – 9x2 + 18x – 12 is (b) Degree of a zero polynomial is not defined.
(a) 1 (b) 0 (c) Degree of a constant polynomial is not defined.
(c) –1 (d) 2 (d) A polynomial of degree n must have n zeroes.
1 27. Which of the following is/are a polynomial?
20. Let P(x) be a polynomial of degree 3 and P(n) = for
2
n = 1, 2, 3, 4. Then the value of P(5) is 1
x2 +
(a)
1 x
(a) 0 (b)
5 2 x2 – 3 x + 1
(b)
2 3

(c) (d) x3 – 3x + 1
(c)
5 5
3
(d)
2 x 2 – 5x
Polynomials M-11
28. Which of the following is/are not graph of a quadratic 1
polynomial ? 31. If a and α
are the zeroes of the quadratic polynomial
Y
2x2 – x + 8k, then k is
1
A B (a) 4 (b)
(a)
X X (b) 4
O
–1
(c) (d) 2
Y 4
Y Y
32. The graph of x2 + 1 = 0
(a) Intersects x-axis at two distinct points.
A B X
(c)
X
O X (d) O
X
(b) Touches x-axis at a point.
Y Y (c) Neither touches nor intersects x-axis.
(d) Either touches or intersects x-axis.

33. If the sum of the roots is –p and product of the roots is


DIRECTIONS : Study the given Case/Passage and answer 1
– , then the quadratic polynomial is
the following questions. p

 x   x 
Case/Passage-I k  px 2 – –1
k  – px 2 + + 1 (b)
(a)
 p   p 
The below picture are few natural examples of parabolic shape
which is represented by a quadratic polynomial. A parabolic arch
 1  1
is an arch in the shape of a parabola. In structures, their curve k  x 2 + px –  (d)
(c) k  x 2 – px + 
 p  p
represents an efficient method of load, and so can be found in
bridges and in architecture in a variety of forms.
Case/Passage-II
An asana is a body posture, originally and still a general term
for a sitting meditation pose, and later extended in hatha yoga
and modern yoga as exercise, to any type of pose or position,
adding reclining, standing, inverted, twisting, and balancing
poses. In the figure, one can observe that poses can be related
  to representation of quadratic polynomial.

TRIKONASANA

[From CBSE Question Bank-2021]
29. In the standard form of quadratic polynomial, ax2 + bx + c,
a, b and c are
ADHOMUKHA SAVASANA   ADHO MUKHA SVANA
(a) All are real numbers.
(b) All are rational numbers. [From CBSE Question Bank-2021]
(c) ‘a’ is a non zero real number and b and c are any real
34. The shape of the poses shown is
numbers.
(a) Spiral (b) Ellipse
(d) All are integers.
(c) Linear (d) Parabola
30. If the roots of the quadratic polynomial are equal, where
the discriminant D = b2 – 4ac, then 35. The graph of parabola opens downwards, if__________.
(a) D > 0 (b) D < 0 (a) a ≥ 0 (b) a = 0
(c) D ≥ 0 (d) D = 0 (c) a < 0 (d) a > 0
M-12 Mathematics
36. In the graph, how many zeroes are there for the polynomial?

–2 4
3
1

2 4

–8

(a) 0 (b) 1
(c) 2 (d) 3 [From CBSE Question Bank-2021]

37. The two zeroes in the above shown graph are 39. The shape of the path traced shown is
(a) 2, 4 (b) –2, 4 (a) Spiral (b) Ellipse (c) Linear (d) Parabola
(c) –8, 4 (d) 2, –8 40. The graph of parabola opens upwards, if____________.
(a) a = 0 (b) a < 0 (c) a > 0 (d) a ≥ 0
38. The zeroes of the quadratic polynomial 4 3 x 2 + 5 x – 2 3
are 41. Observe the following graph and answer
2 3 2 3
(a) , –
(b) ,
3 4 3 4 6

2 3 2 3
(c) , – (d) – ,− 2
3 4 3 4 –4 –3 –2 –1 1 2 3 4
–2
Case/Passage-III
Basketball and soccer are played with a spherical ball. Even –6
though an athlete dribbles the ball in both sports, a basketball
player uses his hands and a soccer player uses his feet. Usually,
soccer is played outdoors on a large field and basketball is In the above graph, how many zeroes are there for the
played indoor on a court made out of wood. The projectile (path polynomial?
traced) of soccer ball and basketball are in the form of parabola
(a) 0 (b) 1 (c) 2 (d) 3
representing quadratic polynomial.
42. The three zeroes in the above shown graph are
(a) 2, 3, –1 (b) –2, 3, 1
(c) –3, –1, 2 (d) –2, –3, –1
v = 8.552 m/s 43. What will be the expression of the polynomial?
(a) x3 + 2x2 − 5x − 6 (b) x3 + 2x2 − 5x + 6
θ = 51.89°

R = 7.239 m x3 + 2x2 + 5x − 6
(c) (d) x3 + 2x2 + 5x + 6

h = 3.048 m Assertion & Reason

DIRECTIONS : Each of these questions contains an Assertion


followed by Reason. Read them carefully and answer the
question on the basis of following options. You have to select
the one that best describes the two statements.
Polynomials M-13
(a) If both Assertion and Reason are correct and Reason is
the correct explanation of Assertion. Fill in the Blanks
(b) If both Assertion and Reason are correct, but Reason is
not the correct explanation of Assertion. DIRECTIONS : Complete the following statements with an
(c) If Assertion is correct but Reason is incorrect. appropriate word / term to be filled in the blank space(s).
(d) If Assertion is incorrect but Reason is correct.
51. Polynomials of degrees 1, 2 and 3 are called ...............,
44. Assertion : If one zero of polynomial p(x) = (k 2 + 4) x2 + .................. and ............. polynomials respectively.
13x + 4k is reciprocal of other, then k = 2.
52. The zeroes of a polynomial p(x) are precisely the
Reason : If (x – α) is a factor of p(x), then p(α) = 0 i.e. α
x-coordinates of the points, where the graph of y = p(x)
is a zero of p(x).
intersects the ...............-axis.
45. Assertion : x3 + x has only one real zero.
53. A quadratic polynomial can have at most 2 zeroes and a
Reason : A polynomial of nth degree must have n real cubic polynomial can have at most ............. zeroes.
zeroes.
54. If α and β are the zeroes of the quadratic polynomial
46. Assertion : Degree of a zero polynomial is not defined. −b c
ax2 + bx + c, then α + β = & αβ =
Reason : Degree of a non-zero constant polynomial is ...... .....
‘0’. 55. Zero of a polynomial is always ...............

47. Assertion : Zeroes of f(x) = x2 – 4x – 5 are 5, – 1. 56. A polynomial of degree n has at the most ........... zeroes.

Reason : The polynomial whose zeroes are 2 + 3, 2 – 3


is x2 – 4x + 7. True / False
48. Assertion : x2 + 4x – 5 has two zeroes.
DIRECTIONS : Read the following statements and write your
Reason : A quadratic polynomial can have at the most answer as true or false.
two zeroes.
57. Sum of zeroes of quadratic polynomial
Match the Following (coefficient of x )
= −
(coefficient of x 2 )
1
DIRECTIONS : Each question contains statements given in 1
58. x 2 + 1 is a polynomial
two columns which have to be matched. Statements (A, B, C, D) 5
in column-I have to be matched with statements (p, q, r, s) in 3
column-II. 6 x + x2
59. is a polynomial, x ≠ 0
49. Column-I Column-II x
(Zeroes) (Quadratic 60. Product of zeroes of quadratic polynomial
polynomial) constant term
= −
(A) 3 and –5 (p) x2 – 25 (coefficient of x 2 )
(B) 5 + 2 and 5 − 2 (q) + 2x – 15 x2 61. A polynomial cannot have more than one zero.
(C) – 9 and 1/9 2
(r) x + (80/9)x – 1
62. The degree of the sum of two polynomials each of degree
(D) 5 and – 5 (s) x2 – 10x + 21 5 is always 5.
50. Column-I Column-II
63. 3, –1, 1/3 are the zeroes of the cubic polynomial
(Polynomials) (Zeroes)
(A) 4 – x2 (p) 7 p(x) = 3x3 – 5x2 – 11x – 3.
x3 –
(B) 2x2 (q) –2
64. Zeroes of quadratic polynomial x2 + 7x + 10 are 2 and –5
(C) 6x2 – 3 – 7x (r) 3/2
(D) –x + 7 (s)  0 65. Sum of zeroes of 2x2 – 8x + 6 is – 4
M-14 Mathematics

ANSWER KEY & SOLUTIONS


1. (a) Since zeroes are reciprocal of each other, so product So, p(x) = 0 when x = 15 or x = –5. Therefore
k+2 required zeroes are 15 and –5.
of the roots will be 1, so =1,
k2 9. (c) We have, sum of zeroes
k2 – k – 2 = 0 ⇒ (k – 2)(k + 1) = 0
( −4)
= a + b = − =2
k = 2, k = –1, Since k > 0 ∴ k = 2 2
3
5 Product of zeroes = ab =
2. (c) Let α, β be two zeroes of 2x2 – 8x – m, where a = . 2
2
3

( −Coefficient of x ) ∴ a2b + ab2 = ab (a + b) =
2
×2 = 3
a+b=
Coefficient of x 2
10. (b) x2 – (m +3)x + mx – m(m + 3) = 0
5 8 ⇒ x[x – (m + 3)] + m[x – (m + 3)] = 0
⇒ +b=
2 2
⇒ (x + m) [x – (m + 3)] = 0
8 5 3
⇒ b= − = . \ x + m = 0   x – (m + 3) = 0
2 2 2
3. (d) Substitute y = 1 x = –m x=m+3

4. (a) Substitute x = 3 in polynomial ax3 + 4x2 + 3x – 4 and 11. (d)


The expressions (x – 1) (x + 1) and (x – 1) (x – 1)
x3 – 4x + a to obtain remainder and equate. which vanish if x = 1.

a(3)3 + 4(3)2 + 3(3) – 4 = (3)3 – 4(3) + a ⇒ a = –1 12. (b) 10 x = 7.7 or x = 0.7

5. (b) Equate value of polynomial at x = a with a 7


Subtracting, 9x = 7  \  x =
9
a2 – 27a + 196 = a ⇒ a2 – 28a + 196 = 0 ⇒ a = 14
14
2x = = 1.555........ = 1.5
6. (b) Substitute x = 1 in f (x) and x = –2 in g(x), and add 9
f (1) = 2(1) – 6(1) + 4(1) – 5 = –5 ⇒ g(–2) = 3(4) – 9 x2 + 4x + 2 = (x2 + 4x + 2) – 2 = (x + 2)2 – 2
13. (b)
=3
Lowest value = – 2 when x + 2 = 0
f (1) + g(–2) = –2
14. (d) Put a = b in given polynomial. Remainder comes to be
7. (a) x3 – 3x2 – 10x + 24 0.
∵ Last term = (product of roots) 15. (c)
∴ Factorising 24 = 2 × 4 × 3 16. (b) P(x) = ax3 + 4x2 + 3x – 4
Also sum of roots must be “3” P(3) = 27a + 36 + 9 – 4 = 27a + 41
∴ Possible factors are (2, 4, –3) P(x) = x3 – 4x + a; P(3) = 27 – 12 + a = 15 + a
∴ Factorization of x3 – 3x2 – 10x + 24 \ 27a + 41 = 15 + a ⇒ a = –1

= (x – 2) (x + 3) (x – 4) 17. (a) x = – 1 is the root of the quadratic polynomial p(x)

8. (c) We have, p(x) = x2 –10x –75 = x2 – 15x + 5x – 75 So, quadratic polynomial p(x) = k(x + 1)2
p(–2) = k(–2 + 1)2 = 2 ⇒ k = 2 ∴ p(x) = 2(x + 1)2
= x (x – 15) + 5 (x –15) = (x – 15) (x + 5)
Also, p(2) = 2(2 + 1)2 = 2 × 3 × 3 = 18
∴ p(x) = (x –15) (x + 5)
Polynomials M-15
18. (c) x2 – 4 = (x – 2)(x + 2) are the factors 24. (b) Coefficient of all the terms are positive. So, both
roots will be negative.
∴ x = 2, –2 are roots of polynomial
25. (d) In (a) power of x is –1 i.e. negative
∴ at x = 2; P(2) = 2(2)3 + k1(2)2 + k2(2) + 12 = 0
\ (a) is not true.
⇒ 16 + 4k1 + 2k2 + 12 = 0 ⇒ 2k1 + k2 = –14...(i)
at x = 2; P(–2) = 2(–2)3 + k1(–2)2 + k2(–2) + 12 = 0 1
In (b) power of x = , not an integer. \ (b) is not
2
⇒ –16 + 4k1 – 2k2 + 12 = 0 true

⇒ 2k1 – k2 = 2 ...(ii) In (c) Here also power of x is not an integer \ (c) is


not true
From (i) & (ii), k1 = –3 ∴ k1 + k2 = –11
(d) holds [Q all the powers of x are non-negative
19. (b) x = 3 + 32/3 + 31/3 integers.]
3
 2 1 26. (b)
3
(x – 3) =  3 + 33 
3
  (a) is not true [By def.]
(b) holds
x3 – 27 – 9x2 + 27x = 32 + 3 + 3 × 32/3 × 31/3 (32/3 + [Q degree of a zero polynomial is not defined]
1/3
3 ) (c) is not true
x3 – 27 – 9x2 + 27x – 9 – 3 = 9(x – 3) [Q degree of a constant polynomial is ‘0’]
(d) is not true
x3 – 39 – 9x2 + 27x – 9x + 27 = 0
[Q a polynomial of degree n has at most n zeroes].
x3 – 9x2 + 18x – 12 = 0
27. (c)
20. (a) P(x) is a polynomial of degree 3.
1
x2 +
(a) = x 2 + x –1 is not a polynomial since the
1 x
and P(n) = ⇒ n P(n) – 1 = 0
n exponent of variable in 2nd term is negative
n(P(n)) is a polynomial of degree 4 1

\ n P(n) – 1 = k(n – 1)(n – 2)(n – 3)(n – 4) (b) 2 x 2 – 3 x + 1 = 2 x 2 – 3x 2 + 1 is not a polynomial,


since the exponent of variable in 2nd term is a
−1 rational number.
For n = 0; –1 = 24 k ⇒ k =
24
x3 – 3x + 1 is a polynomial.
(c)
−1
For n = 5; 5 × P(5) – 1 = (4)(3)(2)(1) 3
24
2 x 2 – 5 x is also not a polynomial, since the
(d)
⇒ 5 ⋅ P(5) – 1 = – 1 ⇒ P(5) = 0 exponents of variable in 1st term is a rational
number
f(x) = (x – 1)2 + (x – 2)2 + (x – 3)2 + (x – 4)2
21. (d)
2 Hence, (a), (b) and (d) is not a polynomial.
 5
= 4  x −  + 5 28. (d)
 2
5 (a) It is quadratic polynomial
f(x) is minimum at x = = 2.5
2 [∵ the graph meets the x-axis in two points]
22. (b) Since, 2 is the zero of x2 + 3x + k, (b) It is a quadratic polynomial
[∵ the graph meets the x-axis in two points]
\ (2)2 + 3(2) + k = 0  ⇒  k + 10 = 0 ⇒ k = – 10
(c) It is a quadratic polynomial
23. (a) Since –3 is the zero of (k –1) x2 + kx + 1, [∵ the graph meets the x-axis in two points]
\ (k – 1) (–3)2 + k(–3) + 1 = 0 (d) It is a not quadratic polynomial
4 [∵ the graph meets the x-axis in one point]
⇒ 9k – 9 – 3k + 1 = 0 ⇒ 6k – 8 = 0 ⇒ k =
3
M-16 Mathematics
29. (c) a ≠ 0, a, b, c are real numbers x3 – (– 3 – 1 + 2) x2 + ((–3)(–1) + (–1)(2) + (2)(–3))x
30. (d) For roots are equal – (–3)(–1)(2)
b2 – 4ac = 0 x3 + 2x2 + (3 – 2 – 6)x – 6
or D = 0 x3 + 2x2 – 5x – 6
31. (b) For value of k,
44. (b) Reason is true.
1 c c
α. = (Product of roots = ) 1
α a a Let α, be the zeroes of p(x), then
α
8k
1=
2 1 4k 4k
α. = 2 ⇒1= 2
1 α k +4 k +4
k=
or
4
\ k2 – 4k + 4 = 0
32. (c) For x2 + 1 = 0
roots are not real. ⇒ (k – 2)2 = 0 \ k = 2

So, graph of x2 + 1 = 0, neither touches nor intersects \ Assertion is true.


x-axis.
Since, reason is not correct explanation for assertion.
33. (c) We know, for a quadratic polynomial
45. (c) Reason is false [\a polynomial of nth degree has at
k(x2 – (Sum of roots) x + Product of roots)
most x zeroes.]
k(x2 – (–p) + (–1/p))
k (x2 + p – 1/p) Again, x3 + x = x (x2 + 1)
34. (d) Parabola. which has only one real zero (x = 0)
35. (c) a < 0, Graphs look like
[Q x2 + 1 ≠ 0 for all x ∈ R]
open downwards
\ Assertion is true.
36. (c) According to graph, there are two zeros
one at (–2) and 2nd at 4, –2, 4 46. (b)
37. (b) –2, 4 47. (c)
−b ± b 2 − 4ac
38. (b) For zeros D = 48. (d)
2a
Here, a = 4 3, b = 5, c = −2 3 49. (A) → q; (B) → s; (C) → r; (D) → p
−5 ± 25 + 4 × 4 3 × 2 3 −5 ± 11 50. (A) → q ; (B) → s ; (C) → r ; (D) → p
= =
8 3 8 3 (A) 4 – x2 = 0 ⇒ x = ±2
−2 3 x3 – 2x2 = 0 ⇒ x2(x – 2) = 0
(B)
⇒ ,
3 4
x = 0 or x = 2
39. (d) Parabola
(C) 6x2 – 7x – 3 = 0 ⇒ 6x2 – 9x + 2x – 3 = 0 
40. (c) If a > 0, Graph of parabola looks like
41. (d) Here graph cuts x-axis at 3 points 3x(2x – 3) + 1(2x – 3) = 0 ⇒ (3x + 1) (2x – 3) = 0
so it has three zeros. x = 3/2 or x = – 1/3
42. (c) Observing the graph we find –3, –1, 2 as zeros.
x=7
(D)
43. (a) Given zeros are –3, –1, 2, then
Expression is (x – (–3)) (x – (–1)) (x – 2) 51. linear, quadratic, cubic

= (x + 3)(x + 1)(x – 2) 52. x


= x3 + 2x2 – 5x – 6 53. 3
x3 – (Sum of zeros)x2 + (Sum of zeros taking two at a
54. a, a
time)x – (Product of zeros)
Polynomials M-17
55. zero 61. False, a polynomial can have any number of zeroes. It
depends upon the degree of the polynomial.
56. n
62. False, x5 + 1 and – x5 + 2x + 3are two polynomials of
57. True
degree 5 but the degree of the sum of the two polynomials
58. False, because the exponent of the variable is not a whole is 1.
number.  1
63. True, p(3) = 0, p(–1) = 0, p   = 0
3 3
6 x + x2 64. False
59. True, because = 6 + x, which is a polynomial.
x Coefficient of x −( −8)
60. False 65. False, sum of zeroes = = =4
Coefficient of x 2 2
Pair of Linear
3 Acids, Bases and
Equations in Two
Salts
Variables
5. Which of the following is the other name for a pair of
Multiple Choice Questions (MCQs) linear equations in two variables?
(a) Consistent equations (b) Simultaneous equations
DIRECTIONS : This section contains multiple choice
(c) Inconsistent equations (d) Dependent equations
questions. Each question has 4 choices (a), (b), (c) and (d) out
of which only one is correct. 6. The difference between two numbers is 26 and one number
is three times the other. Find them.
1.
For which value of p, will the lines represented by the
(a) 39, 13 (b) 41, 67
following pair of linear equations be parallel
(c) 96, 70 (d) 52, 26
3x – y – 5 = 0
6x – 2y – p = 0 7. Sanjay starts his job with a certain monthly salary and
earns a fixed increment every year. If his salary was
(a) all real values except 10 ` 4500 after four years of service and ` 5400 after 10 years,
(b) 10 find his initial salary and annual increment.
(c) 5/2 (a) 4000, 200 (b) 3900, 150
(d) 1/2 (c) 4500, 100 (d) 3800, 250

2. For what values of k will the following pair of linear 8. The pair of linear equations x + 2y = 5 and 3x + 12y = 10 has
equations have infinitely many solutions? (a) unique solution
kx + 3y – (k – 3) = 0 (b) no solution
12x + ky – k = 0 (c) more than two solutions
k = 4
(a) (b) k = 3 (d) infinitely many solutions
k = 6
(c) (d) k = 2 9. ` 49 was divided among 150 children. Each girl got 50 paise
3.
Which of the following is true if following pair of linear and each boy got 25 paise. How many boys were there?
equations has unique solution? (a) 100 (b) 102
3x – 2y = – 8 (c) 104 (d) 105
(2m – 5)x + 7y – 6 = 0 10. If the sum of the ages (in years) of a father and his son is
11 11 65 and twice the difference of their ages (in years) is 50,
m=
(a) (b) m = – what is the age of the father?
4 4
(a) 45 years (b) 40 years
11 11
m≠–
(c) (d) m≠ (c) 50 years (d) 55 years
4 4
4. The perimeter of a rectangle is 40 cm. The ratio of its sides 11. The value of k for which the system of linear equations
is 2 : 3. Find its length and breadth. x + 2y = 3, 5x + ky + 7 = 0 is inconsistent is
(a) l = 10 cm, b = 8 cm (b) l = 12 cm, b = 8 cm 14 2
(a) − (b)
3 5
l = 12 m, b = 8 m
(c) (d) l = 40 m, b = 30 m
(c) 5 (d) 10
Pair of Linear Equations in Two Variables M-19
12. A can do a piece of work in 24 days. If B is 60% more (a) 4 : 1 (b) 1 : 4
efficient than A, then the number of days required by B to (c) 7 : 1 (d) 1 : 7
do the twice as large as the earlier work is
21. In a number of two digits, unit’s digit is twice the tens digit.
(a) 24 (b) 36
If 36 be added to the number, the digits are reversed. The
(c) 15 (d) 30 number is
13. X’s salary is half that of Y’s. If X got a 50% rise in his (a) 36 (b) 63
salary and Y got 25% rise in his salary, then the percentage (c) 48 (d) 84
increase in combined salaries of both is
1 22. A man can row a boat in still water at the rate of 6 km per
(a) 30 (b) 33 hour. If the stream flows at the rate of 2 km/hr, he takes
3
1 half the time going downstream than going upstream the
(c) 37 (d) 75 same distance. His average speed for upstream and down
2
stream trip is
14. The points (7, 2) and (–1, 0) lie on a line
(a) 6 km/hr
(a) 7y = 3x – 7 (b) 4y = x + 1
(b) 16/3 km/hr
y = 7x + 7
(c) (d) x = 4y + 1
(c) Insufficient data to arrive at the answer
15. At present ages of a father and his son are in the ratio 7 : 3,
and they will be in the ratio 2 : 1 after 10 years. Then the (d) none of the above
present age of father (in years) is 23. A boat travels with a speed of 15 km/hr in still water. In
(a) 42 (b) 56 a river flowing at 5 km/hr, the boat travels some distance
(c) 70 (d) 77 downstream and then returns. The ratio of average speed
to the speed in still water is
16. A fraction becomes 4 when 1 is added to both the numerator
(a) 8 : 3 (b) 3 : 8
and denominator and it becomes 7 when 1 is subtracted
from both the numerator and denominator. The numerator (c) 8 : 9 (d) 9 : 8
of the given fraction is 24. x and y are two non-negative numbers such that 2x + y = 10.
(a) 2 (b) 3 The sum of the maximum and minimum values of (x + y)
is
(c) 5 (d) 15
(a) 6 (b) 9
17. A motor boat takes 2 hours to travel a distance 9 km
down the current and it takes 6 hours to travel the same (c) 10 (d) 15
distance against the current. The speed of the boat in still 25. The average incomes of the people in two villages are P
water and that of the current (in km/hour) respectively and Q respectively. Assume that P ≠ Q. A person moves
are from the first village to the second village. The new average
(a) 3, 1.5 (b) 3, 2 incomes are P’ and Q’ respectively. Which of the following
is not possible?
(c) 3.5, 2.5 (d) 3, 1
P’ > P and Q’ > Q (b)
(a) P’ > P and Q’ < Q
18. The 2 digit number which becomes (5/6)th of itself when
its digits are reversed. The difference in the digits of the P’ = P and Q’ = Q (d)
(c) P’< P and Q’ < Q
number being 1, then the two digits number is 26. The graphs of the equations x – y = 2 and kx + y = 3,
(a) 45 (b) 54 where k is a constant, intersect at the point (x, y) in the
(c) 36 (d) None of these first quadrant, if and only if k is
(a) equal to –1 (b) greater than –1
19. x and y are 2 different digits. If the sum of the two digit
numbers formed by using both the digits is a perfect square, (c) less than 3/2 (d) lying between –1 and 3/2
then value of x + y is 27. For what value of p, the following pair of linear equations
(a) 10 (b) 11 in two variables will have infinitely many solutions ?
(c) 12 (d) 13 px + 3y – (p – 3) = 0, 12x + py – p = 0
20. If 3x + 4y : x + 2y = 9 : 4, then 3x + 5y : 3x – y is equal (a) 6 (b) – 6
to (c) 0 (d) 2
M-20 Mathematics
28. In a classroom, one-fifth of the boys leave the class and 36. If x = a, y = b is the solution of the equations x – y = 2 and
the ratio of the remaining boys to girls is 2 : 3. If further x + y = 4, then the values of a and b are, respectively.
44 girls leave the class, then the ratio of boys to girls is 5: (a) 3 and 5 (b) 5 and 3
2. How many more boys should leave the class so that the
(c) 3 and 1 (d) – 1 and – 3
number of boys equals that of girls?
5
(a) 16 (b) 24 37. I. If x – y = xy = 1 – x – y, then x + y is
3
(c) 30 (d) 36 II. The system of equations 3x + 2y = a and 5x + by
1 1 = 4 has infinitely many solutions for x and y, then
1 1
29. The equations + = 15 and − = 5 are such that a = 4, b = 3
x y x y
x y
ax = 1 and by = 1. The values of ‘a’ and ‘b’ respectively are III. If+ = 2 and ax – by = a2 – b2, then x = a, y = b
a b
(a) 10, 5 (b) 10, –5
Which is true?
(c) –5, 10 (d) 5, 10
(a) I only (b) II only
30. Consider the following two statements: (c) III only (d) None of these.
I. Any pair of consistent linear equations in two variables 38. I. If 3x – 5y = –1 and x – y = – 1, then x = –2, y = –1
must have a unique solution.
II. 2x + 3y = 9, 3x + 4y = 5 ⇒ x = –21, y = 17
II. There do not exist two consecutive integers, the sum
2x y x y
of whose squares is 365. III. + = 2, – = 4 ⇒ x = 2a, y = 2b
a b a b
Then,
Which is true?
(a) both I and II are true (b) both I and II are false
(a) I (b) II
(c) I is true and II is false (d) I is false and II is true
(c) III (d) None of these
31. In village Madhubani 8 women and 12 girls can paint a
39. If a pair of linear equations is inconsistent, then the lines
large mural in 10 hours. 6 women and 8 girls can paint it
will be
in 14 hours. The number of hours taken by 7 women and
14 girls to paint the mural is (a) parallel (b) always coincident
(a) 10 (b) 15 (c) intersecting (d) coincident
(c) 20 (d) 35 40. For what values of k, do the equations 3x – y + 8 = 0 and
32. A boat takes 3 hours to travel 30 km downstream and takes 6x – ky = –16 represent coincident lines?
5 hours to return to the same spot upstream. Find the speed (a) solution of 3k – 9 = 0 (b) solution of 2k – 8 = 0
of the boat in still water. (km/hr)
(c) 2 (d) 3
(a) 10 km/hr (b) 8 km/hr
(c) 6 km/hr (d) 5 km/hr
24
33. The pair of equations 5x – 15y = 8 and 3 x – 9 y = has
5 DIRECTIONS : Study the given Case/Passage and answer the
(a) one solutio
following questions.
(b) two solutions
(c) infinitely many solutions Case/Passage-I
A test consists of ‘True’ or ‘False’ questions. One mark is
(d) no solution
awarded for every correct answer while 1/4 mark is deducted
34. The sum of the digits of a two-digit number is 9. If 27 for every wrong answer. A student knew answers to some of the
is added to it, the digits of the number get reversed. The questions. Rest of the questions he attempted by guessing. He
number is answered 120 questions and got 90 marks.
(a) 25 (b) 72 Type of Question Marks given for Marks deducted for
(c) 63 (d) 36 correct answer wrong answer
True/False 1 0.25
35. The value of c for which the pair of equations cx – y = 2
[From CBSE Question Bank-2021]
and 6x + 2y = 3 will have infinitely many solutions is
41. If answer to all questions he attempted by guessing were
(a) 3 (b) – 3 wrong, then how many questions did he answer correctly?
(c) – 12 (d) no value 42. How many questions did he guess?
Pair of Linear Equations in Two Variables M-21
43. If answer to all questions he attempted by guessing were Refer situation 1
wrong and answered 80 correctly, then how many marks 50. If the fixed charges of auto rickshaw be ` x and the
he got? running charges be ` y km/hr, the pair of linear equations
44. If answer to all questions he attempted by guessing were representing the situation is
wrong, then how many questions answered correctly to (a) x + 10y = 110, x + 15y = 75
score 95 marks?
(b) x + 10y = 75, x + 15y = 110
Case/Passage-II
(c) 10x + y = 110, 15x + y = 75
Amit is planning to buy a house and the layout is given below.
The design and the measurement has been made such that areas (d) 10x + y = 75, 15x + y = 110
of two bedrooms and kitchen together is 95 sq.m. 51. A person travels a distance of 50km. The amount he has
x 2 y to pay is
(a) ` 155 (b) ` 255 (c) ` 355 (d) ` 455
5m Bath
Bedroom 1 room Kitchen Refer situation 2
52. What will a person have to pay for travelling a distance
2m of 30km?
Living Room (a) ` 185 (b) ` 289 (c) ` 275 (d) ` 305
5m Bedroom 2 53. The graph of lines representing the conditions are:
(situation 2)
Y
15 m 25 (20, 25)
[From CBSE Question Bank-2021] 20
Based on the above information, answer the following questions: 15
45. Form the pair of linear equations in two variables from (a) 10
this situation. 5 (0, 5) (30, 5)
46. Find the length of the outer boundary of the layout.
47. Find the area of each bedroom and kitchen in the layout. X¢ –5 0 5 10 15 20 25 30 35 X
–5
48. Find the area of living room in the layout.
–10
49. Find the cost of laying tiles in kitchen at the rate of ` 50 Y¢
per sq.m Y
25
Case/Passage-III
20
It is common that Governments revise travel fares from time to
15
time based on various factors such as inflation ( a general increase
in prices and fall in the purchasing value of money) on different (b) 10 (0, 10) (20, 10)
types of vehicles like auto, rickshaws, taxis, radio cab etc. The auto 5 (12.5, 0)
charges in a city comprise of a fixed charge together with the charge
X¢ –5 0 5 10 15 20 25 30 35 X
for the distance covered. Study the following situations. –5
(5, –10) (25, –10)
–10

50
45
40
35
Name of the city Distance travelled (km) Amount paid (`)
30
City A 10 75
25
15 110
20
City B 8 91 15 (11, 10) (19, 9)
14 145 (47, 7)
(c) 10 (5, 10)
Situation 1: In city A, for a journey of 10 km, the charge paid 5 (27, 8)
is ` 75 and for a journey of 15 km, the charge paid is ` 110. 0 5 10 15 20 25 30 35 40 45 50 55
Situation 2: In a city B, for a journey of 8 km, the charge paid
is ` 91 and for a journey of 14km, the charge paid is ` 145.
[From CBSE Question Bank-2021]
M-22 Mathematics
Y 58. Assertion : If kx – y – 2 = 0 and 6x – 2y – 3 = 0 are
25 inconsistent, then k = 3
20
Reason : a1x + b1y + c1 = 0 and a2x + b2y + c2 = 0 are
15 (15, 15)
(35, 10) a1 b1 c1
(d)
10 inconsistent if = ≠
5 (0, 10) a2 b2 c2
59. Assertion : 3x – 4y = 7 and 6x – 8y = k have infinite
X¢ –5 0 5 10 15 20 25 30 35 X number of solution if k = 14
–5 (15, –5)
–10
Y¢ Reason : a1x + b1y + c1 = 0 and a2x + b2y + c2 = 0 have
a1 b1
a unique solution if ≠
Assertion & Reason a2 b2
60. Assertion : The linear equations x – 2y – 3 = 0 and
DIRECTIONS : Each of these questions contains an Assertion 3x + 4y – 20 = 0 have exactly one solution
followed by Reason. Read them carefully and answer the Reason : The linear equations 2x + 3y – 9 = 0 and
question on the basis of following options. You have to select 4x + 6y – 18 = 0 have a unique solution
the one that best describes the two statements.
Match the Following
(a) If both Assertion and Reason are correct and Reason is
the correct explanation of Assertion.
DIRECTIONS : Each question contains statements given in
(b) If both Assertion and Reason are correct, but Reason is
two columns which have to be matched. Statements (A, B, C, D)
not the correct explanation of Assertion. in column-I have to be matched with statements (p, q, r, s) in
(c) If Assertion is correct but Reason is incorrect. column-II.
(d) If Assertion is incorrect but Reason is correct.
61. Column-II give value of x and y for pair of equation given
54. Assertion : 3x + 4y + 5 = 0 and 6x + ky + 9 = 0 in Column-I.
represent parallel lines if k = 8 Column-I Column-II
Reason : a1x + b1y + c1 = 0 and a2x + b2y + c2 = 0 (A) 2x + y = 8, x + 6y = 15 (p) (3, 4)
(B) 5x + 3y = 35, 2x + 4y = 28 (q) (1/14, 1/6)
a1 b1 c1
represent parallel lines if = ≠ 1 1 1 1
a2 b2 c2 (C) + = 3, − = 5 (r) (4, 5)
7x 6 y 2x 3y
55. Assertion : x + y – 4 = 0 and 2x + ky – 3 = 0 has no solution (D) 15x + 4y = 61 (s) (3, 2)
if k = 2
4x + 15y = 72
Reason : a1x + b1y + c1 = 0 and a2x + b2y + c2 = 0 are 62. Column-I Column-II
a1 b1 (A) 5y – 4x = 14, y – 2x = 1 (p) Infinite solutions
consistent if ≠
a2 b2 (B) 6x – 3y + 10 = 0, (q) Consistent
56. Assertion : If the system of equations 2x + 3y = 7 and 2x – y + 9 = 0
2ax + (a + b) y = 28 has infinitely many solutions, then (C) 3x – 2y = 4, 9x – 6y = 12 (r) No solution
2a – b = 0
(D) 2x – 3y = 8, 4x – 6y = 9 (s) Inconsistent
Reason : The system of equations 3x – 5y = 9 and
63. Column-I Column-II
6x – 10y = 8 has a unique solution.
24
(A) No solution (p) 5x – 15y = 8, 3x – 9y =
57. Assertion : If the pair of lines are coincident, then we 5
say that pair of lines is consistent and it has a unique
solution. (B) Infinitely many solutions (q) 2x + 4y = 10, 3x + 6y = 12

Reason : If the pair of lines are parallel, then the (C) Unique solution (r) x + y = 6, x – y = 2
pair has no solution and is called inconsistent pair of (D) Integral solution (s) 2x + y = 6, 4x – 2y – 2 = 0
equations.
Pair of Linear Equations in Two Variables M-23

Fill in the Blanks True / False

DIRECTIONS : Complete the following statements with an DIRECTIONS : Read the following statements and write your
appropriate word / term to be filled in the blank space(s). answer as true or false.
64. If the lines intersect at a point, then that point gives the 72. If a pair of linear equations is given by a1x + b1y + c1 = 0
unique solution of the two equations. In this case, the pair
a1 b1
of equations is ................. and a2x + b2y + c2 = 0 and ≠ . In this case, the pair
a2 b2
65. If the lines are parallel, then the pair of equations
has no solution. In this case, the pair of equations of linear equations is consistent.
is .............. 73. If a pair of linear equations is given by a1x + b1y + c1 = 0 and
66. Two distinct natural numbers are such that the sum of one a1 b1 c1
a2x + b2y + c2 = 0 and = ≠ . In this case, the pair
number and twice the other number is 6. The two numbers a2 b2 c2
are .............. of linear equations is consistent.
67. If p + q = k, p – q = n and k > n, then q is ................... 74. If a pair of linear equations is given by a1x + b1y + c1 = 0 and
(positive/negative).
a1 b1 c1
68. Sum of the ages of X and Y, 12 years, ago, was 48 years a2x + b2y + c2 = 0 and = = . In this case, the pair
a2 b2 c2
and sum of the ages of X and Y, 12 years hence will be
96 years. Present age of X is .............. of linear equations is consistent.

69. The number of common solutions for the system 75. 3x – y = 3, 9x – 3y = 9 has infinite solution.
of linear equations 5x + 4y + 6 = 0 and 10x + 8y = 12
76. 2x + 3y = 0 , 3 x − 8 y = 0 has no solution.
is .............
77. 3x + 2y = 5, 2x – 3y = 7 are consistent pair of equation.
70. If 2x + 3y = 5 and 3x + 2y = 10, then x – y = ............... .
1 1 1 1 78. In a ∆ ABC, ∠C = 3 ∠B = 2 (∠ A + ∠ B), then angles are
71. If + = k and – = k , then the value of y is ........... 20°, 40°, 100°.
x y x y
M-24 Mathematics

ANSWER KEY & SOLUTIONS


1. (a)
If the lines are parallel, then on solving, x = 12, y = 8
a1 b1 c1 ∴ length = 12 cm and breadth = 8cm.
a = b ≠ c 5. (b) The pair of linear equations in two variables is also
2 2 2
known as simultaneous equations.
Here, a1 = 3, b1 = – 1, c1 = –5,
6. (a) Let the two numbers be x and y (x > y). Then,
a2 = 6, b2 = – 2, c2 = – p x – y = 26 ... (i)
3 −1 −5 x = 3y ... (ii)
⇒ = ≠  ... (i)
6 −2 − p Substituting value of x from equation (ii) in (i)
Taking II and III part of equation (i), we get 3y – y = 26 ⇒ 2y = 26  ⇒  y = 13
1 −5 Substituting value of y in equation (ii), x = 3 × 13 = 39
⇒ ≠ ⇒ − p ≠ −10   ⇒  p ≠ 10
2 −p Thus, two numbers are 13 and 39.
So, option (a) is correct. 7. (b) Let the annual increment be ` y and initial salary be ` x
a1 k b 3 c k −3 ∴ x + 4y = 4500 ... (i)
2. (c)
Here, = , 1 = , 1 =
a2 12 b2 k c2 k x + 10y = 5400
and ... (ii)
For a pair of linear equations to have infinitely many Solving eqs. (i) and (ii), we get
solutions: x = 3900 and y = 150
a1 b1 c1 ∴ Initial salary = ` 3900
= =
a2 b2 c2 and increment = ` 150
k 3 k −3 k 3 8. (a) The pair of linear equations are
So, we have = = or = which gives
12 k k 12 k x + 2y – 5 = 0 and
k2 = 36 i.e., k = ± 6 3x + 12y – 10 = 0
3 k −3 Here, a1 = 1, b1 = 2, c1 = – 5
Also, = gives 3k = k2 – 3k, i.e., 6k = k2,
k k and a2 = 3, b2 = 12,
which means k = 0 or k = 6. a1 1 b1 2 1
c2 = – 10. Now, = , = = .
Therefore, the value of k that satisfies both the a2 3 b2 12 6
conditions, is k = 6. For this value, the pair of linear a1 b1
equations has infinitely many solutions. As ≠
a2 b2
3. (c) For a pair of linear equations having unique solution So, pair of equations has a unique solution.
a1 b1 3 −2
≠ ⇒ ≠ 9. (c) Let the no. of girls be ‘x’ and the no. of boys be ‘y’.
a2 b2 2m − 5 7
Given, 0.50x + 0.25y = 49
or – 4m + 10 ≠ 21
and x + y = 150 ...(i)
or – 4m ≠ 11
x x
11 ⇒ + = 49 ...(ii)
or m ≠ − 2 4
4
From (i) & (ii), x = 46, y = 104
4. (b) Let length and breadth be x cm and y cm respectively.
Hence, number of boys (y) = 104
According to problem,
2 (x + y) = 40 ... (i) 10. (a) Let the age of father be ‘x’ years and the age of son
be ‘y’ years
y 2
and = ... (ii)
x 3 According to question, x + y = 65 ...(i)
Pair of Linear Equations in Two Variables M-25
and 2(x – y) = 50 ⇒ x – y = 25 ...(ii) 9
and time (t2) = = 6 (for up = rate) ... (ii)
Adding eqs. (i) and (ii), we get, 2x = 90 ⇒ x = 45 x− y
Hence, the age of father = 45 years Solving equations (i) & (ii), we get
x = 3 km/hr and y = 1.5 km/hr
1 2 −3 Q For inconsistent 
11. (d) = ≠  
5 k 7  a1 b1 c1  Speed of the boat = 3 km/hr
= ≠
 a2 b2 c2  Speed of the current = 1.5 km/hr
⇒ k = 10
18. (b) If the two digits are x and y, then the number is 10x + y.
12. (d)
Work ratio of A : B = 100 : 160 or 5 : 8
5
∴ time ratio = 8 : 5 or 24 : 15 Given that, (10x + y) = 10y + x. Solving it,
6
x 5
If A takes 24 days, B takes 15 days. Hence, B takes we get 44x + 55y ⇒ = .
30 days to do double the work. y 4
A Also x – y = 1. Solving them, we get x = 5 and y = 4.
13. (b)
Let salary of Y be = A and of X is = Therefore, number is 54.
2
3A 19. (b)
The numbers that can be formed are xy and yx. Hence
∴ Total salary of X and Y = ... (i)
2 (10x + y) + (10y + x) = 11(x + y). If this is a perfect
Let X’ and Y ’ be the new salary after increment, then square then x + y = 11.
we get
3x + 4 y 9
3A 5A 20. (c) =
X ' = and Y ' = ⇒ X '+ Y ' = 2A ... (ii) x + 2y 4
4 4
 2 A − 3A ⇒ 4(3x + 4y) = 9(x + 2y)
  × 100
2  Hence, 12x + 16y = 9x + 18y or 3x = 2y
∴ Required percentage increase =
3A
2
2 \ x= y.
3
[from (i) & (ii) eqns.] 2
1 1 Substitute x = y in the required expression.
= × 100 ⇒ 33 % 3
3 3
i.e. 3 x + 5 y : 3 x − y
14. (b) The point satisfy the line 4y = x + 1 2  2 
= 3  y  + 5 y : 3  y  − y
15. (c) Let the ages of father and son be 7x, 3x 3  3 
After 10 years, = 2y + 5y : 2y – y
\ (7x + 10) : (3x + 10) = 2 : 1 or x = 10 = 7y : y = 7 : 1
\ Age of the father is 7x i.e. 70 years.
21. (c) Let unit’s digit : x, tens digit : y
x
16. (d)
Let the fraction be then x = 2y, number = 10y + x
y
According to given conditions, Also 10y + x + 36 = 10x + y

x +1 ∴ 9x – 9y = 36 or x – y = 4
= 4 ... (i)
y +1 Solve, x = 2y, x–y=4

x −1 Substitute x = 2y in x – y = 4
and = 7 ... (ii)
y −1 we get, 2y – y = 4 ⇒ y = 4
Solving (i) and (ii), we have x = 15, y = 3 and x = 8
i.e. numbers = 15 So, the number = 10y + x = 48
17. (a) Let speed of boat in still water = x km/hr 22. (b)
Upstream speed = 4 km/hr and time = x hrs.
and speed of stream = y km/hr Downstream speed = 8 km/hr and
According to question,
time taken = x/2 hrs.
9 4 x + 8 × x / 2 16
time (t1) = = 2 (for down = rate) ... (i) Hence average speed = = km/hr.
x+ y x+ x/2 3
M-26 Mathematics
23. (c) Let distance = d, Adding (i) and (ii), we have
d d 5
Time taken upstream = = kx + x = 5 ⇒ x (k + 1) = 5 ⇒ x =
15 − 5 10 k +1
d d Putting the value of x in equation (i), we have
Time taken downstream = = 5
15 + 5 20 −y=2
Hence, average speed k + 1

2d 2d × 20 40 5 5 − 2k − 2 3 − 2k
= = = km/hr ⇒ −2= y⇒ = y⇒ y=
d d 3d 3 k + 1 k +1 k +1
+
10 20 y should be positive as they intersect in 1st quadrant
40 Therefore, y > 0
Ratio = :15 = 40 : 45 = 8 : 9
3 3 − 2k 2k − 3
>0⇒ <0
24. (d) Given 2x + y = 10 k + 1 k +1
on adding y both sides, we get, 2x + y + y = 10 + y + – +
y –∞ –1 3 +∞
⇒ 2(x + y) = 10 + y ⇒ x + y = 5 + 2
2
Now, (x + y)max when y is maximum & maximum \ k should lie between – 1 and 3/2
value of y will be 10. (Q y = 10 – 2x) 27. (a) Condition for infinite many solutions.
So (x + y)max = 5 + 5 = 10 & (x + y)min when y = 0 p 3 p – 3  a1 b1 c1 
= =  = = 
\ minimum value of x + y = 5 12 p p  a2 b2 c2 

So, sum of (x + y)max & (x + y)min = 15 p2 = 36 ; p = {From I and II}
25. (c) Let the number of people in first and second village p2 – 3p = 3p {From II and III}
be x and y respectively. p=6
According to given condition, \ p = 6
average income of x people = P and 28. (b) Let the number of boys and girls in classroom is x
and y.
average income of y people = Q, where P ≠ Q
According to question
\ Total income of people in two villages are Px

x −x/5 2 4x 2 x 5
and Qy respectively. = ⇒ = ⇒ =  ...(i)
y 3 5y 3 y 6
When, one person moves from first village to second
village. x − x/5 5 4x 5
Also, = ⇒ =
Then, number of people in first village = x – 1 and in y − 44 2 5 (y − 44) 2
second village = y + 1. ⇒ 8x = 25y – 1100 ...(ii)
New average income = P′ and Q′ From Eqs. (i) and (ii), we get, x = 50, y = 60
(Total income = no. of persons × average income) Let n number of boy leaves the class so number of
boys and number of girls become equal.
\ Total income = P′(x – 1) and Q′(y + 1)
\ 50 – 10 – n = 60 – 44
Total income in both cases are same n = 40 – 16 = 24
\ Px + Qy = P′(x – 1) + Q′(y + 1) 1 1
29. (a) x = ⇒ a = 10 and y = ⇒ b = 5
⇒ Px – P′(x – 1) = Q′(y + 1) – Qy 10 5
⇒ x(P – P′) + P′ = y(Q′ – Q) + Q′ 30. (b) (I) Statement I is false. Consistent Linear equations
\ P′ ≠ P and Q′ ≠ Q may have unique or infinite solutions.
(II) Statement or is also false
Hence, option (c) is not possible.
\ Q 132 + 142 = 365
26. (d)
x – y = 2 .... (i)
31. (a) Let one woman can paint a large mural in W hours
kx + y = 3 .... (ii) and one girl can paint it in G hours
Pair of Linear Equations in Two Variables M-27
According to question, 3 –1 8
40. (c) For coincident lines, = =
8 12 1 2 3 1 6 – k 16
+ = fi + = ...(i)
W G 10 W G 40 1 1
= ⇒ k = 2
6 8 1 3 4 1 2 k
Also, + = fi + = ...(ii)
W G 14 W G 28 Sol. (41-44):
On solving equation (i) and (ii), we get Let x be number of known questions and y be number of
W = 140 and G = 280 questions cheating by the student.
7 14 1 1 Here, x + y = 120
Now, + = = (say)
140 280 Time taken t 1
x − y = 90
4
1 1 1
⇒ = + ⇒ t = 10 hours On solving these two equations
t 20 20
32. (b) Let speed of boat in still water be x km/hr We have, x = 96 and y = 24
and speed of stream be y km/hr 41. No. of correct questions are 96
30 42. He guessed 24 questions.
= 3  ⇒  x + y = 10 …(i)
x+y 1
43. Marks = 80 – of 40 = 70
4
30
= 5  ⇒  x – y = 6 …(ii) 44. Here, x + y = 120 ...(i)
x−y
1
From solving equations (i) and (ii) x − y = 95
4 ...(ii)
–x + y = 10 On solving (i) & (ii) x = 100
–x – y = 6
45. Given area of two bedrooms and a kitchen is 95 sq m.
  +  –
2 × Area of bedroom + Area of kitchen = 95
2y = 4   y = 2 km/hr. and
2 × 5 x + 5y = 95
x = 8 km/hr or 2x + y = 19 ...(i)
a1 b1 c1 5 and x + 2 + y = 15
33. (c) a = b = c = 3
2 2 2 or x + y = 13 ...(ii)
34. (d)
Let x & y be the unit and tenth digits respectively of 46. Length of outer boundary = 12 + 15 + 12 +15 = 54 m
a two digit number. Then,
47. On solving x + y = 13
x + y = 9 (∵ Given) ... (i)
2x + y = 19
and according to given condition,
10x + y = 10 y + x + 27 x = 6m, y = 7m
⇒ 9x – 9y = 27 Area of a bedroom = 5x = 5 × 6 = 30 sq m
⇒ x – y = 3 ... (ii) Area of kitchen = 5y = 5 × 7 = 35 sq m
On adding (i) & (ii) 48. Area of living room = 9 × 5 + 2 × 15 = 75 sq m
2x = 12 ⇒ x = 6 49. Total cost of laying tiles in the kitchen = ` 50 × 35 = ` 1750
Hence, from equation (i), 50. (b) Given, fixed charges of auto rickshaw be ` x and
6 + y = 9 ⇒ y = 3 running charges be ` y km/hr, so representing situation 1
So number will be 10 × 3 + 6 = 36 x + 10y = 75
−c −1 −2 x + 15y = 110
35. (d) For solution to be infinite, = = must satisfy. 51. (c) On solving x + 10y = 75
6 2 −3
−1 2  x + 15y = 110
but ≠ , so, infinite solution don’t exist, for given
2 3 we get x = 5 km,
equations.
 y = ` 7/km
36. (c) On adding both the equations, we get x = 3, y = 1 Charges to go 50 km.
37. (c) 38. (a) 39. (a) x + 50y = 5 + 50 × 7 = ` 355
M-28 Mathematics
52. (b) To cover 30 km distance, 57. (d) Assertion is clearly false.
x + 30y = 19 + 30 × 9 = 289 [Q If the lines are coincident, then it has infinite
53. (c) number of solutions]

54. (a) Reason is true. Reason is clearly true.


In assertion, given lines represent parallel lines if 58. (a) 59. (b) 60. (c)
3 4 5 6×4
= ≠ ⇒k= = 8 \ Reason is also true 61. (A) → (s); (B) → (r); (C) → (q); (D) → (p)
6 k 9 3
Also, reason is the correct explanation for assertion. 62. (A) → (q); (B) → (s); (C) → (p); (D) → (r)
55. (b) Reason is true. 63. (A) → (q); (B) → (p); (C) → (s); (D) → (r)
For assertion, given equation has no solution if 64. consistent 65. inconsistent.
1 1 –4 4 1 4
= ≠ i.e. ⇒ k = 2  ≠ holds  66. 4 and 1 67. positive
2 k –3 3 2 3 
Assertion is true. 68. Cannot be determined 69.
zero
Reason does not give result of assertion.
70. 5 71. Does not exist
56. (c) Assertion: given system of equations has infinitely
many solutions if 72. True 73. False 74. True
2 3 –7
= = 75. True 76. False 77. True
2a a + b –28
1 1 3 1
i.e. ⇒ = = ⇒ 3a = a + b ⇒ 2a – b = 0 78. False
4 a a+b 4
Also clearly a = 4, and a + b = 12 ⇒ b = 8
\ 2a – b = 8 – 8 = 0
\ Assertion is true
3 –5
But reason is false ∵ =
6 –10
[Q3(–10) = (–5)(6) = –30]
a1 b1
For unique solution if a1x + b2y + c2 = 0, then ≠
a2 b2
4 Acids, Bases and
Coordinate
Salts
Geometry
7. If P (x , y) is any point on the line joining the points A (a, 0)
Multiple Choice Questions (MCQs) and B (0, b), then
x y x y
DIRECTIONS : This section contains multiple choice (a) + = 1 (b) − = 1
questions. Each question has 4 choices (a), (b), (c) and (d) out b a a b
of which only one is correct. x y x y
(c) + = 1 (d) − = 1
a b b a
1. If P = (2, 5), Q = (x, –7) and PQ = 13, what is the value of
‘x’? 8. The perimeter of a triangle with vertices (0, 4),
(0, 0) and (3, 0)is
(a) 5 (b 3
(a) 5 (b) 12
(c) –3 (d) –5
(c) 11 (d) 7 + 5
2. The points (a, b), (a1, b1) and (a – a1, b – b1) are collinear
if 9. The point P on x-axis equidistant from the points A(–1, 0)
(a) ab = a1b1 (b) ab1 = a1b and B(5, 0) is
(c) a = b (d) a1 = b1 (a) (2, 0) (b) (0, 2)
(c) (3, 0) (d) (2, 2)
a 
3. If  , 4  is the midpoint of the line segment joining
3  10. The coordinates of the point which is reflection of point
A(–6, 5) and B(–2, 3), then what is the value of ‘a’? (–3, 5) in x-axis are
(a) –4 (b) –12 (a) (3, 5) (b) (3, –5)
(c) 12 (d) –6 (c) (–3, –5) (d) (–3, 5)
4. If the mid point of the line joining (3, 4) and (k, 7) is 11. If the point P(6, 2) divides the line segment joining
(x, y) and 2x + 2y + 1 = 0. Find the value of k. A(6, 5) and B(4, y) in the ratio 3 : 1, then the value
(a) 10 (b) –15 of y is
(c) 15 (d) –10 (a) 4 (b) 3 (c) 2 (d) 1
5. In what ratio is the line segment joining the points (3, 5) 12. P, Q, R are three collinear points. The coordinates of P
& (–4, 2) divided by y–axis? and R are (3, 4) and (11, 10) respectively and PQ is equal
(a) 3 : 2 (b) 3 : 4 to 2.5 units. Coordinates of Q are
(c) 2 : 3 (d) 4 : 3 (a) (5, 11/2) (b) (11, 5/2)
6. In what ratio does the point (–2, 3) divide the line-segment (c) (5, –11/2) (d) (–5, 11/2)
joining the points (–3, 5) and (4, –9) ? 13. C is the mid-point of PQ, if P is (4, x), C is (y, –1) and Q is
(a) 2 : 3 (b) 1 : 6 (–2, 4), then x and y respectively are
(c) 6 : 1 (d) 2 : 1 (a) – 6 and 1 (b) – 6 and 2
(c) 6 and – 1 (d) 6 and – 2
M-30 Mathematics
14. The ratio in which the point (2, y) divides the join of 23. P is a point on the graph of y = 5x + 3. The coordinates of
(– 4, 3) and (6, 3) and hence the value of y is a point Q are (3, –2). If M is the mid point of PQ, then M
must lie on the line represented by
(a) 2 : 3, y = 3 (b) 3 : 2, y = 4
y = 5x + 1
(a) (b) y = 5x – 7
(c) 3 : 2, y = 3 (d) 3 : 2, y = 2
5 7 5 1
15. Ratio in which the line 3x + 4y = 7 divides the line segment y = x –
(c) (d) y = x+
2 2 2 2
joining the points (1, 2) and (–2, 1) is
24. A line l passing through the origin makes an angle q
(a) 3 : 5 (b) 4 : 6 3
with positive direction of x-axis such that sin θ = . The
(c) 4 : 9 (d) None of these 5
coordinates of the point, which lies in the fourth quadrant
16. The point on the X-axis which is equidistant from the points at a unit distance from the origin and on perpendicular to
A(–2, 3) and B(5, 4) is l, are

(a) (0, 2) (b) (2, 0)  3 4  4 3


 ,− 
(a) (b)  ,− 
5 5 5 5
(c) (3, 0) (d) (–2, 0)
(c) (3, –4) (d) (4, –3)
17. The point which divides the line joining the points A (1, 2)
25. The centre of the circle passing through the ponts (6, – 6),
and B(–1, 1) internally in the ratio 1 : 2 is (3, – 7) and (3, 3) is
 –1 5  1 5 (a) (3, 2) (b) (–3, –2)
(a)
 ,  (b)
 , 
 3 3 3 3 (c) (3, – 2) (d) (–3, 2)
(c) (–1, 5) (d) (1, 5) 26. A circle passes through the vertices of a triangle ABC.
18. The centroid of the triangle whose vertices are (3, –7), If the vertices are A(–2, 5), B(–2, –3), C(2, –3), then the
centre of the circle is
(–8, 6) and (5, 10) is
(a) (0, 0) (b) (0, 1)
(a) (0, 9) (b) (0, 3)
(c) (–2, 1) (d) (0, –3)
(c) (1, 3) (d) (3, 5)
27. Which of the following points is 10 units from the origin?
19. The points A (– 4, – 1), B (–2, – 4), C (4, 0) and D (2, 3)
(a) (– 6, 8) (b) (– 4, 2)
are the vertices of a
(c) (– 6, 5) (d) (6, 4)
(a) Parallelogram (b) Rectangle
28. The distance between which of the following two points
(c) Rhombus (d) Square is 2 units?
20. If the point P (p, q) is equidistant from the points (a) (–2, –3) and (–2, –4) (b) (0, 4) and (0, 6)
A (a + b, b – a) and B (a – b, a + b), then (c) (7, 2) and (6, 2) (d) (4, –3) and (2, 3)
ap = by
(a) (b) bp = ay 29. Which of the following is / are not correct ?
ap + bq = 0
(c) (d) bp + aq = 0 Three points will form :
21. The distances of a point from the x-axis and the y-axis (a) an equilateral triangle, if all the three sides are equal.
are 5 and 4 respectively. The coordinates of the point (b) an isosceles triangle, if any two sides are equal.
can be (c) a collinear or a line, if sum of two sides is equal to
third side.
(a) (5, 4) (b) (5, 0)
(d) a rhombus, if all the four sides are equal.
(c) (0, 4) (d) (4, 5)
30. Which of the following is / are correct?
22. If the points (a, 0), (0, b) and (1, 1) are collinear then which
of the following is true : Four points will form :

1 1 1 1 (a) a rectangle, if opposite sides and diagonals are not equal.


(a) + = 2 (b) − = 1
a b a b (b) a parallelogram, if opposite sides are not equal.
1 1 1 1 (c) a square, if all the four sides and diagonals are equal.
(c) − = 2 (d) + = 1
a b a b (d) a right angle triangle, if sum of squares of any two
sides is equal to square of third largest side.
Coordinate Geometry M-31

Assertion & Reason

DIRECTIONS : Study the given Case/Passage and answer the DIRECTIONS : Each of these questions contains an Assertion
following questions. followed by Reason. Read them carefully and answer the
question on the basis of following options. You have to select
Case/Passage-I the one that best describes the two statements.
In order to conduct Sports Day activities in your School, (a) If both Assertion and Reason are correct and Reason is
lines have been drawn with chalk powder at a distance the correct explanation of Assertion.
of 1 m each, in a rectangular shaped ground ABCD, 100
(b) If both Assertion and Reason are correct, but Reason is
flowerpots have been placed at a distance of 1 m from
not the correct explanation of Assertion.
each other along AD, as shown in given figure below.
Niharika runs 1/4 th the distance AD on the 2nd line and (c) If Assertion is correct but Reason is incorrect.
posts a green flag. Preet runs 1/5 th distance AD on the (d) If Assertion is incorrect but Reason is correct.
eighth line and posts a red flag.
[From CBSE Question Bank 2021] 36. Assertion : If A(2a, 4a) and B(2a, 6a) are two vertices of
D C an equilateral triangle ABC then, the vertex C is given by
(2a + a 3,5a) .

Reason : In equilateral triangle, all the coordinates of three


G
R vertices can be rational.

37. Assertion : The points (k, 2 – 2k), (– k+ 1, 2k) and

1
2
(– 4 – k, 6 – 2k) are collinear if k = .
1 2
A
1 2 3 4 5 6 7 8 9 10
Reason : Three points A, B and C are collinear in same
straight line, if AB + BC = AC.
31. Find the position of green flag
38. Assertion : Mid-point of a line segment divides line in the
(a) (2, 25) (b) (2, 0.25)
ratio 1 : 1.
(c) (25, 2) (d) (0, –25)
Reason : If area of triangle is zero that means points are
32. Find the position of red flag
collinear.
(a) (8, 0) (b) (20, 8)
(c) (8, 20) (d) (8, 0.2) Match the Following
33. What is the distance between both the flags?
(a) √41 (b) √11 DIRECTIONS : Each question contains statements given in
(c) √61 (d) √51 two columns which have to be matched. Statements (A, B, C, D)
in column-I have to be matched with statements (p, q, r, s) in column-II.
34. If Rashmi has to post a blue flag exactly halfway between
the line segment joining the two flags, where should she
39. Column-II gives distance between pair of points given in
post her flag?
Column-I
(a) (5, 22.5) (b) (10, 22)
Column-I Column-II
(c) (2, 8.5) (d) (2.5, 20)
(A) (–5, 7), (–1, 3) (p) 17
35. If Joy has to post a flag at one-fourth distance from green
flag ,in the line segment joining the green and red flags, (B) (5, 6), (1, 3) (q) 8
then where should he post his flag?
( 3 + 1,1), (0, 3) (r) 6
(C)
(a) (3.5, 24) (b) (0.5, 12.5)
(c) (2.25, 8.5) (d) (25, 20) (0,0) (− 3, 3) (s)
(D) 4 2
M-32 Mathematics
40. Column-II gives the coordinates of the point P that divides 48. (1, 2), (4, y), (x, 6) and (3, 5) are the vertices of a
the line segment joining the points given in Column-I. parallelogram taken in order, then the value of x and y are
Column-I Column-II ...........

A (–1, 3) and
(A) (p) (7, 3) 49. Relation between x and y if the points (x, y), (1, 2) and
(7, 0) are collinear is ...........
B (5, –6) internally
in the ratio 1 : 2 50. The distance of the point (x1, y1) from the origin is ............

A (–2, 1) and
(B) (q) (0, 3)
True / False
B (1, 4) internally
in the ratio 2 : 1 DIRECTIONS : Read the following statements and write your
A (–1, 7) and
(C) (r) (1, 3) answer as true or false.
B (4, –3) internally
51. The distance between P (x1, y1) and Q (x2, y2) is
in the ratio 2 : 3
A (4, –3) and
(D) (s) (1, 0) ( x2 + x1 )2 + ( y2 + y1 )2

B (8, 5) internally 52. The coordinates of the point P(x, y) which divides the
in the ratio 3 : 1 line segment joining the points A(x1, y1) and B(x2, y2)
internally in the ratio m1 : m2 are
Fill in the Blanks  m1 x2 − m2 x1 m1 y2 − m2 y1 
 m + m , m + m 
1 2 1 2
DIRECTIONS : Complete the following statements with an
appropriate word / term to be filled in the blank space(s). 53. The mid-point of the line segment joining the points

41. Points (3, 2), (–2, –3) and (2, 3) form a ............. triangle. x +x y + y 
P (x1, y1) and Q (x2, y2) is  1 2 , 1 2  .
 2 2 
42. If x – y = 2, then point (x, y) is equidistant from (7, 1) and 54. Points (1, 7), (4, 2), (–1, –1) and (– 4, 4) are the vertices
(.........) of a square.
43. Distance between (2, 3) and (4, 1) is .............. 55. Coordinates of the point which divides the join of (–1, 7)
and (4, –3) in the ratio 2 : 3 is (1, 3).
44. Points (1, 5), (2, 3) and (– 2, – 11) are ...........
56. Ratio in which the line segment joining the points (– 3, 10)
45. (5, – 2) (6, 4) and (7, – 2) are the vertices of an ..............
and (6, – 8) is divided by (– 1, 6) is 3 : 7.
triangle.
57. The ratio in which the point (3, 5) divides the join of
46. Point on the X-axis which is equidistant from (2, –5) and (1, 3) and (4, 6) is 2 : 1.
(–2, 9) is .............
58. The distance of the point (5, 3) from the X-axis is 5 units
47. Point (– 4, 6) divide the line segment joining the points
A(– 6, 10) and B(3, – 8) in the ratio .............. 59. The distance of a point (2, 3) from Y-axis is y-units.
Coordinate Geometry M-33

ANSWER KEY & SOLUTIONS


1. (c) PQ = 13 ⇒ PQ2 = 169 ∴ Its x-cordinates = 0

⇒ (x – 2)2 + (–7 – 5)2 = 169 −4 K + 3


∴ = 0 ⇒ −4 K + 3 = 0
K +1
⇒ x2 – 4x + 4 + 144 = 169
3
⇒ x2 – 4x – 21 = 0 ⇒K =
4
⇒ x2 – 7x + 3x – 21 = 0 3
⇒ Required ratio = :1
⇒ (x – 7) (x + 3) = 0 4

⇒ x = 7, –3 ∴ ratio = 3 : 4

2. (b) We have, 6. (b) Suppose the required ratio is m1 : m2


a(b1 – b + b1) + a1(b – b1 – b) + (a – a1) (b – a1) (b – b1) = 0 Then, using the section formula, we get
⇒ 2ab1 – ab – a1b1 + ab – ab1 – a1b + a1b1 = 0 m1 (4) + m 2 ( –3)
–2 =
⇒ ab1 – a1b = 0 m1 + m 2

⇒ ab1 = a1b. ⇒ – 2m1 – 2m 2 = 4m1 – 3m 2

3. (b) Coordinates of mid-point are given by ⇒ m2 = 6 m1 ⇒ m1 : m2 =1:6


 x1 + x 2 y1 + y 2 
 ,  7. (c) As the point P (x, y) lies on the line joining the points
 2 2 
A (a, 0) and B (0, b), the points A, B and P are collinear
a  ⇒ a (b – y) + 0 (y – 0) + x (0 – b) = 0
Here, coordinates of mid-point are  , 4 
3 
a −6 − 2 ⇒ ab – ay – bx = 0 ⇒ bx + ay = ab
So, =
3 2 x y
⇒ + =1
∴ a = – 12 a b
4. (b) Since (x, y) is midpoint of (3, 4) and (k, 7) 8. (b) A(0, 4), B(0, 0), C(3, 0)

3+ k 4+7
∴ x= and y = AB = (0 − 0)2 + (0 − 4)2 = 4
2 2
Also 2x + 2y + 1 = 0 putting values we get BC = (3 − 0)2 + (0 − 0)2 = 3
3+k+4+7+1=0
CA = (0 − 3)2 + (4 − 0)2 = 5
⇒ k + 15 = 0 ⇒ k = – 15
AB + BC + CA = 12
5. (b) Let the required ratio be K : 1
 5 −1 
∴ The coordinates of the required point on the y-axis is 9. (a) P(x, 0) =  , 0  = (2, 0)
 2 
K(−4) + 3(1) K(2) + 5(1) [Q A and B both lies on x-axis]
x= ; y=
K +1 K +1
Three or more points lies in same line are called collinear.
Since, it lies on y-axis
M-34 Mathematics
10. (c) For reflection of a point with respect to x-axis change 19. (b)
sign of y-coordinate and with respect to y-axis change sign
20. (b)
of x-coordinate.
21. (d) 4
P (4, 5)
 4 × 3 + 1× 6 3 × y + 1× 5 
11. (d) P(6, 2) =  , 
 3 +1 3 +1  5
18
Q 6 ≠  (Question is wrong)
4
22. (d) As (a, 0), (0, b) and (1, 1) are collinear
3y + 5
2=   ⇒ 3y + 5 = 8 \ a(b – 1) + 0(1 – 0) + 1(0 – b) = 0
4
3y = 3  ⇒  y = 1 ab – a – b = 0

12. (a) ab = a + b

13. (a) Since, C (y, – 1) is the mid-point of P (4, x) and Q (–2, 4). 1 1
1= +
a b
23. (b) Let coordinate of point p be (h, 5h + 3)
P(h, 5h + 3)
y = 5x + 3
4−2 4+ x
We have, = y and = −1
2 2
∴ y = 1 and x = – 6 M

14. (c) Let the required ratio be k : 1


Q (3, –2)
6k − 4(1) 3
Then, 2 = or k =
k +1 2 Since, M is the mid-point of PQ, therefore by mid-point
3  h + 3 5h + 3 − 2 
∴ The required ratio is :1 or 3 : 2 formula, we have M =  ,
2  2  .
2
3(3) + 2(3)
Also, y = =3
3+ 2 Clearly by observing the options, we can say that M must
3(1) + 4(2) − 7 4 4 lie on the line
15. (c) − = − =
3(−2) + 4(1) − 7 −9 9 y = 5x – 7
16. (b) Let P (x, 0) be a point on X-axis such that AP = BP
A
⇒ AP2 = BP2
⇒ (x + 2)2 + (0 – 3)2 = (x – 5)2 + (0 + 4)2
24. (a)
⇒ x2 + 4x + 4 + 9 = x2 – 10x + 25 + 16 90 – θ θ
B
⇒ 14x = 28 ⇒ x = 2 O 90 – θ
y
Hence, required point is (2, 0).

17. (b) x
C
x + x + x y + y + y3 
18. (b) Centroid is  1 2 3 , 1 2 
 3 3  ∠AOB = q
3 + (–8) + 5 –7 + 6 + 10   0 9 
i.e.  ,  =  3 , 3  = (0, 3) Q CO ^ OA
 3 3   
Coordinate Geometry M-35
\ ∠BOC = (90° – q)
⇒ ( x + 2) 2 + ( y + 3) 2 = ( x - 2) 2 + ( y + 3) 2
3 4 
sin θ = cos θ = Q cos θ = 1 −sin 2 θ  ⇒ x2 + 4 + 4x + (y + 3)2 = x2 + 4 – 4x + (y + 3)2
5; 5 
⇒ 8x = 0 ⇒ x = 0
−y
Now, sin(90° − θ) =
1 \ centre of the circle is (0, 1).

−4 27. (a)
⇒ y = – cos q  ⇒  y =
5
28. (b)
x 3
cos(90° – q) =   ⇒  x = sin q, x = 29. (d) 
All the statements given in option ‘a’, ‘b’ and ‘c’ are
1 5
correct.
25. (c) (x – 6)2 + (y + 6)2 = (x – 3)2 + (y + 7)2...(i) 30. (c)

Also, (x – 3)2 + (y – 3)2 = (x – 3)2 + (y + 7)2  1 


31. (a) (2, 25) ∵ x = 2, y = 4 × 100 = 25
 
y2 – 6y + 9 = y2 + 14y + 49
 1 
– 20y = 40 ⇒ y = – 2 32. (c) (8, 20) ∵ x = 8, y = 5 × 100 = 20 
 

Putting y = – 2 in equation (i), we have 33. (c) (8 − 2) 2 + (25 − 20) 2 = 36 + 25 = 61


(x – 6)2 + (4)2 = (x – 3)2 + (5)2
 8 + 2 25 + 20 
34. (a)  ,  = (5, 22.5)
x2 – 12x + 36 + 16 = x2 – 6x + 9 + 25 2 2 

 2 + 5 25 + 22.5 
–6x = – 18 ⇒ x = 3 35. (a)  ,  = (3.5, 24)
 2 2 
26. (b) A (–2, 5)
36. (c) Let A(x1, y1), B(x2, y2) & C(x3, y3) are all rational
coordinates of a triangle ABC.

x1 y1 1
O 1
(x, y) ar (∆ ABC) = x2 y2 1
2
x3 y3 1
B C 3
(–2, 3) (2, –3) = [(x1 – x2)2 + (y1 – y2)2 ]
4
LHS = rational, RHS = irrational
Let O(x, y) is the centre of the given circle.
Hence, (x1, y1) (x2, y2) & (x3, y3) cannot be all rational.
Join OA, OB & OC.
37. (a) Both assertion and reason are correct. Reason is correct
Q OA = OB = OC
explanation of assertion.
\ OA2 = OB2
38. (b) Both statements are individually correct.

⇒ ( x + 2) 2 + ( y - 5) 2 = ( x + 2) 2 + ( y + 3) 2 39. (A) → (s); (B) → (p); (C) → (q); (D) → (r)

⇒ x2 + 4 + 4x + y2 + 25 – 10y = x2 + 4 + 4x + y2 + 9 + 6x 40. (A) → (s); (B) → (q); (C) → (r); (D) → (p)
41. right angle
⇒ 16y = 16 ⇒ y = 1
42. (3, 5)
Again: OB2 = OC2
M-36 Mathematics
51. False
43. 2 2
52. False
44. Non-collinear
53. True
45. isosceles
54. True
46. (–7, 0)
55. True
47. 2 : 7
56. False
48. (6, 3)
57. True
49. x + 3y = 7
58. False
50. x12 + y12
59. False
5 Acids, Bases and
Triangles
Salts
A
Multiple Choice Questions (MCQs)

DIRECTIONS : This section contains multiple choice


questions. Each question has 4 choices (a), (b), (c) and (d) out
of which only one is correct

1. In the given figure, AD is the bisector of ∠A. If BD = 4 cm, 70° 50°


DC = 3 cm and AB = 6 cm, determine AC B D C
A (a) 30° (b) 40°
(c) 50° (d) 45°

6 cm 5. If ∆ABC ~ ∆DEF such that BC = 2.1cm and EF = 2.8 cm. If


the area of triangle DEF is 16 cm2, then the area of triangle
ABC (in sq. cm) is
(a) 9 (b) 12
B C (c) 8 (d) 13
4 cm D 3 cm
6. From the given figure, then length of the sides AB and BD.
(a) 4.5 cm (b) 3.5 cm A
(c) 4.8 cm (d) 3.2 cm
24 cm
2. ∆ABC is an isosceles triangle right angled at B. 20 cm
Similar triangles ACD and ABE are constructed on D
sides AC and AB. Ratio between the areas of ∆ABE
B
and ∆ACD is C 15 cm

(a) 1 : 4 (b) 2 : 1 (a) 25 cm and 7 cm (b) 25 cm and 17 cm


(c) 1 : 2 (d) 4 : 3 (c) 7 cm and 15 cm (d) 18 cm and 7 cm
3. Two isosceles triangles have their corresponding angles 7. The diagonal BD of a parallelogram ABCD intersects the
equal and their areas are in the ratio 25 : 36. The ratio of segment AE at the point F, where E is any point on the side
their corresponding height is BC. Then
D C
(a) 25 : 35 (b) 36 : 25
4
(c) 5 : 6 (d) 6 : 5
AB BD E
4. In ∆ ABC, = , ∠B = 70° and ∠C = 50°. Then, 1 F 2
AC DC
3
∠BAD = _______.
A B
M-38 Mathematics
EF FB 15. The perimeters of two similar triangles ABC and PQR are
(a)  (b) DF × EF = FB × FA respectively 36 cm and 24 cm. If PQ = 10 cm, then AB =
FA AB
(a) 10 cm (b) 20 cm
(c) DF × EF = (FB)2 (d) None of these
(c) 25 cm (d) 15 cm
8. ABC is an isosceles triangle in which AB = AC = 10 cm,
16. In the given figure, DE || BC. The value of EC is
BC = 12 cm. PQRS is a rectangle inside the isosceles
A
triangle. Given PQ = SR = y cm and PS = QR = 2x cm, cm 1 cm
.5
then x = D1 E
3y cm
(a) 6 − (b) 6 + 6y 3
4
4y 7x + 8y B C
(c) 6 + (d)
3 4 (a) 1.5 cm (b) 3 cm
9. If ABC and EBC are two equilateral triangles such that D (c) 2 cm (d) 1 cm
is mid-point of BC, then the ratio of the areas of triangles 17. In the given figure, express x in terms of a, b and c.
ABC and BDE is L
(a) 2 : 1 (b) 1 : 2 (c) 1 : 4 (d) 4 : 1
10. Given ∆ABC ~ ∆DEF, if AB = 2DE and area of ∆ABC is a P
56 cm2, find the area of ∆DEF. x
(a) 14 sq.cm (b) 5 sq.cm 46° 46°
M N K
(c) 18 sq.cm (d) 56 sq.cm b c
11. The area of a right angled triangle is 40 sq. cm. and its ab ac
(a) x = (b) x =
perimeter is 40 cm. The length of its hypotenuse is a+b b+c
(a) 16 cm (b) 18 cm bc ac
(c) x = (d) x =
(c) 17 cm (d) Data insufficient b+c a+c
12. In the given figure, P and Q are points on the sides AB and 18. Which of the following statement is false?
AC respectively of a triangle ABC. PQ is parallel to BC (a) All isosceles triangles are similar.
and divides the triangle ABC into 2 parts, equal in area. (b) All quadrilateral triangles are similar.
The ratio of PA : AB = (c) All circles are similar.
A (d) None of the above
19. If ∆ABC ~ ∆APQ and ar (∆APQ) = 4 ar (∆ABC), then the
P Q ratio of BC to PQ is
(a) 2 : 1 (b) 1 : 2
B C
(c) 1 : 4 (d) 4 : 1
(a) 1 : 1 (b) ( 2 − 1) : 2
20. The areas of two similar triangles ABC and PQR are in the
(c) 1: 2 (d) ( 2 − 1) :1 ratio 9 : 16. If BC = 4.5 cm, then the length of QR is
(a) 4 cm (b) 4.5 cm
BC 1
13. It is given that ∆ABC ~ ∆PQR with = . Then (c) 3 cm (d) 6 cm
QR 3
ar(∆PQR) 21. The length of the side of a square whose diagonal is 16 cm, is
is equal to
ar(∆ABC )
(a) 8 2 cm (b) 2 8 cm
1 1
(a) 9 (b) 3 (c) (d) (c) 4 2 cm (d) 2 2 cm
3 9
22. ∆ABC is an equilateral triangle with each side of length
14. The area of a right angled isosceles triangle whose
2p. If AD ⊥ BC, then the value of AD is
hypotenuse is equal to 270 m is-
(a) 3 (b) 3p
(a) 19000 m2 (b) 18225 m2
(c) 2p (d) 4p
(c) 17256 m2 (d) 18325 m2
Triangles M-39
23. The areas of two similar triangles are 81 cm2 and 49 cm2 29. Let D be a point on the side BC of a triangle ABC such
respectively, then the ratio of their corresponding medians that ∠ADC = ∠BAC. If AC = 21 cm, then the side of an
is equilateral triangle whose area is equal to the area of the
(a) 7 : 9 (b) 9 : 81 rectangle with sides BC and DC is
1/2 –1/2
(a) 14 × 3 (b) 42 × 3
(c) 9 : 7 (d) 81 : 7 3/4 1/2
(c) 14 × 3 (d) 42 × 3
24. In the figure, ABC is a triangle in which AD bisects
30. In a triangle ABC, ∠BAC = 90°; AD is the altitude from
∠A, AC = BC, ∠B = 72° and CD = 1cm. Length of BD
A on to BC. Draw DE perpendicular to AC and DF
(in cm) is
perpendicular to AB. Suppose AB = 15 and BC = 25.
C
Then the length of EF is
(a) 12 (b) 10
(c) 5 3 (d) 5 5
D 31. If ∆ABC is an equilateral triangle such that AD ­⊥ BC, then
AD2 =
3a 2 3a 2
A. B.
A B 4 2
1 3 3
(a) 1 (b) C. BC2 D. a
2 4 2
(a) A and C (b) A
5 –1 3 +1
(c) (d)
2 2 (c) D (d) B and C
25. Let P be an interior point of a DABC. Let Q and R be the 32. Which among the following is/are correct?
reflections of P in AB and AC, respectively. If Q, A, R are (a) The ratios of the areas of two similar triangles is equal
collinear, then ∠A equals to the ratio of their corresponding sides.
(a) 30° (b) 60° (b) The areas of two similar triangles are in the ratio of
(c) 90° (d) 120° the corresponding altitudes.
26. Let ABC be a triangle and M be a point on side AC closer (c) The ratio of area of two similar triangles are in the
to vertex C than A. Let N be a point on side AB such that ratio of the corresponding medians.
MN is parallel to BC and let P be a point on side BC such (d) If the areas of two similar triangles are equal, then the
that MP is parallel to AB. If the area of the quadrilateral triangles are congruent.
5
BNMP is equal to of the area of DABC, then the ratio 33. Which among the following is/are correct?
AM/MC equals 18 (I) If the altitudes of two similar triangles are in the ratio
(a) 5 (b) 6 2 : 1, then the ratio of their areas is 4 : 1.
18 15 (II) PQ || BC and AP : PB = 1 : 2.
(c) (d)
5 2 area ( ∆APQ )1
Then, =
27. In DABC, AB = AC, P and Q are points on AC and area ( ∆ABC )4
AB respectively such that BC = BP = PQ = AQ. Then, (III) The areas of two similar triangles are respectively
∠AQP is equal to (use p =180º) 9cm2 and 16cm2. The ratio of their corresponding
2π 3π sides is 3 : 16.
(a) (b)
7 7 (a) I (b) II
4π 5π (c) III (d) None of these
(c) (d)
7 7 34. In a right angled triangle ∆ABC, length of two sides are
28. Consider a DPQR in which the relation QR2 + PR2 = 5 8cm and 6cm, then which among the given statements is/
PQ2 holds. Let G be the points of intersection of medians are correct?
PM and QN. Then ∠QGM is always A
(a) less than 45°
(b) obtuse
(c) a right angle
B C
(d) acute and larger than 45°
M-40 Mathematics
(a) Length of greatest side is 10cm 39. What is the height of Ajay’s house?
(b) ∠ACB = 45° (a) 30m (b) 40m
(c) ∠BAC = 45° (c) 50m (d) 20m
(d) Pythagoras theorem is not applicable here. 40. When the tower casts a shadow of 40m, same time what
35. Two triangles are similar if will be the length of the shadow of Ajay’s house?
(a) their corresponding angles are equal. (a) 16m (b) 32m
(b) their corresponding sides are equal. (c) 20m (d) 8m
(c) both are right triangle. 41. When the tower casts a shadow of 40m, same time what
(d) None of the above will be the length of the shadow of Vijay’s house?
36. Which of the following is/are not correct? (a) 15m (b) 32m
(a) If the diagonals of a quadrilateral divide each other (c) 16m (d) 8m
proportionally, then it is a trapezium.
(b) The line segments joining the mid-points of the Case/Passage-II
adjacent sides of a quadrilateral form a parallelogram.
Rohan wants to measure the distance of a pond during the visit
(c) If corresponding sides of two similar triangles are to his native. He marks points A and B on the opposite edges of a
in the ratio 4 : 5, then corresponding medians of the
pond as shown in the figure below. To find the distance between the
triangles must be in the ratio 4 : 5.
points, he makes a right-angled triangle using rope connecting B
(d) None of the above with another point C are a distance of 12m, connecting C to point D
at a distance of 40m from point C and the connecting D to the point
A which is are a distance of 30m from D such the ∠ADC=90° .
[From CBSE Question Bank-2021]
DIRECTIONS : Study the given Case/Passage and answer the B 12 m
following questions. A C

Case/Passage-I
Vijay is trying to find the average height of a tower near his
30

m
m

40
house. He is using the properties of similar triangles.The height
of Vijay’s house if 20m when Vijay’s house casts a shadow 10m
long on the ground. At the same time, the tower casts a shadow D
50m long on the ground and the house of Ajay casts 20m shadow
on the ground. [From CBSE Question Bank-2021] 42. Which property of geometry will be used to find the
distance AC?
(a) Similarity of triangles (b) Thales Theorem
(c) Pythagoras Theorem (d) Area of similar triangles
43. What is the distance AC?
(a) 50m (b) 12m
(c) 100m (d) 70m
Vijay's House Tower 44. Which is the following does not form a Pythagoras triplet?
Ajay's House
(a) (7, 24, 25) (b) (15, 8, 17)
37. What is the height of the tower? (c) (5, 12, 13) (d) (21, 20, 28)
(a) 20m (b) 50m 45. Find the length AB?
(c) 100m (d) 200m (a) 12m (b) 38m
38. What will be the length of the shadow of the tower when (c) 50m (d) 100m
Vijay’s house casts a shadow of 12m? 46. Find the length of the rope used.
(a) 75m (b) 50m (a) 120m (b) 70m
(c) 45m (d) 60m (c) 82m (d) 22m
Triangles M-41
51. In figure, the line segment XY is parallel to the side AC of
Assertion & Reason ∆ABC and it divides the triangle into two parts of equal
areas, then,
DIRECTIONS : Each of these questions contains an Assertion A
followed by reason. Read them carefully and answer the question
on the basis of following options. You have to select the one that
X
best describes the two statements.
(a) If both Assertion and Reason are correct and Reason is
the correct explanation of Assertion.
(b) If both Assertion and Reason are correct, but Reason is
B Y C
not the correct explanation of Assertion.
(c) If Assertion is correct but Reason is incorrect. Column-I   Column-II
(d) If Assertion is incorrect but Reason is correct. (A) AB : XB (p)  2 :1
47. Assertion : If in a ∆ABC, a line DE || BC, intersects AB (B) ar (∆ ABC) : ar (∆ XBY) (q)  2 : 1
AB AC (C) AX : AB (r)  ( 2 − 1) 2 : 2
in D and AC in E, then = .
AD AE (D) ∠ X : ∠ A (s)  1 : 1
Reason : If a line is drawn parallel to one side of a triangle 52. Column-I Column-II
intersecting the other two sides, then the other two sides A
are divided in the same ratio.
(A) (p) 36 : 49
48. Assertion : ABC is an isosceles, right triangle, right angled
at C. Then AB2 = 3AC2.
C B
Reason : In an isosceles triangle ABC if AC = BC and ABC is an isosceles
AB2 = 2AC2, then ∠C = 90°. right angled triangle.
49. Assertion : ABC and DEF are two similar triangles such AB2 =?
that BC = 4 cm, EF = 5 cm and area of ∆ABC = 64 cm2, (B) ∆ABC ~ ∆DEF, (q) AB2 = 2AC2
then area of ∆DEF = 100 cm2.
such that
Reason : The areas of two similar triangles are in the ratio
AB = 1.2 cm and
of the squares of the corresponding altitudes.
DE = 1.4 cm
Match the Following area ( ∆ABC )
= ?
area ( ∆DEF )
DIRECTIONS : Each question contains statements given in (C) ∆ABC ~ ∆APQ and (r) 36 : 49
two columns which have to be matched. Statements (A, B, C, D) area ( ∆APQ ) 36
=
in column-I have to be matched with statements (p, q, r, s) in area ( ∆ABC ) 49
column-II.
BC
= ?
50. If in a ∆ ABC, DE || BC and intersects AB in D and AC in PQ
E, then. A
Column-I Column-II
AD AC (D) D E (s) 6 : 7
(A) (p)
DB AE A
AB
(B) (q) AE B C
AD EC If DE || BC and
AE AD 6
(C) DB
D E
(r) =
AB AC DB 7

AD EC B C AE 2
(D) (s) then, =?
AB AC EC 2
M-42 Mathematics

Fill in the Blanks True / False

DIRECTIONS : Complete the following statements with an DIRECTIONS : Read the following statements and write your
appropriate word / term to be filled in the blank space(s). answer as true or false.
53. All circles are .................. 66. Two figures having the same shape but not necessarily the
54. All squares are .................. same size are called similar figures.
55. All ........... triangles are similar. 67. All the congruent figures are similar but the converse is
56. Two polygons of the same number of sides are similar, not true.
if their corresponding angles are ......... and their 68. If in two triangles, corresponding angles are equal, then
corresponding sides are in the same ........... their corresponding sides are in the same ratio and hence
57. If a line is drawn parallel to one side of a triangle to intersect the two triangles are similar.
the other two sides in distinct points, then the other two
69. If in two triangles, two angles of one triangle are
sides are divided in the ............. ratio.
respectively equal to the two angles of the other triangle,
58. If a line divides any two sides of a triangle in the same then the two triangles are similar.
ratio, then the line is parallel to the .............. side.
70. If in two triangles, corresponding sides are in the same
59. All congruent figures are similar but the similar figures
ratio, then their corresponding angles are equal and hence
need ............. be congruent.
the triangles are similar.
60. Two polygons of the same number of sides are similar, if
71. If one angle of a triangle is equal to one angle of another
all the corresponding angles are .............
triangle and the sides including these angles are in the
61. The diagonals of a quadrilateral ABCD intersect each other same ratio (proportional), then the triangles are similar.
AO CO 72. The ratio of the areas of two similar triangles is equal to
at the point O such that = . ABCD is a .............
BO DO the square of the ratio of their corresponding sides.
62. A line drawn through the mid-point of one side of a triangle
73. In a right triangle, the square of the hypotenuse is equal to
parallel to another side bisects the ............. side.
the sum of the squares of the other two sides.
63. Line joining the mid-points of any two sides of a triangle
74. If, in a triangle, square of one side is equal to the sum of
is ............. to the third side.
the squares of the other two sides, then the angle opposite
64. In fig., MN || BC and AM : MB = 1 : 2, then A the first side is a right angle.
ar(∆AMN) 75. Diagonals AC and BD of a trapezium ABCD with
= ..................
ar(∆ABC) M N OA OB
AB || DC intersect each other at the point O, = .
OC OD
B C
65. In DABC, AB = 6 3 m, AC = 12 cm and BC = 6 cm,
then ∠B = .................. .
OR
Two triangles are similar if their corresponding sides are
.................. .
Triangles M-43

ANSWER KEY & SOLUTIONS


1. (a)
It is given that AD is the bisector of ∠A.
ar (∆ABC) BC2
5. (a) =
AB BD 6×3 ar (∆DEF) EF2
= ⇒ AC = = 4.5 cm
AC DC 4
2
2.1 
D ⇒ ar (∆ABC) =   × ar (∆DEF) = 9cm
2
 2.8 

6. (a)
From the right angled ∆ACB,
2. (c)
E A
AB2 = AC2 + CB2
= (20)2 + (15)2 = 400 + 225 = 625
x
∴ AB = 625  25cm
2x
Again, from right angled ∆ABD
x
AB2 = AD2 + BD2
B C

Let AB = BC = x. ⇒ 625 = (24)2 + (BD)2
Since, ∆ABC is right-angled with ∠B = 90°
⇒ (BD)2 = 625 – 576 = 49
∴ AC2 = AB2 + BC2 = x2 + x2 = 2x2
⇒ BD = 7 cm
⇒ AC = 2x
7. (a) In DAFD & DFEB,
Since, ∆ABE ~ ∆ACD ∠1 = ∠2 (V.O.A)
Area ( ∆ABE ) AB x 1 2 2
∠3 = ∠4 (Alternate angle)
∴ = = 2 = .
Area ( ∆ACD ) AC 2
2x 2
\ ∆ FBE ~ ∆ FDA
Area (∆ABE) 1 EF FB
Thus, = So, 
Area (∆ACD) 2 FA DF

Thus, required ratio is 1 : 2. 8. (a) In DABC, AB = AC


3. (c) Here, the two triangles are similar. Draw AL ⊥ BC,
Ratio of areas of two similar triangles is equal to the then L is the mid-point of BC
ratio of squares of their corresponding altitudes. Using Pythagoras theorem in ∆ABL, we get
h12 25 AL = 8cm
So, =
h22 36 Also, ∆BPS ≅ ∆CQR,
h1 5 \ BS = RC
∴ =
h2 6 SL = LR = x cm
\ BS = CR = 6 – x
4. (a) In DABC, ∠A = 180° – (70° + 50°) = 60°.
In DABL, PS || AL
BD AB
= . It means AD is the bisector of ∠A.
DC AC PS BS y 6− x
\ = ⇒ =
AL BL 8 6
1
∴ ∠BAD = × 60° = 30° 3
2 or x = 6 − y
4
M-44 Mathematics
A Perimeter of ∆ABC AB BC AC
15. (d) ∵ = = =
Perimeter of ∆PQR PQ QR PR

10
m
P

cm
16. (c) Since, DE || BC ∴ ∆ADE ~ ∆ABC
10
P 2x Q
AD AE 1.5 1
y ∴ = ⇒ = ⇒ EC = 2 cm
y
2x DB EC 3 EC
B S L R C
17. (b) In ∆KPN and ∆KLM, we have
9. (d) All equilateral triangles are similar ∠KNP = ∠KML = 46°
∴ ∆ ABC~ ∆EBD ∠K = ∠K (Common)
2 A ∴ ∆KNP ~ ∆KML (By A A criterion of similarity)
Area of ABC BC
⇒ 
Area of BDE BD 2 E ⇒ KN = NP ⇒ c = x
D is mid-point of BC KM ML b+c a
B C
2 D 18. (a) Statement given in option (a) is false.
( 2BD) 4
\ BC = 2BD = = 19. (b) Since, ∆ABC ~ ∆APQ
BD 2 1
⇒ Area (∆ABC) : Area (∆BDE) = 4 : 1 ar( ∆ABC ) BC 2
∴ =
ar( ∆APQ) PQ 2
10. (a) Given, AB = 2DE and ∆ABC ~ ∆DEF
2
ar( ∆ABC ) BC 2  BC  1
area(∆ABC ) AB 2 ⇒ = ⇒  =
Hence, = 4 ⋅ ar( ∆ABC ) PQ 2  PQ  4
area(∆DEF ) DE 2
BC 1
56 4 DE 2 ⇒ =
or = = 4 [∵ AB = 2DE] PQ 2
area(∆DEF ) DE 2
20. (d) Since, ∆ABC ~ ∆PQR
56
area (∆DEF) = = 14sq.cm. ar(∆ABC ) BC 2 9 (4.5) 2
4 ∴ = ⇒ =
ar(∆PQR) QR 2 16 QR 2
11. (b)
12. (c) As PQ is parallel to BC ⇒ ∆ABC ~ ∆APQ 16 × (4.5) 2
⇒ QR 2 = ⇒ QR = 6 cm
9
Area of ∆ABC 2
⇒ = 21. (a) Let side of a square = x cm
Area of ∆APQ 1
∴ By Pythagoras theorem, x2 + x2 = (16)2 = 256
Ratio of sides = AB = 2 \ AP : AB = 1 : 2 ⇒ 2x2 = 256 ⇒ x2 = 128 ⇒ x = 8 2 cm.
AP 1
22. (b) Given an equilateral triangle ABC in which
13. (a) Since, ∆ABC ~ ∆PQR
AB = BC = CA = 2p A
ar(∆PQR) PR 2 QR 2 9  QR 3 
\ = = = ∵ = =9 and AD ⊥ BC.
ar(∆ABC ) AC 2 BC 2 1  BC 1  2p 2p
∴ In ∆ADB,
14. (b) Hypotenuse = 270m AB2 = AD2 + BD2
⇒ Hypotenuse2 = Side2 + Side2 =2 Side2 B D C
(By Pythagoras theorem)
⇒ Side2 = (270)2/2 = 72900/2 = 36450 ⇒ (2p)2 = AD2 + p2 ⇒ AD2 =
3 p.
or Side = 190.91m
23. (c) Given, area of two similar triangles,
⇒ Required area = 1/2 × 190.91 × 190.91
A1 = 81cm2 , A2 = 49 cm2
= 36446.6/2 = 18225 m2 (approx). A1 81 9
Ratio of corresponding medians = = =
A2 49 7
Triangles M-45
24. (c) Let BD = x cm \ ∠QAB = ∠BAP
R is reflection of P on AC
Since AC = BC, therefore DABC is an isoscele triangle.
\ ∠RAC = ∠CAP
⇒ ∠B = ∠CAB = 72°
∠QAR = 180°
Since AD bisects ∠A
\ 2∠BAP + 2∠CAP = 180°
\ ∠DAB = 36° so, In DADB, ∠ADB = 72°
∠BAP + ∠CAP = 90°  ⇒  ∠BAC = 90°
⇒ DADB is an isoscele triangle
26. (a) DABC ~ DANM
\ AB = AD = 1cm
⇒ AB = 1 cm Area of ∆ABC AC 2
\ =  ...(i)
Similarly, DADC is also an isoscele triangle. Area of ∆ANM AM 2

\ AD = CD ⇒ AD = 1 cm DABC ~ DMPC
2
\ Area of ∆ABC = AC 
C
...(ii)
Area of ∆MPC MC 2
36° 1
From Eqs. (i) and (ii,) we get

Area of ∆ANM AM 2
D =
x
1+

Area of ∆MPC MC 2
72°
36° x Area of ∆ANM + Area of ∆MPC AM 2 + MC 2
=
36° Area of ∆MPC MC 2
72°
A B Now, Area of DANM + Area of DMPC
AC CD = Area of DABC – Area of BNMP
Now   =
AB BD 5
Using Area of BNMP = of area of DABC
1+ x 1 18
⇒ =   ⇒  x + x2 – 1 = 0
1 x 2 2
2 \ 13 (Area of ∆ABC ) = AM + MC  ...(iii)
– 1 ± (1) – 4(1)(–1) –1 ± 5 18 (Area of ∆MPC ) MC 2
⇒ x = =
2 2
13  AC 2  AM 2 + MC 2
From Eq. (iii), =
BD =
5 –1 18  MC 2  MC 2
2
⇒ 13 (AM + MC)2 = 18 (AM2 + MC2)
25. (c) Here, ABC is a triangle & P be interior point of a
AM 1
DABC, Q and R be the reflections of P in AB and AC, ⇒ = 5, . Hence, option (a) is correct.
MC 5
respectively.
27. (d) A
R

A
b
Q θ

φ b
Q P
P 2b a
2b
a
a C
B C B
In DABC
As QAR are collinear
AB = AC
\ ∠QAR = 180°
⇒  ∠C = ∠B ⇒  ∠B = ∠C = a
Q is reflection of P on AB
M-46 Mathematics
By angle sum properly in DABC,
1  2QR 2 + 7QR 2  1 2 2
b + a + a = 180 =
9   = QR = QM
 4  4
⇒  b + 2a = 180°  ...(i) QG 2 + GM 2 = QM 2 ∴ ∠QGM = 90
In  DQPB
29. (c) ∵ ∠BAC = ∠ADC (given)
⇒  ∠QPB = 180 – 4b
∠C = ∠C  (common)
Since ‘APC’ is a straight line A
⇒  180 – 4b + a + b = 180
⇒  a = 3b  ...(ii)
From equations (i) & (ii)
180 B D C
b + 2(3b) = 180 ⇒ b =
7 \ DABC ~ DDAC  (by AA similarity criterion)
 180  5
∠AQP = 180° – 2  = p BC AC
 7  7
2
⇒ = ⇒ BC × DC = AC
AC DC
28. (c) Let DPQR 2
⇒ BC × DC = (21) = area of rectangle with sides BC
Given, QR2 + PR2 = 5PQ2 & DC
Median PM and QN intersect at G.
Now, Area of equilateral triangle = area of rectangle
1
⇒ PN = NR = PR &
⇒ 3 (side) = (21) ⇒ Side = 14 × 3
2 2 3/4
2
1 4
QM = MR = QR
2 30. (a) Here, BAC is a right angle triangle
P B
AB = 15 & BC = 25
N D
∴ AC = BC 2 − AB 2 = 20 F
G
1 A
Area of ∆ABC =
BC. AD
Q R 2 E C
M
1
= AB. AC
2 1 2
QG = QN , GM = PM
3 3 ⇒ BC. AD = AB. AC
2 2
2 2 2  1  ⇒ 25(AD) = 15(20) ⇒  AD = 12
⇒ QG + GM =  QN  +  PM 
3 3
∵ AEDF is rectangle then, AD = EF = 12
4 1
= QN 2 + PM 2
9 9 31. (a) We know that height of an
4  2 PQ 2 + 2QR 2 − PR 2  3 A
= a,
equilateral triangle
9  4 
2
1  2 PQ 2 + 2 PR 2 − QR 2  where a is the side of 30° 30°
+  
9 4 equilateral triangle
 8 PQ + 8QR − 4 PR 
2 2 2
3 3
∴ AD 2 = a 2 = BC 2
60° 60°
  B C
1  +2 PQ 2 + 2 PR 2 − QR 2  4 4  D
=
9 4 
32. (d) 33. (a) 34. (a)
 

  35. (a) (By definition of similar triangles).
1 10 PQ + 7QR − 2 PR 
2 2 2
36. (d) All the statements given in option (a, b, c) are correct.
= 9  4

 
Triangles M-47
Sol. (37-41) DB EC
P
⇒ 1 + =1+ A
AD AE
X
AD + DB AE + EC
A
⇒ =
AD AE D E

20 m AB AC
⇒ =
B C Q R Y Z AD AE B C
10 m 50 m 20 m
[Vijay’s house] [Tower] [Ajay’s house] ∴ Assertion is true.
37. (c) Q DABC ~ DPQR Since reason gives Assertion.
AB BC 20 10 48. (d) In right angled ∆ABC,
∴ = ⇒ =
PQ QR PQ 50
AB2 = AC2 + BC2 (By Pythagorus Theorem)
⇒ PQ = 100 = AC2 + AC2 [∵ BC = AC]
\ Height of the tower = 100 m = 2AC2 A
38. (d) Let BC = 12 m and PQ = 100 m \ AB2 = 2AC2
AB BC 20 12 ∴ Assertion is false.
= ⇒ =
PQ QR 100 QR
Again since C B
⇒ QR = 60 AB2 = 2AC2 = AC2 + AC2
39. (b) Q DABC ~ DXYZ = AC2 + BC2 (∵ AC = BC given)
AB BC 20 10 \ ∠C = 90° (By converse of Pythagoras Theorem)
∴ = ⇒ =
XY YZ XY 20 \ Reason is true.
⇒ XY = 40 49. (b) Reason is true [standard result]
40. (a) Let QR = 40 m, PQ = 100 m and XY = 40 m For Assertion, since ∆ABC ~ ∆DEF
PQ QR 100 40 area ( ∆ABC )
∴ = ⇒ = BC 2 (4)2 16
XY YZ 40 YZ \ = = =
area ( ∆DEF ) EF 2 (5)2 25
⇒ YZ = 16 m.
(∵ ratio of areas of two similar ∆s is equal to the ratio
41. (d) Let QR = 40m, PQ =100m and AB = 20 m of the squares of corresponding sides)
AB BC 20 BC 64
∵ = ⇒ = 16 64 × 25
PQ QR 100 40 \ = ⇒ area (∆DEF) =
area (∆DEF ) 25 16
⇒ BC = 8 m. = 4 × 25 = 100 cm2
42. (c) Pythagoras theorem \ Assertion is true. But reason is not the correct
43. (a) AC2 = 302 + 402 = 2500 ⇒ AC = 50m explanation for assertion.
44. (d) (21, 20, 28) Q 282 ≠ (21)2 + (20)2 50. (A) → q; (B) → p; (C) → s; (D) → r
45. (b) AB = 50 – 12 = 38m 51. (A) → p; (B) → q; (C) → r; (D) → s
46. (c) 82m
52. (A) → q; (B) → p; (C) → s; (D) → r
47. (a) Reason is true. [This is Thale’s Theorem]
(A) AB2 = AC2 + BC2
For Assertion
Since, ∆ABC is an isosceles right angled triangle.
Since DE || BC ∴ by Thale’s Theorem
\ AC = BC
AD AE DB EC
= ⇒ = Now, AB2 = AC2 + AC2 = 2AC2
DB EC AD AE
M-48 Mathematics
2 2 1
area ( ∆ABC ) ( AB ) (1.2) 1.44 64.  
(B) = = = 9
area ( ∆DEF ) ( DE ) 2
(1.4) 2 1.96
AM AM 1 1
36 (36 × 2) 72 = = =
= = = AB AM + BM 1 + 2 3
49 (49 × 2) 98 2 2
ar ( ∆AMN)  AM   1 1
2 \ =  =   =
area ( ∆APQ ) ( BC ) 36 BC 6 ar ( ∆ABC)  AB 3 9
(C) = = = =
area ( ∆ABC ) ( PQ ) 2 49 PQ 7 65. [90°]
AD AE 6 Q AB2 + BC2 = 108 + 36 = 144 = AC2
∵ DE || BC \
(D) = =
DB EC 7 So, AC is hypotenuse and ∠B = 90°.

53. similar 54. similar 55. equilateral OR


[Proportional]
56. equal , ratio 57. same 58. third 59. not
66. True 67. True 68. True 69. True
60. equal 61. trapezium 62. third
70. True 71. True 72. True 73. True
63. parallel
74. True 75. True
6 Acids, Bases and
Introduction
Salts
Trigonometry
to

7. If x = a cos2θ + b sin2θ, then (x – a) (b – x) is equal to


Multiple Choice Questions (MCQs) (a) (a – b) sinθ cosθ (b) (a – b)2 sin2θ cos2θ
(c) (a – b)2 sinθ cosθ (d) (a – b) sin2θ cos2θ
DIRECTIONS : This section contains multiple choice
questions. Each question has 4 choices (a), (b), (c) and (d) out 3
8. , find the value of 9 cot2A – 1.
If cos A =
of which ONLY ONE is correct. 5
16
(a) 1 (b)
65
1. If (sec2θ) (1 + sinθ) (1 – sinθ) = k, then find the value of k.
65
(a) sinθ (b) secθ (c) (d) 0
(c) 1 (d) cotθ 16
9. cos 1° . cos 2°. cos 3° ......... cos 179° is equal to
 15  (2 + 2sin θ) (1 − sin θ) (a) –1 (b) 0
2. If cot θ =   , then evaluate (c) 1 (d) 1/ 2
8 (1 + cos θ) (2 − 2 cos θ)
225 10. sin2q + cosec2q is always
(a) 1 (b) (a) greater than 1
64
(b) less than 1
156
(c) (d) –1 (c) greater than or equal to 2
7
(d) equal to 2
b(1 − cos θ)
3. If x = a (cosec θ + cot θ) and y = , then xy = 11. If x = p sec q and y = q tan θ , then
2 2 sin θ
a +b (a) x2 – y2 = p2q2
(a) a2 – b2
(b)
2 2
a −b (b) x2q2 – y2p2 = pq
a
ab
(c) (d) 1
b (c) x2q2 – y2p2 = 2 2
4. If p sin θ + q cos θ = a and p cos θ – q sin θ = b, then p q
p+a q−b (d) x2q2 – y2p2 = p2q2
+ =
q+b p−a
a sin θ − b cos θ
(a) 1 (b) a2 + b2 12. If b tan θ = a, the value of is
a sin θ + b cos θ
(c) 0 (d) 2
a−b a+b
5. If x = r sinA cos C, y = r sin A sin C, z = r cos A, then (a) (b) 2
a 2 + b2 a + b2
(a) r2 = x2 + y2 + z2 (b) r2 = 2xy

a 2 + b2 a 2 − b2
(c) r2 = x + y + z (d) r2 = y2 + z2 + 2xy (c) (d)
a 2 − b2 a 2 + b2
6. If tan2θ = 1– a2, then the value of 13. (cos4A – sin4A) is equal to
sec θ + tan3θ cosec θ is
(a) 1 – 2 cos2A (b) 2 sin2 A – 1
(a) (2 – a2) (b) (2 – a2)1/2
(c) sin2A – cos2A (d) 2 cos2A – 1
(c) (2 – a2)2/3 (d) (2 – a2)3/2
M-50 Mathematics
a sin φ b sin θ a cos θ cos θ
14. If tan θ = and tan φ = , then = 22. If + = 4, then
1 − a cos φ 1 − b cos θ b 1 − sin θ 1 + sin θ

sin θ sin θ 3 1
(a) (b) (a) cos θ = (b) sin θ =
1 − cos θ 1 − cos φ 2 2
sin θ 1
sin φ
(c) (d) (c)
θ = 60° (d) tan θ =
sin φ 3
sin θ
1 tan θ − cot θ
23. is equal to
15. If cosec x – cot x = , where x ≠ 0, then the value of sin θ cos θ
cos2x – sin2x is 3
(a) sec2 θ + cosec2 θ (b) cot2 θ – tan2 θ
16 9 (c) cos2 θ – sin2 θ (d) tan2θ – cot2θ
(a) (b)
25 25
2 tan 30°
8 7 24. =
(c) (d) 1 + tan 2 30°
25 25 (a) sin 60° (b) cos 60°
16. If cosec x + sin x = a and sec x + cos x = b, then
(c) tan 60° (d) sin 30°
2 2
2 3 2 3
(a) ( a b) + (ab ) =1 1 − tan 2 45°
25. =
2 2 1 + tan 2 45°
(ab 2 ) 3
(b) + ( a 2b 2 ) 3 =1 (a) tan 90° (b) 1
(c) a2 + b2 = 1 (c) sin 45° (d) 0
(d) b2 – a2 = 1 26. sin 2A = 2 sin A is true when A =
17. If tan2q = 1 – e2, then the value of (a) 0° (b) 30°
secq + tan3q cosec q is equal to (c) 45° (d) 60°
(a) (1 – e2)1/2 (b) (2 – e2)1/2 2 tan 30°
27. =
(c) (2 – e2)3/2 (d) (1 – e2)3/2 1 − tan 2 30°
18. If sinq + sin3q = cos2q, then the value of (a) cos 60° (b) sin 60°
cos6q – 4cos4q + 8cos2q is (c) tan 60° (d) sin 30°
(a) 1 (b) 4
28. 9 sec2 A – 9 tan2 A =
(c) 2 (d) 0 (a) 1 (b) 9
11 (c) 8 (d) 0
19. If cosec A + cot A = , then tan A
2 29. (1 + tan θ + sec θ) (1 + cot θ– cosec θ) =
21 15 (a) 0 (b) 1
(a) (b)
22 16 (c) 2 (d) –1
44 11 30. (sec A + tan A) (1 – sin A) =
(c) (d)
117 117 (a) sec A (b) sin A
(c) cosec A (d) cos A
2 tan 30°
20. is equal to
1 + tan 2 30° 1 + tan 2 A
31. =L
(a) sin 30° (b) cos 60° 1 + cot 2 A
1 3 (a) sec2 A (b) –1
(c) (d)
2 2 (c) cot2 A (d) tan2 A
32. The value of (sin 30° + cos 30°) – (sin 60° + cos 60°) is
sin θ − 2sin 3 θ
21. is equal to (a) –1 (b) 0
2 cos3 θ − cos θ (c) 1 (d) 2
(a) sec θ (b) tan θ
(c) sec θ − 1 (d) cot θ
Introduction to Trigonometry M-51
tan 30° 39. The value of sec C is
33. The value of is 4 5
cot 60° (a) (b)
1 1 3 3
(a) (b) 1
2 3 (c) (d) None of these
3
(c) 3 (d) 1
40. sin2C + cos2C =
34. The value of (sin 45° + cos 45°) is (a) 0 (b) 1
1 (c) –1 (d) None of these
(a) (b) 2
2
3 Assertion & Reason
(c) (d) 1
2
a DIRECTIONS : Each of these questions contains an Assertion
35. Given that sin θ = , then cos θ is equal to
b followed by reason. Read them carefully and answer the
b question on the basis of following options. You have to select
b
(a) 2 2
(b) the one that best describes the two statements.
b −a a
(a) If both Assertion and Reason are correct and Reason is
b −a 2 2 a the correct explanation of Assertion.
(c) (d)
b b2 − a 2 (b) If both Assertion and Reason are correct, but Reason is
not the correct explanation of Assertion.
36. If sin A + sin2A = 1, then the value of the expression (c) If Assertion is correct but Reason is incorrect.
(cos2A + cos4A) is (d) If Assertion is incorrect but Reason is correct.
1 3
(a) 1 (b) 41. Assertion: In a right angled triangle, if tan θ = , the
2 4
greatest side of the triangle is 5 units.
(c) 2 (d) 3
Reason: (greatest side)2 = (hypotenuse)2
3 1 = (perpendicular)2 + (base)2.
37. If sin (A + B) =
and sin 2B = , then
2 2 1
(a) tan B = 1 (b) B = 30° 42. Assertion : In a right angled triangle, if cos θ = and
2
1 3
(c) B = 45° (d) cos A = sin θ = , then tan θ = 3
2 2
sin θ
Reason: tan θ = cos θ

DIRECTIONS : Study the given Case/Passage and answer the Match the Following
following questions.
Case/Passage DIRECTIONS : Each question contains statements given in
In ∆ABC, right angled at B two columns which have to be matched. Statements (A, B, C, D)
C in column-I have to be matched with statements (p, q, r, s) in
column-II.

3cm 43. In ∆ ABC, ∠ B = 90°, AB = 3 cm and BC = 4 cm, then


match the column.
A B Column-I Column-II
AB + AC = 9 cm and BC = 3cm. (A) sin C (p) 3/5
38. The value of cot C is (B) cos C (q) 4/5
[From CBSE Question Bank-2021] (C) tan A (r) 5/3
3 1 (D) sec A (s) 4/3
(a) (b)
4 4
5
(c) (d) None of these
4
M-52 Mathematics
44. Column-I Column-II 50. In ∆ ABC, right-angled at B, AB = 24 cm, BC = 7 cm.
cos A 1 + sin A sin A = ...........
(A) + (p) cosec A + cot A
1 + sin A cos A 51. If 15 cot A = 8, sec A = ..............
cos A − sin A + 1 52. In ∆ PQR, right-angled at Q, PR + QR = 25 cm and
(B) (q) 2 sec A
cos A + sin A − 1 PQ = 5 cm. The value of tan P is ...........
1 + sin A 53. sin 60° cos 30° + sin 30° cos60° = ................
(C) (r) sec A + tan A
1 − sin A 54. 2 tan2 45° + 3 cos2 30° – sin2 60° = .............
sin 2 A 1 + sec A cos 45°
(D) (s) 55. = ..............
1 − cos A sec A sec30° + cosec 30°
7
45. If sin A = , then
25 True / False
Column-I Column-II
(A) cos A (p) 24/25 DIRECTIONS : Read the following statements and write your
(B) tan A (q) 7/24 answer as true or false.
(C) cosec A (r) 25/7 56. The value of tan A is always less than 1.
(D) sec A (s) 25/24 57. sec A = 12/5, for some value of angle A.
58. cos A is the abbreviation used for the cosecant of angle A.
Fill in the Blanks 59. cot A is the product of cot and A.
4
DIRECTIONS : Complete the following statements with an 60. sin θ = , for some angle θ.
3
appropriate word / term to be filled in the blank space(s).
61. sin (A + B) = sin A + sin B.
46. The value of sin A or cos A never exceeds ...... 62. cot A is not defined for A = 0°.
47. sin2 A+ cos2 A = .............. 63. If ∠B and ∠Q are acute angles such that sin B = sin Q,
48. If tan A = 4/3, then sin A ............... then ∠B ≠ ∠Q.
49. In a right trianlge ABC, right angled at B, if tan A = 1,
sin A cos A = ..........
Introduction to Trigonometry M-53

ANSWER KEY & SOLUTIONS


sec2θ (1 + sinθ) (1– sinθ) = k
1. (c) 7. (b) x – a = b sin2θ – a sin2θ = (b – a) sin2θ

 1  b – x = b cos2θ – a cos2θ = (b – a) cos2θ


 (1 – sin2θ) = k
 cos 2 θ  ∴ (x – a) (b – x) = (b – a)2 sin2θ cos2θ
= (a – b)2 sin2θ cos2θ
 1 
⇒  (cos2θ) = k ⇒ 1 = k.
 cos 2 θ  8. (c) cos A =
3
⇒ sin A = 1 −
9
=
4
(2 + 2sin θ) (1 − sin θ) 2(1 + sin θ) (1 − sin θ) 5 25 5
2. (b) = Consider,
(1 + cos θ) (2 − 2 cos θ) (1 + cos θ) (2) (1 − cos θ)

2(1 − sin 2 θ) 2cos 2 θ


2 9 cos 2 A 9 cos 2 A − sin 2 A
 15  225 2 9 cot 2 A − 1 = − 1 =
= 2
= 2
= cot θ =   =
8 64 sin 2 A sin 2 A
2(1 − cos θ) 2sin θ
3. (c)
We have, x = a (cosec θ + cot θ)  9   16 
9  −  
 25   25  (81 − 16) 25 65
x = = × =
⇒ = (cosec θ + cot θ) ...(1) 16 25 16 16
a
25
 1 − cos θ  y 1 cos θ
and y = b  ⇒ = −
 sin θ  b sin θ sin θ 9. (b) 10.  (c)
y x
⇒ = cosec θ – cot θ ...(2) 11. (d) We know that sec2θ – tan2θ = 1 and sec θ = ,
b p
x y y
⇒ × = (cosec θ + cot θ) (cosec θ – cot θ) tan θ =
a b q
xy ∴ x2q2 – p2y2 = p2q2
⇒ = (cosec2θ – cot2 θ) ∴ xy = ab
ab
4. (c) By squaring and adding both the given equations, we a
12. (d) Given, tan θ =
get b
p2 (sin2θ + cos2θ) + q2 (cos2θ + sin2θ) a sin θ − b cos θ a tan θ − b a 2 − b 2
∴ = =
= a2 + b2 a sin θ + b cos θ a tanθ + b a 2 + b 2

⇒ p2 + q2 – a2 – b2 = 0 13. (d) (cos 4 A − sin 4 A) = (cos 2 A) 2 − (sin 2 A) 2


⇒ (p – a) (p + a) + (q – b) (q + b) = 0
= (cos 2 A − sin 2 A)(cos 2 A + sin 2 A)
p+a q−b
⇒ + =0
q+b p−a = (cos 2 A − sin 2 A)(1) = cos 2 A − (1 − cos 2 A)
5. (a) x = r sin A cos C, y = r sin A sin C, z = r cos A = 2 cos2A – 1

x2+ y2+ z2 = r2sin2A cos2C + r2 sin2A sin2C + r2 cos2A a sin φ


14. (d) We have, tan θ =
1 − a cos φ
( )( )
= r 2 sin 2 A cos 2 C + sin 2 C + r 2 cos 2 A
1 1
⇒ cot θ = − cot φ ⇒ cot θ + cot φ = ...(i)
2 2 2 2 2
= r sin A(1) + r cos A = r (sin A + cos A) =
2 2
r2 a sin φ a sin φ

6. (d) sec θ + tan3θ cosec θ b sin θ


and tan φ =
1 − b cos θ
sin θ
sec θ +
= tan2 θ cosec θ = sec θ (1 + tan2θ) 1
cos θ ⇒ cot φ = − cot θ
b sin θ
(1 + tan2θ)3/2 = [1+ (1 – a2)]3/2
=
M-54 Mathematics
1 1 sin q 1
⇒ cot φ + cot θ = ...(ii) \ secq + tan3q cosecq = + tan 2 q. .  
b sin θ cos q cos q sin q
From (i) and (ii), we have 1 sec 2 q
= (1 + tan 2 q) = sec3 q = (2 – e2)3/2  [from (i)]
=
1 1 a sin θ cos q cos q
⇒ =
a sin φ b sin θ b sin φ 18. (b) sinq + sin3q = cos2q
1 sinq(1 + 1 – cos2q) = cos2q
15. (d) Let cosec x – cot x =
3 ⇒ sin2q(2 – cos2q)2 = cos4q
1 cos x 1 ⇒ (1 – cos2q)(4 + cos4q – 4cos2q) = cos4q
⇒ – =
sin x sin x 3 ⇒ 4 + cos4q – 4cos2q – 4cos2q – cos6q + 4 cos4q = cos4q
x
2sin 2 ⇒ cos6q – 4 cos4q + 8 cos2q = 4
1 – cos x 1 1
2
⇒ = ⇒ =
sin x 3 x x 3 19. (c)
2 sin cos
2 2
2 tan 30°
x 1 20. (d)
We have,
⇒ tan = 1 + tan 2 30°
2 3
Consider 2 1

x 2 3 2×3
3 3
2 tan = = = =
2 = 3 =3  1 
2
1+
1 3×4 2
tan x = 1+  
x 1 4 3
1 – tan 2 1–  3
2 9
Alternate method:
3 4
Thus sin x = , cos x =
5 5  2 tan A 
 Using identity,sin 2 A = 
∴ cos2 x – sin2x =
16 9
– =
7  1 + tan 2 A 
25 25 25
2 tan 30° 3
16. (a) cosec x – sin x = a & sec x – cos x = b sin 60° = 2
=
1 + tan 30° 2
1 1
cosec x − = a & sec x − =b 21. (b) We have,
cosec x sec x
2 2 sin θ − 2sin 3 θ sin θ(1 − 2sin 2 θ)
cosec x − 1 sec x − 1 =
⇒ =a& =b 2 cos3 θ − cos θ cos θ(2 cos 2 θ − 1)
cosec x sec x
1 − 2(1 − cos 2 θ)   (2 cos 2 θ − 1) 
cot 2 x tan 2 x = tan θ   = tan θ  
⇒ = a& =b 2 2
cosec x sec x  2 cos θ − 1   2 cos θ − 1 
2 2 = tan q
cos x = a & sin x = b
sin x cos x cos θ cos θ
22. (c) We have, + =4
1 − sin θ 1 + sin θ
cos 4 x sin 2 x
Now, a 2b = 2
. = cos3 x  1 + sin θ + 1 − sin θ 
sin x cos x ⇒ cos θ   = 4
 1 − sin 2 θ
⇒ cos x = (a2b)1/3 ⇒ cos2 x = (a2b)2/3
2 cos θ 1
Similarly, sin2 x = (ab2)2/3 ⇒ 2
= 4 ⇒ cos θ = ⇒ θ = 60°
cos θ 2
We know that, sin2x + cos2x = 1 tan θ − cot θ
23. (d) We have,
sin θ cos θ
⇒ (ab2)2/3 + (a2b)2/3 =1
tan θ cot θ
= −
17. (c) ∵ tan2 q = 1 – e2 sin θ cos θ sin θ cos θ

⇒ secq = 1 + tan 2 q = 1 + 1 - e2 sin θ cos θ


= −
cos θ sin θ cos θ sin θ cos θ cos θ
⇒ secq = 2 - e 2 ...(i)
Introduction to Trigonometry M-55

=
1

1
= sec2 θ − cosec2θ = (1 + sin A)(1 – sin A)
2 2 cos A
cos θ sin θ
= 1 + tan θ − 1 − cot 2 θ = tan 2 θ − cot 2 θ
2 1 – sin 2 A cos 2 A
= =  (∵ cos 2 A = 1 – sin 2 A)
cos A cos A
 1  = cosA.
2 
2 tan 30°  3
24. (a) = 1 + tan 2 A (sec2 A – tan 2 A) + tan 2 A
1 + tan 2 30°  1 
2 31. (d) =
1+   1 + cot 2 A (cosec2 A – cot 2 A) + cot 2 A
 3
2
sec2 A sin 2 A sin A  2
2 = = =
  = tan A.
cosec2 A 2
cos A cos A
2 3 3
= 3 = × = = sin 60°. 32. (b) (sin 30° + cos 30°) – (sin 60° + cos 60°)
1 3 4 2
1+
3 1 3  1 3
2 2 =  2 + 2  −  2 + 2  = 0
1 – tan 45° 1 – (1)    
25. (d) = = 0.
1 + tan 45°2
1 + (1) 2 1
tan 30° 3
26. (a) Here, when A = 0° 33. (d) = =1
cot 60° 1
LHS = sin 2 A = sin 0° = 0 3
and RHS = 2 sin A= 2 sin 0° = 2 × 0 = 0 1 1 2
34. (b) sin 45° + cos 45° = + = = 2
In the other options, we will find that 2 2 2
(on rationalizing)
LHS ≠ RHS
a2 b2 − a 2
 1  35. (c) cos θ = 1 − sin 2 θ = 1 − =
2  b2 b
2 tan 30°  3
27. (c) = 36. (a) Given, sin A + sin2A =1
1 – tan 2 30°  1 
2
1–   ⇒ sin A = 1 – sin2A = cos2 A
 3
Consider, cos2A + cos4A = sinA + (sin A)2 = 1
2
3
= 3 = 2 × 3 = 3 = tan 60°. 37. (c) We have, sin ( A + B) =
1 3 2 2
1– ⇒ A + B = 60° ...(i)
3
and 2B = 30° ∴ B = 15°
28. (b) 9sec2 A – 9 tan 2 A = 9(sec2 A – tan 2 A)
= 9 × 1 = 9. Putting B in (i), we get

29. (c) (1 + tan θ + sec θ)(1 + cot θ – cosec θ) A + 15° = 60° ⇒ A = 45°

 sin θ 1   cos θ 1  Sol. (38-40):


= 1 + +  × 1 + – 
 cos θ cos θ   sin θ sin θ  In ∆ABC, by Pythagoras theorem,
{(cos θ + sin θ) + 1} × {(cos θ + sin θ) –1} AC2 = AB2 + BC2 ⇒ AB = 4 cm.
=
cos θ × sin θ AC = 5 cm.
2 2 2 2 BC 3
= (cos θ + sin θ) – (1) {∵ (a + b)(a – b) = a – b } 38. (a) cot C = =
cos θ × sin θ AB 4
1 + 2 cos θ sin θ –1 AC 5
= = 2. 39. (b) sec C = =
cos θ × sin θ BC 3
30. (d) (sec A + tan A) (1 – sin A) 4 3
40. (b) sin C = , cos C =
 1 sin A  5 5
=  + × (1 – sin A)
 cos A cos A 
M-56 Mathematics
2 2 44. (A) → q; (B) → p; (C) → r; (D) → s
2 2  4 3
L.H.S = sin C + cos C =   +   45. (A) → p; (B) → q; (C) → r; (D) → s
5 5
16 + 9 46. 1 47. 1 48. 4/5
= = 1 = R.H.S
25 1
41. (a) Both Assertion and Reason are correct and Reason is 49. 50. 7/25 51. 17/8
2
the correct explanation of the assertion.
52. 12/5 53. 1
greatest side = (3)2 + (4)2 = 5 units.
7 3( 3 − 1)
54. 55.
42. (a) Both assertion and reason are correct and reason is 2 4
the correct explanation of the assertion.
56. False 57. True 58. False
3
tan θ = ×2= 3 . 59. False 60. False 61. False
2
43. (A) → p; (B) → q; (C) → s; (D) → r 62. True 63. False
7 Acids,
Areas Bases
Salts
Circles
and
Related to

5. Mrs. Vidya bought a piece of cloth as shown in the figure.


Multiple Choice Questions (MCQs) The portion of the cloth that is not coloured consists of 6
identical semi-circles.
DIRECTIONS : This section contains multiple choice 42 cm
questions. Each question has 4 choices (a), (b), (c) and (d) out
of which only one is correct.

1. Find area of minor segment made by a chord which


subtends right-angle at the centre of a circle of radius
10 cm.
(a) 24.5 cm2 (b) 25.5 cm2
Find the area of the coloured portion.
(c) 24.5 cm2 (d) 28.5 cm2
(a) 144 cm2 (b) 126 cm2
2. The figure shows two concentric circles B
(c) 195 cm2 (d) 243 cm2
with centre O and radii 3.5 m and 7 D A
6. The figure given shows a rectangle with a semi-circle and
m. If ∠BOA = 40°, find the area of the C
2 identical quadrants inside it.
shaded region. O
28 cm
77 2 76
(a) cm (b)
6 5
73
(c) (d) None of these 16 cm
6 23 cm
3. A drain cover is made from a square metal plate of side
40 cm having 441 holes of diameter 1 cm each drilled in
it. Find the area of the remaining square plate.
22
(a) 1250.5 cm2 (b) 1253.5 cm2 What is the shaded area of the figure? (Use π = )
7
(c) 1240.2 cm2 (d) 1260.2 cm2 (a) 363 cm2 (b) 259 cm2
4. The figure given shows two 15 cm 4 cm (c) 305 cm2 (d) 216 cm2
identical semi-circles cut out 7. The perimeter of a sector of a circle with central angle 90° is
from a piece of coloured paper. 25 cm. Then the area of the minor segment of the circle is.
Find the area of the remaining (a) 14 cm2 (b) 16 cm2
22 (c) 18 cm2 (d) 24 cm2
piece of paper (Use π = ) 20 cm
7
(a) 296.1 cm 2 8. The sum of the areas of two circles, which touch each other
externally, is 153 π. If the sum of their radii is 15, then the
(b) 265.4 cm2
ratio of the larger to the smaller radius is
(c) 221.5 cm2
(a) 4 : 1 (b) 2 : 1
(d) 201.7 cm2 7 cm
(c) 3 : 1 (d) None of these
M-58 Mathematics
9. A race track is in the form of a ring whose inner and outer
circumference are 437m and 503m respectively. The area
of the track is
(a) 66 sq. cm (b) 4935 sq. cm
(c) 9870 sq. cm (d) None of these
10. If the sum of the circumferences of two circles with
diameters d1 and d2 is equal to the circumference of a
circle of diameter d, then
2 2 2 π 3π
(a) d1 + d 2 = d (b) d1+ d2 = d (a) (b)
4 2
(c) d1+ d2 > d (d) d1+ d2 < d
π
11. In the adjoining figure, OABC is a O (c) (d) p
C 2
square of side 7 cm. OAC is a
quadrant of a circle with O as centre. 17. If a circular grass lawn of 35m in radius has a path 7m
The area of the shaded region is wide running around it on the outside, then the area of the
(a) 10.5 cm2 path is
(b) 38.5 cm2 (a) 1450 m2 (b) 1576 m2
(c) 49 cm2 (c) 1694 m2 (d) 3368 m2
A B
(d) 11.5 cm2 18. In the adjoining figure, OACB is a quadrant of a circle of
12. The area of a circular ring formed by two concentric circles radius 7 cm. The perimeter of the quadrant is
whose radii are 5.7 cm and 4.3 cm respectively is B
(Take π = 3.1416) C
(a) 43.98 sq.cm (b) 53.67 sq. cm
(c) 47.24 sq.cm (d) 38.54 sq.cm
13. A sector is cut from a circular sheet of radius 100 cm, the
angle of the sector being 240º. If another circle of the area O A
same as the sector is formed, then radius of the new circle (a) 11 cm (b) 18 cm
is (c) 25 cm (d) 36 cm
(a) 79.5 cm (b) 81.6 cm
19. If the circumference of a circle increases from 4π to 8π,
(c) 83.4 cm (d) 88.5 cm then its area is
14. The area of a sector of angle p (in degrees) of a circle with (a) halved (b) doubled
radius R is (c) tripled (d) quadrupled
p p
(a) × 2πR (b) × πR 2 20. If the radius of a circle is diminished by 10%, then its area
360° 180° is diminished by
p p (a) 10% (b) 19%
(c) × 2πR (d) × 2πR 2
720° 720° (c) 36% (d) 20%
15. If the sector of a circle of diameter 10 cm subtends an 21. If the perimeter of a semi-circular protractor is 36 cm, then
angle of 144° at the centre, then the length of the arc of its diameter is
the sector is
(a) 10 cm (b) 14 cm
(a) 2π cm (b) 4π cm (c) 12 cm (d) 16 cm
(c) 5π cm (d) 6π cm
22. The area of a circular path of uniform width ‘d’ surrounding
16. The figure below shows two concentric circles with centre a circular region of radius ‘r’ is
O. PQRS is a square inscribed in the outer circle. It also
(a) πd(2r + d) (b) π(2r + d) r
circumscribes the inner circle, touching it at point B, C,
(c) π(d + r)d (d) π(d + r)r
D and A. The ratio of the perimeter of the outer circle to
that of polygon ABCD is
Areas Related to Circles M-59
23. If Anish is moving along the boundary of a triangular field (c) The ratio between the circumference and area of a
of sides 35 m, 53 m and 66 m and you are moving along circle of radius 5 cm is 2 : 5.
the boundary of a circular field whose area is double the (d) Area of a circle whose radius is 6 cm, when the length
area of the triangular field, then the radius of the circular of the arc is 22 cm, is 66 cm2.
22
field is (Take π = ) 32. Which of the following statement is/are not correct?
7
(a) A chord divides the interior of a circle into two parts.
(a)
14 3 m (b) 3 14 m (c) 28 3 m (d) 7 3 m (b) An arc of a circle whose length is less than that of a
24. If the area of a square inscribed in a semicircle is 2cm2, semicircle of the same circle is a called a minor arc.
then the area of the square inscribed in a full circle of the (c) Circles having the same centre but different radii are
same radius is ______ called concentric circles.
(a) 5 cm2 (b) 10 cm2 (c) 5 2 cm2 (d) 25 cm2 (d) A line segment joining any two points of a circle is
called an arc.
25. Suppose we have two circles of radius 2 each in the plane
such that the distance between their centers is 2 3 . The 33. Tick the correct answer in the following and justify
area of the region common to both circles lies between your choice. If the perimeter and the area of a circle are
numerically equal, then the radius of the circle is
(a) 0.5 and 0.6 (b) 0.65 and 0.7
(a) 2 units (b) π units
(c) 0.7 and 0.75 (d) 0.8 and 0.9
26. A circle is inscribed in a right angled triangle of perimeter (c) 4 units (d) 7 units
7p . Then the ratio of numerical values of circumference 34. If θ is the angle (in degree) of a sector of a circle of
of the circle to the area of the right angled triangle is radius r, then, area of the sector is
(a) 4 : 7 (b) 3 : 7 (c) 2 : 7 (d) 1 : 7
πr 2θ πr 2θ
(a) (b)
27. How much time the minute hand of a clock will take to 360° 180°

describe an angle of radians? 2πrθ 2πrθ
3 (c) (d)
360° 180°
(a) 15 minutes (b) 20 minutes
35. If the sum of the areas of two circles with radii R1 and R2
(c) 10 minutes (d) 25 minutes
is equal to the area of a circle of radius R, then
7 2 2 2
28. If the radius of a circle is cm, then the area of the circle (a) R1 + R2 = R (b) R1 + R 2 = R
π
is equal to
(c) R1 + R2 < R (d) R12 + R 22 < R 2
49
(a) cm2 p cm2
(b) 36. It is proposed to build a single circular park equal in area
π
to the sum of areas of two circular parks of diameters
(c) 154 cm2 (d) 49 cm2
16 m and 12 m in a locality. The radius of the new park
29. If the ratio of the areas of the two circles is 25 : 16, then would be
the ratio of their circumferences is (a) 10 m (b) 15 m (c) 20 m (d) 24 m
25 4 5 500
(a) (b) (c) (d)
16 5 4 625

30. If the sector of a circle of diameter 14cm subtends an angle


of 30° at the centre, then its area is DIRECTIONS : Study the given Case/Passage and answer the
following questions.
(a) 49π (b) 49π (c) 242
(d)
121
12 3π π Case/Passage-I
Pookalam is the flower bed or flower pattern designed during
31. Which of the following is/are not correct?
Onam in Kerala. It is similar as Rangoli in North India and
(a) Area of a circle with radius 6 cm, if angle of sector is Kolam in Tamil Nadu.
132 2
60°, is cm . During the festival of Onam, your school is planning to conduct
14 a Pookalam competition. Your friend who is a partner in
(b) If a chord of circle of radius 14 cm makes an angle competition , suggests two designs given below.
of 60° at the centre of the circle, then area of major Observe these carefully. [From CBSE Question Bank-2021]
sector is 512.87 cm2.
M-60 Mathematics
A Refer to Design A
42. The total length of silver wire required is
A B
(a) 180 mm (b) 200 mm
(c) 250 mm (d) 280 mm
43. The area of each sector of the brooch is
(a) 44 mm2 (b) 52 mm2
B
C (c) 77 mm2 (d) 68 mm2
D C
Refer to Design B
I II 44. The circumference of outer part (golden) is
Design I: This design is made with a circle of radius 32cm (a) 48.49 mm (b) 82.2 mm
leaving equilateral triangle ABC in the middle as shown in the
(c) 72.50 mm (d) 62.86 mm
given figure.
45. The difference of areas of golden and silver parts is
Design II: This Pookalam is made with 9 circular design each
(a) 18 p (b) 44 p
of radius 7cm.
Refer Design I: (c) 51 p (d) 64 p
46. A boy is playing with brooch B. He makes revolution
37. The side of equilateral triangle is with it along its edge. How many complete revolutions
(a) 12√3 cm (b) 32√3 cm must it take to cover 80 p mm ?
(c) 48 cm (d) 64 cm (a) 2 (b) 3
38. The altitude of the equilateral triangle is (c) 4 (d) 5
(a) 8 cm (b) 12 cm
(c) 48 cm (d) 52 cm Assertion & Reason
Refer Design II:
39. The area of square is DIRECTIONS : Each of these questions contains an Assertion
(a) 1264 cm2 (b) 1764 cm2 followed by Reason. Read them carefully and answer the
(c) 1830 cm2 (d) 1944 cm2 question on the basis of following options. You have to select
40. Area of each circular design is the one that best describes the two statements.
(a) 124 cm2 (b) 132 cm2
2
(a) If both Assertion and Reason are correct and Reason is
(c) 144 cm (d) 154 cm2
the correct explanation of Assertion.
41. Area of the remaining portion of the square ABCD is
(b) If both Assertion and Reason are correct, but Reason is
(a) 378 cm2 (b) 260 cm2
not the correct explanation of Assertion.
(c) 340 cm2 (d) 278 cm2
(c) If Assertion is correct but Reason is incorrect.
Case/Passage-II (d) If Assertion is incorrect but Reason is correct.
A brooch is a small piece of jewellery which has a pin at the
47. Assertion : If the circumference of a circle is 176 cm, then
back so it can be fastened on a dress, blouse or coat.
its radius is 28 cm.
Designs of some brooch are shown below. Observe them
carefully. [From CBSE Question Bank-2021] Reason : Circumference = 2π × radius.
48. Assertion : If the outer and inner diameter of a circular
path is 10m and 6m, then area of the path is 16π m2.
Reason : If R and r be the radius of outer and inner circular
path respectively then area of path = π (R2 – r2).
A B 49. Assertion : If a wire of length 22 cm is bent in the
Design A: Brooch A is made with silver wire in the form of shape of a circle, then area of the circle so formed is
a circle with diameter 28mm. The wire used for making 4 40 cm 2 .
diameters which divide the circle into 8 equal parts.
Reason : Circumference of the circle = length of the wire.
Design B: Brooch b is made two colours-Gold and silver. Outer
part is made with Gold. The circumference of silver part is 44mm
and the gold part is 3mm wide everywhere.
Areas Related to Circles M-61
C
Match the Following

DIRECTIONS : Each question contains statements given in (B)


A B (q) 115 m2
two columns which have to be matched. Statements (A, B, C, D) D
in column-I have to be matched with statements (p, q, r, s) in
column-II.
AD = 6.5 cm, CA = 5 cm.
50. For circle shown, match the column. Y
Area of ∆ABC = ?
Column-I Column-II A M B

441 120°
21
cm
D
(A) Area of (p) 3 C
segment AYB 4 O O
(C) (r) 114.7 cm2
21
(B) Area of (q) (88 − 21 3) A B
sector OAYB 4

BC = 6 cm, AB = 8 cm.
(C) Area of ∆ (r) 462
Area of the shaded region = ?
OAB
(D) OM (s) 21/2
51. Two circular flower beds have been shown on two sides O
(D) (s) 30.57 cm2
of a square lawn ABCD of side 56m. If the centre of each
circular flowered bed is the point of intersection O of the 60°
diagonals of the square lawn, then match the column. A B
Column-I Column-II OB = 14.8 m.
Area of the shaded region = ?
(A) Area of ∆OAB (p) 4032
A B
(B) Area of flower bed (q) 784 O
56m Fill in the Blanks
(C) Area of sector OAB (r) 448 D C
(D) Total area (s) 1232 DIRECTIONS : Complete the following statements with an
appropriate word / term to be filled in the blank space(s).
52. Column - I Column - II
θ 54. A sector of a circle is called a ................. sector if the minor
(A) Circumference (p) 2r + × 2πr
360° arc of the circle is a part of its boundary.
θ 55. The boundary of a sector consists of an arc of the circle
(B) Area of a quadrant (q) × πr 2
360° and the two .................. .

πr 2 56. The region enclosed by an arc and a chord is called the


(C) Length of the arc of the (r) ................. of the circle.
4
sector
57. Circumference of a circle is .................. .
θ
(D) Perimeter of the sector (s) × 2πr 58. Area of a circle is .................. .
360°
(E) Area of the sector (t) 2πr 59. Length of an arc of a sector of a circle with radius r and
53. Column-I Column-II angle with degree measure θ is ................. .
60. The area of a circle is the measurement of the region
enclosed by its ................. .
(A) (p) 30 cm2
61. If the area of a circle is 154 cm2, then its circumference is
................. .
62. Area of a sector of a circle with radius 6 cm, if angle of
the sector is 60°, is ................. .
OA = 26m, OC = 23m.
Area of shaded region = ?
M-62 Mathematics
67. Distance moved by a rotating wheel in one revolution is
True / False equal to the circumference of the wheel.
68. In a circle of radius 21 cm, an arc subtends an angle of 60°
DIRECTIONS : Read the following statements and write your
at the centre, then the length of the arc is 22 cm.
answer as true or false.
69. If the circumference of a circle is 88 cm, then its radius is
63. A segment corresponding a major arc of a circle is known 14 cm.
as the major segment.
70. The length of an arc of a sector of a circle of radius r units
64. If the boundary of a segment is a minor arc of a circle, θ
and of centre angle θ is × πr 2 .
then the corresponding segment is called a minor 360°
segment. 71. The length of a rope by which a cow must be tethered in
order that it may be able to graze of an area of 616cm2 is
65. A minor sector has an angle ‘θ’ subtended at the centre
18m.
of the circle, whereas a major sector has no angle.
66. The perimeter of a circle is generally known as its
circumference.
Areas Related to Circles M-63

ANSWER KEY & SOLUTIONS


1. (d) Let AB be the chord of circle such that ∠AOB = 90° πr 2
Let OA = 10 cm Area of 6 semi-circle = 6 × = 3pr2
2
22
∴ AB = 10 2 cm = 3× × 7× 7 = 462 cm2
7
Area of minor segment A X B
Area of cloth piece = 42 × 14 = 588 cm2
= Area of the sector AOB – Area of ∆AOB
Area of the coloured portion = 588 – 462 = 126 cm2
90° 1
= × π(10)2 − × 10 × 10 6. (b) Area of rectangle = 28 × 23 = 644 cm2
360° 2
Radius of semi-circle = 28 ÷ 2 = 14 cm
Radius of quadrant = 23 – 16 = 7 cm
O
Area of unshaded region
m
c

 1 22   1 22 
10

=  × × 14 × 14  + 2
 2 × 4 × 7 × 7 × 7  = 385 cm
A B 2 7   
X
\ Shaded area = (644 – 385) = 259 cm2
= 25 π – 50 = 25 × 3.14 – 50 = 78.5 – 50 = 28.5 cm2.
7. (a) Perimeter of sector = 25 cm
2. (a) Area of the shaded region
θ
40° 22 40° 22 ⇒ 2r + × 2pr = 25
= × × (7)2 − × × (3.5)2 360°
360° 7 360° 7
90° 22
1 22 1 22 49
= × × (7 2 − 3.52 ) = × ×  49 −  ⇒ 2r + 360° × 2 ×
7
× r = 25
9 7 9 7  4 
1 22 49 77 2 11 25
= × × × 3 = cm ⇒ 2r + r = 25 ⇒ r = 25 ⇒ r = 7
9 7 4 6 7 7
3. (b) We have,  πθ sinθ  2
Area of minor segment =  − r
Area of square metal plate = 40 × 40 = 1600 cm2  360° 2 
2
22  1  11  22 90° sin 90°  2
Area of each hole = πr 2 = × = cm 2 =  × −  (7)
7  2  14  7 360° 2 

11  11 1  4
∴ Area of 441 holes = 441× = 346.5cm 2 =  −  × 49 = × 49 = 14 cm2.
14  14 2  14
Hence, area of the remaining square plate 8. (a) Let the radii of the two circles be r1 and r2, then
= (1600 – 346.5) = 1253.5 cm2 r1 + r2 = 15       (given) ..... (i)
4. (c) Area of the remaining piece of paper πr12 + πr22 = 153π   
and (given)
= Area of trapezium – Area of 2 semi-circles
1 1 ⇒ r12 + r22 = 153  ..... (ii)
= (7 + 19)× 20 – 2 × pr2
2 2 On solving, we get
1 22 7 7 r1 = 12, r2 = 3
= × 26 × 20 – × × Required ratio = 12 : 3 = 4 : 1
2 7 2 2
= 260 – 38.5 = 221.5 cm2 9. (b)
2 πr1 = 503 and 2 πr2 = 437
42 503 437
5. (b) Diameter of each semi-circle = = 14 cm ∴ r1 = and r2 =
3 2π 2π
Radius of each semi-circle = 7 cm Area of ring = π (r1 + r2 ) (r1 − r2 )
M-64 Mathematics
 503 + 437   503 − 437  2
= π   r  2  81 
    then, area = π  r −  = πr  
2π 2π  10   100 
940  66  66  81 
=   = 235 × × 7 = 235 × 21 = 4935 sq. cm. Thus, area is diminished by 1 −  % = 19%
2  2π  22  100 
10. (b) πd1 + πd2 = πd ⇒ d1 + d2 = d 2πr
21. (b) Perimeter = + 2r
2
 2 1 22 2  = πr + 2r
11. (a) Required area =  7 – × × 7  cm2
 4 7 
⇒ (π + 2) r = 36
= (49 – 38.5) cm2 = 10.5 cm2
 36  r r
12. (a) Let the radii of the outer and inner circles be r1 and ⇒   – r = 36
r2 respectively; we have  7 
Area = pr12 – pr22 = p(r12 – r22) ⇒ r = 7 cm
= p(r1 – r2) (r1 + r2) Hence, diameter = 7 × 2 = 14 cm.
= p(5.7 – 4.3) (5.7 + 4.3) = p × 1.4 × 10 sq. cm
22. (a) Required area = π (r + d )2 − r 2 
= 3.1416 × 14sq. cm. = 43.98 sq. cms.  
13. (b) Area of sector = 240°/360° × p(100)2 = 20933 cm2. d
Let r be the radius of the new circle, then
20933
20933 = pr2 ⇒ r = = 81.6 cm. r
π
14. (d) 15. (b)
16. (c) Joining B to O and C to O
Let the radius of the outer cirlce be r = π[r2 + d2 + 2rd – r2]
∴ perimeter = 2πr = π[d2 + 2rd] = πd[d + 2r]
But OQ = BC = r [diagonals of the square BQCO] 23. (a) A
∴ Perimeter of ABCD = 4r. 53
m
m

2πr π
Hence, ratio = =
35

4r 2
17. (c) Radius of outer concentric circle = (35 + 7) m = 42 m.
B 66 m C
22
Area of path = π (422 – 352) m2 = (422 – 352) m2 Here, a = 66 m, b = 53 m & c = 35 m
7
a + b + c 66 + 53 + 35
1 s= = = 77m
18. (c) Perimeter = × 2πr + 2r 2 2
4
Area of ∆ = s(s − a)(s − b)(s − c)
 1 22 
=  × × 7 + 2 × 7  cm = 25 cm
2 7  So, area of ∆ = 77(11)(24)(42) = 924
19. (d) 2πr = 4π ⇒ r = 2
πr2 = 2(924)
Area = π(2)2 = 4π
When, 2πr = 8π 2 × 924 + 7 r
⇒ r 2 = ⇒ r2 = 588
⇒r=4 22

Area = 16π ⇒ r = 14 3 m

20. (b) Let r be the radius of circle, then area = πr2 24. (a) Area of square = 2 cm2
Side of square = 2 cm
When r is diminished by 10%
Areas Related to Circles M-65
We know,
2
OP = cm, OQ = x cm Area of common region
2
Q = 2 (Area of sector – Area of ∆APQ)

 60° 1 
x = 2  × π(2) 2 − × (2) 2 × sin 60°
2
360° 2 
P O  4π 4 3 
x
= 2  − 
2  6 4 
 2
( 2)
2
⇒ x2 = +   2 
 2  = 2  (3.14) − (1.73)
3
2
⇒ x2 = 2 + = 2 (2.09 – 1.73) = 2 (0.36) = 0.72.
4
∴ Area of region lie between 0.7 and 0.75.
5 2 5 26. (a) A
⇒ x = ⇒x= cm.
2 2

5 r
AC = 2 cm (AC = Diameter) r
2 O
r
D a C
B C

1  circumference of circle = 2pr ...(i)


  × d1 × d 2 
a a 2  Area of DABC = [ar(DAOB) + ar(DBOC) + ar(DAOC)]
O
1 1
= AB × r + × BC × r + AC × r
2 2
A a B
1 1
= r [AB + BC + AC] = r × 7p...(ii)
1 2 2
Area of square = × AC × BD
2 From (i) and (ii),
1 Circmference of circle 2pr 4
Area of square = × d1 × d 2 = =
Area of triangle 1
2 r ¥ 7p 7
2
1 5 5
= ×2 ×2 = 5 cm2 180°
2 2 2 27. (b) As 1 radian = 1 degree ×
π
25. (c) Given,
2π  2π 180° 
Two circle each of radius is 2 and difference between their \ radian =  × 
3  3 π 
centre is 2 3
120
1 \ Time = = 20 min.
AB = 2 3 ⇒ AC = AB 6
2 2
 7  π ( 49)
AC = 3 = CB 28. (d) Area of the circle = π   = = 49 cm2.
 π π
P
154 154 × 7
Now, consider = = 49 cm2
2 2
π 22
 C
A
3 3
B
πr12 25
29. (c) =
πr22 16
Q
r1 5
⇒ =
r2 4
AC 3
In ∆APC, cos θ = = (∠C = 90°) 2πr1 5 5 × 125
AP 2 625
⇒ = = =
⇒ θ = 30° 2πr2 4 4 × 125 500
M-66 Mathematics
θ 30° 2 49π 39. (b)
Side of square = 6 × 7 = 42 cm.
30. (b) Area = × πr 2 = × π ( 7) =
360° 360° 12 Area of square = 42 × 42 = 1764 cm2
θ 60° 22 2 132
40. (d)
Area of each circular
31. (a) (a) Area= × πr 2 = × × (6 ) = cm2
360° 360° 7 7 22
= p(7)2 = × 49
θ 7
(b) Area of minor sector = × πr 2
360° = 154 cm2
60° 22 41. (a) Area of remaining
= × × 14 × 14 = 102.57 cm2
360° 7 portion = 1764 – 9 × 154 = 378 cm2
Area of major sector
42. (b)
Here r = 14 mm
= Area of circle – Area of minor sector
Length of silverwire
22
= (14) 2 –102.57 = 2pr + 8r
7
22
= 615.44 – 102.57 = 512.87 cm2 =2× × 14 + 8 × 14
7
C 2π ( 5) 2 = 200 mm
(c) = 2
=
A π ( 5) 5 43. (c) Area of each sector
1 22
 θ  = × × 14 × 14 = 77 mm2
(d) Given,   2πr = 22 8 7
 360° 
44. (d)
Circumference of inner part = 44 mm
 θ  2  θ  πr
∴ Area of sector =   πr =   ( 2r ) 22
 360°   360°  2 ⇒2× × r = 44
7
 θ   r  22 × 6
=   2πr   = = 66 cm2 ⇒ r = 7 mm
 360°  2 2
outer radius = 7 + 3 = 10 mm
32. (d)
outer circumference
33. (a) Justification : Let the radius of the circle be r units.
22
When 2 π r = π r2, (numerically) =2× × 10 = 62.86 mm
7
We have, 2 r = r2, i.e. r = 2 45. (c) Difference of areas
34. (a) 35. (b) 36. (a) 22
= ( 102 – 72)
Sol. (37-41) A 7
= 51 p mm2
46. (c) Number of revolution
Distance
=
O Outer circumference

cm 80π
32 = = 4.
20π
30° 47. (a) Both assertion and reason are correct. Also, reason is
B D C
the correct explanation of the assertion.
BD 22
cos 30° =
37. (b) C = 2 × × r = 176
32 7
BD = 16 3 cm. 176 × 7
⇒ r = = 28 cm.
side BC = 32 3 cm 2 × 22
48. (a) Both assertion and reason are correct. Also, reason is
38. (c) AD = AB2 − BD 2
the correct explanation of the assertion.
= (32 3) 2 − (16 3) 2  10 2  6 2 

π
Area of the path =   –   
= 48 cm 2 2
    
= π(25 – 9) = 16π
Areas Related to Circles M-67
49. (d) Assertion is not correct, but reason is true. 1 1
Area (∆ABC) = × AC × BC = (5) (12) = 30 cm2
2πr = 22 2 2
⇒ r = 3.5 cm = 0.003 m2
22 (C) (AC)2 = (AB)2 + (BC)2 = 64 + 36 = 100
∴ Area of the circle = × 3.5 × 3.5 = 38.5 cm2.
7 AC = 10 cm
50. (A) → (q); (B) → (r); (C) → (p); (D) → (s) Area of the shaded region
51. (A) → (q); (B) → (r); (C) → (s); (D) → (p) = (area of the circle) – (area of the rectangle ABCD)
52. (A) → (t); (B) → (r); (C) → (s); (D) → (p); (E) → (q)  22  10 2 
53. (A) → (q); (B) → (p); (C) → (s); (D) → (r) =  ×    – (8 × 6)
 7  2  
(A) Area of the shaded region
= (78.57 – 48) = 30.57cm2
= 90° π ( 26)2 – ( 23)2  60° 22 2
360°   (D) Area of the shaded region = × × (14.8)
360° 7
1 22 
= × ( 26) 2 – ( 23) 2  = 115.5 ≈ 115 m2 = 114.7 m2
4 7 
54. minor 55. radii 56. segment
AB = 2 × AD = 2 × 6.5 = 13 cm
(B)
θ
2 2
57. 2πr. 58. πr2 59. × 2πr
BC = ( AB ) – ( AC ) 360°
60. boundary 61. 44 cm 62. 132/7 cm2
2 2
= (13) – ( 5) = 12 cm 63. True 64. True 65. True
66. True 67. True 68. True
69. True 70. False 71. False
8 Acids, Bases and
Probability
Salts
3 5
Multiple Choice Questions (MCQs) (a) (b)
5 12
7 4
DIRECTIONS : This section contains multiple choice (c) (d)
12 5
questions. Each question has 4 choices (a), (b), (c) and (d) out
of which only one is correct. 6.
Two fair dice are thrown. Find the probability that both
dice show different numbers.
1. Two dice are thrown at a time, then find the probability that 1 5
the difference of the numbers shown on the dice is 1. (a) (b)
6 6
3 5 32 29
(a) (b) (c) (d)
16 18 36 36
7 7
(c) (d) 7. A bag contains 40 coins, consisting of `2, `5 and
36 18
`10 denominations. If a coin is drawn at random, the
2. A book containing 100 pages is opened at random. Find 5
probability of drawing a `2 coin is . If x number of `2
the probability that a doublet page is found. 8
8 9 coins are removed from the bag and then a coin is drawn
(a) (b)
25 100 1
at random, the probability of drawing a `2 coin is . Find
7 11 the value of x. 2
(c) (d)
100 100 (a) 5 (b) 2
(c) 10 (d) 8
3.
When two dice are thrown, find the probability of getting
a number always greater than 4 on the second dice. 8.
A factory has 120 workers in January, 90 of them are
2 1 female workers. In February, another 15 male workers
(a) (b)
3 3 were employed. A worker is then picked at random.
3 2 Calculate the probability of picking a female worker.
(c) (d) 4
5 5 3
(a) (b)
4 9
4.
Two numbers are chosen from 1 to 5. Find the probability
2 1
for the two numbers to be consecutive. (c) (d)
3 2
3 2
(a) (b)
5 5 9.
A box contains a number of marbles with serial number 18
4 2 to 38. A marble is picked at a random. Find the probability
(c) (d) that it is a multiple of 3.
5 3
3 7
5. An unbiased die is rolled twice. Find the probability of (a) (b)
5 20
getting the sum of two numbers as a prime
3 1
(c) (d)
4 3
Probability M-69
10. One card is drawn from a well shuffled deck of 52 cards. 18. A single letter is selected at random from the word
I. The probability that the card will be diamond, is 1/2. “PROBABILITY”. The probability that the selected letter
II. The probability of an ace of heart is 1/52. is a vowel is
III. The probability of not a heart is 3/4. 2 3
(a) (b)
IV. The probability of king or queen is 1/26. 11 11
Which of the statement(s) is/are true? 4
(c) (d) 0
(a) I and II (b) II and III 11
(c) III and IV (d) None of these
19. A three digit number is to be formed using the digits 3, 4,
11. A fair die is thrown once. The probability of getting a
7, 8 and 2 without repetition. The probability that it is an
composite number less than 5 is
odd number is
1 1
(a) (b) 2 1
3 6 (a) (b)
5 5
2 4 3
(c) (d) 0 (c) (d)
3 5 5
12. If a letter is chosen at random from the letter of English
20. An urn contains 6 blue and ‘a’ green balls. If the probability
alphabet, then the probability that it is a letter of the word
of drawing a green ball is double that of drawing a blue
‘DELHI’ is
ball, then ‘a’ is equal to
1 1
(a) (b) (a) 6 (b) 18
5 26 (c) 24 (d) 12
5 21
(c) (d) 21. Two coins are tossed simultaneously. The probability of
26 26
getting at most one head is
13. The probability of raining on day 1 is 0.2 and on day 2 is 1 1
(a) (b)
0.3. The probability of raining on both the days is 4 2
(a) 0.2 (b) 0.1 3
(c) (d) 1
(c) 0.06 (d) 0.25 4
14. Which of the following cannot be the probability of an 22. If in a lottery, there are 5 prizes and 20 blanks, then the
event? probability of getting a prize is
(a) 2/3 (b) – 1/5 2 4
(c) 15% (d) 0.7 (a) (b)
5 5
15. The probability that a two digit number selected at random 1
(c) (d) 1
will be a multiple of ‘3’ and not a multiple of ‘5’ is 5
2 4
(a) (b) 23. Which of the following relationship is the correct ?
15 15
P (E) + P ( E ) = 1
(a)
1 4
(c) (d)
15 90 (b) P ( E ) – P(E) = 1

16. The probability of getting a number greater than 2 in P(E) = 1 + P ( E )


(c)
throwing a die is (d) None of these
(a) 2/3 (b) 1/3
(c) 4/3 (d) 1/4 24. For an event E, P (E) + P ( E ) = q, then
17. Out of one digit prime numbers, one number is selected at 0 ≤ q <1
(a) (b) 0 < q ≤ 1
random. The probability of selecting an even number is
(c) 0 < q < 1 (d) None of these
1 1
(a) (b)
2 4 25. A girl calculates that the probability of her winning the
first prize in a lottery is 0.08. If 6000 tickets are sold, how
4 2
(c) (d) many tickets has she bought?
9 5
(a) 40 (b) 240
(c) 480 (d) 750
M-70 Mathematics
26. Three - digit numbers formed by using digits 0, 1, 2 and 33. Two dice are rolled simultaneously. Find the probability
5 (without repetition) are written on different slips with that they show different faces.
distinct number on each slip, and put in a bowl. One slip 3 1
(a) (b)
is drawn at random from the bowl. The probability that 4 6
the slip bears a number divisible by 5 is 1 5
5 4 (c) (d)
(a) (b) 3 6
9 9
34. A die is thrown once then,
2 1
(c) (d) 2
3 3 (a) the probability of getting an odd number is
3
27. A box contains four cards numbered as 1, 2, 3 and 4 and (b) the probability of getting multiple of 3 is 1/3
another box contains four cards numbered as 1, 4, 9 and (c) the probability of getting a prime number is 2/3
16. One card is drawn at random from each box. What is (d) the probability of getting number greater than 5 is 1/3
the probability of getting the product of the two numbers
so obtained , more than 16? 35. The probability of an event can not be
5 1 (a) positive (b) negative
(a) (b) (c) zero (d) one
8 2
3 1 36. If the probability of an event is 0, then it is called a
(c) (d)
8 4 (a) sure event (b) certain event
28. From the data (1, 4, 7, 16, 27, 29) if 29 is removed, the (c) impossible event (d) none of these
probability of getting a prime number is 37. Which of the following statement(s) is/are not correct ?
1 1 (a) An event which can never happen is called impossible
(a) (b)
2 5 event.
2 1 (b) A n event which has more than one (favourable)
(c) (d) outcomes is called a compound event.
5 3
(c) T he collection of all possible outcomes of an
29. A bag contains card numbers 3, 4, 5, 6, 7....27. One card
experiment is called sample space.
is drawn, then probability of prime number card is
(d) None of these
9 8
(a) (b)
25 27
8 1
(c) (d)
25 5
DIRECTIONS : Study the given Case/Passage and answer the
30. Two dice are rolled, then probability of getting a total of following questions.
9 is
1 1 Case/Passage-I
(a) (b)
3 9 On a weekend Rani was playing cards with her family. The
9 8 deck has 52 cards. If her brother drew one card .
(c) (d)
10 9
31. From a bag containing 100 tickets numbered 1, 2, 3, .........,
100 one ticket is drawn. If the number on this ticket is x,
1
then the probability that x + > 2 is ......
x
(a) 0 (b) 0.99
(c) 1 (d) None of these
32. A coin is tossed. Then the probability of getting either head
or tail is
1 [From CBSE Question Bank-2021]
(a) 1 (b)
3 38. Find the probability of getting a king of red colour.
1 1 1 1 1
(c) (d) 1
2 4 (a) (b) (c) (d)
26 13 52 4
Probability M-71
39. Find the probability of getting a face card.
1 1 2 3
Assertion & Reason
(a) (b) (c) (d)
26 13 13 13
40. Find the probability of getting a jack of hearts. DIRECTIONS : Each of these questions contains an Assertion
followed by Reason. Read them carefully and answer the
1 1 3 3
(a) (b) (c) (d) question on the basis of following options. You have to select
26 52 52 26
the one that best describes the two statements.
41. Find the probability of getting a red face card.
(a) If both Assertion and Reason are correct and Reason is
3 1 1 1
(a) (b) (c) (d) the correct explanation of Assertion.
26 13 52 4
(b) If both Assertion and Reason are correct, but Reason is
42. Find the probability of getting a spade.
not the correct explanation of Assertion.
1 1 1 1 (c) If Assertion is correct but Reason is incorrect.
(a) (b) (c) (d)
26 13 26 4 (d) If Assertion is incorrect but Reason is correct.
Case/Passage-II 48. Assertion : If a box contains 5 white, 2 red and 4 black
Rahul and Ravi planned to play Business (board game) in marbles, then the probability of not drawing a white marble
which they were supposed to use two dice. 5
from the box is .
11

Reason : P ( E ) = 1 – P( E ), where E is any event.

49. Assertion : In rolling a dice, the probability of getting


number 8 is zero.
Reason : Its an impossible event.
[From CBSE Question Bank-2021]
50. Assertion : An event is very unlikely to happen. Its
43. Ravi got first chance to roll the dice. What is the
probability is 0.0001
probability that he got the sum of the two numbers
appearing on the top face of the dice is 8? Reason : If P(A) denote the probability of an event
1 5 1 A, then 0 < P(A) < 1.
(a) (b) (c) (d) 0
26 36 18 51. Assertion : If the probability of an event is P then
44. Rahul got next chance. What is the probability that he got probability of its complementary event will be 1 – P.
the sum of the two numbers appearing on the top face of Reason : When E and E are complementary events, then
the dice is 13? P(E) + P ( E ) = 1
5 1
(a) 1 (b) (c) (d) 0 52. Assertion : If a die is thrown, the probability of getting a
36 18 number less than 3 and greater than 2 is zero.
45. Now it was Ravi’s turn. He rolled the dice. What is Reason : Probability of an impossible event is zero.
the probability that he got the sum of the two numbers
appearing on the top face of the dice is less than or equal
to 12 ? Match the Following
5 1
(a) 1 (b) (c) (d) 0 DIRECTIONS : Each question contains statements given in
36 18
46. Rahul got next chance. What is the probability that he got two columns which have to be matched. Statements (A, B, C,
the sum of the two numbers appearing on the top face of D,............) in Column-I have to be matched with statements
the dice is equal to 7 ? (p, q, r, s,...............) in Column-II.
5 5 1
(a) (b) (c) (d) 0 53. Match the proposed probability under Column-I with the
9 36 6
appropriate written description under column-II :
47. Now it was Ravi’s turn. He rolled the dice. What is Column -I Column-II
the probability that he got the sum of the two numbers
appearing on the top face of the dice is greater than 8 ? (Probability) (Written Description)
5 1 5 (A) 0.95 (p) An incorrect assignment
(a) 1 (b) (c) (d)
36 18 18 (B) 0.02 (q) No chance of happening
M-72 Mathematics
(C) –0.3 (r) A
 s much chance of happening 62. If A is an event of a random experiment, then AC or A or
as not A′ is called the ..................of the event.
(D) 0.5 (s) Very likely to happen
63. A set of events which have no pair in common are called
(E) 0 (t) Very little chance of happening .............
54. Two unbiased coins are tossed simultaneously. Match 64. An outcome of a random experiment is called an .............
Column-I with the probabilities given in Column-II. event.
Column-I Column-II
3
(A) The probability (p) True / False
4
of getting one head is
1 DIRECTIONS : Read the following statements and write your
(B) The probability of (q)
4 answer as true or false.
getting at least one
head is
65. The sum of the probabilities of all the elementary events
1
(C) The probability of (r) of an experiment is 1.
2
getting two heads is
66. For any event E, P (E) + P ( E ) = 1, where E stands for
‘not E’. E and E are called complementary events.
Fill in the Blanks
67. The probability of an event can be greater than 1.

DIRECTIONS : Complete the following statements with an 68. If the probability of an event is 1, then it is an impossible
appropriate word / term to be filled in the blank space(s). event.
69. If A is any event in a sample space, then P( A) = 1 + P( A)
55. Probability of an event E + Probability of the event ‘not
E’ = .................... 70. The sum of probabilities of two students getting distinction
in their final examinations is 1.2.
56. The probability of an event that cannot happen is
................. Such an event is called .................. 71. A bag contains 5 red balls and some blue balls. If the
probability of drawing a blue ball is double that of a red
57. The probability of an event that is certain to happen is
ball, the number of blue balls in the bag is 10.
............... Such an event is called ................
72. A box contains 90 discs which are numbered from 1 to 90.
58. The sum of the probabilities of all the elementary events
If one disc is drawn at random from the box, the probability
of an experiment is ................
that it bears a two-digit number is 0.9
59. The probability of an event is greater than or equal to .........
73. An event A associated to a random experiment is said to
and less than or equal to ..............
occur if any one of the elementary events associated to the
60. If P(E) = 0.05, the probability of ‘not E’ is ........... event A is an outcomes.
61. A die is thrown once, the probability of getting a prime 74. An event associated to a random experiment is a
number is .............. compound event if it is obtained by combining two or more
elementary events associated to the random experiment.
Probability M-73

ANSWER KEY & SOLUTIONS


1. (b) n(S) = 6 × 6 = 36 6. (b) S = {(1, 1), ..., (1, 6), (2, 1), ..., (2, 6), (3, 1), ..., (3, 6),
(4, 1), ..., (4, 6), (5, 1), ..., (5, 6), (6, 1), ...., (6, 6)}
E= {(1, 2), (2, 1), (2, 3), (3, 2), (3, 4), (4, 3), (4, 5),
(5, 4), (5, 6), (6, 5)} n(S) = 36
n(E) = 10 Let E be the event that both dice show different
n(E) 10 5 numbers.
\ P(E) = = =
n(S) 36 18 E {(1, 2), (1, 3),...., (1, 6), (2, 1), (2, 3), (2, 4),...., (2, 6),
2. (b) S = {1, 2, 3, ......, 100} (3, 1), (3, 2), (3, 4), (3, 5), (3, 6), (4, 1), (4, 2), (4, 3),
(4, 5), (4, 6), (5, 1), (5, 2), (5, 3), (5, 4), (5, 6), (6, 1),
n(S) = 100 (6, 2), (6, 3), (6, 4), (4, 5)}
E = {11, 22, 33, 44, 55, 66, 77, 88, 99} n(E) = 30
n(E) = 9
n ( E) 30 5
9 \ P ( E) = = =
\ P(E) = n (S) 36 6
100
7. (c) n(S) = 40, let n(C) = C
3. (b) n(S) = 6× 6 = 36, E = {(1, 5), (2, 5), (3, 5), (4, 5),
(5, 5), (6, 5), (1, 6), (2, 6), (3, 6), (4, 6), (5, 6), (6, 6)} 5 C 5
P(C) = ⇒ = or C = 25
8 40 8
n(E) = 12
n(E) 12 1 25 − x 1
\ P(E) = = = Now, = ⇒ x = 10
n(S) 36 3 40 − x 2

4. (b) E = {(1, 2), (2, 3), (3, 4), (4, 5)} 8. (c) Initial number of workers = 120

n(E) = 4 When 15 male workers are added, then the total


number of workers = 120 + 15 = 135
n(E) 4 2
n(S) = 5 × 2 = 10 \ P(E) = = =
n(S) 10 5 Number of female workers = 90

5. (b) The sum of the two numbers lies between 2 and 12. 90 2
\ Probability of female workers = =
So, the primes are 2, 3, 5, 7, 11. 135 3
9. (d) Total number of marbles = 38 – 18 + 1 = 21
No. of ways for getting 2 = (1, 1) = 1
The multiples of 3 from 18 to 38 are 18, 21, 24, 27,
No. of ways of getting 3 = (1, 2), (2, 1) = 2
30, 33, 36.
No. of ways of getting 5
These are 7 in numbers
= (1, 4), (4, 1), (2, 3), (3, 2) = 4 7 1
∴ Required probability = =
No. of ways of getting 7 21 3
10. (b) Total number of cards = 52
= (1, 6), (6, 1), (2, 5), (5, 2), (3, 4), (4, 3) = 6
Total number of diamond cards = 13
No. of ways of getting 11 = (5, 6), (6, 5) = 2
I. P(diamond cards) = 13/52 = 1/4
No. of favourable ways = 1 + 2 + 4 + 6 + 2 = 15 II. P(an ace of heart) = 1/52
No. of exhaustive ways = 6 × 6 = 36 1 3
III. P(not a heart) = 1 − =
\ Probability of getting the sum as a prime 4 4
15 5 4 4 8 2
= = IV. P(king or queen) = + = =
36 12 52 52 52 13
M-74 Mathematics
11. (b) [Hint. The outcomes are 1, 2, 3, 4, 5, 6. Out of these, So, required probability of getting the product of the
4 is the only composite number which is less than 5]. 6 3
two numbers so obtained is =
12. (c) [Hint. The English alphabet has 26 letters in all. 16 8
The word ‘DELHI’ has 5 letter, so the number of No. of favourable outcomes 1
28. (b) Probability = =
favourable outcomes = 5.] Total number of outcomes 5
13. (c) P(raining on both day) = 0.2 × 0.3 = 0.06 29. (c) Total number of cards = 25
(Because both independent event) Prime number are 3, 5, 7, 11, 13, 17, 19, 23,
14. (b) 8
∴ Probability of prime number card =
15. (b) 24 out of the 90 two digit numbers are divisible by 25
‘3’ and not by ‘5’. 30. (b) No. of sample space = 6 × 6 = 36
24 4 Sum total of 9 = (3, 6), (4, 5), (5, 4), (6, 3)
The required probability is therefore, = .
90 15
4 1
∴ P = =
4 2 36 9
16. (a) Required probability = = .
6 3
31. (b) n(S) = [1, 2, 3, ..., 100] = 100
17. (b) [Hint. One digit prime numbers are 2, 3, 5, 7. Out of 1
these numbers, only the number 2 is even.] Q x + >2
x
1+ 2 +1 4
18. (c) Required probability = = . \ x2 + 1 > 2x
11 11
⇒ x2 – 2x + 1 > 0
19. (a) There are 2 favourable choice (3, 7) for unit place.
⇒ (x – 1)2 > 0
2 2
P = 1× 1× = x = [2, 3, ... ,100]
5 5
n(E) = [2, 3, 4, ..., 100] = 99
20. (d)
99
21. (c) Total outcomes = HH, HT, TH, TT P(E) = = 0.99
100
Favourable outcomes = HT, TH, TT 32. (a) 33. (d)
3 34. (b) A die is thrown once therefore, total number of
P(at most one head) = .
4 outcomes are {1, 2, 3, 4, 5, 6}
P(odd number) = 3/6 = 1/2
(a)
5 1
22. (c) Required probability = = .
25 5 (b) P(multiple of 3) = 2/6 = 1/3
P(prime number) = 3/6 = 1/2
(c)
23. (a) P (E) + P ( E ) = 1
P(greater than 5) = 1/6
(d)
24. (d) 25. (c)
35. (b) The probability of an event can never be negative.
26. (a) Total three digit number are : 3 × 3 × 2 = 18
36. (c)
Now, numbers divisible by 5 are :
37. (d) All the three statements are correct.
2 × 3 × 1 + 2 × 2 × 1 = 10
2 1
So, probability that the slip bears a number divisible 38. (a) P(king of red colour) = =
52 26
10 5 12 3
by 5 = = 39. (d) P(getting a face card) = =
18 9 52 13
27. (c) Possible products are 1, 4, 9, 16, 2, 8, 18, 32, 3, 12, 1
40. (b) P(getting a jack of hearts) =
27, 48, 4, 16, 36, 64 52
Probability M-75
3 49. (a) Assertion and Reason both are correct. Also Reason
41. (a) P(getting a red face card) = is the correct explanation of the Assertion.
26
13 1 50. (b) Assertion and Reason is correct but Reason is not
42. (d) P(getting a spade) = =
correct explanation for Assertion.
52 4
43. (b) Sum of the two numbers appearing on the top face 51. (a) Both statements are correct and Reason is the correct
of dice is 8. for Assertion.
52. (a) Both statements are correct. Event given in Assertion
(2, 6), (3, 5), (4, 4) (5, 3), (6, 2)
is an impossible event.
5 53. (A) → (s); (B) → (t); (C) → (p); (D) → (r); (E) → (q)
\ Required probability =
36 54. (A) → (r) ; (B) → (p) ; (C) → (q)
44. (d) Since, the sum of two numbers appearing on the top
Sample space = {HH, HT, TH, TT}
face of dice cannot be 13.
2 1
So, required probability = 0. A : P (one head) = =
4 2
45. (a) Since, the pair of number whose sum is less than 0
(∵ favourable event = HT, TH)
or equal to 12 in a pair of dice is 36.
3
36 B : P (at least one head) =
\ Required probability = =1 4
36
46. (c) Since, the pair of numbers on the top of dice whose (∵ favourable event = HH, HT, TH)
sum is 7 are (1, 6), (2, 5), (3, 4), (4, 3), (5, 2) , (6, 1)
6 1 1
\ Required probability = = C : P (two heads) = (∵ favourable event = HH)
36 6 4
5 55. 1 56. 0, impossible event
47. (d)
18 57. 1, sure or certain event 58. 1
59. 0, 1 60. .95
48. (d) Assertion is not correct, but reason is correct.
61. 1/2 62. complement
5 5
P(white marble) = = . 63. mutually exclusive 64. elementary
5 + 2 + 4 11
65. True
66. True 67. False 68. False
5 11 – 5 6 69. False 70. True 71. True 72. True
P(not white marble) = 1 – = = .
11 11 11
73. True 74. True
9 Acids, Bases and
Quadratic
Salts
Equations
7. Find the product of the roots of x2 + 8x – 16 = 0
Multiple Choice Questions (MCQs) (a) 8 (b) –8
(c) 16 (d) –16
DIRECTIONS : This section contains multiple choice
questions. Each question has 4 choices (a), (b), (c) and (d) out If the roots of the equation ax2 + bx + c = 0 are α and β,
8.
of which ONLY ONE is correct. then the quadratic equation whose roots are –α and –β
is _____ .
1. If the product of roots of the equation x3 – 3x + k = 10 is (a) ax2 – bx – c = 0 (b) ax2 – bx + c = 0
–2, then the value of k is
ax2 + bx – c = 0
(c) (d) ax2 – bx + 2c = 0
(a) –2 (b) –8
(c) 8 (d) 12 9. If the equation
(1 + m2) x2 + (2mc) x + (c2 – a2) = 0 has equal roots,
2. If one root of 5x2 + 13x + k = 0 be the reciprocal of the
then
other root, then the value of k is
(a) c2 – a2 = 1 + m2 (b) c2 = a2 (1 + m2)
(a) 0 (b) 1
c2a2 = (1 + m2) (d)
(c) c2 + a2 = 1 + m2
(c) 2 (d) 5
10. Which of the following satisfy the equation
3.
If the sum of the roots of a quadratic equation is 6 and their
product is 6, the equation is a2b2x2 + b2x – a2x – 1 = 0
(a) x2 – 6x + 6 = 0 (b) x2 + 6x – 6 = 0 1 1
(a) 2 (b) 2
x2 – 6x – 6 = 0
(c) (d) x2 + 6x + 6 = 0 a b
4. If the equation x2 + 2(k + 2)x + 9k = 0 has equal roots, then −1
(c) (d) None of these
k=? b2
(a) 1 or 4 (b) –1 or 4
11. The roots of the quadratic equation x2 – 0.04 = 0 are
(c) 1 or – 4 (d) –1 or – 4
(a) ± 0.2 (b) ± 0.02
5. If the roots of 5x2 – kx + 1 = 0 are real and distinct, then
(c) 0.4 (d) 2
−2 5 < k < 2 5
(a)
12. One of the two students, while solving a quadratic equation
(b)
k > 2 5 only in x, copied the constant term incorrectly and got the roots
(c)
k < −2 5 only 3 and 2. The other copied the constant term and coefficient
of x2 correctly as –6 and 1 respectively. The correct roots
(d) either k > 2 5 or k < −2 5 are
If a – b, b – c are the roots of ax2 + bx + c = 0, then find
6. (a) 3, –2 (b) –3, 2
(a − b)(b − c) (c) –6, –1 (d) 6, –1
the value of
c−a 13. The condition for one root of the quadratic equation
b c ax2 + bx + c = 0 to be twice the other, is
(a) (b)
c b b2 = 4ac
(a) (b) 2b2 = 9ac
ab bc c2 = 4a + b2
(c) (d) c2 = 9a – b2
(c) (d)
c a
Quadratic Equations M-77
2 2 24. If the equation (m2 + n2) x2 –2 (mp + nq) x + p2 + q2 = 0
1 3
14. If  x −  −  x −  = x + 2 , then x = has equal roots, then
 2  2
mp = nq
(a) mq = np
(b)
(a) 3 (b) 2
(c) 4 (d) None of these mn = pq
(c) mq =
(d) np

15. If x2 + y2 = 25, xy = 12, then x = 25. Each root of x2 – bx + c = 0 is decreased by 2. The resulting
(a) {3, 4} (b) {3, –3} equation is x2 – 2x + 1 = 0, then
(c) {3, 4, –3, –4} (d) {–3, –3} b = 6, c = 9
(a) (b) b = 3, c = 5

1 b = 2, c = –1
(c) (d) b = – 4, c = 3
16. If x = 7 + 4 3 , then x + = 26. Two distinct polynomials f (x) and g(x) are defined as
x
(a) 4 (b) 6 follows:
f (x) = x2 + ax + 2; g (x) = x2 + 2x + a.
(c) 3 (d) 2
If the equations f (x) = 0 and g(x) = 0 have a common
17. If the roots of the equation px 2 + 2qx + r = 0 and root, then the sum of the roots of the equation
qx 2 − 2 pr x + q = 0 be real, then f (x) + g(x) = 0 is

(a) p = q (b) q2 = pr 1
-
(a) (b) 0
2 2
(c) p = qr (d) r2 = pq
18. The equation 2x2 + 2(p + 1) x + p = 0, where p is real, 1
(c) (d) l
always has roots that are 2
(a) Equal 27. If a and b are the roots of the quadratic equation
x2 – 6x – 2 = 0 and if an = an – bn, then the value of
(b) Equal in magnitude but opposite in sign a10 − 2a 8
(c) Irrational is
2a 9
(d) Real
(a) 6.0 (b) 5.2
19. If the ratio of the roots of the equation x2 + bx + c = 0 is
the same as that of x2 + qx + r = 0, then (c) 5.0 (d) 3.0
(a) r2b = qc2 (b) r2c = qb2 28. Consider the quadratic equation nx 2 + 7 nx + n = 0,
(c) c2r = q2b (d) b2r = q2c where n is a positive integer. Which of the following
statements are necessarily correct?
20. The real roots of the equation x 2/3 + x1/3 − 2 = 0 are
I. For any n, the roots are distinct.
(a) 1, 8 (b) –1, –8
II. There are infinitely many values of n for which both
(c) –1, 8 (d) 1, –8
roots are real.
21. Which of the following is not a quadratic equation?
III. The product of the roots is necessarily an integer.
(a) x2 – 2x + 2 (3 – x) = 0
(a) III only (b) I and III
x (x + 1) + 1 = (x – 2) (x – 5)
(b)
(c) (2x – 1) (x – 3) = (x + 5) (x – 1) (c) II and III (d) I, II and III

(d) x3 – 4x2 – x + 1 = (x –2)3 29. Two quadratic equations x2 – bx + 6 = 0 and x2 – 6x + c – 0


have a common root. If the remaining roots of the first and
22. If one root of the quadratic equation ax2 + bx + c = 0 is
the reciprocal of the other, then second equations are positive integers and are in the ration
3 : 4 respectively, then the common root is
(a) b = c (b) a=b
(c) ac = 1 (d) a = c (a) 1 (b) 2

1 1 (c) 3 (d) 4
23. The roots of the equation x + = 3 , x ≠ 0, are 30. The values of k, so that the equations 2x2 + kx – 5 = 0 and
x 3
x2 – 3x – 4 = 0 have one root in common, are
1
(a) 3, 1 (b) 3, 27 27
3 (a) 3, (b) 9,
2 4
1 1 − 27 4
(c) 3, − (d) – 3, − (c) – 3, (d) 3,
3 3 4 27
M-78 Mathematics
38. Which of the following equations has 2 as a root?
3+ 5
31. If x = and y = x3, then y satisfies the quadratic (a) x2 – 4x + 5 = 0
2
equation x2 + 3x – 12 = 0
(b)
(a) y2 – 18y + 1 = 0 (b) y2 + 18y + 1 = 0 (c) 2x2 – 7x + 6 = 0
(c) y2 – 18y – 1 = 0 (d) y2 + 18y – 1 = 0 (d) 3x2 – 6x – 2 = 0
32. Let b be a non-zero real number. Suppose the quadratic 39. Values of k for which the quadratic equation 2x2– kx + k = 0
1 has equal roots is
equation 2x2 + bx + = 0 has two distinct real roots.
Then b (a) 0 only (b) 4 only

1 5 1 5 (c) 8 only (d) 0, 8


b + >
(a) (b) b+ < 40. If α, β are roots of the equation x2 – 5x + 6 = 0, then the
b 2 b 2
equation whose roots are α + 3 and β + 3 is
1
b2 – 3b > –2
(c) (d) b2 + <4 (a) 2x2 – 11x + 30 = 0 (b) –x2 + 11x = 0
b2
x2 – 11x + 30 = 0
(c) (d) 2x2 – 22x + 40 = 0
33. If the quadratic equations 2x2 + 4x + (a + 5) = 0 have equal
roots and (a + 4)x2 + ax – 3b = 0 have distinct real roots 41. If equation x2 – (2 + m) x + 1 (m2 – 4m + 4) = 0 has equal
then which of the following is true: roots, then:
3 3 (a) m = 0 (b) m = 6
(a) a = –3, b < (b) a = 3, b >
4 4 m = 2
(c) (d) m = 3
3 3 42. Which of the following equations have no real roots?
a = –3, b > – (d)
(c) a = 3, b <
4 4
x 2 − 2 3 x + 5 = 0 (b)
(a) 2 x2 + 6 2 + 8 = 0
34. The value of l such that sum of the squares of the roots of
(c) x 2 − 2 3 x − 5 = 0 (d) 2 x2 − 6 2x − 9 = 0
the quadratic equation, x2 + (3 – l)x + 2 = l has the least
value is: 43. Two numbers whose sum is 8 and the absolute value of
whose difference is 10 are roots of the equation
15
(a) (b) 1 x2 – 8x + 9 = 0
(a) (b) x2 – 8x – 9 = 0
8
x2 + 8x – 9 = 0
(c) (d) –x2 + 8x + 9 = 0
(c) (d) 2 44. If α, β are roots of x2 + 5x + a = 0 and 2α + 5β = –1,
then
35. If α and β be two roots of the equation x2 – 64x + 256 = 0.
1 1 (a)
α = 8 (b) β = –3
 α 3  8  β3  8 (c) α = 9 (d) a = – 24
Then the value of  5  +  5  is:
β  α  45. The value of p for which the difference between the roots
of the equation x2 + px + 8 = 0 is 2, are
(a) 2 (b) 3
(a) 4 (b) 8
(c) 1 (d) 4
(c) 6 (d) – 4
36. Which one of the following is not a quadratic equation? 2
46. If the roots of x + px + 12 = 0 are in the ratio 1 : 3, then
(a) (x + 2)2 = 2(x + 3) value(s) of p are
x2 + 3x = (–1) (1 – 3x)2
(b) (a) 3 (b) 8
(c) (x + 2) (x – 1) = x2 – 2x – 3 (c) 6 (d) – 3
x3 – x2 + 2x + 1 = (x + 1)3
(d) 47. Roots of quadratic equation x2 – 3x + 2 = 0 are
(a) 3 (b) –1
37. Which constant should be added and subtracted to solve
(c) 2 (d) 4
the quadratic equation 4 x 2 – 3 x – 5 = 0 by the method
48. If x = 2 and x = 3 are roots of the equation
of completing the square?
3x2 – 2px + 2q = 0, then
9 3
(a) (b) 2
16 64 p=
(a) (b) p = 15
15
3 3
(c) (d) q = 9
(c) (d) 6p + 2q = 27
4 4
Quadratic Equations M-79
55. Which is the correct quadratic equation for the speed of
the current ?
(a) x2+ 30x − 200 = 0 (b) x2+ 20x − 400 = 0
DIRECTIONS : Study the given Case/Passage and answer the (c) x 2+ 30x − 400 = 0 (d) x2− 20x − 400 = 0
following questions. 56. What is the speed of current ?
Case/Passage-I (a) 20 km/hour (b) 10 km/hour
Raj and Ajay are very close friends. Both the families decide to
(c) 15 km/hour (d) 25 km/hour
go to Ranikhet by their own cars. Raj’s car travels at a speed of
x km/h while Ajay’s car travels 5 km/h faster than Raj’s car. Raj 57. How much time boat took in downstream?
took 4 hours more than Ajay to complete the journey of 400 km. (a) 90 minute (b) 15 minute
[From CBSE Question Bank-2021] (c)
30 minute (d)
45 minute

Assertion & Reason

DIRECTIONS : Each of these questions contains an Assertion


followed by Reason. Read them carefully and answer the
question on the basis of following options. You have to select
the one that best describes the two statements.
(a) If both Assertion and Reason are correct and Reason is
49. What will be the distance covered by Ajay’s car in two the correct explanation of Assertion.
hours? (b) If both Assertion and Reason are correct, but Reason is
(a) 2(x +5)km (b) (x – 5)km not the correct explanation of Assertion.
(c) 2( x + 10)km (d) (2x + 5)km (c) If Assertion is correct but Reason is incorrect.
50. Which of the following quadratic equation describe the (d) If Assertion is incorrect but Reason is correct.
speed of Raj’s car? 58. Assertion : If roots of the equation x2 – bx + c = 0 are two
(a) x2 – 5 x – 500 = 0 (b) x2 + 4x - 400 = 0 consecutive integers, then b2 – 4c = 1.
(c) x2 + 5x – 500 = 0 (d) x2 – 4x + 400 = 0 Reason : If a, b, c are odd integer then the roots of the
51. What is the speed of Raj’s car? equation 4abc x2 + (b2 – 4ac) x – b = 0 are real and distinct.
(a) 20 km/hour (b) 15 km/hour 59. Assertion : (2x – 1)2 – 4x2 + 5 = 0 is not a quadratic
(c) 25 km/hour (d) 10 km/hour equation.
52. How much time took Ajay to travel 400 km? Reason : x = 0, 3 are the roots of the equation 2x2 – 6x = 0.
(a) 20 hour (b) 40 hour 60. Assertion : The equation 9x2 + 3kx + 4 = 0 has equal roots
for k = ± 4.
(c) 25 hour (d) 16 hour
Reason : If discriminant ‘D’ of a quadratic equation is
Case/Passage-II equal to zero then the roots of equation are real and equal.
The speed of a motor boat is 20 km/hr. For covering the 61. Assertion : 4x2 – 12x + 9 = 0 has repeated roots.
distance of 15 km the boattook 1 hour more for upstream than
Reason : The quadratic equation ax2 + bx + c = 0 have
downstream. [From CBSE Question Bank-2021]
DOWNSTREAM(a) UPSTREAM(b)
repeated roots if discriminant D > 0.
62. Assertion : A quadratic equation ax2 + bx + c = 0, has two
distinct real roots, if b2 – 4ac > 0.
Reason : A quadratic equation can never be solved by
using method of completing the squares.
63. Assertion : Sum and product of roots of 2x2 – 3x + 5 = 0
53. Let speed of the stream be x km/hr. then speed of the 3 5
motorboat in upstream will be are and respectively.
2 2
(a) 20 km/hr (b) (20 + x) km/hr
Reason : If α and β are the roots of ax2 + bx + c = 0, a ≠ 0,
(c) (20 - x) km/hr (d) 2 km/hr
54. What is the relation between speed ,distance and time? b
then sum of roots = α + β = – and product of roots
(a) speed = (distance )/time (b) distance = (speed )/time a
c
(c) time = speed x distance (d) speed = distance x time = αβ = .
a
M-80 Mathematics
70. The equation ax2 + bx + c = 0, a ≠ 0 has no real roots, if
Match the Following .........
71. The values of k for which the equation 2x2 + kx + x + 8 = 0
DIRECTIONS : Each question contains statements given in
will have real and equal roots are .............
two columns which have to be matched. Statements (A, B, C, D)
in column-I have to be matched with statements (p, q, r, s) in 72. If α, β are roots of the equation ax2 + bx + c = 0, then the
column-II. quadratic equation whose roots are aα + b and aβ + b is
.............
64. Column-II give roots of quadratic equations given in
Column-I. 73. If r, s are roots of ax2 + bx + c = 0, then is ..........
Column-I Column-II
74. The quadratic equation whose roots are the sum and
(A) 6x2 + x – 12 = 0 (p) (– 6, 4) difference of the squares of roots of the equation
(B) 8x2 + 16x + 10 = 202 (q) (9, 36) x2 – 3x + 2 = 0 is....
x2 – 45x + 324 = 0
(C) (r) (3, –1/2) 75. If a, b are the roots of x2 + x + 1 = 0, then a2 + b2 = ...........
(D) 2x2 – 5x – 3 = 0 (s) (–3/2, 4/3)
76. If α, β are the roots of x2 + bx + c = 0 and α + h, β + h are
65. Column-I Column-II the roots of x2 + qx + r = 0, then h = ..........
(A) (x – 3) (x + 4) + 1 = 0 (p) Forth degree polynomial
77. A quadratic equation cannot have more than ........ roots.
(B) (x + 2)3 = 2x (x2 – 1) (q) Quadratic equation
78. Let ax2 + bx + c = 0, where a, b, c are real numbers,
(C) (2x – 2)2 = 4x2 (r) Non-quadratic equation
a ≠ 0, be a quadratic equation, then this equation has no
(D) (2x2 – 2)2 = 3 (s) Linear equation real roots if and only if ........... .
66. Column-I Column-II
(A) If α, β are roots of (p) a < 0, b > 0 True / False
ax2 + bx + c = 0,
then one of the roots DIRECTIONS : Read the following statements and write your
of the equation answer as true or false.
ax2 – bx (x –1)
+ c (x –1)2 = 0 79. A quadratic equation cannot be solved by the method of
completing the square.
(B) If the roots of (q) real and equal
ax2 + b = 0 are real, 80. If we can factorise ax2 + bx + c, a ≠ 0, into a product of
two linear factors, then the roots of the quadratic equation
then β ax2 + bx + c = 0 can be found by equating each factor to
(C) Roots of 4x2 – 4x + 1 = 0 (r)
1+ β zero.
(D) Roots of (s) Real 81. (x – 2) (x + 1) = (x – 1) (x + 3) is a quadratic equation.
(x – a) (x – b) + (x – b) 82 (x2 + 3x + 1) = (x – 2)2 is not a quadratic equation.
(x – c) + (x – c) (x – a) = 0
83. x2 + x – 306 = 0 represent quadratic equation where product
are always
of two consecutive positive integer is 306.

Fill in the Blanks 84. The roots of the equation (x – 3)2 = 3 are 3 ± 3
85. If sum of the roots is 2 and product is 5, then the quadratic
DIRECTIONS : Complete the following statements with an equation is x2 – 2x + 5 = 0
appropriate word/ term to be filled in the blank space(s).
86. Sum of the reciprocals of the roots of the equation
67. A quadratic equation in the variable x is of the form x2 + px + q = 0 is 1/p.
ax2 + bx + c = 0, where a, b, c are real numbers and a ......
87. The nature of roots of equation x 2 + 2 x 3 + 3 = 0 are real
68. A quadratic equation ax2 + bx + c = 0 has two distinct real
and equal.
roots, if b2 – 4ac ..............
69. The altitude of a right triangle is 7 cm less than its 88. For the expression ax2 + 7x + 2 to be quadratic, the possible
base. If the hypotenuse is 13 cm, the other two sides values of a are non-zero real numbers.
are ...............
Quadratic Equations M-81

ANSWER KEY & SOLUTIONS


11. (a) x2 – 0.04 = 0
1. (c) Given equation is x 2 − 3 x + (k − 10) = 0 .
⇒ x2 = 0.04
\ Product of roots = (k – 10).
⇒ x = ± 0.2
So, k – 10 = –2 ⇒ k = 8.
1 12. (d) Let a, b be the roots of the equation. Then a + b = 5
2. (d) Let the roots be α and . Then, and ab = –6. So, the equation is x2 – 5x – 6 = 0.
α
 1 The roots of the equation are 6 and –1.
product of roots =  α ×  = 1. b c
 α b
k 13. (b) α + 2α = − and α × 2α = ⇒ 3α = −
So, = 1 ⇒ k = 5. a a a
5 b 2 c  −b 
2
c
⇒ α = − and 2α = ⇒ 2   =
Required equation is x2 – 6x + 6 = 0.
3. (a) 3a a  3a  a
4. (a)
Since the roots are equal, we have D = 0. 2b 2 c
⇒ = ⇒ 2ab2 – 9a2c = 0 ⇒ a(2b2 – 9ac) = 0
2 2 a
∴ 4(k + 2) − 36k = 0 ⇒ (k +2)2 – 9k = 0 9a
Since a ≠ 0, ∴ 2b2 = 9ac
∴ k2 – 5k + 4 = 0 ⇒ k2 – 4k – k + 4 = 0
Hence, the required condition is 2b2 = 9ac
⇒ k(k – 4) – (k – 4) = 0
⇒ (k – 4)(k – 1) = 0 ⇒ k = 4 or k = 1. 14. (c) Use options or apply the formula

5. (d) The roots of 5x2 – kx + 1= 0 are real and distinct. a2 – b2 = (a – b) (a + b), we get x = 4

∴ (k2 – 4 × 5 × 1) > 0 ⇒ k2 > 20 15. (c) x2 + y2 = 25, xy = 12


2
⇒ k > 20 or k < – 20 ⇒ k > 2 5 or k < −2 5 .  12 
⇒ x 2 +   = 25 ⇒ x 4 + 144 − 25 x 2 = 0
c  x
6. (b)
(i) (a – b) (b – c) = product of the roots =
.
a ⇒ (x2 – 16) (x2 – 9) ⇒ x2 = 16 and x2 = 9
(ii) c – a = –(a – b + b – c) = – (sum of the roots) ⇒ x = ± 4 and x = ± 3
= b . 16. (a) We have, x = 7 + 4 3
a
x2 + 8x – 16 = 0
7. (d) 1 1 7−4 3
\ = = = 7−4 3
c x 7+4 3 7 + 4 3. 7 − 4 3
The product of the roots = = – 16.
a
1
8. (b) Use x2 – (α + β)x + αβ = 0 \ x + = 7+4 3 + 7−4 3
x
9. (b) Since the equation has two equal roots, D = 0 = ( 3 + 2) + (2 − 3) = 4
⇒ (2mc)2 – 4(1 + m2) (c2 – a2) = 0
17. (b) Equation px2 + 2qx + r = 0 and qx 2 − 2 pr x + q = 0
⇒ 4m2c2 – 4c2 + 4a2 – 4m2c2 + 4m2a2 = 0
have real roots then from first
⇒ –4c2 + 4a2 + 4m2a2 = 0 ⇒ 4c2 = 4a2 + 4m2a2
4q 2 − 4 pr ≥ 0 ⇒ q 2 ≥ pr ... (i)
⇒ 4c2 = 4a2 (1 + m2) ⇒ c2 = a2 (1 + m2)
10. (b) a2b2x2 + b2x – a2x – 1 = 0 and from second 4( pr ) − 4q 2 ≥ 0 (for real root)

⇒ b2x (a2x + 1) – 1(a2x + 1) = 0 ⇒ pr ≥ q 2 ... (ii)


⇒ (a2x + 1) (b2x – 1) = 0 From (i) and (ii), we get result q2 = pr
1 1 18. (d) The discriminant of a quadratic equation
⇒ x=− 2
,
a b2
ax2 + bx + c = 0 is given by b2 – 4ac.
M-82 Mathematics
a = 2, b = 2(p + 1) and c = p 26. (c) Given,
b2 – 4ac = [2(p + 1)]2 – 4(2p) = 4(p + 1)2 – 8p f(x) = x2 + ax + 2 and g(x) = x2 + 2x + a
= 4[(p + 1)2 – 2p] = 4[(p2 + 2p + 1) – 2p] = 4(p2 + 1) Let a be the common root of f(x) = 0 and g(x) = 0.

For any real value of p, 4(p2 + 1) will always be \ a2 + aa + 2 = 0 ...(i)


positive as p2 cannot be negative for real p. and a2 + 2a + a = 0 ...(ii)
Hence, the discriminant b2 – 4ac will always be positive. Using elimination method,
When the discriminant is greater than ‘0’ or is positive, a2 + aa = – 2
then the roots of a quadratic equation will be real. a2 + 2a = – a
and
19. (d) Let 1, 2 be the roots of equations (i), 2 and 4 be the – + +
roots of equation (ii). aa – 2a = – 2 + a
\ equations are x2 – 3x + 2 = 0 and x2 – 6x + 8 = 0. -2 + a
⇒ a(–2 + a) = –2 + a ⇒ a = =1
Comparing with x2 + bx + c = 0 and x2 + qx + r = 0, -2 + a
we get b = – 3, c = 2, q = – 6 and r = 8. Substitute value of a in (i) eqn., we get
Putting these values in the options, we find that option \ – (a + 2) = 1 ⇒ a + 2 = –1 ⇒ a = –3
(d) is satisfied.
Now f (x) + g(x) = 0
20. (d) The given equation is x2/3 + x1/3 – 2 = 0 \ x2 – 3x + 2 + x2 + 2x – 3 = 0 ⇒ 2x2 – x – 1 = 0
Put x1/3 = y, then y2 +y–2=0 1
So, sum of roots =
⇒ (y – 1) (y + 2) = 0 ⇒ y = 1 or y = – 2 2
⇒ x1/3 = 1 or x1/3 = – 2 27. (d) x2 – 6x – 2 = 0
∴ x = (1)3 or x = (–2)3 = – 8 a and b are the roots of the above equation.
Hence, the real roots of the given equations are 1, –8. So, a2 – 2 = 6a
21. (b) x (x + 1) + 1 = (x – 2) (x – 5) Similarly, b2 – 2 = 6b
⇒ x2 + x + 1 = x2 – 7x + 10 We can see that, a + b = 6 and ab = – 2
⇒ 8x – 9 = 0, which is not a quadratic equation. Given: an = an – bn

22. (d) If one root is α, then the other is


1
So,
10 10
a10 − 2a 8 α − β − 2 α − β
=
8 8
( )
1 c c
α 2a 9 2 α 9 − β9 ( )
∴ α. = product of roots = ⇒ 1= ⇒ a = c
α a a
=
α10 − β10 + αβ α8 − β8 ( )
23. (b) x +
1 10
=
x 3

x + 1 10
x
=
2

3
⇒ 3 x 2 − 10 x + 3 = 0 (
2 α 9 − β9 )
⇒ (x – 3) (3x – 1) = 0 ⇒ x = 3, x =
1
=
α10 − α 9β − αβ9 + β10 ( )
24. (b) ‘b2 = 4ac’
3
(
2 α −β 9 9
)


⇒ 4 (mp +
⇒ m2q2 +
nq)2
n2p2
=4 (m2
– 2mnpq = 0
+ n2) (p2 + q2)
α 9 ( α + β ) − β9 ( α − β ) (
( α + β ) α 9 − β9 )
⇒ (mq – np)2 = 0 ⇒ mq – np = 0.
=
(
2 α −β 9 9
) =
( 9
2 α −β 9
)
25. (a) α + β = b, αβ = c 6
= = 3 (Q a + b = 6)
Sum of roots of resulting equation = (α – 2) + (β – 2) 2
⇒ (α + β – 4 ) = b – 4 ; 28. (b) The given quadratic equation is, nx2 + 7 n x + n = 0
Product of roots resulting equation Now, the discriminant,
= (α – 2) (β – 2) = αβ – 2 (α + β ) + 4 D = 49n – 4n2 = n (49 – 4n)
= c – 2b + 4 D ≠ 0; \ ∀ n ∈ I+
Now, 2 = b – 4 ; 1 = c – 2b + 4 etc.
⇒ Roots of the quadratic equation are distinct.
Quadratic Equations M-83
For real roots D ≥ 0 18 – 3k (k + 6) – 5 (k + 6)2 = 0
49 18 – 3k2 – 18k – 5k2 – 180 – 60k = 0
⇒ n(49 – 4n) ≥ 0 ⇒ n ≤
4
–8k2 – 78k – 162 = 0
So, n ∈ {1, 2, 3, 4, ..., 12}
8k2 + 78k + 162 = 0
So, x have only finite value. 4k2 + 39x + 81 = 0
n 4k2 + 27k + 12k + 81 = 0
Product of roots = =1
n
k(4k + 27) + 3 (4k + 27) = 0
⇒ Products of root is necessarily integer.
(k + 3) (4k + 27) = 0
Hence, option (b) is correct.
27
29. (b) Let a, b be the roots of x2 – bx + 6 = 0 and a, g be the k = −3, −
4
roots of x2 – 6x + c = 0
3+ 5
x2 – bx + 6 = 0; α + β = b, α + γ = 6 31. (a) x =
2
x2 – 6x + c = 0; αβ = 6, αγ = c 3
3 Ê3+ 5ˆ 27 + 5 5 + 9 5(3 + 5)
β 3 fi x = Á ˜ = = 9+4 5
Given, = Ë 2 ¯ 8
γ 4
αβ 6 ⇒ y = x3 = 9 + 4 5
=
αγ c ⇒ One root is 9 + 4 5 \ other is root 9 – 4 5
β 6 3 6
= ⇒ = ∴ c=8 \ Sum of roots = 9 + 4 5 + 9 – 4 5 = 18
γ c 4 c
Product of roots = (9 + 4 5 ) (9 – 4 5 ) = 1
ab = 6; ag = 6
\ Required equation is: y2 – 18y + 1 = 0
HCF (α β, α γ ) = α
32. (c) D > 0 (for real roots)
HCF (6, 8) = 2
1 b3 − 8
a=2 ⇒ b2 − 4 × 2 × > 0 ⇒ >0
b b
30. (c) Let the common root be t 1.5
(b − 2)(b 2 + 2b + 4) 0 3
Then, the equation becomes ⇒ >0
b
2t2 + kt – 5 = 0 ...(i)
⇒ b ∈ ( −∞, 0) ∪ (2, ∞)
t2 – 3t – 4 = 0 ...(ii)
Clearly options A and B are wrong
Multiply equation (ii) by 2 and then subtract from
Let f(b) = b2 – 3b
equation (i)
range of f (b) when b ∈ (–∞, 0) ∪ (2, ∞)
2t2 + kt – 5 = 0
= (f(2), ∞) = (–2, ∞)
2t2 – 6t – 8 = 0
⇒ b2 – 3b > – 2 is correct
– + +
So, b ∈ (–∞, 0) ∪ (2, ∞) is subset of solution set of
(k + 6)t + 3 = 0 b2 – 3b + 2 > 0
3 1
t= −
Also D is wrong as b 2 + 2 ∈ (0, ∞)
k +6 b
Now, put the value of t in equation (i) 33. (c) (i) For equal roots,

 −3   −3 
2 D = 0 ⇒ b2 – 4ac = 0
2  +k −5 = 0
 k + 6   k + 6  16 – 8 (a + 5) = 0
18 −3 a+5=2
2
+ −5 = 0
k+6 a = –3
(k + 6)
M-84 Mathematics
(ii) For distinct real roots, For equal roots, discriminant, D = 0
D > 0 Þ b2 – 4ac > 0 ⇒ b2 – 4ac = 0 ⇒ ( − k )2 − 4(2) (k ) = 0
a2 + 12b(a + 4) > 0 ⇒ k2 – 8k = 0 ⇒ k = 0, 8
9 + 12b > 0
40. (c) Let α + 3 = x ∴ α = x – 3 (replace x by x – 3)
9
b>– So the required equation
12
(x – 3)2 – 5 (x –3) + 6 = 0
3
b> − ⇒ x2 – 6x + 9 – 5x + 15 + 6 = 0 ⇒ x2 – 11x + 30 = 0
4
(x2 – 11x + 30) × 2 = 0 ⇒ 2x2 – 22x + 60 = 0
34. (d)
The given quadratic equation is
x2 + (3 – l) x + 2 = l
41. (b) 42. (a)

Sum of roots = a + b = l – 3 43. (b) Let the roots be α and β.


Product of roots = ab = 2 – l α + β = 8 , |α – β| = 10
a2 + b2 = (a + b)2 – 2ab
(α – β)2 = 100 ⇒ (α + β)2 – 4αβ = 100 ⇒ αβ = –9
= (l – 3)2 – 2 (2– l) ∴ x2 – 8x – 9 = 0 ⇒ (x2 – 8x – 9) = 0
= l2 – 4l + 5 or – (– x2 + 8x + 9) = 0
= (l – 2)2 + 1 44. (d) Since, α and β are roots of x2 + 5x + a = 0
For least (a2 + b2), l = 2. ∴ α + β = – 5 and αβ = a
35. (a) ∵ α + β = 64, αβ = 256 Consider 2α + 5β = – 1
⇒ 2α + 5 (– 5 – α) = –1 ⇒ 2α – 25 – 5α = – 1
α3/8 β3/8 α+β 64 64
5/8
+ 5/8 = 5/8
= 8 5/8 = =2 ⇒ – 3α = 24 ⇒ α = – 8 and β = – 5 + 8 = 3
β α (αβ ) (2 ) 32

Also, αβ = (3) (– 8) = – 24 = a
36. (c) ( x + 2) ( x − 1) = x 2 − 2 x − 3
45. (c) Let α and β be the roots of x2 + px + 8 = 0
⇒ x2 − x + 2 x − 2 = x2 − 2 x − 3
⇒ α + β = – p and αβ = 8
−1
⇒ x − 2 = −2 x − 3 ⇒ 3 x = −1 ⇒ x = ⇒ α(– p – α) = 8 ⇒ – pα – α2 = 8
3
2
37. (b) 4 x − 3 x − 5 = 0 ⇒ α2 + pα + 8 = 0 ... (i)
3 5 Also, given α – β = 2
⇒ x2 − x− = 0
4 4 ∴ α + β = – p and α – β = 2 together gives
2 2 2α = 2 – p ⇒ p = 2 – 2α
3  3  3 5
⇒ x 2 − 2. .x +   −   − = 0 Put value of ‘p’ in equation (i), we get
8 8
  8
  4
α2 + (2 – 2α) α + 8 = 0 ⇒ α2 + 2α – 2α2 + 8 = 0
2 2
 3 3 5  3  83
⇒  x −  − − = 0 ⇒  x −  − =0 ⇒ – α2 + 2α + 8 = 0
 8  64 4  8  64
On solving this, we get, α = – 2, 4
Hence, the required constant that should be added and ∴ p = ± 6 (when α = – 2, 4)
3
subtracted is . 46. (b) Let the roots of the given equation be α and 3α.
64
Now, α + 3α = – p and α(3α) = 12
38. (c) 2 x 2 − 7 x + 6 = 2(2)2 − 7(2) + 6
⇒ 4α = – p and α2 = 4 ⇒ α = ± 2
= 8 − 14 + 6 = 14 − 14 = 0
Now, 4(2) = – p and 4(–2) = – p ⇒ p = ± 8
39. (d) Compare the quadratic equation 2x2 – kx + k = 0 with
the standard form of quadratic equation 47. (c) Given equation is
ax2 + bx + c = 0; a = 2, b = –k, c = k x2 – 3x + 2 = 0
Quadratic Equations M-85
⇒ x2 – 2x – x + 2 = 0 ⇒ x(x – 2) – 1 (x – 2) = 0 Assertion : (2x – 1)2 – 4x2 + 5 = 0 ⇒ – 4x + 6 = 0
⇒ (x – 1) (x – 2) = 0 ⇒ x = 1, 2 Reason : 2x2 – 6x = 0 ⇒ 2x(x – 3) = 0
48. (c) Since x = 2 and x = 3 are roots of given equation ⇒ x = 0 and x = 3.

∴ 3(2)2 – 2p(2) + 2q = 0 60. (a) Assertion : 9x2 + 3k x + 4 = 0


⇒ 12 – 4p + 2q = 0 ⇒ – 2p + q = – 6 ... (i) ⇒ D = b2 – 4ac = (3k)2 – 4(9) (4) = 9k2 – 144
and 3(3)2 – 2p(3) + 2q = 0 For equal roots : D = 0 ⇒ 9k2 = 144

⇒ 27 – 6p + 2q = 0 ⇒ – 6p + 2q = – 27 ... (ii) 12
⇒ k = ± ⇒ k=±4
3
On solving (i) and (ii), we get
61. (c) Reason is false.
15
p = and q = – 6 + 15 = 9
2 Assertion : 4x2 – 12x + 9 = 0
49. (a) Speed of Ajay's car = (x + 5) km/h. ⇒ D = b2 – 4ac = (– 12)2 – 4(4) (9) = 144 – 144 = 0
⇒ Roots are repeated.
Distance = Speed × Time = 2(x + 5) km
400 400 62. (c) Assertion is correct. Reason is incorrect.
50. (c) – =4
x x+5 63. (a) Assertion and Reason both are correct and Reason is
⇒ x2 + 5x – 500 = 0 correct explanation.
51. (a) (x + 25)(x – 20) = 0 Assertion : 2x2– 3x + 5 = 0
⇒ x = 20 km/hour – b – (–3) 3 c 5
⇒ α+β = = = and αβ = =
52. (d) Speed of Ajay = x + 5 = 25 km/h. a 2 2 a 2
400 64. (A) → s; (B) → p; (C) → q; (D) → r
Time = = 16 hours.
25 (A) 6x2 + x – 12 = 0
53. (c) (20 – x) km/hr 6x2 + 9x – 8x – 12 = 0
54. (a) Speed = Distance/Time 3x (2x + 3) – 4 (2x + 3) = 0
(3x – 4)(2x + 3x) = 0
15 15
55. (c) – =1 4 −3
20 – x 20 + x x= ,
⇒ x2 + 30x – 400 = 0 3 2
(B) 8x2 + 16x – 192 = 0
56. (b) (x – 10)(x + 40) = 0
8x2 + 48x – 32x – 192 = 0
⇒ x = 10 km/hour 8x (x + 6) – 32(x + 6) = 0
57. (c) Speed in downstream = 20 + 10 = 30 km/h x = 4, – 6
Distance 15 (C) x2 – 45x + 324 = 0
Time = = = 30 minutes
Speed 30 x2 – 36x – 9x + 324 = 0
58. (b) Assertion : Given equation x2 – bx + c = 0 x (x – 36) – 9 (x – 36) = 0
2x (x – 3) + 1 (x – 3) = 0
Let α, β be two consecutive roots such that | α – β | = 1
x = 9, 36.
⇒ (α + β)2 – 4αβ = 1 ⇒ b2 – 4c = 1
(D) 2x2 – 5x – 3 = 0
Reason : Given equation : 2x2 – 6x + x – 3 = 0
4abc x2 + (b2 – 4ac) x – b = 0 −1
x = ,3
D= (b2 – 4ac)2 + 16ab2c 2
D= (b2 + 4ac)2 >0 65. (A) → q; (B) → r; (C) → s; (D) → p
Hence, roots are real and unequal. 66. (A) → (r,); (B) → (p,); (C) → (q) ; (D) → (s)

59. (b) Assertion and Reason both are true statements. But 67. ≠ 0
Reason is not the correct explanation.
M-86 Mathematics
68. > 0 1
76. (b − q )
69. 5 cm, 12 cm. 2
70. b2 < 4ac 77. two 78. b2 < 4ac
71. 7 and –9 79. False 80. True
72. x2 – bx + ca = 0 81. False 82. True

b 2 − 2ac 83. True 84. True


73.
c2 85. True 86. False
87. True 88. True
74. x2 – 8x + 15 = 0
75. –1
10 Acids, Bases and
Arithmetic
Salts
Progressions
1 1− p 1− 2 p
Multiple Choice Questions (MCQs) 9. The common difference of the A.P. , , , ........
p p p
is
DIRECTIONS: This section contains multiple choice questions. 1 1
Each question has 4 choices (a), (b), (c) and (d) out of which (a) 1 (b) (c) –1 (d) −
p p
only one is correct.
10. The nth term of the A.P. a, 3a, 5a, ......., is
1. In an A.P. if a = 5, an = 81 and Sn = 860, then n is (a) na (b) (2n – 1)a
(a) 10 (b) 15 (c) 20 (d) 25 (c) (2n + 1)a (d) 2na
2.
What is the value of k if (k + 2), (4k – 6) and (3k – 2) are 11. If the sum of the series 2 + 5 + 8 + 11 ........... is 60100,
three consecutive terms of an A.P.? then the number of terms are
(a) k = –3 (b) k = 2 (c) k = –2 (d) k = 3 (a) 100 (b) 200
3. The first term of an A.P. is 5 and its 100th term is –292. (c) 150 (d) 250
The 50th term of this A.P. will be
12. What is the common difference of four terms in A.P. such
(a) 142 (b) –142 (c) 130 (d) –140
that the ratio of the product of the first and fourth term to
4.
If a, b, c are in A.P., then the value of (a + 2b – c) (2b + c – a) that of the second and third term is 2 : 3 and the sum of
(c + a – b) will be all four terms is 20 ?
(a) 4abc (b) 2abc (a) 3 (b) 1 (c) 4 (d) 2
(c) abc (d) None of these 13. There are 60 terms in an A.P. of which the first term is 8
5. Sum of n terms of the series and the last term is 185. The 31st term is
2 + 8 + 18 + 32 + .... is (a) 56 (b) 94 (c) 85 (d) 98

n ( n + 1) 14. There are four arithmetic means between 2 and –18. The
(a) (b) 2n( (n + 1) means are
2
(a) –4, –7, –10, –13 (b) 1, –4, –7, –10
n ( n + 1) (c) –2, –5, –9, –13 (d) –2, –6, –10, –14
(c) (d) 1
2 15. If the first, second and the last terms of an A.P. are a, b, c
respectively, then the sum is
6. If eight times the8th term of an A.P. is equal to 12 times
the 12th term of the A.P. then its 20th term will be (a + b) (a + c − 2b) (b + c) (a + b − 2c)
(a) (b)
(a) –1 (b) 1 (c) 0 (d) 2 2 (b − a ) 2 (b − a )

7. The 10th term of an AP is 20 and the 19th term is 101. (a + c) (b + c − 2a )


(c) (d) None of these
Then, the third term is 2 (b − a )
(a) – 43 (b) – 61 (c) – 52 (d) 1
16. The sum of 11 terms of an A.P. whose middle term is 30,
8. Given that the sum of the first ‘n’ terms of an arithmetic is
progression is 2n2 + 3n, find the 12th term. (a) 320 (b) 330
(a) 72 (b) 36 (c) 625 (d) 56 (c) 340 (d) 350
M-88 Mathematics
17. The first and last term of an A.P. are a and  respectively. (nr – 1)nr n (nr + 1)
If S is the sum of all the terms of the A.P. and the common (c) (d)
2 2
2 − a 2 28. First term of an arithmetic progression is 2. If the sum of
difference is , then k is equal to
k − ( + a ) its first five terms is equal to one-fourth of the sum of the
(a) S (b) 2S next five terms, then the sum of its first 30 terms is
(c) 3S (d) None of these (a) 2670 (b) 2610 (c) –2520 (d) –2550
18. If four numbers in A.P. are such that their sum is 50 and 29. The odd natural numbers have been divided in groups as
the greatest number is 4 times the least, then the numbers (1, 3) ; (5,7, 9, 11) ; (13, 15, 17, 19, 21, 23), .....
are
(a) 5, 10, 15, 20 (b) 4, 10, 16, 22 Then the sum of numbers in the 10th group is
(c) 3, 7, 11, 15 (d) None of these (a) 4000 (b) 4003 (c) 4007 (d) 4008
30. Suppose the sum of the first m terms of an arithmetic
19. Let Tr be the rth term of an A.P. for r = 1, 2, 3, .... If
progression is n and the sum of its first n terms is m,
1
for some positive integers m, n we have, Tm = and where m ≠ n. Then, the sum of the first (m + n) terms of
1 n the arithmetic progression is
Tn = , then Tmn equals
m (a) 1 – mn (b) mn – 5
1 1 1 (c) – (m + n) (d) m+n
(a) (b) + (c) 1 (d) 0
mn m n 31. Let an, n ≥ 1, be an arithmetic progression with first term
2 and common difference 4. Let Mn be the average of the
20. If the sum of the first 2n terms of 2, 5, 8, ....... is equal to first n terms. 10
the sum of the first n terms of 57, 59, 61......., then n is Then the sum is  M n
equal to n =1

(a) 10 (b) 12 (c) 11 (d) 13 (a) 110 (b) 335 (c) 770 (d) 1100
21. The number of terms of the series 5, 7, 9, .... that must be 32. Which of the following represents an A.P. ?
taken in order to have the sum 1020 is (a) 0.2, 0.4, 0.6, .... (b) 29, 58, 116....
(a) 20 (b) 30 (c) 40 (d) 50 (c) 15, 45, 135, 405... (d) 3, 3.5, 4.5, 8.5 ....
22. If the nth term of an A.P. is 4n + 1, then the common 33. If tn = 6n + 5, then tn+1 =
difference is : (a) 6(n + 1) + 17 (b) 6(n – 1) + 11
(a) 3 (b) 4 (c) 5 (d) 6 (c) 6n + 11 (d) 6n – 11
23. If a, b, c, d, e, f are in A.P., then e – c is equal to: 34. Summation of n terms of an A.P. is
(a) 2(c – a) (b) 2(d – c) n n
(c) 2(f – d) (d) (d – c)
(a) ( 2a + l ) (b)  2a + ( n − 1) d 
2 2
24. The number of common terms of the two sequences
17, 21, 25, ....., 417 and 16, 21, 26, ........, 466 is
a rn −1
(c)
(
) (d)
(
a 1− rn )
(a) 19 (b) 20 (c) 21 (d) 91
( r − 1) (1 − r )
25. The number of two digit numbers which are divisible by 35. Sn = 54 + 51 + 48 + ........ n terms = 513. Least value of n is
3 is (a) 18 (b) 19
(a) 33 (b) 31 (c) 30 (d) 29 (c) 15 (d) None of these
26. If the nth term of an A.P. is given by an = 5n – 3, then the 36. If the nth term of an A.P. be (2n – 1), then the sum of its
sum of first 10 terms is first n terms will be
(a) 225 (b) 245 (c) 255 (d) 270 (a) n2 – 1 (b) (n – 1)2
2
(c) (n – 1) – (2n – 1) (d) n2
27. If S1, S2, S3, ......., Sr are the sum of first n terms of r
arithmetic progressions respectively. Whose first terms b+c−a c+a−b a+b−c
37. If , , are in A.P., then which of
are 1, 2, 3, ......... and whose common differences are 1, a b c
3, 5, ........ respectively, then the value of S1 + S2 + S3 + the following is in A.P.?
...... Sr is (a) a, b, c (b) a2, b2, c2
(nr – 1 )(nr +1 ) (nr + 1)nr 1 1 1
(a) (b) (c) , , (d) a3, b3, c3
2 2 a b c
Arithmetic Progressions M-89
46. If nth term of an AP is given by
an = 2n + 3 then common difference of an AP is
(a) 2 (b) 3 (c) 5 (d) 1
DIRECTIONS : Study the given Case/Passage and answer 47. The value of x, for which 2x, x+ 10, 3x + 2 are three
the following questions. consecutive terms of an AP
Case/Passage-I (a) 6 (b) –6 (c) 18 (d) –18
India is competitive manufacturing location due to the low cost
of manpower and strong technical and engineering capabilities
contributing to higher quality production runs. The production of
Assertion & Reason
TV sets in a factory increases uniformly by a fixed number every
year. It produced 16000 sets in 6th year and 22600 in 9th year. DIRECTIONS : Each of these questions contains an Assertion
[From CBSE Question Bank 2021] followed by Reason. Read them carefully and answer the
question on the basis of following options. You have to select
the one that best describes the two statements.

(a) If both Assertion and Reason are correct and Reason is


the correct explanation of Assertion.
(b) If both Assertion and Reason are correct, but Reason is
not the correct explanation of Assertion.
(c) If Assertion is correct but Reason is incorrect.
(d) If Assertion is incorrect but Reason is correct.
Based on the above information, answer the following questions:
38. Find the production during first year. 48. Assertion : Let the positive numbers a, b, c be in A.P.,
39. Find the production during 8th year. 1 1 1
then , , are also in A.P.
40. Find the production during first 3 years. bc ac ab
41. In which year, the production is Rs 29,200. Reason : If each term of an A.P. is divided by abc, then
42. Find the difference of the production during 7th year and 4th
the resulting sequence is also in A.P.
year.
Case/Passage-II 49. Assertion : The sum of the series with the nth term,
Your friend Veer wants to participate in a 200m race. He can tn = (9 – 5n) is (465), when no. of terms n = 15.
currently run that distance in 51 seconds and with each day of Reason : Given series is in A.P. and sum of n terms of an
practice it takes him 2 seconds less.He wants to do in 31 seconds . n
[From CBSE Question Bank 2021] A.P. is Sn =  2a + ( n − 1) d  .
2
50. Assertion : Sum of first 10 terms of the arithmetic
progression – 0.5, – 1.0, – 1.5, ............................ is 27.5.
Reason : Sum of n terms of an A.P. is given as
n
Sn = [2a + (n − 1)d ] where a = first term, d = common
2
difference.
51. Assertion : Sum of first hundred even natural numbers
divisible by 5 is 500.
Reason : Sum of first n-terms of an A.P. is given by
43. Which of the following terms are in AP for the given n
Sn = [a + ] where  = last term.
situation 2
(a) 51,53,55…. (b) 51, 49, 47…. 52. Assertion : If nth term of an A.P. is 7 – 4n, then its common
(c) –51, –53, –55…. (d) 51, 55, 59… difference is –4.
44. What is the minimum number of days he needs to Reason : Common difference of an A.P. is given by
practice till his goal isachieved d = an + 1 – an.
(a) 10 (b) 12 (c) 11 (d) 9 53. Assertion : If Sn is the sum of the first n terms of an A.P.,
45. Which of the following term is not in the AP of the above then its nth term an is given by an = Sn – Sn – 1 .
given situation Reason : The 10th term of the A.P.
(a) 41 (b) 30 (c) 37 (d) 39 5, 8, 11, 14, ................... is 35.
M-90 Mathematics
54. Assertion : Arithmetic between 8 and 12 is 10. 61. Sum of 1 + 3 + 5 + .... + 1999 is ......... .
Reason : Arithmetic between two numbers ‘a’ and ‘b’ is 62. The sum of 8 A.Ms between 3 and 15 is ................... .
a+b
given as . 63. The sum of n terms of an A.P. is 4n2 – n. The common
2
difference = .................. .

Match the Following 64. The difference of corresponding terms of two A.P’s will
be .................... .

DIRECTIONS : Each question contains statements given in two 65. Sum of all the integers between 100 and 1000 which are
columns which have to be matched. Statements (A, B, C, D) in divisible by 7 is ................... .
column-I have to be matched with statements (p, q, r, s) in column-II.
True / False
55. Column -I Column-II
(A.P.) (Common Difference)
DIRECTIONS : Read the following statements and write your
3 5 answer as true or false.
(A) 1, , 2, ,....... (p) – 4
2 2
1 5 9 13 66. In an AP with first term a and common difference
(B) , , , ,....... (q) 0.2 d, the n th term (or the general term) is given by
3 3 3 3
(C) 1.8, 2.0, 2.2, 2.4 (r) 4/3 an = a + (n – 1)d.
(D) 0, – 4, –8, –12 (s) 1/2 67. If  is the last term of the finite AP, say the nth term, then
the sum of all terms of the AP is given by :
56. Column-I Column-II
(A.P.) (nth term) S = n (a + )
(A) 119, 136, 153, 170 ...... (p) 13 – 3n 2
68. The balance money ( in `) after paying 5% of the total
(B) 7, 11, 15, 19, ....... (q) 9 – 5n loan of ` 1000 every month is 950, 900, 850, 800, . . . 50.
(C) 4, –1, –6, –11, ...... (r) 3 + 4n represented A.P.
(D) 10, 7, 4, 3, ...... (s) 17n + 102 69. 2, 4, 8, 16, ............. is not an A.P.
70. 10th term of A.P. 2, 7, 12, ......... is 45.
Fill in the Blanks
71. 301 is a term of A.P. 5, 11, 17, 23, ............. .

DIRECTIONS : Complete the following statements with an 72. The general form of an A.P. is a, a + d, a + 2d, a + 3d,
appropriate word / term to be filled in the blank space(s). .............

57. 4, 10, 16, 22, ........, .......... . 73. In an Arithmetic progression, the first term is denoted by
58. 1, –1, –3, – 5, ....... , ......... . ‘a ’ and ‘d ’ is called the common difference.
59. 11th term from last term of an A.P. 10, 7, 4......... , – 62, is 74. If an+1 – an = same for all ‘n’, then the given numbers
......... . form an A.P.
60. In a flower bed, there are 23 rose plants in the first row, 21 75. If Sn of A.P. is 3n2 + 2n, then the first term of A.P. is 3.
in the second, 19 in the third, and so on. There are 5 rose
plants in the last row. Number of rows in the flower bed
is ............... .
Arithmetic Progressions M-91

ANSWER KEY & SOLUTIONS


n ⇒ 8a + 56d = 12a + 132d
1. (c) Sn = (a + an )
2 ⇒ 8a – 12a + 56d – 132d = 0
n ⇒ –4a – 76d = 0
⇒ 860 = (5 + 81)
2 ⇒ a + 19d = 0 …(i)
n = 860 ÷ 43 = 20
∴ t20 = a + 19d = 0 using (i)
2. (d) (k + 2), (4k – 6) and (3k – 2) are in A.P.
∴ t20 = 0
⇒ 4k – 6 – k – 2 = 3k – 2 – 4k + 6
7. (a) Given that,
⇒ 3k – 8 = –k + 4 ⇒ 3k + k = 4 + 8
t10 = a + 9d = 20 ....(i)
12
⇒ 4k = 12 ⇒ k = =3 and t19 = a + 18d = 101 ....(ii)
4
By solving equations (i) and (ii), we get
3. (b) a = 5, t100 = –292
a = –61, d = 9
t100 = 5 + (100 – 1)d
t3 = a + 2d = –61 + 2 × 9 = – 43
[using tn = a + (n – 1)d]
8. (a) Sn = 2n2 + 3n
⇒ –292 = 5 + 99d
an = Sn – Sn – 1
⇒ –292 – 5 = 99d
a12 = S12 – S11
−297
⇒ d = ⇒ d = −3 = 2(12)2 + 3(12) – (2(11)2 + 3(11))
99
= 288 + 36 – (242 + 33) = 288 + 36 – 242 – 33
∴ t50 = 5 + (50 – 1) (–3) = 5 + (–147)
= 46 + 3 = 49 = 72
= 5 – 147 ⇒ t50 = –142
1− p 1 1− p −1 − p
4. (a) Let a, b, c are in A.P. 9. (c) d = − = = = –1
p p p p
∴ b – a = c – b ⇒ 2b = a + c
10. (b) an = a + (n – 1)d = a +(n – 1)2a
So the given expression becomes [Q d = 3a – a = 2a]
(a + a + c − c)(a + c + c − a )(2b − b) = a + 2an – 2a = 2an – a = (2n – 1)a
11. (b)
= (2a ) (2c) (b) = 4abc
12. (d) Take the four terms as a – 3x, a – x, a + x, a + 3x
5. (c) Here, a1 = 2, a 2 = 8 = 2 2 The sum = 4a = 20 ⇒ a = 5
∴ d = 2 2 − 2 = 2, a = 2 Also, 3 (a2 – (3x)2) = 2 (a2 – x2)

n ⇒ x = 1
Sn =  2a + ( n − 1) d 
2 However, the common difference is 2x and not x
n n ( n + 1) ∴ When x = 1, d = 2x = 2
= 2 × 2 + ( n − 1) 2  =
2 2 13. (d) Let d be the common difference

6. (c) t8 = a + 7d, t12 = a + 11d then 60th term = 8 + 59d = 185

According to question, 8t8 = 12t12 (given) ⇒ 59d = 177

⇒ 8(a + 7d) = 12(a + 11d) ⇒ d = 3


⇒ 31st term = 8 + 30 × 3 = 98.
M-92 Mathematics
14. (d) Let the means be X 1 , X 2 , X 3 , X 4 and the common 25. (c) Two digit numbers which are divisible by 3 are 12,
difference be b; then 2, X 1 , X 2 , X 3 , X 4 , –18 are 15, 18,..., 99;
in A.P.; So, 99 = 12+ (n – 1) × 3.
⇒ –18 = 2 + 5b 26. (b) Putting n = 1, 10, we get a = 2, l = 47.
⇒ 5b = – 20 10
\ S10 = ( 2 + 47 ) = 5 × 49 = 245.
⇒ b = – 4 2
Hence, X1 = 2 + b = 2 + (–4) = – 2; n
27. (b) S1 =
[2(1) + (n − 1)(1)]
2
X2 = 2 + 2b = 2 – 8 = –6 n
S2 = [2(2) + (n − 1)(3)]
X3 = 2 + 3b = 2 – 12 = –10;
 2
X4 = 2 + 4b = 2 – 16 = – 14 n
S3 = [2(3) + (n − 1)(5)]
The required means are –2, –6, –10, –14. 2
15. (c) ..................................................
..................................................
16. (b) ..................................................

n 2S n
17. (b) We have, S = ( a + ) ⇒ = n …(i) Sr = [2(r ) + (n − 1)(2r − 1)]
2 a+ 2
Adding S1, S2, S3, ......., Sr, we have
−a −a
Also,  = a + (n − 1) d ⇒ d = = n r (r + 1) r 
n −1 2S S1 + S2 + ... + Sr = (2) + (n − 1) [1 + 2r − 1]
−1 2 2 2 
a+
n
2 − a 2 =  r (r + 1) + (n − 1)r 2 
= \ k = 2S 2  
2 S − ( + a )
nr nr
= [ r + 1 + nr − r ] = [ nr + 1]
18. (a) 2 2

19. (c) 28. (d) a = 2, d = d,

2n n According to question,
20. (c) Given, {2.2 + (2n − 1)3} = {2.57 + (n − 1)2}
2 2 1
S5 = ( S10 – S5 ) , 4S5 = S10 – S5, 5S5 = S10
4
or 2 (6n + 1) = 112 + 2n
or 10n = 110 ∴ n = 11 5  10
5  {2 × 2 + (5 –1)d } = [2 × 2 + (10 –1)d ]
21. (b)  2  2

22. (b)  n 
 Sn = ( 2a + (n – 1) d 
23. (b) Let x be the common difference of the A.P.  2 
a, b, c, d, e, f. 5 10
⇒ 5 × (4 + 4d ) [4 + 9d ]
∴ e = a + (5 – 1)x 2 2
[∵ an = a + (n – 1)d] ⇒ 20 + 20d = 8 + 18d
⇒ e = a + 4x ...(i) ⇒ d = – 6
and c = a + 2x ...(ii) 30 30
S30 = [2 × 2 + (30 – 1) (–6)] = [4 + 29 × (–6)]
∴ Using equations (i) and (ii), we get 2 2
e – c = a + 4x – a – 2x 30 –5100
= × (–170) = = – 2550
⇒ e – c = 2x = 2(d – c). 2 2
24. (b) Common terms will be 21, 41, 61, ........ 29. (a) Since, the general term of sum of odd natural
21 + (n – 1) 20 ≤ 417 number in the group is = n (2n)2 = 4n × n2 = 4n3
Hence, the required sum of numbers in the 10th group =
⇒ n ≤ 20.8 ⇒ n = 20
4 × 103 = 4000
Arithmetic Progressions M-93
30. (c) Given, Sm = n and Sn = m Dividing each term by (a + b + c),
m a+b+c a+b+c a+b+c
Sm = [2a + (m − 1)d ] = n ...(i) , , are in A.P.
2 a (a + b + c) b (a + b + c) c (a + b + c)
n 1 1 1
Sn = [2a + (n − 1)d ] = m ...(ii) , , are in A.P.
2 a b c

On subtracting Eq. (ii) from Eq. (i), we get 38. Given that
a6 = a + 5d = 16000 ..... (i)
(m − n ) d a9 = a + 8d = 22600 ..... (ii)
2a + [m(m − 1) − n(n − 1)] = n − m
2 2 – – –
d –3d = – 6600 ⇒ d = 2200
(m – n)a + (m – n)(m + n –1) = – (m – n) ⇒ a = 5000
2
\ Production during first year = 5000
⇒ 2a + (m + n –1) d = –2 [m ≠ n]
39. Production during 8th year is (a + 7d) = 5000 + 2(2200)
m+n
\ Sm + n = [(2a + (m + n − 1)d ]
2 = 20400
m+n 40. Production during first 3 year = 5000 + 7200 + 9400
= ( − 2) = −(m + n)
2
= 21600
31. (a) a1 = 2, d = 4
41. 5000 + (n – 1) 2200 = 29200 ⇒ n = 12th year
n
[ 2a1 + (n − 1)d ] 42. Difference = (a + 6d) – (a + 3d) = 3d = 6600
Mn = 2 = 2(n + 1) − 2 = 2n
n 43. (b)
10 10 44. (c) an = 51 – (n – 1)2 = 31 ⇒ n = 11
\ ∑ M n = 2 ∑ n = 110 45. (b) an = 51 – (n – 1)2 = 30 ⇒ n = 11.5 (not possible)
n =1 n =1
32. (a) Since there is a common difference option (a), 46. (a) d = a2 – a1 = [2(2) +3] – [2(1) + 3] = 2
d = 0.4 – 0.2 = 0.6 – 0.4 = 0.2 47. (a) 2(x + 10) = 2x + 3x + 2 ⇒ x = 6
33. (c) Put n + 1 in place of n in Tn = 6n + 5 48. (a) 49. (d)
34. (b)
n 50. (a) Both are correct. Reason is the correct reasoning for
35. (a) Sn = 513; [2(54) + (n – 1)(– 3)] = 513 assertion.
2
10
⇒ n(108 – 3n + 3) = 1026 Assertion : S10 = [ 2(−0.5) + (10 − 1) (−0.5)]
⇒ n2 – 37n + 342 = 0 2
⇒ n2 – 19n – 18n + 342 = 0 = 5 [−1 − 4.5] = 5(−5.5) = 27.5
⇒ n(n – 19) – 18(n – 19) = 0 51. (d) Assertion is incorrect.
⇒ (n – 18) (n – 19) = 0 ⇒ n = 18 or n = 19
Assertion : Even natural numbers divisible by 5 are
36. (d) t1 = 2(1) – 1 = 1
10, 20, 30, 40, ..........
t2 = 2(2) – 1 = 3, t3 = 2(3) – 1 = 5 and so on.
They form an A.P. with a = 10, d = 10
\ t1 + t2 + t3 +…+ tn = 1 + 3 + 5 +…[2(n) – 1]
100
n n
= [2 + (n – 1)2] = (2 + 2n – 2) = n2
S100 = [ 2(10) + 99(10)] = 50500
2
2 2
Reason is correct.
b+c−a c+a−b a+b−c 52. (a) Both are correct. Reason is the correct explanation.
37. (c) , , are in A.P.
a b c
Assertion : an = 7 – 4n
Adding 2 to each term
d = an + 1 – an= 7 – 4 (n + 1) – (7 – 4n)
b+c−a c+a−b a+b−c
+2, +2, + 2 are in A.P. = 7 – 4n – 4 – 7 + 4n = – 4.
a b c
a+b+c a+b+c a+b+c 53. (c) Assertion is correct. Reason is incorrect.
, , are in A.P.
a b c a10 = a + 9d = 5 + 9(3) = 5 + 27 = 32.
M-94 Mathematics
54. (a) 
Both are correct and Reason is the correct
  3 + 15  
explanation for the Assertion. 62. 72 8   etc.
  2  
55. (A) → (s); (B) → (r); (C) → (q); (D) → (p)
63. 11 [S2 = 4(2)2 – 2 ⇒ 14
3 1
(A) Common difference = d = −1 = S1 = 4(1)2 – 1 ⇒ 3 etc.]
2 2
5 1 4 64. another A.P.
d= − =
(B)
3 3 3 65. 70336 [Hint : S = 105 + 112 + ... 994 and 105 + (n – 1)7
d = 2 – 1.8 = 0.2
(C) = 994 ⇒ 105 + 7n – 7 = 994 ⇒ n = 128 etc.]
d = – 4 – 0 = – 4.
(D) 66. True
56. (A) → (s); (B) → (r); (C) → (q); (D) → (p) 67. True
13 – 3n = 13 – 3(1) = 10 68. True
9 – 5n = 9 – 5(1) = 4 69. True
3 + 4n = 3 + 4(1) = 7 70. False
17n + 102 = 17(1) + 102 = 119 71. False
57. 28, 34 72. True
58. –7, –9 73. True
59. –32 74. True
60. n = 10 75. False
First term = a = a1 = 3 (1)2 + 2(1) = 5
61. 1000 [ 2(1) + (1000 − 1)2]
2
11 Acids, Bases and
Circles
Salts
(a)
∠AQB = 70°, ∠AMB = 110°
Multiple Choice Questions (MCQs) (b)
∠AQB = 110°, ∠AMB = 70°
(c)
∠AQB = 100°, ∠AMB = 50°
DIRECTIONS : This section contains multiple choice
(d)
∠AQB = 60°∠AMB = 40°
questions. Each question has 4 choices (a), (b), (c) and (d) out
of which only one is correct. 5. A tangent CQ touches a circle
A
with centre O at P. Diameter AB
1. Two circles with centres O Q
is produced to meet the tangent O
R
and P, and radii 8 cm and at C. If ∠ACP = a° and
T
4 cm touch each other S ∠BPC = b°, the relation
B

externally. Find the length O P connecting a and b is


C
P
Q
of their common tangent
(a) a° + b° = 180° (b) a° + 2b° = 90°
QR.
(c) a° – b° = 60° (d) 2a° + b° = 100°
(a) 8 cm (b) 7 cm (c) 8 2 cm (d) 7 3 cm
6. PQ is a tangent drawn from a point P to a circle with
2. Two chords AB and CD
A
5 centre O and QOR is a diameter of the circle such that
of a circle intersect each B ∠POR = 120°, then ∠OPQ is
other at P outside the
3
(a) 60° (b) 45° (c) 30° (d) 90°
circle. If AB = 5 cm, C x D 2
P

BP = 3 cm and PD = 2 cm, 7. If a regular hexagon is inscribed in a circle of radius r,


find CD. then its perimeter is
(a) 4 cm (b) 5 cm (c) 8 cm (d) 10 cm (a) 3r (b) 6r (c) 9r (d) 12r

3. A ball is in the rest position 8. AB and CD are two chords of a circle intersecting at the
against a step PQ. If PQ = 10 point P outside the circle. If PA = 12 cm, CD = 7cm and
cm and QR = 15 cm, then find PC = 15 cm, then AB is equal to
diameter of the ball. (a) 15.5 cm (b) 4 cm
P
(a) 16 cm (c) 8 cm (d) 10 cm
(b) 32.5 cm Q R 9. In the figure below (not to scale), AB = CD and AB and
(c) 28 cm CD are produced to meet at the point p.
(d) 42 cm A

4. In the given figure, PA and A B


P
PB are two tangents to the Q

circle with centre O. If O D

∠APB = 40°, find ∠AQB M C


and ∠AMB. 40° If ∠BAC = 70°, then ∠P is
P
B
(a) 30° (b) 40° (c) 45° (d) 50°
M-96 Mathematics
10. In the adjoining figure, TP and TQ are the two tangents to (c) only parallel chords are of same length.
a circle with centre O. If ∠POQ = 110°, then ∠PTQ is (d) only perpendicular chords are of same length.
P
T
15. Number of tangents to a circle which are parallel to a
secant, is
O (a) 3 (b) 2 (c) 1 (d) infinite
Q
16. AB and CD are two common tangents to circles which
touch each other at a point C. If D lies on AB such that
(a) 60° (b) 70° (c) 80° (d) 90°
CD = 4 cm, then AB is
11. In the diagram below, if l and m are two tangents and AB (a) 12 cm (b) 8 cm (c) 4 cm (d) 6 cm
is a chord making an angle of 60° with the tangent l, then
17. In the figure, ∆APB is formed by three tangents to the
the angle between l and m is
m
circle with centre O. If ∠APB = 40°, then the measure of
∠BOA is

C B
R
B
O
60°
40°
l P
A P A T
(a) 50° (b) 55° (c) 60° (d) 70°
(a) 45° (b) 30° (c) 60° (d) 90°
18. Three circles with radii R1, R2 and r touch each other
12. In the diagram, O is the centre of the circle and D, E and F externally as shown in the adjoining figure. If PQ is their
are mid points of AB, BO and OA respectively. If ∠DEF common tangent and R1 > R2, then which of the following
= 30°, then ∠ACB is relations is correct?

C (a) R1 – R2 = r 
P
(b) R1 + R2 = 2r
R1 R
1 1 1 Q
O (c) + =
F E R1 R 2 r R2
1 1 1
A B (d) + =
D R1 R2 r
19. Two circles, both of radii a touch each other and each of them
(a) 30° (b) 60° (c) 90° (d) 120º touches internally a circle of radius 2a. Then the radius of the
circle which touches all the three circles is
13. In the below diagram, O is the centre of the circle, AC is
the diameter and if ∠APB = 120°, then ∠BQC is 1 2 3
(a) a (b) a (c) a (d) a
2 3 4
C 20. In the figure, O is the centre of the circle and OA = CD,
O
then ∠CPD is
Q
O
A  B
A
B
P
C D
(a) 30° (b) 150° (c) 90° (d) 120°

14. In two concentric circles, if chords are drawn in the outer
P

circle which touch the inner circle, then (a) 45° (b) 30° (c) 70° (d) 60°
(a) all chords are of different lengths. 21. In figure ABCD is a cyclic quadrilateral and ∠ADC = 80°,
(b) all chords are of same length. ∠ACD = 50°, then ∠CBD is:
Circles M-97
(b) In a circle, the perpendicular from the centre to a chord
C B bisects the chord.
(c) The point common to a circle and its tangent is called
the point of contact.
O (d) Adjacent angles of a cyclic quadrilateral are
D A supplementary.
29. Which of the following statement(s) is / are not correct ?
(a) 60° (b) 130° (c) 50° (d) 40° (a) The length of tangent from an external point P on
circle with centre O is always less than OP.
22. In the given figure, AC is the diameter of the circle.
(b) The tangent to the circumcircle of an isosceles triangle
ED || AC, ∠CBE = 65°, then ∠DEC is ABC at A, in which AB = AC, is parallel to BC.
B
(c) If angle between two tangents drawn from a point
P to a circle of radius ‘a’ and centre ‘O’is 90°, then
A C OP = a 2.
O
(d) None of these
E D 30. Which of the following statement(s) is/are correct?
(a) If a chord AB subtends an angle of 60° at the centre
(a) 35° (b) 25° (c) 65° (d) 30°
of a circle, then angle between the tangents at A and
23. Let ABCD be a square of side length 1, and G a circle passing B is also 60°.
through B and C, and touching AD. The radius of G is (b) The length of tangent from an external point on a circle
1 is always greater than the radius of the circle.
3 1 5
(a) (b) (c) (d) (c) If a number of circle touch a given line segment PQ at
8 2 2 8
a point A, then their centres lie on the perpendicular
24. Three circles of radii 1, 2 and 3 units respectively touch bisector of PQ.
each other externally in the plane. The circumradius of the (d) None of these
triangle formed by joining the centers of the circles is
31. Which of the following statement(s) is/are incorrect?
(a) 1.5 (b) 2 (c) 2.5 (d) 3
(a) Angle between the tangent line and the radius at the
25. Circles A, B and C are externally D point of contact is 90°.
tangent to each other and internally (b) A circle can have two parallel tangents atmost.
tangent to circle D. Circles A and B A (c) The distance between two parallel tangents drawn to
are congruent. Circle C has radius C a circle is equal to twice of radius.
1 unit and passes through the centre B (d) A line intersecting a circle in two points is called a chord.
of circle D. Then are radius of circle
32. A tangent PQ at a point P of a circle of radius 5 cm meets a
B is ________ units.
line through the centre O at a point Q so that OQ = 12 cm.
3 6 8 9
(a) (b) (c) (d) Length PQ is :
7 9 9 8
(a) 12 cm (b) 13 cm
26. The length of tangent drawn from a point Q to a circle
(c) 8.5 cm (d) 119 cm
is 24 cm and distance of Q from the centre of circle is
25 cm. The radius of circle is 33. In fig. if TP and TQ are the two tangents to a circle with
(a) 7 cm (b) 12 cm centre O so that ∠POQ = 110°, then ∠PTQ is equal to
(c) 15 cm (d) 24.5 cm T

27. Which of the following is a cyclic quadrilateral? P


(a) Rhombus (b) Rectangle
(c) Parallelogram (d) Trapezium 110°
Q
O
28. Which of the following is/are not correct?
(a) A secant is a line that intersects a circle in two distinct
points. (a) 60° (b) 70° (c) 80° (d) 90°
M-98 Mathematics
34. If tangents PA and PB from a point P to a circle with centre 39. Find ∠RSQ
O are inclined to each other at an angle of 80°, then ∠POA (a) 60 (b) 75
is equal to (c) 100 (d) 30
(a) 50° (b) 60° (c) 70° (d) 80°
40. Find ∠ORP
35. If angle between two radii of a circle is 130°, the angle (a) 90 (b) 70
between the tangents at the ends of the radii is :
(c) 100 (d) 60
(a) 90° (b) 50° (c) 70° (d) 40°
Case/Passage-II
36. In fig. the pair of tangents AP and AQ drawn from an Varun has been selected by his School to design logo for Sports
external point A to a circle with centre O are perpendicular Day T-shirts for students and staff . The logo design is as given
to each other and length of each tangent is 5 cm. in the figure and he is working on the fonts and different colours
according to the theme.
Then, the radius of the circle is
P
m
5c
A
O

In given figure, a circle with centre O is inscribed in a ΔABC,


such that it touches the sides AB, BC and CA at points D, E and
DIRECTIONS : Study the given Case/Passage and answer the
F respectively. The lengths of sides AB, BC and CA are 12 cm,
following questions.
8 cm and 10 cm respectively. [From CBSE Question Bank 2021]
Case/Passage-I C

A Ferris wheel (or a big wheel in the United Kingdom) is an F E


amusement ride consisting of a rotating upright wheel with
multiple passenger-carrying components (commonly referred
to as passenger cars, cabins, tubs, capsules, gondolas, or pods)
attached to the rim in such a way that as the wheel turns, they A D B

are kept upright, usually by gravity. 41. Find the length of AD


After taking a ride in Ferris wheel, Aarti came out from the (a) 7 (b) 8
crowd and was observing her friends who were enjoying the
ride . She was curious about the different angles and measures (c) 5 (d) 9
that the wheel will form. She forms the figure as given below. 42. Find the Length of BE
[From CBSE Question Bank 2021]
R (a) 8 (b) 5
S
(c) 2 (d) 9
S R 43. Find the length of CF
O (a) 9 (b) 5
30°
P (c) 2 (d) 3
Q 30°
P
Q 44. If radius of the circle is 4cm, Find the area of ∆OAB
37. In the given figure find ∠ROQ (a) 20 (b) 36
(a) 60 (b) 100 (c) 24 (d) 48
(c) 150 (d) 90 45. Find area of ∆ABC
38. Find ∠RQP (a) 50 (b) 60
(a) 75 (b) 60
(c) 100 (d) 90
(c) 30 (d) 90
Circles M-99
50. If two tangents PA and PB are drawn to a circle with
Assertion & Reason centre O from an external point P (figure), then match
the column.
DIRECTIONS : Each of these questions contains an Assertion
followed by Reason. Read them carefully and answer the
question on the basis of following options. You have to select A
the one that best describes the two statements.

(a) If both Assertion and Reason are correct and Reason is P  O


Y
the correct explanation of Assertion.
(b) If both Assertion and Reason are correct, but Reason is B
not the correct explanation of Assertion.
(c) If Assertion is correct but Reason is incorrect.
Column-I Column-II
(d) If Assertion is incorrect but Reason is correct.
(A) ∠ PAB (p) 90°
46. Assertion: If in a circle, the radius of the circle is 3 cm (B) ∠ OAP (q) q/2
and distance of a point from the centre of a circle is 5 cm, θ
then length of the tangent will be 4 cm. (C) ∠ OAB (r) 90° −
2
Reason:(hypotenuse)2 = (base)2 + (height)2 (D) ∠ AOB (s) 180° – θ

47. Assertion: If in a cyclic quadrilateral, one angle is 40°, 51. If an isosceles ∆ABC in which AB = AC = 6 cm is inscribed
then the opposite angle is 140° in a circle of radius 9 cm, then
A
Reason: Sum of opposite angles in a cyclic quadrilateral 6 cm 6 cm
P
is equal to 360° B C

48. Assertion: If length of a tangent from an external point 9 cm 9 cm


O
to a circle is 8 cm, then length of the other tangent from
the same point is 8 cm.
Reason: length of the tangents drawn from an external
Column-I Column-II
point to a circle are equal.
AP
(A) (p)
8 2
Match the Following CP
(B) (q) 4 2
OB
(C) (r) 2
DIRECTIONS : Each question contains statements given in
(D) Area of ∆ABC (s) 9
two columns which have to be matched. Statements (A, B, C, D)
in Column-I have to be matched with statements (p, q, r, s) in 52. A circle is inscribed in a A
Column-II. ∆ABC having sides
AB = 8 cm, BC = 10 cm F D
49. If AB is a chord of length 6 cm A and CA = 12 cm as shown
of a circle of radius 5cm, the in figure. Observe the
X O C B
tangents at A and B intersect Y diagram and match the E
at a point X (figure), then B
columns.
match the columns. Column-I Column-II
(A) AD (p) 15
Column-I Column-II
(A) AY (p) 4 cm (B) BE (q) 7 cm
(C) CF (r) 3 cm
(B) OY (q) 3.75 cm
(D) AD + BE + CF (s) 5 cm
(C) XA (r) 5 cm
(D) OA (s) 3 cm
M-100 Mathematics
53. Column-I Column-II
63. In given Fig., the length PB = .................. cm.
Definition Term
(A) A line segment which join (p) Secant
any two points on a circle. 5 cm
A
O
(B) A line which intersect the (q) Tangent
3 cm
circle in two points. P
(C) A line that intersects the circle (r) Chord B
at only one point.
True / False
Fill in the Blanks
DIRECTIONS : Read the following statements and write your
DIRECTIONS : Complete the following statements with an answer as true or false.
appropriate word / term to be filled in the blank space(s).
64. The tangent to a circle is a special case of the secant.
54. A tangent to a circle touches it at ............... point (s). 65. The perpendicular at the point of contact to the tangent to
a circle does not pass through the centre.
55. A line intersecting a circle at two points is called a ...........
66. A circle can have at the most two parallel tangents.
56. A circle can have .............. parallel tangents at the most.
67. If P is a point on a circle with centre C, then the line drawn
57. The common point of a tangent to a circle and the circle through P and perpendicular to CP is the tangent to the
is called .................. . circle at the point P.
58. There is no tangent to a circle passing through a point lying 68. The centre of the circle lies on the bisector of the angle
............ the circle. between the two tangents.

59. The tangent to a circle is .............. to the radius through 69. A tangent to a circle is a line that intersects the circle at
the point of contact. only one point.
70. Two equal chords of a circle are always parallel.
60. There are exactly two tangents to a circle passing through
a point lying ........... the circle. 71. A line drawn from the centre of a circle to a chord always
bisects it.
61. The lengths of the two tangents from an external point to
72. Line joining the centers of two intersecting circles always
a circle are ............. .
bisect their common chord.
62. The tangents drawn at the ends of a diameter of a circle 73. In a circle, two chords PQ and RS bisect each other. Then
are .................. . PRQS is a rectangle.
Circles M-101

ANSWER KEY & SOLUTIONS


1. (c) Join O to P and Q. Join P to R. Draw SP ⊥ OQ. 6. (c) Since, PQ is tangent from a point P
Now SP = QR, as they are opposite sides of rectangle PRQS.
OP = 8 cm + 4 cm = 12 cm; OS = 8 cm – 4 cm = 4 cm Q
In right triangle POS,
O
2 2 = 2 2
SP = OP − OS 12 − 4 = 8 2 cm
120°
∴ QR = 8 2 cm P
R
2. (d) Since, two chords AB and CD of the circle are
∴ ∠OQP = 90° ⇒ ∠POR = 120°
intersecting at P, when produced.
∴ ∠POR + ∠POQ = 180° [linear pair]
∴ PA. PB = PC.PD
∠POQ = 180° – 120° = 60°
[Each = (length of the tangent from P)2] In ∆OPQ
⇒ (AB + PB). (PB) = (PD + DC) ⋅ PD ⇒ ∠OPQ + ∠OQP +∠ POQ = 180°
⇒ (5 + 3) (3) = (2 + x) 2 [Q Sum of angles of ∆ = 180°]
⇒ 24 = (2 + x) (2) ⇒ 12 = 2 + x ∴ ∠OPQ + 90° + 60° = 180° ⇒ ∠OPQ = 30°
⇒ x = 10 ⇒ CD = 10 cm 7. (b) Side of the regular hexagon inscribed in a circle of
3. (b) In right ∆OSP,  radius r is also r, the perimeter is 6r.
OP2 = PS2 + OS2 r
O
8. (a)
r2 = 225 + (r – 10)2 P S
2 2
10 15 10 9. (b) Exterior angle of a cyclic quadrilateral is equal to
⇒ r = 225 + r – 20r + 100
Q 15 R its interior opposite angle.
⇒ 20 r = 325 ⇒ 2r = 32.5
Hence, diameter = 32.5 cm. ∠BAC = ∠DCA and proceed.
4. (a) Since, OA ⊥ PA and OB ⊥ PB, 10. (b) [Hint. OP ⊥ PT, OQ ⊥ QT.
∴ In quadrilateral AOBP, In quad. OPTQ, ∠POQ + ∠OPT + ∠PTQ + ∠OQT = 360°
⇒ 40° + 90° + 90° + ∠AOB = 360° ⇒ 110° + 90° + ∠PTQ + 90° = 360°
⇒ ∠AOB = 140°
⇒ ∠ PTQ = 70°]
1 1
Also, ∠AQB = of ∠AOB = 70° and ∠AMB = 11. (c) Tangents drawn to a circle from an external point
2 2
are equal.
1 1
of reflex ∠AOB = × (360° – 140°) = × 220 = 110° 12. (b) (i) ADEF is a parallelogram.
2 2
[Q The angle subtended by an arc at the centre is double (ii) ∠FAD = 30° and ∠OAD = ∠OBA
the angle subtended by the arc at any point on the (angles opposite to equal sides)
remaining part of the cirlce.]
13. (b) (i) APBC is a cyclic quadrilateral.
5. (b) Given, ∠BPC = b° and ∠ACP = a°.
Also, ∠OPA = ∠OAP = b° (Angles in an isosceles (ii) ∠ABC is an angle in a semi circle.
triangle OAP, angle in alternate segment.) (iii) ABQC is a cyclic quadrilateral.
∠CPO = 90° ∴ ∠CPA = 90° + b° 14. (b) All chords are of same length.
In ∆ ACP, ∠ACP = 180° – [(b° + 90°) + b°]
15. (b) Only two tangents are parallel to a secant.
⇒ a° + 2b° = 90°
M-102 Mathematics
16. (b) AD = CD A D B In ∆ABP,
and BD = CD C ∠APB = 180° − ( x + y ) = 60°
∴ AB = AD + BD = CD + CD 21. (c) In cyclic quadrilateral ABCD,
= 2CD = 2 × 4 = 8 cm ∠ADC = 80° and ∠ABC = 100°
17. (d) We redraw the figure. (Q opposite angles of a cyclic quadrilateral are
L supplementary)
B
y
y
O Now, ∠ACD = ∠DBA = 50°
xx
20° (angles in same segment of a circle are equal).
P 20°
A S \ ∠CBD = ∠ABC – ∠ABD = 50°
In ∆OPS, using Pythagoras theorem, ∠POS = 70° 22. (b) ∠ABC = ∠ABE + ∠CBE = 90°
and In ∆POL, ∠POL = 70° (angle in a semi circle is a right angle)
B
From figure, 2x + 2y = 140°
∠BOA = x + y = 70° 65°

18. (d) Here, PQ is the common tangent to the three circles A  C


2 2 O
So, PR = (R1 + r) − (R1 − r) = 4R1r ...(i)
E D
RQ = 4R 2 r ....(ii) PQ = 4R1R 2 ....(iii)
We know that PQ = PR + RQ ⇒ ∠ABE = 90° – 65° = 25°
⇒ 4R1R 2 = 4R1r + 4R 2 r (QFrom (i), (ii) and (iii)) ⇒ ∠ACE = ∠ABE = 25°
(∵ angles in same segment of a circle are equal)
1 1 1
⇒ R1R 2 = R1r + R 2 r ∴ = + ⇒ ∠ACE = ∠DEC = 25°
r R2 R1
[alternate interior angles are equal]
19. (b)
23. (d) ABCD is a square of sides
A AB = BC = CD = AD = 1 unit
r
2a
A circle G passing through B and C and touching AD at N,
a where BC is chord of circle.
C
D
a O a B a a
1
r

Since, in figure, DAOB is a right angled triangle r


2
N M
O 1–r
\ OA2 = AB2 + OB2
⇒ (a + r)2 = (2a – r)2 + a2 A B

⇒ a2 + 2ar + r2
+ = 4a2 r2 – 4ar + a2
\ OM bisects the chord BC
2 a
⇒ 6ar = 4a2 ⇒ r = 1 1
3 \ CM = MB = BC =  (∵ BC = 1)
2 2
20. (d) In the given figure, ODC is
⇒ OM = MN – ON = 1 – r
equilateral triangle
A x
O
B In DOMC, OC2 = OM2 + CM2
⇒ ∠ODC = ∠OCD = 60° y
2
Ê 1ˆ
⇒ r2 = (1 – r)2 + Á ˜
x y
Now, in quadrilateral ABCD C
60° 60°
Ë 2¯
D
⇒ x + x + 60° + y + y + 60° = 360°
1 5
⇒ x + y = 120° ⇒ r2 = 1 – 2r + r2 + ⇒r=
P 4 8
Circles M-103
24. (c) Since, radii of circles are 1, 2 and 3 units. 26. (a) Here, O is the centre of circle.
∴ Side of ∆ABC are AB = 5, BC = 3, AC = 4

C
1
1
2
3
B
3
2
Let r be the radius of circle r = (25) 2 − (24) 2
A
= 625 − 576 = 49
= 7 cm
∴ ∆ABC is formed a right angled triangle, where AB is
27. (b) 28. (d)
hypotenuse of triangle.
29. (d) All the three statements are correct.
Since, circumradius of a right angled triangle is the half
of the hypotenuse. 30. (d) All the three statements are false.
1 1 31. (d)
∴ Circumradius = × AB = × 5 = 2.5
2 2
32. (d) O is the centre of the circle. The radius of the circle
25. (c) In ∆MAN is 5 cm.
(2 – r)2 = x2 + r2
⇒ 4 + r2 – 4r = x2 + r2 O

4(1 – r) = x2 ⇒ 4 – 4r = x2 12 c m 5 cm
Q
2
4− x
⇒ r= P
4
PQ is tangent to the circle at P. Then,
OP = 5 cm and ∠OPQ = 90°.
r B
1
1M
r
x N We are given that OQ = 12 cm.
C
1 2–r r
r A
By Pythagoras Theorem, we have
PQ2 = OQ2 – OP2 = (12)2 – (5)2 = 144 – 25 = 119
⇒ PQ = 119 cm .
In ∆CAN, (1 + x)2 + r2 = (1 + r)2
33. (b) In figure, TPOQ is a quadrilateral.
1 + x2 + 2x + r2 = 1 + r2 = 2r
Here, ∠OPT = ∠OQT = 90° ⇒ ∠PTQ + ∠POQ = 180°
⇒ x2 +2x = 2r ⇒ x2 = 2r – 2x
⇒ ∠PTQ + ∠110° = 180° ⇒ ∠PTQ = 70°.
 4 − x2 
⇒ x2 = 2  − 2x 34. (a) In figure
 4 
P
2
4− x
⇒ x2 = − 2x
°
40 40°
2
⇒ 2x2 = 4 – x2 – 4x ⇒ 3x2 + 4x – 4 = 0
A
B

⇒ 3x2 + 6x – 2x – 4 = 0 ⇒ 3x(x + 2) – 2(x + 2) = 0 O

2
⇒ x = , x = −2.
3
∆OAP ≅ ∆OBP (SSS congruence)
2
 2 4
  4− ⇒ ∠POA = ∠POB = ∠AOB ... (i)
r = 4−
3
= 9 ⇒ 36 − 4 = 32 ⇒ 8 .
4 4 36 36 9
Also, ∠AOB + ∠APB = 180°
M-104 Mathematics
⇒ ∠AOB + 80° = 180° B

⇒ ∠AOB = 100° ... (ii) 3cm

Then, from (i) and (ii), A


5cm O

1
∠POA = × 100° = 50°.
2
47. (c) Angle + 40° = 180°
35. (b)
Angle = 180° – 40° = 140°.
36. (c)
48. (a)
37. (c) ∵ ∠ROQ + ∠RPQ = 180°
⇒ ∠ROQ = 180° − 30° = 150° 49. (A) → (s); (B) → (p); (C) → (q); (D) → (r)
38. (a) ∵ ∠RQP = ∠QRP 50. (A) → (r); (B) → (p); (C) → (q); (D) → (s)
and ∠RQP + ∠QRP + ∠QPR = 180° 51. (A) → (r); (B) → (q); (C) → (s); (D) → (p)
⇒ 2∠RQP = 180° − 30° ⇒ ∠RQP = 75° OP ⊥ BC, Let AP = x cm and PB = CP = y cm
1 1
39. (b) ∠RSQ = ∠ROQ = × 150° = 75° On applying Pythagoras in ∆APB and ∆OPB, we have
2 2
36 = y2 + x2 and 81 = (9 – x)2 + y2
40. (a) 90° [∵ OR ⊥ RP]
On solving these, we get, x = 2 cm and y = 4 2 cm.
Sol. (41-45)
C 1 1
Area of ∆ABC = (BC × AP) = × 8 2 × 2 = 8 2 cm2.
10-x 2 2
10-x
52. (A) → (s); (B) → (r); (C) → (q); (D) → (p)
F E
O AD = AF = x cm, BD = BE = y cm, CE = CF = z cm
x 12-x
(∵ tangents drawn from an exterior point to a circle are
B equal in length).
A x D 12-x
AB = 8 cm ⇒ AD + BD = 8 ⇒ x + y = 8  ... (i)
BC = 10 – x + 12 – x = 8 Similarly, BE + CE = 10
⇒ x = 7.
⇒ y + z = 10 ... (ii)
41. (a) AD = 7 cm
z + x = 12 
and ... (iii)
42. (b) BE = 12 – x = 12 – 7 = 5 cm
Adding equations (i) + (ii) + (iii),
43. (d) CF = 10 – x = 10 – 7 = 3 cm
⇒ x + y + z = 15  ... (iv)
1
44. (c) Ar DOAB = × AB × OD Thus, on solving (i), (ii), (iii) and (iv)
2
we get, AD = x cm = 5 cm
1
= × 12 × 4 = 24 cm 2 BE = y cm = 3 cm
2
45. (b) Ar DABC = Ar DAOB + Ar DOBC + Ar DAOC CF = z cm = 7 cm

1 1 53. (A) → (r); (B) → (p); (C) → (q)


= 24 + × 8 × 4 + × 10 × 4 = 60 cm 2
2 2 54. One
46. (a) (OA)2 = (AB)2 + (OB)2 55. Secant
AB = 25 − 9 = 4 cm. 56. Two
Both Assertion and Reason are correct. 57. Point of contact
Also, Reason is the correct explanation of 58. inside
the Assertion.
Circles M-105
59. perpendicular 64. True 65.  False 66.  True 67. True
60. outside 68. True 69.  True 70.  False 71. False
61. equal 72. True 73. True
62. Parallel
63. 4 cm
PB = AP = 52 − 32 (Q OP ^ AB)

= 25 − 9 = 4 cm
12 Acids, Bases and
Constructions
Salts
equal distance points are marked on the ray AX such that
Multiple Choice Questions (MCQs) the minimum number of these point is
(a) 2 (b) 5
DIRECTIONS : This section contains multiple choice (c) 4 (d) 7
questions. Each question has 4 choices (a), (b), (c) and (d) out 6. The sides of a triangle (in cm) are given below. In which
of which only one is correct. case, the construction of triangle is not possible.
(a) 8, 7, 3 (b) 8, 6, 4
1. To draw tangent from an exterior point ‘P’ to a circle using
(c) 8, 4, 4 (d) 7, 6, 5
the centre ‘O’ of the circle, first we
7. Given a triangle with side AB = 8 cm. To get a line segment
(a) join P to O.
3
(b) draw tangent from P to the circle. AB' = of AB, it is required to divide the line segment
4
(c) draw secant from P which intersects the circle at two
points. AB in the ratio
(d) draw perpendicular at P to PO. (a) 3 : 4 (b) 4 : 3
(c) 1 : 3 (d) 3 : 1
2. To divide a line segment AB of any length in the ratio
8. To draw a pair of tangents to a circle which are inclined
2 : 5, first draw a ray AX (or BX) making an acute angle
to each other at an angle of 70°, it is required to draw
with AB. Then, draw 7 arcs intersecting the ray at X1,
tangents at end points of those two radii of the circle, the
X2, X3, X4, X5, X6 and X7 such that AX1 = X1X2 =
angle between them should be
X2X3 = X3X4 = X4X5 = X5X6 = X6X7. Then join X7B,
(a) 110° (b) 20°
then after
(c) 90° (d) 120°
(a) join X2B
9.
To divide a line segment AB in the ratio p : q (p, q are
(b) draw X2B′ parallel to X7 B, which intersects AB at B′.
positive integers), draw a ray AX so that ∠BAX is an acute
(c) join X5B angle and then mark points on ray AX at equal distances
(d) draw perpendicular from X2 to AB. such that the minimum number of the points is
3. To draw a pair of tangents to a circle which are inclined to (a) greater of p and q (b) p+q
each other at an angle of 45° it is required to draw tangents (c) p + q – 1 (d) pq
at end points of those two radii of the circle, the angle 10. Draw a pair of tangents to a circle which are inclined to
between them should be- each other at an angle of 35°, it is required to draw tangents
(a) 60° (b) 90° (c) 45° (d) 135° at the end points of those two radii of the circle, the angle
4. Which is true? between which is
In order to divide a line segment AB = 6 cm, in the radio (a) 105° (b) 70°
3 : 5, we draw a ray AX making an acute angle ∠BAX. (c) 140° (d) 145°
Along AX mark off.... points A1, A2, A3 etc. such that 11. To divide a line segment AB in the ratio 4 : 7, a ray AX
AA1 = A1A2 = etc., The number of points is is drawn first such that ∠BAX is an acute angle and then
(a) 3 (b) 5 (c) 8 (d) 2 points A1, A2, A3, ..... are located at equal distance on the
ray AX and the point B is joined to
5. To divide a line segment AB in the ratio 2 : 5, first a ray
(a) A12 (b) A11 (c) A10 (d) A9
AX is drawn, so that ∠BAX is an acute angle and then at
Constructions M-107

Match the Following

DIRECTIONS : Given below question contains statements given in two columns which have to be matched. Statements
(A, B, C, D,....) in Column-I have to be matched with statements (p, q, r, s,....) in Column-II.

12. To draw tangents from an exterior point P to a circle, drawn using a bangle, each step of continue diagram in random order is
given in column I and how to draw each steps of continue diagram is written in different random order in column II match
the items in two columns.
     Column-I       Column-II

(A)      (p) Draw a circle using a bangle

(B) (q) Draw perpendicular bisector of RB, which intersects RB


at O.
P A B

(C) (r) Draw a secant PAB intersecting the circle at A and B.

O A
R B
P
T

(D) (s) Produce AP to R, such that PA = PR.

O A
R B
P

(E) O A (t) Draw PQ perpendicular to RB at P which intersects the


R B
P semi-circle drawn with centre O and radius OR at Q.
M-108 Mathematics
Q

(F) (u) Draw ray PS and PT.

O A
R B
P
T

(G) (v) With centre O and radius OB (or OR), draw a semi-circle.
A
R B
P

(H) (w) With centre P and radius PQ draw arcs intersecting the
circle drawn using the bangle at S and T.
O A
R B
P

Fill in the Blanks True / False

DIRECTIONS : Complete the following statements with an DIRECTIONS : Read the following statements and write your
appropriate word / term to be filled in the blank space(s). answer as true or false.

13. Two circles are called concentric if both have ............ 16. We can not draw the tangent to a circle at a given point on
centre and ............ radii. it, when the centre of the circle is not known.
14. To construct a triangle we must know atleast its ............ 17. We can draw the tangents to a circle from a point outside
parts. it (external point), only when its centre is known.
15. To construct the tangents to a circle from a external point
18. Construction of a triangle is not possible if AB + BC < AC.
if distance of point from centre of circle is ............ radius
of circle. 19. To draw the perpendicular bisector of line segment PQ,
1
we open the compass more than PQ.
2
Constructions M-109

ANSWER KEY & SOLUTIONS


1. (a) Join P to O 6. (c) We know that, in a triangle sum of two sides of
2. (b) draw X2B′ parallel to X7 B, which intersects AB at triangle is greater than the third side. Here, the sides
B′. of triangle given in option (c) does not satisfy this
condition. So, with these sides the construction of a
3. (d) Here PA and PB are two tangents inclined at an angle
45º triangle is not possible.
Then, x + 90º + 90° + 45º = 360º 7. (d) 8. (a) 9. (b)
x + 225º = 360º
x = 360° – 225° 10. (d) 11. (b)
x = 135º
12. (A) → (p); (B) → (r); (C) → (u); (D) → (t); (E) → (q);
A
(F) → (w); (G) → (s); (H) → (v)
13. Same, different
O x 45º P
14. Three
15. More than
B
16. False
4. (c) 17. False
5. (d) We know that, to divide a line segment AB in the 18. True
ratio m : n, first draw a ray AX which makes an acute
19. True
∠BAX then, marked m + n points at equal distance.
Here, m = 2, n = 5
∴  Minimum number of these points = 2 + 5 = 7
Some
13 Acids, Bases
Applications
Salts
Trigonometry
and
of

4. A 15m long ladder placed vertically along a wall broke


Multiple Choice Questions (MCQs) in such a way that its top touches the ground making an
angle of 60° with it. At what height from the ground did
DIRECTIONS : This section contains multiple choice the ladder break?
questions. Each question has 4 choices (a), (b), (c) and (d) out (a) 7.34 m (b) 8 m
of which only one is correct.
(c) 9.46 m (d) 6.96 m
1. From a point on the ground, the angles of elevation of the 5. A tree 6 m tall casts a 4 cm long shadow. At the same time,
bottom and the top of a transmission tower fixed at the top a flag pole casts a shadow 50 m long. How long is the flag
pole?
of a 20 m high building are 45° and 60° respectively. Find
the height of the tower. (a) 75 m (b) 100 m
(c) 150 m (d) 50 m
(a) 
20 3 –1 m (b)
20 3m 6. The height of a tower is h and the angle of elevation of the
(c) 20 m (d) 40 m top of the twoer is α. On moving a distance h/2 towards
the tower, the angle of elevation becomes β. What is the
tan θ
2. From the figure, find the value of and it is given value of cotα – cotβ?
tan φ
that ‘D’ be the mid point of BC. 1 2
(a) (b)
A 2 3
(c) 1 (d) 2
 7. A bridge across a river makes an angle of 45° with the

river bank. If the length of the bridge across the river is
150 m, width of the river is
(a)
75 2m (b) 150 m

75
B D C (c) m (d) 75 m
2
3 1 8. A professor standing on one end of a football field observes
(a) (b)
4 2 the elevation of the top of a flood light tower at an angle
1 4 of α. He then walks a distance equal to twice the height
(c) (d) of the tower and finds that the elevation of the top is now
2 7
at an angle of 90°– α. What is the value of tanα ?
3. The angles of elevation of the top of a tower, as seen
from two points A and B situated in the same line and at 1 − 2
(a) (b) 2 −1
distance x and y respectively. from the foot of the tower,
(c) 2 (d) 1
are complementary. Find the height of the tower.
(a) x + y (b) xy 9. An electrician has a repair an electric fault on a pole of
height 5 cm. He has to reach a point 1.3 m below the top
xy
(c) (d) x+ y of the pole to undertake the repair work. What should
Some Applications of Trigonometry M-111
be the length of the ladder that he should use which, 16. In the given figure, the positions of the observer and the
when inclined at an angle of 60° to the horizontal, would object are mentioned, the angle of depression is
enable him to reach the required position? Also, how Observer
far from the foot of the pole should he place the foot of
the ladder?
[Take 3 = 1.73].
(a) 4.28 m, 2.14 m
(b) 4.28 m, 2.83 m
(c) 4.6 7m, 2.14 m Object

(d) 3.67 m, 2.14 m (a) 30° (b) 90°


10. An aeroplane when flying at a height of 3125 m from (c) 60° (d) 45°
the ground passes vertically below another plane at an 17. A person walks towards a tower. Initially when he starts,
instant when the angles of elevation of the two planes angle of elevation of the top of tower is 30°. On travelling
from the same point on the ground are 30° and 60° 20 metres towards the tower, the angle changes to 60°. How
respectively.Find the distance between the two planes much more has he to travel to reach the tower?
at that instant.
(a)
10 3 metres (b) 10 metres
(a) 3920 m (b) 6250 m
10
(c) 5125 m (d) 4444 m (c) 20 metres (d) metres
3
11. In the adjoining figure, the length of BC is
18. ABC is a field in the form of an equilateral triangle. Two
(a)
2 3 cm
C vertical poles of heights 45m and 20m are erected at A
m and B respectively. The angles of elevation of the tops of
(b)
3 3 cm 6c the two poles from C are complementary to each other.
There is a point D on AB such that from it, the angles of
(c)
4 3 cm elevation of the tops of the two poles are equal.Then AD
30°
(d) 3 cm A B is equal to –
5 10
12. If the angle of depression of an object from a 75 m high (a)
17 m (b)
20 m
tower is 30°, then the distance of the object from the tower 12 13
is 5 10
(c)
20 m (d)
17 m
(a)
25 3 m (b)
50 3 m 13 12
(c)
75 3 m (d) 150 m 19. A vertical pole of height 10 metres stands at one corner of
a rectangular field. The angle of elevation of its top from
13. The angles of elevation of the top of a tower from two the farthest corner is 30º, while that from another corner
points at distances m and n metres are complementary. If is 60º. The area (in m2) of rectangular field is
the two points and the base of the tower are on the same
200 2 (b) 400
straight line, then the height of the tower is (a)
3 3
(a) mn (b) mn
m 200 2 400 2
(c) (d) None of these (c) (d)
n 3 3
14. If the height and length of the shadow of a man are the 20. From the top of a building of height ‘h’ meter, the angle
same, then the angle of elevation of the sun is of elevation of the top of the tower is ‘a’ and angle of
(a) 45° (b) 60° depression of the foot of the tower is ‘b’. The height of
the tower is
(c) 90° (d) 120°
h + (tan α + tan β) h(tan α + tan β)
15. The height of a tree, if it casts a shadow 15 m long on the (a) (b)
level of ground, when the angle of elevation of the sun is tan β tan β
45°, is
h tan( α + β) h + tan( α + β)
(a) 10 m (b) 14 m (c) (d)
tan β tan β
(c) 8 m (d) 15 m
M-112 Mathematics
21. Which of the following is/are incorrect? 23. What is the angle of elevation if they are standing at a
on A distance of 42m away from the monument?
ati
s erv (a) 30° (b) 45°
f ob
eo (c) 60° (d) 0°
Lin  Horizontal line
O 24. They want to see the tower at an angle of 60°. So, they
C
Li
ne   want to know the distance where they should stand and
of hence find the distance.
ob
ser
va (a) 25.24 m (b) 20.12 m
tio
n B (c) 42 m (d) 24.64 m
(a)
θ1 is the angle of elevation. 25. If the altitude of the Sun is at 60°, then the height of
the vertical tower that will cast a shadow of length
(b)
θ2 is the angle of depression. 20 m is
(c) The angle of elevation or depression is always measured (a) 20√3 m (b) 20/√3 m
from horizontal line through the point of observation. (c) 15/√3 m (d) 15√3 m
(d)
θ1 and θ2 are always equal. 26. The ratio of the length of a rod and its shadow is 1:1 . The
angle of elevation of the Sun is
22. Which of the following statements are incorrect?
(a) 30° (b) 45°
(a) Line of sight is the line drawn from the eyes of the (c) 60° (d) 90°
observer to a point in the object where the person is
27. The angle formed by the line of sight with the horizontal
viewing. when the object viewd is below the horizontal level is
(b) Angle of elevation is the angle formed by the line of (a) corresponding angle
sight with horizontal through the eyes of observer (b) angle of elevation
when the object is above the horizontal level. (c) angle of depression
(c) Angle of depression is the angle formed by the line (d) complete angle
of sight with the horizontal when the object is below
the horizontal level. Assertion & Reason
(d) None of these
DIRECTIONS : Each of these questions contains an Assertion
followed by Reason. Read them carefully and answer the
question on the basis of following options. You have to select
DIRECTIONS : Study the given Case/Passage and answer the the one that best describes the two statements.
following questions. (a) If both Assertion and Reason are correct and Reason is
the correct explanation of Assertion.
Case/Passage-I
(b) If both Assertion and Reason are correct, but Reason is
A group of students of class X visited India Gate on an not the correct explanation of Assertion.
education trip. The teacher and students had interest in history (c) If Assertion is correct but Reason is incorrect.
as well. The teacher narrated that India Gate, official name (d) If Assertion is incorrect but Reason is correct.
Delhi Memorial, originally called All-India War Memorial,
28. Assertion :
monumental sandstone arch in New Delhi, dedicated to the
troops of British India who died in wars fought between In the figure,
A
1914 and 1919.The teacher also said that India Gate, which is
located at the eastern end of the Rajpath (formerly called the
Kingsway), is about 138 feet (42 metres) in height.
30°
B C

if BC = 20 m, then
height AB is 11.56 m.
AB perpendicular
Reason : tan θ = = ,
BC base
where θ is the angle ∠ACB.
[From CBSE Question Bank 2021]
Some Applications of Trigonometry M-113
29. Assertion : If the length of shadow of a vertical pole is
equal to its height, then the angle of elevation of the sun Fill in the Blanks
is 45°.
Reason : According to pythagoras theorem, h2 = l2 + b2, DIRECTIONS : Complete the following statements with an
where h = hypotenuse, l = length and b = base appropriate word / term to be filled in the blank space(s).

Match the Following 32. The ............... is the line drawn from the eye of an observer
to the point in the object viewed by the observer.
DIRECTIONS : Each question contains statements given in two
33. The ............. of an object viewed, is the angle formed by
columns which have to be matched. Statements (A, B, C, D) in
the line of sight with the horizontal when it is above the
column-I have to be matched with statements (p, q, r, s) in column-II.
horizontal level, i.e., the case when we raise our head to
30. From a window, h metres high above the ground, of a house look at the object.
in a street, the angles of elevation and depression of the top
and bottom of another house on the opposite side of the street 34. The ....................... of an object viewed, is the angle formed
are α and β respectively, then match the column. by the line of sight with the horizontal when it is below
E the horizontal level, i.e., the case when we lower our head
to look at the object.
35. In the adjoining figure, the positions of observer and object
α
D
β A are marked.
O
(Observer)
hm

β B
30°
C
P (Object) Horizontal
Column-I Column-II
(A) DB (p) h(1 + tana cotb) The angle of depression is ...............
(B) DE (q) h / sinb
(C) CE (r) h tan a cot b 36. The top of a building from a fixed point is observed at an
(D) AD (s) h cot b angle of elevation 60° and the distance from the foot of
31. Column-I Column-II the building to the point is 100 m. then the height of the
(A) A (p) 60º building is................

10 37. In fig. the angles of depressions from the observing


45°
positions O1 and O2 respectively of the object A are
B
BC = ?
C .................., ................... .
(B)
A (q) 10 O2 O1

60°
60°
B C
3
AB = ?
1
(C)
A
(r) 45°
5 A
40 B C


B
20
C True / False
=?
A
DIRECTIONS : Read the following statements and write your
(D) (s) 3
answer as true or false.

45°  A straight highway leads to the foot of a tower of height 50 m.


B D
C From the top of the tower, the angles of depression of two cars
2
10 standing on the highway are 30° and 60°. [ 3 = 1.73]
tan  = ?
M-114 Mathematics
Now, based on the above information, mark the given statements 38. Distance between the cars is 57.67 m.
as true or false.
39. First car is at a distance of 38.90 m from the tower.
A
40. Second car is at a distance of 86.50 m. from the tower.
41. Car at point C is at a distance of 200 m away from the top

50 m
of the tower.
30° 60°
C D B
Some Applications of Trigonometry M-115

ANSWER KEY & SOLUTIONS


1.
(a) Let AB be the building of height 20m and BC be the
transmission tower of height h metres.
AB 20
In ∆ OAB, tan 45° = ⇒1=
OA OA

⇒ OA = 20 m
In ∆OAC,

h + 20
tan 60° =
20
h
h + 20 In rt. ∆BDC, tan (90 – θ) =
⇒ 3 = y
20
h
⇒ h = 20 ( 3 –1 m ) ⇒ cot θ =
y
 ... (ii)

2. (b) BD = DC given as D is the mid point of BC. Multiplying (i) by (ii), we get
A h h
tan θ × cot θ = ×
x y

 1 h2
⇒ tan θ × =
tan θ xy

2
⇒ 1 = h ⇒ h 2 = xy
C xy
B D
⇒ h = xy
AC
In ∆ADC, cot θ = ... (i) 4. (d)
CD
AC
In ∆ABC, cot φ = ... (ii)
BC
AC
cot θ CD
∴ =
cot φ AC
BC
AC
= CD [∵ 2CD = BC] Let the broken part of the ladder, AC = x metres
AC
2CD ∴ length of ladder = AC + AB = 15 m
cot θ 2
⇒ = ⇒ x + AB = 15 ⇒ AB = (15 – x)m
cot φ 1
In ∆ ABC,
tan θ 1
⇒ =
tan φ 2 AB 15 − x 3 15 − x
sin 60° = = ⇒ =
3. (b) Let DC be the tower of height ‘h’ metres. AC x 2 x

In rt. ∆ACD, tan θ =


h
… (i)
⇒ 3 x = 30 − 2 x ⇒ x ( )
3 + 2 = 30
x
M-116 Mathematics

⇒ x=
30
=
(
30 2 − 3 ) From (i) and (ii)
tan2α + 2tanα – 1 = 0
( )
3 + 2 ( 2 )2 − 3 2
∴ tan α = 2 − 1

⇒ x = 30 ( 2 − 3 ) = 8.04 m
9. (a)
In figure, the electrician is required to reach the point
Hence, AB = 15 – x = (15 – 8.04)m = 6.96 m B on the pole AD.
5. (a) Let h be the length of the pole. A

6 h
By the given condition =
4 50
B
6 × 50
⇒ h = = 75m
4
60°
6. (a)
Here, In rt. ∆ABD,  A
D C
h
+ BC So, BD = AD – AB = (5 – 1.3)
BD
cot α = = 2 ....(i) = 3.7 m
AB h
also, In rt. ∆ABC,
hm BD 3.7 3
So, sin 60° = ⇒ =
BC BC BC BC 2
cot β = =
AB h 3.7 × 2
α β ∴ BC = = 4.28 m (Approx.)
Now, cot α – cot β 3
D C B
h m
h 2 i.e., the length of the ladder should be 4.28 m
+ BC BC
2 1
= − = DC 1
h h 2 Now, = cot 60° =
BD 3
7. (a)
A
3.7
i.e., DC = = 2.14m (Approx )
3
15 dge
0m
i

Therefore, he should place the foot of the ladder at a


Br

River River
distance of 2.14 m from the pole.
45°
C B
10. (b)
Let A1 and A2 be the position of the two aeroplanes.
and A1A2 = x m
x 1
= sin 45° = OM = y m
150 2

150 2 y
⇒ x= . = 75 2 m. = cot 30° = 3
2 2 3125

8. (b) In the following figure, ⇒ ⇒ y = 3 × 3125


A1

Let OB = x m.
A2
h
tan α = ...(i) 60° 3125 m
2h + x
30°
h O M
tan(90° − α ) = ⇒ x = tanα  ... (ii) y
x
Some Applications of Trigonometry M-117
y 1 C
Also, = cot 60° =
3125 + x 3


(3125) 3
=
1
3125 + x 3
⇒ 3125 + x = (3125) (3) ⇒ x = 6250 m.

BC
11. (d) Hint: sin 30° = A 15 m B
AC
1 BC In ∆ ABC,
⇒ = ⇒ BC = 3 cm.
2 6
BC
AB = tan 45°
12. (c) Hint: tan30° = BA
OB
⇒ BC = AB = 15 m.
A
30° 16. (c) ∠ XCA = ∠CAB = 60°

Hence, angle of depression = 60°


Tower 17. (b)
D
30°
B
O (object)
x
1 75
⇒ =
3 OB 30o 60o
A 20 m B y C
⇒ OB = 75 3 m
x
13. (a) In ∆DBC, tan 60o = ⇒  x = 3y  ... (i)
y
14. (a) Let AB be the height of a man and BC be the shadow
x
of a man. In ∆ΑDC, tan 30o =
A 20 + y

1 3y
⇒ = (Q From (i))
3 20 + y
⇒ y + 20 = 3y
⇒ 2y = 20 ⇒ y = 10 m
18. (b) R
A
C B

∴ AB = BC D
Q
In ∆ ABC,
B
C
AB AB Taking DARC and DBQC
tan θ = ⇒ = tan θ ⇒ tan θ = 1 ⇒ θ = 45°
BC AB
R Q
⇒ tan q = 1 ⇒ q = 45°
45 20
15. (d) Let BC be the tree of height h metre.
 90 – 
Let AB be the shadow of tree. A C B C
M-118 Mathematics
tan(90 – q) = cotq 20. (b) In figure,
20 AC y
=  (let AC = BC = AB = x m) tan a = and
BC 45 x
h
x2 = 900 ⇒ x = 30 tan b =
x
Now, in D ARD and BQD
tan α y
\ =
45 20 AD 9 tan β h
= = tan θ ⇒ =
AD BD BD 4
 tan α 
h + y = h 1 +
9  10   tan β 
AD = × 30 = 20   m
13  13 
21. (d)
19. (a) Let PQRS be the rectangular field with length l and 22. (d) All the statements given in option a, b and c are correct.
width b and RT be the vertical pole.
23. (b) A
T

42 m
60°
S R
C 42 m B
30°
Since h = 42
P Q d = 42
So q = 45°
RT 1 10
In ∆PRT, tan 30° = ⇒  = 42
PR 3 PR 24. (a) tan 60° =
x
PR = l 2 + b 2 = 10 3 42 42
x= = 3 = 14 3 = 24.24 m
3 3
l2 + b2 = 100 × 3 = 300 ... (i)
25. (a) A
10
In ∆SRT, tan 60° =
SR
10
⇒ 3= h
SR
10
SR = =l 60°
3 C 20 m B
100 h
⇒ l2 = ... (ii) tan 60° =
3 20
Use equation (ii) in (i), then h = 20 3 m
100 800 26. (b)
b2 = 300 − = A
3 3
100 800 80000
l2b2 = × =
3 3 9 x
200 2
⇒ lb = 45°
3 C B
x
Hence, the required area of rectangular field is
Since ratio of length of a rod: Shadow of rod = 1:1
200 2 .
3 so Angle = 45°
Some Applications of Trigonometry M-119
27. (a) Corresponding angle 32. line of sight
28. (a) Both the assertion and reason are correct, reason is 33. angle of elevation
the correct explanation of the assertion.
34. angle of depression
AB AB
tan30° = = 35. 30°
BC 20
1 20 36. 100 3 m
AB = × 20 = = 11.56 m.
3 1.73 37. [30°, 45°]
29. (b)
Both assertion and reason are correct, but reason is O2 O1
F E
not the correct explanation of the assertion.
60°
30. (A) → q; (B) → r; (C) → p; (D) → s
31. (A) → q ; (B) → s ; (C) → p ; (D) → r
AB 45°
(A) tan 45° = ⇒ BC = 10 A
BC B C
AB AB Depression angle at O1 = 90° – 60° = 30°
tan 60° =
(B) = ⇒ AB = 3 × 3 = 3
BC 3 Depression angle at O2 = 45° = ∠BAO2
20 1 38. True
(C)
cos θ = = = cos 60º ⇒ θ = 60º
40 2
AB 39. False
(D) In ∆ABC, tan 45° = ⇒ AB = 2, and
BC 40. True
AB 2 1 41. False
tan θ = = = .
BD 10 5
14 Acids,
Surface
Salts
Bases and
Areas
and Volumes
20
Multiple Choice Questions (MCQs) (a) 
π 10 cm
50
DIRECTIONS : This section contains multiple choice (b)
questions. Each question has 4 choices (a), (b), (c) and (d) out π
of which only one is correct. 25
(c)
π 2 cm
1. The volume of a cylinder is 448 p cm3 and height 7 cm. (d) 40 π
Find its lateral surface area and total surface area is
7. If h be the height and α the semi-vertical angle of a right
(a) 253 cm2 (b) 352 cm2
2
circular cone, then its volume is given by
(c) 532 cm (d) 325 cm2
1 3 1 2
2. The diameter of a garden roller is 1.4 m, and 2m long. (a) πh tan 2 α (b) πh tan 2 α
3 3
How much area will it cover in 5 revolutions.
1 1 3
(a) 44 m2 (b) 140 m2 (c) πh 2 tan 3 α (d) πh tan 3 α
2 3 3
(c) 440 m (d) 220 m2
8. Cubes A, B, C having edges of 18 cm, 24 cm and 30 cm
3. If a sphere and a cube have equal surface areas, then the respectively are melted and moulded into a new cube D.
ratio of the diameter of the sphere to the edge of the cube is Find the edge of the bigger cube D.
(a) 1 : 2 (b) 2 : 1 (a) 32 (b) 28
(c) 6: ≠
≠ : 6 (d) (c) 39 (d) 36
4. The internal and external diameter of a hollow 9. The height of a conical tent is 14 m and its floor area is 346.5
hemispherical vessel are 42 cm and 45.5 cm. respectively. m­­­2. The length of canvas, 11m wide, required for it is.
Find its capacity (volume) and also its outer curved (a) 490 m (b) 525 m
surface area. (c) 665 m (d) 860 m
(a) 5.27 litres, 3253.25 cm2
10. Three identical cones with base radius r are placed on
(b) 5.20 litres, 3253.25 cm2 their bases so that each is touching the other two. The
(c) 5.27 litres, 3200.18 cm2 radius of the circle drawn through their vertices is –
(a) smaller than r
(d) 5.27 litres, 3250.25 cm2 (b) equal to r
5. If h, c, v are respectively the height, the C.S.A and the (c) larger than r
(d) depends on the height of the cones
volume of a cone, find the value of 3πvh3 – c2h2 + 9v2
(a) 1 (b) 2 11. The diameter of hollow cone is equal to the diameter of a
(c) 0 (d) 3 spherical ball. If the ball is placed at the base of the cone,
what portion of the ball will be outside the cone?
6. The diagram shows the parts of a right cylinder. The
(a) 50% (b) less than 50%
volume of the cylinder, in cm3 is
(c) more than 50% (d) 100%
Surface Areas and Volumes M-121
12. A slab of ice 8 inches in length, 11 inches in breadth, and 21. In the adjoining figure, the bottom of the
2 inches thick was melted and resolidified in the form of glass has a hemispherical raised portion. If
a rod of 8 inches diameter. The length of such a rod, in the glass is filled with orange juice, the
inches, is nearest to quantity of juice which a person will

15cm
(a) 3 (b) 3.5 get is
(c) 4 (d) 4.5 (a) 135 π cm3
13. The base radii of a cone and a cylinder are equal. If their (b) 117 π cm3
curved surface areas are also equal, then the ratio of the (c) 99 π cm3
slant height of the cone to the height of the cylinder is
6cm
(d) 36 π cm3
(a) 2 : 1 (b) 1 : 2
(c) 1 : 3 (d) 3 : 1 22. Ratio of lateral surface areas of two cylinders with equal
height is
14. If the perimeter of one face of a cube is 20 cm, then its
(a) 1 : 2 (b) H : h
surface area is
(c) R : r (d) None of these
(a) 120 cm2 (b) 150 cm2
2
(c) 125 cm (d) 400 cm2 23. Ratio of volumes of two cylinders with equal height is
(a) H : h (b) R:r
15. A cube of side 12 cm is painted red on all the faces and
(c) R2 : r2 (d) None of these
then cut into smaller cubes, each of side 3 cm. What is the
total number of smaller cubes having none of their faces 24. Ratio of volumes of two cones with same radii is
painted? (a) h1 : h2 (b) s1 : s2
(a) 16 (b) 8 (c) r1 : r2 (d) None of these
(c) 12 (d) 24 25. Volume of a spherical shell is given by
16. If the diameter of the sphere is doubled, the surface (a) 4π (R2 – r2) (b) π (R3 – r3)
area of the resultant sphere becomes x times that of the 4
(c) 4π (R3 – r3) (d) π (R3 – r3)
original one, then x would be 3
(a) 2 (b) 3
(c) 4 (d) 8 26. The volume of a largest sphere that can be cut from
cylindrical log of wood of base radius 1m and height 4 m, is
17. If h be the height and α the semi-vertical angle of a right
16 8
circular cone, then its volume is given by (a) ≠ m3 (b) ≠ m3
3 3
1 1 2
(a) πh3 tan 2 α (b) πh tan 2 α 4 10
3 3 (c) ≠ m3 (d) ≠ m3
3 3
1 1
(c) πh 2 tan 3 α (d) πh3 tan 3 α
3 3 27. If four times the sum of the areas of two circular faces
of a cylinder of height 8 cm is equal to twice the curve
18. If the radius of the sphere is increased by 100%, the surface area, then diameter of the cylinder is
volume of the corresponding sphere is increased by (a) 4 cm (b) 8 cm
(a) 200% (b) 500% (c) 2 cm (d) 6 cm
(c) 700% (d) 800%
28. A rectangular sheet of paper 40 cm × 22 cm, is rolled to
19. A sphere is melted and half of the melted liquid is used form a hollow cylinder of height 40 cm. The radius of the
to form 11 identical cubes, whereas the remaining half is cylinder (in cm) is
used to form 7 identical smaller spheres. The ratio of the (a) 3.5 (b) 7
side of the cube to the radius of the new small sphere is (c) 80/7 (d) 5
1/ 3 1/ 3
4 8 29. A right circular cylinder has its height equal to two times
(a)   (b)  
3 3 its radius. It is inscribed in a right circular cone having
(c) (3)1/3 (d) 2 its diameter equal to 10 cm and height 12 cm, and the
axes of both the cylinder and the cone coincide. Then, the
20. If a solid of one shape is converted to another, then the
volume (in cm3) of the cylinder is approximately
volume of the new solid
(a) 107.5 (b) 118.6
(a) remains same (b) increases
(c) 127.5 (d) 128.7
(c) decreases (d) can’t say
M-122 Mathematics
30. If the ratio of volumes of two cubes is 27 : 64, then the 36. The number of solid cones with integer radius and integer
ratio of their surface area is: height each having its volume numerically equal to its
(a) 3 : 4 (b) 4 : 3 total surface area is
(c) 9 : 16 (d) 16 : 9 (a) 0 (b) 1
(c) 2 (d) infinite
31. Volumes of two spheres are in the ratio 125 : 64. The ratio
of their surface areas will be 37. A solid metallic cylinder of height 10 cm and diameter 14
(a) 5 : 4 (b) 25 : 16 cm is melted to make two cones in the proportion of their
(c) 16 : 25 (d) 125 : 64 volumes as 3 : 4, keeping the height 10 cm, what would
be the percentage increase in the flat surface area?
32. A solid metallic block of volume one cubic metre is (a) 9 (b) 16
melted and recast into the form of a rectangular bar of (c) 50 (d) 200
length 9 metres having a square base. If the weight of the
block is 90 kg and biggest cube is cut off from the bar, 38. Given three cubes with integer side lengths, if the sum of
then the weight of the cube is surface areas of three cubes is 498 sq. cm, then the sum
of the volumes of the cubes in all possible solutions is
1 2
(a)
6 kg (b) 5 kg (a) 731 (b) 495
3 3 (c) 1226 (d) None of these
2 1
(c)
4 kg (d) 3 kg 39. Which one of the following is/are incorrect ?
3 3
(a) Total surface area of cuboid is 2(lb + bh + hl)
33. Consider a cuboid all of whose edges are integers and (b) Total surface area of a cube is 4l2
whose base is a square. Suppose the sum of all its edges (c) Area of four walls = 2h (l + b)
is numerically equal to the sum of the areas of all its six (d) Area of four walls = Height × Perimeter of the room
faces. Then, the sum of all its edges is
40. A Circular Cylinder can not be separated into
(a) 12 (b) 18
(a) circular end at the bottom
(c) 24 (d) 36
(b) curved surface
34. Shyam wants to make a solid brick shape structure from (c) circular end at the top
400 wooden cubes of unit volume each. If the sides of (d) None of these
the solid brick have the ratio 1 : 2 : 3, then the maximum
41. Which one of the following is / are incorrect ?
number of cubes, which can be used, will be _______.
(a) Total surface area of cylinder is 2πr2 + 2πrh.
(a) 400 (b) 288
(b) Total surface area of a sphere is 4πr2.
(c) 300 (d) 384
(c) Total surface area of cone is πr2 + πrl.
35. Sealed bottle containing some water is made up of two (d) None of these
cylinders A and B of radius 1.5 cm and 3 cm respectively,
42. Which one of the following is/ are made up of
as shown in the figure. When the bottle is placed right up
combinations of two or more of the basic solids?
on a table, the height of water intit is 15 cm, but when
(a) Buildings (b) Funnel
placed upside down the height of water is 24 cm. The
(c) Monuments (d) Test-tube
height of the bottle is
43. Among the following, which one is/are correct?
(a) The slant height is the longest side of a pyramid.
(b) The section between the base and a plane parallel to
the base of a solid is known as frustum.
(c) All the surfaces of a cuboid are square.
15 cm (d) For a cylinder, the top, the bottom and the walls of
24 cm
the cylinder determine the total surface area.
44. If a marble of radius 2.1 cm is put into a cylindrical cup
(a) 25 cm (b) 26 cm full of water of radius 5cm and height 6 cm, then how
(c) 27 cm (d) 28 cm much water flows out of the cylindrical cup?
(a) 38.8 cm3 (b) 55.4 cm3
(c) 19.4 cm3 (d) 471.4 cm3
Surface Areas and Volumes M-123
45. A cubical ice-cream brick of edge 22 cm is to be distributed
among some children by filling ice-cream cones of radius
2 cm and height 7 cm upto its brim. How many children
will get the ice-cream cones? Area = 551 m2
(a) 163 (b) 263 (c) 363 (d) 463 r=7m
46. The volume of the largest right circular cone that can be
cut out from a cube of edge 4.2 cm is
(a) 9.7 cm3 (b) 77.6 cm3 49. The volume of cylindrical cup is
3
(c) 58.2 cm (d) 19.4 cm3 (a) 295.75cm3 (b) 7415.5cm3
3
(c) 384.88cm (d) 404.25cm3
47. If two solid hemispheres of same base radius r are joined 50. The volume of hemispherical cup is
together along their bases, then curved surface area of (a) 179.67cm3 (b) 89.83 cm3
this new solid is 3
(c) 172.25 cm (d) 210.60 cm3
(a) 4πr2 (b) 6πr2 (c) 3πr2 (d) 8πr2
51. Which container had more juice and by how much?
48. A right circular cylinder of radius r cm and height h cm (a) Hemispherical cup, 195 cm3
(h > 2r) just encloses a sphere of diameter (b) Cylindrical glass, 207 cm3
(a) r cm (b) 2r cm (c) h cm (d) 2h cm (c) Hemispherical cup, 280.85 cm3
(d) Cylindrical glass, 314.42 cm3
52. The height of the conical tent prepared to accommodate
four students is
DIRECTIONS : Study the given Case/Passage and answer (a) 18m (b) 10m
the following questions. (c) 24m (d) 14m
53. How much space on the ground is occupied by each
Case /Passage-I student in the conicaltent
Adventure camps are the perfect place for the children to (a) 54m2 (b) 38.5m2
practice decision making for themselves without parents and (c) 86m 2 (d) 24m2
teachers guiding their every move. Some students of a school
reached for adventure at Sakleshpur. At the camp, the waiters Case /Passage-II
served some students with a welcome drink in a cylindrical glass
and some students in a hemispherical cup whose dimensions are
shown below. After that they went fora jungle trek. The jungle
trek was enjoyable but tiring. As dusk fell, it was time to take
shelter. Each group of four students was given a canvas of area
551m2. Each group had to make a conical tent to accommodate
all the four students. Assuming that all the stitching and wasting
incurred while cutting, would amount to 1m2, the students put
the tents. The radius of the tent is 7 m.
[From CBSE Question Bank 2021]

[From CBSE Question Bank 2021]


The Great Stupa at Sanchi is one of the oldest stone structures
in India, and an important monument of Indian Architecture. It
was originally commissioned by the emperor Ashoka in the
3rd century BCE. Its nucleus was a simple hemispherical
brick structure built over the relics of the Buddha. .It is a perfect
example of combination of solid figures. A big hemispherical
22
dome with a cuboidal structure mounted on it.  Take π = 
 7
54. Calculate the volume of the hemispherical dome if the
height of the dome is21 m –
d = 7 cm
h = 10.5 cm (a) 19404 cu. m (b) 2000 cu .m
d = 7 cm
(c) 15000 cu. m (d) 19000 cu. m
M-124 Mathematics
55. The formula to find the Volume of Sphere is- 63. The total surface area of cone with hemispherical ice cream
2 3 4 3 is
(a) πr (b) πr (a) 858 cm2 (b) 885 cm2
3 3
(c) 588 cm2 (d) 855 cm2
(c) 4 pr2 (d) 2 pr2
56. The cloth require to cover the hemispherical dome if the
radius of its base is14m is Assertion & Reason
(a) 1222 sq.m
(b) 1232 sq.m DIRECTIONS : Each of these questions contains an Assertion
(c) 1200 sq.m followed by Reason. Read them carefully and answer the
(d) 1400 sq.m question on the basis of following options. You have to select
57. The total surface area of the combined figure i.e. the one that best describes the two statements.
hemispherical dome withradius 14m and cuboidal shaped
top with dimensions 8m × 6m × 4m is (a) If both Assertion and Reason are correct and Reason is
(a) 1200 sq. m (b) 1232 sq. m the correct explanation of Assertion.
(c) 1392 sq.m (d) 1932 sq. m (b) If both Assertion and Reason are correct, but Reason is
58. The volume of the cuboidal shaped top is with dimensions not the correct explanation of Assertion.
mentioned in question 4
(c) If Assertion is correct but Reason is incorrect.
(a) 182.45 m3 (b) 282.45 m3
(c) 292m 3 (d) 192m3 (d) If Assertion is incorrect but Reason is correct.
Case /Passage-III 64. Assertion: Total surface area of the cylinder having
radius of the base 14 cm and height 30 cm is 3872 cm2.
On a Sunday, your Parents took you to a fair. You could see
lot of toys displayed,and you wanted them to buy a RUBIK’s Reason: If r be the radius and h be the height of the
cube and strawberry ice-cream for you. Observe the figures and cylinder, then total surface area = (2πrh + 2πr2 ).
answer the questions-: [From CBSE Question Bank 2021]
65. Assertion: If the height of a cone is 24 cm and diameter of
the base is 14 cm, then the slant height of the cone is 15 cm.
Reason: If r be the radius and h the slant height of the
cone, then slant height = h2 + r 2 .

66. Assertion: If the radius of a cone is halved and volume is


not changed, then height remains same.
Reason: If the radius of a cone is halved and volume is
not changed then height must become four times of the
59. The length of the diagonal if each edge measures 6cm is original height.
(a) 3√3 (b) 3√6 67. Assertion: If a ball is in the shape of a sphere has a
(c) √12 (d) 6√3
surface area of 221.76 cm2, then its diameter is 8.4 cm.
60. Volume of the solid figure if the length of the edge is 7cm
Reason: If the radius of the sphere be r, then surface
is-
(a) 256 cm3 (b) 196 cm3 1 S
area, S = 4πr2, i. e. r = .
(c) 343 cm3 (d) 434 cm3 2 π

61. What is the curved surface area of hemisphere (ice cream) 68. Assertion: No. of spherical balls that can be made out of
if the base radiusis 7cm? a solid cube of lead whose edge is 44 cm, each ball being
(a) 309 cm2 (b) 308 cm2 4 cm. in diameter, is 2541
(c) 803 cm 2 (d) 903 cm2
Volume of one ball
62. Slant height of a cone if the radius is 7cm and the height Reason : Number of balls = .
volume of lead
is 24 cm
(a) 26cm (b) 25 cm
(c) 52 cm (d) 62cm
Surface Areas and Volumes M-125
Column-I Column-II
Match the Following
(A) Area of bottom of cylinder (p) 10.56
(B) Outer curved surface area (q) 1.54
DIRECTIONS : Each question contains statements given in
two columns which have to be matched. Statements (A, B, C, (C) Curved area of conical cavity (r) 5.5
D)in Column-I have to be matched with statements (p, q, r, s) (D) Total surface area (s) 17.6
in Column-II. 72. Column-I (Object) Column-II (Shape)
69. For figure shown, match the column (A) Dice (p) Right circular cone
(B) Road rollers (q) Sphere
Hemisphere (C) Ice-cream cone (r) Cylinder
(D) Volleyball (s) Cube
3.5cm

5cm. A Fill in the Blanks

DIRECTIONS : Complete the following statements with an


appropriate word / term to be filled in the blank space(s).

73. The volume of a cube with diagonal d is .............


Fig. : Top (Lattu)
74. A cube is a special type of ..............
Column -I Column-II
75. If the heights of two cylinders are equal and their radii
(A) Curved area of hemisphere (p) 3.25
are in the ratio of 7 : 5, then the ratio of their volumes is
(B) Height of cone (q) 77/4 .................
(C) Slant height of cone (r) 3.7
76. The volume of a solid is the measurement of the portion
(D) Surface area of top (s) 39.6 of the ............ occupied by it.
70. For a wooden article was made by scooping out a 77. If the volume of a cube is 64 cm3, then its surface area is
hemisphere from each end of a solid cylinder, as shown ....................
in Fig. If the height of the cylinder is 10 cm, and its base
is of radius 3.5 cm, match the column. 78. If the volume and the surface area of a solid sphere are
numerically equal, then its radius is .........

79. The length of the diagonal of a cube that can be inscribed


in a sphere of radius 7.5 cm is ............

True / False

DIRECTIONS : Read the following statements and write your


Column-I Column-II answer as true or false.
(A) Volume of cylinder (p) 616/3
80. Volume of the solid is measured in cubic units.
(B) Volume of scoops (q) 374
81. Area is the length of the boundary of a closed figure.
(C) Total surface area (r) 122.5 π
82. Area is the total surface covered by a closed figure.
(D) Volume of the article (s) 171.5/3 π
71. From a solid cylinder of height 2.4 cm and diameter πd 3
83. The volume of sphere of diameter is .
1.4cm a conical cavity of the same height and same 6
diameter is hollowed out then match the column.
M-126 Mathematics
84. The total surface area of a solid cylinder of radius r and 87. If the curved surface of a right circular cylinder is
height h is 2π r (h + r). 80
1760 cm2 and its radius is 21 cm, then its height is
85. If a right circular cone and a cylinder have equal circles 3
cm.
as their base and have equal heights, then the ratio of
their volumes is 2 : 3 50 2
88. If the total surface area of a cube is m , then its side
86. If the base area and the volume of a cone are numerically is (5/3) m. 3
equal, then its height is 3 units.
Surface Areas and Volumes M-127

ANSWER KEY & SOLUTIONS


1. (b) Volume of cylinder = πr2h 2
1  1 
448π = πr2 × 7 ⇒ 3π  πr 2 h  h3 − (πr )2 h 2 + 9  πr 2 h 
3  3 
448 448
⇒ r2 = ⇒ r= 1 
⇒ 3π  πr 2 h  h3 − π 2 r 2 (r 2 + h 2 )
7   7 3 

⇒ r = 64 ⇒ r = 8 cm 1
h 2 + 9 × π 2 r 4 h 2  (∵  = h2 + r 2 )
∴ L. S. A or C. S. A = 2ph 9

22 ⇒ p2r2h4 – p2r4h2 – p2r2h4 + p2r4h2 = 0 = R.H.S.


= 2 × × 8 × 7 = 352 cm2
7 6. (b) Height of cylinder, h = 2 cm
1.4 Circumference of base of cylinder = 10 cm
2. (a) r = = 0.7 m and h = 2 m
2
5
Area covered   = C.S.A × number of revolutions ⇒ 2πr = 10  ⇒  r =
π
= 2ph × 5 = 10prh
25 50
Volume of the cylinder = πr 2 h = π × 2 × 2 =
 22  2 π π
⇒ 10   (0.7)(2) = 44 m
 7 
7. (a) In right ∆AOB A
3. (d) Let the diameter of the sphere be d units and the edge
OB
of the cube be a units, then = tan α ⇒ OB = h tan α α
OA h
2 2
d  2 d 6 d 6 ⇒ r = h tan α
4π   = 6a   ⇒  =   ⇒  = . B
2
  a 2 π a π
O r
1 2
Volume of cone = πr h
45.5 42 3
4. (a) =R = 22.75 cm ,=
r = 21 cm
2 2
1 1
Capacity (volume) = π(h tan α) 2 h = πh3 tan 2 α
3 3
2 2 22
= π (R 3 − r 3 ) = × × [(22.75)3 − (21)3 ] 8. (d) Volume of A = 183 = 5832 cm3
3 3 7
Volume of B = 243 = 13824 cm3
= 2 × 22 × 2513.543 = 5266.46 cm3
Volume of C = 27000 cm3
3 7
5266.46 Total volume A, B and C = 46656 cm3
or = lts. = 5.27 litres
1000 Now,volumeofnewcubeD=sumofvolumesofA,BandC.
Outer surface area Let a be the edge of D.
22 2 Then a3 = 46656  ⇒  a = 36 cm.
= 2πR2 = 2 × × (22.75) = 3253.25 cm2
7
 3465 7 
2 2 9. (b) pr2 =346.5  ⇒  r2 =  × 
5. (c) We know that, l = r +h ,  10 22 
2
441  21 
v =
1 2
πr h , c = prl = =     ⇒  r = 21 m
3 4  2 2
441 2 1225 35
L.H.S. 3πvh3 – c2h2 + 9v2 l = r 2 + h2 = + (14 )= = m
4 4 2
M-128 Mathematics
Area of canvas 1 2 1
24. (a) ≠r1 h1 : ≠r22 h2
 22 21 35  3 3
= πrl =  × ×  m 2 = 577.5m 2
 7 2 2 1 2 1
⇒ ≠r1 h1 : ≠r12 h2 (Q r1 = r2)
Area 577.50 3 3
Length of canvas = = m = 525 m
Width 1.1 ⇒ h1 : h2
10. (c) The centres of the bases of the cones form a triangle 25. (d) Volume of spherical shell
of side 2r. The circumference of the circle will be
4 3 4 3 4
identical to a circle drawn through the vertices of the = πR − πr = π( R3 − r 3 )
3 3 3
cones and thus, it will have a radius of 2 / 3 times
r, which is greater than r. 4 3 4
26. (c) Volume of sphere = ≠r = ≠ (1)3
11. (c) Though, it is given that diameter of the 3 3
cone is equal to the diameter of the 27. (b) 28. (a)
spherical ball. But the ball will not fit into 29. (c) Let h be the height and r be the radius of the cylinder.
the cone because of its slant shape.
Hence, more than 50% of the portion of Given h = 2r
the ball will be outside the cone. Since, ∆PSC ~ ∆AOC
12. (b) Volume of the given ice cuboid = 8 × 11 × 2 = 176 ∴ By similarity of two triangles, we have
Let the length of the required rod be l.
PS SC PC h 5 − r PC
= = ⇒ = =
82 AO OC AC 12 5 AC
\ π = 176   \  l = 3.5 inches
4 h 5− r 2r 5 − r
⇒ = ⇒ = ⇒ 10r = 60 – 12r
 2 12 5 12 5
13. (a) πr = 2πrh  ⇒  =
h 1 A

20
14. (b) Edge of cube = cm = 5 cm, Q P 12
4 h
r
surface area = 6×52 cm2 = 150 cm2 B
R O S
C
15. (b)
5
16. (c)
17. (a) 30 60
⇒ r = and h =
11 11
18. (c) When the radius is increased by 100%, the
corresponding volume becomes 800% and thus 22  900  60
Volume of cylinder = πr2h =   ≈ 127.50
increase is 700%. 7  121  11
19. (b) As per the given conditions,
30. (c) Let a1 and a2 be the sides of two cubes and S1 andS2
4 1/ 3 be the surface area of cubes
11a = 7 × × π × r 3
3 a 8
∴ = 
3 r 3 a13 27 a 3 S1 6a12 9
∴ = ⇒ 1 = , = 2 =
20. (a) a23 64 a2 4 S2 6a2 16

21. (b) [Hint. Quantity of juice 4 3


πr
V1 125 3 1 125 r 5
31. (b) = ; = ⇒ 1 = ;
 2 2 3 V2 64 4 πr 3 64 r2 4
=  π × 3 × 15 − π × 3  cm3] 2
 3  3
2
22. (c) 2πRh : 2πrh = R : r SA1 4πr12  r1  25
= =  = = 25 : 16.
23. (c) πR2h : πr2h = R2 : r2 SA2 4πr22  r2  16

Surface Areas and Volumes M-129
32. (d) Volume of metallic block = 1 m3...(1) 35. (c)
Let the side of the square base be x m r

So, volume of the rectangular bar = x2 × 9 ...(2) A h

Volume of metallic block = Volume of rectangular


bar 15 24
B H
1 1
So, 9 x 2 = 1 ⇒ x 2 = ⇒x= m
9 3
1
Side of biggest cube possible = m
2r
3
3 Let the height of cylinder A be h and height of
1 cyilnder B be H. Since, in both the situation, the
So, weight of the cube = weight of block ×  
3 volume of the water in the bottle remains same, then
1 10 1 π(2r)2H + πr2(15 – H) = πr2h + π(2r)2(24 – h)
= 90 × = kg = 3 kg
27 3 3
πr2 [4H + (15 – H)] = πr2[h + 4(24 – h)]
33. (c) Since, all the edges of cuboid are integers and base 4H + 15 – H = h + 4 (24 – h)
is square.
3H + 15 = h + 96 – 4h
Let the length, breadth and height of cuboid be x, x, y.
3H + 15 = 96 – 3h
Sum of all edges of cuboid = 4x + 4x + 4y
H + 5 = 32 – h
Sum of area of all faces = 2x2 + 2xy + 2xy
H + h = 27 cm
Given,
36. (b) Let height and radius of cone is h and r respectively
Sum of all edges of cuboid = Sum of area of all faces such that h, r ∈ I
∴ 4x + 4x + 4y = 2(x2 + xy + xy) Q Volume of cone = Surface area of cone
⇒ x2 + 2xy – 4x – 2y = 0
1 2
∴ πr h = πrl + πr 2
⇒ x2 + 2x (y – 2) – 2y = 0 3

−2( y − 2) ± 4( y − 2)2 + 4(2 y ) ⇒


1 2
πr h = πr h 2 + r 2 + πr 2
⇒ x = 3
2
1
⇒ x = y − 2 ± y 2 − 2 y + 4 ⇒ rh = h 2 + r 2 + r [Q r ≠ 0]
3
Then x is integer, when y = 2 ∴ y = 2, x = 2 2 2
⇒ rh − 3r = 3 h + r
Hence, the required sum of edges of cuboid
⇒ r2h2 + 9r2 – 6hr2 = 9h2 + 9r2
= 8x + 4y = 16 + 8 = 24
6r 2
34. (d) Let sides of the solid brick be x, 2x and 3x. ⇒ h2(r2 – 9) = 6hr2  ⇒  h =
r2 − 9
Volume = Number of bricks = 6x3
 r2 
6x3 < 400 ⇒ h = 6  54
 r2 − 9  ⇒ h = 6+
2
400   r −9
x3 <
6
h and r are integers
2
x3 < 66 Here (r2 – 9) should be factor of 54.
3
Number less than 66 which is perfect cube is 64 than ∴ r2 – 9 = 1, 2, 3, 6, 9, 18, 27, 54.
volume = 6 × 64 = 384 i.e., no. of cube = 384. ⇒ r2 = 10, 11, 12, 15, 18, 27, 36, 63.
M-130 Mathematics
∴ r = 6 (only possible value)
πr 2
53. (b) Space occupied by each student = = 38.5 m 2
54 4
∴ h = 6 + = 6 + 2 = 8 ∴ r = 6, h = 8 54. (a) Volume of hemispherical dome
36 − 9
2 2 22
= πr 3 = × × 21 × 21 × 21 = 19404 cu. m
37. (c) Let r1and r2 be the radii of two cones. 3 3 7
Volume of cylinder = Sum of volume of two cones 4
55. (b) πr 3
3
p 56. (b) Cloth required to cover hemispherical done = curved
p × 72 × 10 = × 10(r12 + r22 )
3 surface area of hemisphere = 2pr2 = 1232 sq. m
57. (c) Surface area of combined figure = 2pr2 + 2(l + b)h
⇒ r 12 + r22 = 49 × 3 ...(i)
= 1232 + 2(6 + 4)8 = 1392 sq. m
\ Percentage increase 58. (d) 192 m3

pr12 + pr22 - 2p ¥ 7 2 r 2 + r22 - 2 ¥ 49 59. (d) Length of diagonal = 3l 2 = 6 3


= ¥ 100 = 1 ¥ 100
2 ¥ p ¥ 72 2 ¥ 49 60. (c) Volume of cube = (side)3 = 343 cm3
61. (b) Curved surface area of hemisphere = 2pr2 = 308 cm2
(49 ¥ 3 - 49 ¥ 2) 62. (b) Slant height = r 2 + h 2 = 25 cm
= ¥ 100  [From (i)]
49 ¥ 2 63. (a) 858 cm2
= 50% 64. (a) Assertion and Reason both are correct and reason is
the correct explanation of the assertion .
38. (c) It is given that 6(x2 + y2 + z2) = 498, where x, y and
z are sides of cubes. Total surface area = 2πrh + 2πr2 = 2πr(h + r)
x2 + y2 + z2 = 83, for x, y, z to be integer 22
= 2× ×14 (30+14) = 88 (44)
7
x = 49 , y = 25 , z = 9 ⇒  x = 7, y = 5, z = 3
= 3872 cm2
Now, sum of volumes= 73 + 53 + 33
65. (d) Assertion is incorrect here, but reason is correct.
⇒  343 + 125 + 27 = 495
For x, y, z to be integer Slant height = (14 / 2)2 + (24)2 = 49 + 576

= 625 = 25 cm
x = 81 , y = 1,z= 1 ⇒  x = 9, y = 1, z = 1
66. (d) Assertion is incorrect and reason is true.
Sum of volumes = 93 + 13 + 13 = 729 + 1 + 1 = 731
V1 (1 / 3)πr 2 h1 4h
So, total sum of volume of cubes is (495 + 731) 1226. = = 1
39. (b) 40. (d) 41. (d) 42. (b) 43. (c) V2 (1 / 3)π(r / 2)2 h2 h2

44. (a) 45. (c) 46. (d) 47. (a) 48. (b)
V1 = V2
As
49. (d) Volume of cylindrical cup = pr2h
22 7 7 ∴ h2 = 4h1
× × × 10.5 = 404.25 cm3
7 2 2 67. (a) Both assertion and reason are correct and reason is
50. (b) Volume of hemispherical cup the correct explanation of the assertion.
3 68. (c) Assertion is correct but reason is not correct.
2 3 3 22  7 
= πr = × ×   = 89.83 cm3
3 2 7  2 69. (A) → (q); (B) → (p); (C) → (r); (D) → (s)
51. (d) Cylindrical glass, 314.42 cm3
70. (A) → (r); (B) → (s); (C) → (q) ; (D) → (p)
52. (c) Curved surface area of cone = 551 ⇒ prl = 441
22 71. (A) → (q); (B) → (p); (C) → (r); (D) → (s)
⇒ × 7 × l = 551
7
72. (A) → (s); (B) → (r); (C) → (p); (D) → (q)
⇒ l = 25.045
d3
2 2
73. cb. units.
3 3
∴ h = l − r = 24 m
Surface Areas and Volumes M-131
74. cuboid 82. True
75. 49 : 25 83. True
76. space 84. True
77. 96 cm2 85. False
78. 3 units 86. True
79. 15 cm 87. False
80. True 88. True
81. False
15 Acids, Bases and
Statistics
Salts
4. The mean weight of a class of 35 students is 45kg.
Multiple Choice Questions (MCQs) If the weight of a teacher be included, the mean
weight increases by 500 grams. Find the weight of
DIRECTIONS : This section contains multiple choice the teacher.
questions. Each question has 4 choices (a), (b), (c) and (d) out (a) 63 kg (b) 61 kg
of which only one is correct.
(c) 64 kg (d) 70 kg
1. For the following grouped frequency distribution 5. In the following distribution
Class 3–6 6–9 9–12 12–15 15–18 18–21 21–24
Monthly income range (in `) Number of families
Frequency 2 5 10 23 21 12 3
Income more than ` 10,000 100
Consider the following. Income more than ` 13,000 85
(I) Lower limit of model class is 12 Income more than ` 16,000 69
(II) Frequency of the class preceding the model class = 10 Income more than ` 19,000 50
(III) Mode = 14.6 Income more than ` 22,000 33
(IV) Width of the model class is 4 Income more than ` 25,000 15
Which of the following is correct? the number of families having income range (in `)
(a) I and II (b) II and III 16000 – 19000 is
(c) I, II and III (d) All of these (a) 15 (b) 16
(c) 17 (d) 19
2. The marks in science of 80 students of class X are given
below. Find the mode of these marks obtained by the 6. Find the mean of the following frequency distribution.
students in science. Class Interval 0–10 10–20 20–30 30–40 40–50
Marks 0–10 10–20 20–30 30–40 40–50 Frequency 8 12 10 11 9
Frequency 3 5 16 12 13
(a) 25. 3 (b) 25.2
Marks 50–60 60–70 70–80 80–90 90–100
Frequency 20 5 4 1 1 (c) 24 (d) 25.5
7. Calculate the mean of the following frequency distribution:
(a) 53.18 (b) 53.25
C. I. 0–80 80–160 160–240 240–320 320–400
(c) 52.25 (d) 53
Frequency 22 35 44 25 24
3. There are three sections A, B and C in class X with 25,
(a) 195.5 (b) 198.8
40 and 35 students respectively. The average marks
obtained by section A, B and C are 70%, 65% and (c) 196.8 (d) 195
50% respectively. Find the average marks of entire 8. If the mean of first ‘n’ odd natural numbers is ‘n’ itself,
class X. what is the value of ‘n’?
(a) 59% (b) 56% (a) 2 (b) 3
(c) 63% (d) 61% (c) 1 (d) Any natural number
Statistics M-133
9. What is the arithmetic mean of 20 fours, 40 fives, 30 sixes 17. In a frequency distribution, the mid value of a class is 10
and 10 tens? and the width of the class is 6. The lower limit of the class
(a) 50 (b) 25 is
(c) 5.6 (d) 33 (a) 6 (b) 7 (c) 8 (d) 12

10. The mean of discrete observations y1, y2, .........., yn is given 18. If the mean of the observations x, x + 3, x + 5, x + 7 and
by x + 10 is 9, then the mean of the last three observations is
1 2
n n
(a)
10 (b) 10
∑y i ∑y i 3 3
(a) i =1
(b) i =1
n 1 2
n (c)
11 (d) 11
∑i
i =1
3 3
19. The mean of fifteen different natural numbers is 13. The
n n
maximum value for the second largest of these numbers
∑y f i i ∑y f i i
is
(c) i =1
(d) i =1
n
n (a) 46 (b) 51 (c) 52 (d) 53
∑f i
i =1 20. For the data (2, 9, x + 6, 2x + 3, 5, 10, 5) if mean is 7, then
11. If the mean of the numbers 27 + x, 31 + x, 89 + x, 107 + x, mode is
156 + x is 82, then the mean of 130 + x, 126 + x, 68 + x, (a) 3 (b) 5 (c) 9 (d) 10
50 + x, 1 + x is 21. The mean of three positive numbers is 10 more than the
(a) 75 (b) 157 smallest of the numbers and 15 less than the largest of the
(c) 82 (d) 80 three. If the median of the three numbers is 5, then the
mean of squares of the numbers is
12. If the class-intervals are 10 – 19, 20 – 29, 30 – 39, ..............,
then the upper limit of the first class-interval is 2 2
(a)
108 (b) 116
(a) 19.5 (b) 19 3 3
(c) 20 (d) None of these 1 2
(c) 208 (d) 216
13. The numbers 3, 5, 7 and 9 have their respective frequencies 3 3
x – 2, x + 2 , x – 3 and x + 3. If the arithmetic mean is 6.5, 22. Positive integers from 1 to 21 are arranged in 3 groups
then the value of x is of 7 integers each, in some particular order. Then the
(a) 3 (b) 4 highest possible mean of the medians of these 3 groups
is ______
(c) 5 (d) 6
(a) 16 (b) 12.5
14. The median of a set of 9 distinct observations is 20.5. If
(c) 11 (d) 14
each of the largest 4 observation of the set is increased by
2, then the median of the new set 23. The mean of 5 observation is 15. If the mean of first three
(a) is increased by 2 observations is 14 and last three observations is 17, then
the third observation is :
(b) is decreased by 2
(a) 18 (b) 19 (c) 17 (d) 20
(c) is two times the original median
24. The mean of a group of eleven consecutive natural numbers
(d) remains the same as that of the original set
is m. What will be the percentage change in the mean when
15. A set of numbers consists of three 4’s, five 5’s, six 6’s, next six consecutive natural numbers are included in the
eight 8’s and seven 10’s. The mode of this set of numbers group ?
is m m 300
(a) 6 (b) 7 (a) m% (b) % (c) % (d) %
3 300 m
(c) 8 (d) 10 25. A 100 mark examination was administered to a class of
16. The mean of a set of 20 observation is 19.3. The mean is 50 students. Despite only integer marks being given, the
reduced by 0.5 when a new observation is added to the set. average score of the class was 47.5. Then, the maximum
The new observation is number of students who could get marks more than the
class average is
(a) 19.8 (b) 8.8
(a) 25 (b) 35 (c) 45 (d) 49
(c) 9.5 (d) 30.8
M-134 Mathematics
26. Let x1, x2 , ..., x11 be 11 distinct positive integers. If we 29. The upper limit of modal class is
replace the largest of these integers by the median of the (a) 15 (b) 25
other 10 integers, then
(c) 35 (d) 45
(a) the median remains the same
30. The mean of the given data is
(b) the mean increases
(a) 26.2 (b) 32.4
(c) the median decreases
(c) 33.5 (d) 35.4
(d) the mean remains the same
Refer to table 2
27. The mean of 15 observations written in some order is
50. If the mean of first eight observations and last eight 31. The mode of the given data is
observations are 48 and 53 respectively, then the eighth (a) 41.4 (b) 48.2
observation is (c) 55.3 (d) 64.6
(a) 35 (b) 80 (c) 72 (d) 58
32. The median of the given data is
(a) 32.7 (b) 40.2
(c) 42.3 (d) 48.6

DIRECTIONS : Study the given Case/Passage and answer the Case/Passage-II


following questions. COVID-19 Pandemic
Electricity energy consumption is the form of energy
Case/Passage-I
consumption that uses electric energy. Global electricity
COVID-19 Pandemic consumption continues to increase faster than world population,
The COVID-19 pandemic, also known as coronavirus pandemic, leading to an increase in the average amount of electricity
is an ongoing pandemic of coronavirus disease caused by the consumed per person (per capita electricity consumption).
transmission of severe acute respiratory syndrome coronavirus 2 [From CBSE Question Bank 2021]
(SARS-CoV-2) among humans. Teriff : LT. Residential
[From CBSE Question Bank 2021]
Type of Supply : Single Phase
Meter Reading Date : 31-11-13
Previous Reading Date : 31-10-13
Bill Number : 384750
Connected Load : 3 kW
The following tables shows the age distribution of case admitted Meter Reading : 65700
during a day in two different hospitals Previous Meter Reading : 65500
Table 1 Units consumed : 269

Age A survey is conducted for 56 families of a Colony A. The


5-15 15-25 25-35 35-45 45-55 55-65 following tables gives the weekly consumption of electricity
(in years)
of these families.
No. of
6 11 21 23 14 5 Weekly
cases
consumption 0-10 10-20 20-30 30-40 40-50 50-60
Table 2 (in units)
Age No. of
(in years) 5-15 15-25 25-35 35-45 45-55 55-65 16 12 18 6 4 0
families
No. of The similar survey is conducted for 80 families of Colony B
8 16 10 42 24 12
cases and the data is recorded as below:
Refer to table 1 Weekly
consumption 0-10 10-20 20-30 30-40 40-50 50-60
28. The average age for which maximum cases occurred is (in units)
(a) 32.24 (b) 34.36 No. of
0 5 10 20 40 5
(c) 36.82 (d) 42.24 families
Statistics M-135
Refer to data received from Colony A
Match the Following
33. The median weekly consumption is
(a) 12 units (b) 16 units
DIRECTIONS : Each question contains statements given in
(c) 20 units (d) None of these two columns which have to be matched. Statements (A, B, C, D)
34. The mean weekly consumption is in column-I have to be matched with statements (p, q, r, s) in
column-II.
(a) 19.64 units (b) 22.5 units
(c) 26 units (d) None of these 41. The table shows a frequency distribution of the life time
35. The modal class of the above data is I of 400 radio tubes tested at a company.

(a) 0-10 (b) 10-20 Life time Number of Life time Number
(c) 20-30 (d) 30-40 (hours) tubes (hours) of tubes
Refer to data received from Colony B 300 – 399 14 800 – 899 62
400 – 499 46 900 – 999 48
36. The modal weekly consumption is
500 – 599 58 1000 – 1099 22
(a) 38.2 units (b) 43.6 units
600 – 699 76 1100 – 1199 6
(c) 26 units (d) 32 units
700 – 799 68
37. The mean weekly consumption is
Column-II gives data for description given in Column-I,
(a) 15.65 units (b) 32.8 units
match them correctly.
(c) 38.75 units (d) 48 units
Column-I Column-II
Assertion & Reason (A) Upper limit of the fifth class (p) 100
(B) Lower limit of the eighth class (q) 949.5
DIRECTIONS : Each of these questions contains an Assertion
(C) Class marks of the seventh class (r) 1000
followed by Reason. Read them carefully and answer the
question on the basis of following options. You have to select (D) Class interval size (s) 799
the one that best describes the two statements. 42. Following is the distribution of heights of students in a
(a) If both Assertion and Reason are correct and Reason is class and the total number of students is 50.
the correct explanation of Assertion.
Height (in cm) Frequency Cumulative
(b) If both Assertion and Reason are correct, but Reason is
frequency
not the correct explanation of Assertion.
(c) If Assertion is correct but Reason is incorrect. 150 – 155 12 a
(d) If Assertion is incorrect but Reason is correct. 155 – 160 b 25
38. Assertion : The arithmetic mean of the following given 160 – 165 10 c
frequency distribution table is 13.81. 165 – 170 d 43
x 4 7 10 13 16 19 170 – 175 e 48
f 7 10 15 20 25 30 175 – 180 2 f

Reason : x =
∑ fi xi . With the help of information given match the columns.
∑ fi
Column-I Column-II
39. Assertion : If the number of runs scored by 11 players of
a cricket team of India are 5, 19, 42, 11, 50, 30, 21, 0, 52, (Unknown) (Values)
36, 27 then median is 30. (A) a= (p) 35
th
 n +1 (B) b= (q) 5
Reason : Median =   value, if n is odd.
 2  (C) c= (r) 13
40. Assertion : If the value of mode and mean is 60 and 66 (D) d= (s) 50
respectively, then the value of median is 64. (E) e= (t) 12
1 (F) f= (u) 8
Reason : Median = (mode + 2 mean).
2
M-136 Mathematics
43. 46. For the given frequency distribution, match the Column-I
Column-I Column-I with Column-II.
(A) The arithmetic th Class 30–35 35–40 40–45 45–50 50–55 55–60 60–65
n +1
mean of a set of (p) Value of the   Frequency 14 16 18 23 18 8 3
observations is observation.  2 
h = width of the class interval
obtained by
c = cumulative frequency
(B) The value of 1 f = frequency of the class interval to which median belongs
the observation (q) × Value of
2 l1 = lower limit of the median class interval
having maximum
frequency is called  n  th  n  th  Column-I Column-II
  +  + 1 
  2   f
(A) (p) 45.4
 2 
observations. c
(B) (q) 45
(C) If n is odd, then (r) Median l1
(C) (r) 48
median is equal to (D) median (s) 23
(D) If n is even, then (s) Mode
median is equal to
Fill in the Blanks
(E) – divides the (t) Dividing the sum of the
arranged series values of observations by
(in ascending or the number of observations DIRECTIONS : Complete the following statements with an
descending order) parts. appropriate word / term to be filled in the blank space(s).
into two equal parts. 47. Median divides the total frequency into .......... equal parts.
44. 48. The algebraic sum of the deviations from arithmetic
Column-I Column-II mean is always ...............
(A) Mean of first five prime numbers is (p) 12 49. The class mark of a class is 25 and if the upper limit of
(B) Mean of all factors of 24 is (q) 7.5 that class is 40, then its lower limit is ...................
(C) Mean of first six multiples of 4 is (r) 5.6 50. The mid-value of 20-30 is .................
(D) If mean of x – 5y, x – 3y, x – y, (s) 14 51. The sum of 12 observations is 600, then their mean is .........
x + y , x + 3y and x + 5y is 12, 52. In the class interval 35-46, the lower limit is ....... and
then x is upper limit is .............
45. For the following marks distribution of 5 students in 53. A class interval of a data has 15 as the lower limit and 25
an examination, match Column-I with the data given in as the size, then the class mark is .............
Column- II. 54. 0–10, 10–20, 20–30 ............ so on are the classes, the
lower boundary of the class 20–30 is ...............
Class interval 0–10 10–20 20–30 30–40
55. The mid-point of a class interval is called its ................
No. of students 1 3 0 1
56. Facts or figures, collected with a definite purpose, are
Here, called ..................
xk = lower limit of the modal class interval 57. Value of the middle-most observation (s) is called ............
h = width of the class interval 58. The ................. is the most frequently occurring observation.
fk = frequency of the modal class 59. 3 median = mode + ............. mean
fk–1 = frequency of the class preceding the modal class 60. To find the mode of a grouped data, the size of the classes
fk+1 = frequency of the class succeeding the modal class is ..................
Column-I Column-II n
61. On an ogive, point A (say), whose Y- co-ordinate is
2
xk
(A) (p) 3
(half of the total observation), has its X-coordinate equal
fk
(B) (q) 10 to .................. of the data.
fk–1
(C) (r) 0 62. Two ogives, for the same data intersect at the point P.
h
(D) (d) 1 Then, Y-coordinate of P represents .................. .
Statistics M-137
66. The modal value is the value of the variate which divides
True / False the total frequency into two equal parts.
67. The mean of x, y, z is y, then x + z = 2y
DIRECTIONS : Read the following statements and write your
68. 2(Median – Mean) = Mode – Mean.
answer as true or false.
69. Mean may or may not be the appropriate measure of
63. The median for grouped data is formed by using the central tendency.
n  70. If 16 observations are arranged in ascending order, then
− cf
 
formula, Median = l +  2  ×h (8th observation + 9th observation )
f median is
  2
71. Median of 15, 28, 72, 56, 44, 32, 31, 43 and 51 is 43.
Upper class limit + Lower class limit
64. Class mark = 72. Mode of 2, 3, 4, 5, 0, 1, 3, 3, 4, 3 is 3.
2
65. The median of grouped data with unequal class sizes 73. Mean of 41, 39, 48, 52, 46, 62, 54, 40, 96, 52, 98, 49, 42,
cannot be calculated. 52, 60 is 55.4
M-138 Mathematics

ANSWER KEY & SOLUTIONS


1. (c) ( 25 × 70) + ( 40 × 65) + ( 35 × 50)
=
Class 3–6 6–9 9–12 12–15 15–18 18–21 21–24 25 + 40 + 35
1750 + 2600 + 1750 6100
fi 2 5 10 23 21 12 3 = = = 61%
100 100
Here the maximum frequency is 23 and the 4. (a) 
Let the mean weight of a class of 35 students
corresponding class is 12-15 be x 1 and that of both students and a teacher be
x2
We have l = 12, h = 13, f = 23 , f1 = 10

and f2 = 21 Then x 1 = 45 kg and


f − f1
Mode = l + ×h
x 2 = 45 +
500
= 45 + 0.5 = 45.5kg
2f − f1 − f 2
1000
23 − 10
⇒ Mode = 12 + ×3 Σx1 Σx
46 − 10 − 21 x 1 = , x2 = 2
n1 n1
39
⇒ Mode = 12 + = 14.6
15 Σx1 Σx
⇒ 45 = , 45.5 = 2
35 36
2. (a) The class (50-60) has maximum frequency i.e. 20
therefore this is the modal class. ⇒ Σx1 = 1575 kg, Σx2 = 1638 kg

Lower Limit of the modal class = l = 50 ⇒ Total weight = weight of students + weight of
teacher
Class-Interval = h = 10
∴ Weight of teacher = Total weight – weight of
Frequency of the modal class = fi = 20
students
Frequency of the class preceding the modal class
∴ Weight of the teacher = Σx2 – Σx1
= f0 = 13
= 1638 – 1575 = 63 kg
Frequency of the class succeeding the modal class
= f2 = 5 5. (d) 
Clearly, the number of families having income
range (in `)
 f1 − f 0 
Mode, N 0 = l +  ×h
 2f1 − f 0 – f 2  16000 – 19000 = 69 – 50 = 19.
6. (b)
 20 − 13 
= 50 + 
 2 × 20 − 13 − 5  C.I. xi fi fixi

7 0 – 10 5 8 40
= 50 + × 10 = 50 + 3.18 = 53.18
22 10 – 20 15 12 180
3. (d) n1 = 25 x1 = 70% 20 – 30 25 10 250
n 2 = 40 x 2 = 65%
30 – 40 35 11 385
n 3 = 35 x 3 = 50%
40 – 50 45 9 405
n x1 + n 2 x 2 + n 3 x 3
x= 1 50 1260
n1 + n 2 + n 3

Statistics M-139
Σ fx 1260 ∴ Required mean is,
We have x =  = = 25.2 130 + x + 126 + x + 68 + x + 50 + x + 1 + x
Σf 50 x=
375 + 5 x 375 + 0 5 375
x= = = = 75
7. (c) Let a = 200 5 5 5

C.I. fi xi di = xi – 200 fidi 12. (a) 13. (c)


th
0 – 80 22 40 – 160 – 3520  9 +1 th
14. (d) Since n = 9, then middle term =   = 5 term.
 2 
80 – 160 35 120 – 80 – 2800
Now, last four observations are increased by 2.
160 – 240 44 200 0 0 Q The median is 5th observation, which is remains


240 – 320 25 280 +80 +2000 unchanged.

\ There will be no change in median.


320 – 400 24 360 + 160 + 3840
15. (c) 
Mode of the data is 8 as it is repeated maximum
150 – 480
number of times.
5
1 1 16. (b)
x = a +
n ∑ fidi   = 200 + 150 ( – 480)
i =1 17. (b) Let x be the upper limit and y be the lower limit.
= 200 – 3.2 = 196.8 Since, the mid value of the class is 10.

8. (d) First find the sum of first ‘n’ odd natural numbers x+ y
∴ = 10 ⇒ x + y = 20 .....(i)
2
1 = 12
and x – y = 6 (width of the class = 6)  .....(ii)
1 + 3 = 4 = 22
By solving equations (i) and (ii), we get y = 7.
1 + 3 + 5 = 9 = 32
Hence, lower limit of the class is 7.
1 + 3 + 5 + ....+ (2n –1) = n2.
Sum of all the observations
2
18. (c) We know, Mean =
Sum of observations n Total no. of observation
Mean = = =n
n n x + x + 3 + x + 5 + x + 7 + x + 10
⇒ Mean =
It is given that the mean is ‘n’ itself. 5
5 x + 25
∴ n=n 9 = ⇒ x=4
5
Thus, ‘n’ is any natural number. So, mean of last three observations i.e; 14, 11 and 9
20 ( 4) + 40 ( 5) + 30 ( 6) + 10 (10) is
9. (c) x=
20 + 40 + 30 + 10 14 + 11 + 9 34 1
= = 11
80 + 200 + 180 + 100 560 3 3 3
= = = 5.6
100 100
x1 + ... + x15
19. (b) Given, = 13
10. (a) 15

11. (a) Given, x1 + x2 + x3 + ... + x15 = 15 × 13 = 195


(27 + x) + (31 + x) + (89 + x) + (107 + x) + (156 + x) In order to set the second largest and largest, first
82 =
5 thirteen natural numbers are
⇒ 82 × 5 = 410 + 57 ⇒ 410 – 410 = 5x ⇒ x = 0 1, 2, 3, 4, 5, 6, 7, 8, 9, 10, 11, 12, 13
M-140 Mathematics
13 × 14 23. (a) 3rd observation = (14 × 3) + (3 × 17) – (5 × 15) = 18
So, x14 + x15 = 195 –
2 24. (d) 
Since, the mean of a group of eleven consecutive
⇒ x14 + x15 = 195 – 91 = 104 natural numbers is m, then
x + x + 1 + ... + x + 10
Now, from the options, we have =m
11
x14 = 51 and x15 = 53
11x + 55 = 11 m; x + 5 = m; x = m – 5
Now, second largest is 51.
Let n be the mean when next six consecutive natural

2 + 9 + x + 6 + 2 x + 3 + 5 + 10 + 5
20. (c) Mean = =7 numbers are included in the group then
7
x + x + 1 + ..... + x + 16
=n;
3 x + 40 = 49 ⇒ x = 3 17
16 × 17
x + 6 = 9 ⇒ 2 x + 3 = 9 17 x + = 17 n
2
Data (2, 9, 9, 9, 5, 10, 5) and Mode = 9 17x + 8 × 17 = 17 n
21. (d) Let the three numbers be a, b and c such that a > b > c m – 5 + 8 = n
According to the given condition ⇒  n = m + 3
a+b+c (∴ x = m – 5)
= c + 10 = a – 15 = k
3 Hence, required percentage change in the mean

⇒ c = k – 10 n−m m +3− m 300


= × 100 = × 100 = %
m m m
Also, a = k + 15; b = 5 (QMedian = 5)
25. (d) Total number of students = 50
We know that, a + b + c = 3k Average marks of student = 47.5
⇒ k + 15 + 5 + k – 10 = 3k ∴ Total marks of students = 50 × 47.5 = 2375
⇒ 10 = k Now, the student get integer marks
Hence, for maximum number of students who
Thus, a = 25, b = 5, c = 0
could get marks more than 47.5,
∴ Mean of squares of the numbers we will divide total marks by 48.
2 2 2
25 + 5 + 0 650 2 2375
= = = 216 ∴ ≈ 49
3 3 3 48

22. (d) We need to maximize the median in each group to
26. (c) Let x1 < x2 < ... < x11
maximize the average of all median.
x5 + x6
Highest possible median is 18 at there should be 3 Median of x1, x2 ,..., x10 =
2
numbers higher than it in a group of 7.
x5 + x6
So we have Now replace x11 by
 and then arrange in
2
order
1 2 3 18 19 20 21
So, new set of numbers are
Similarly, 4 5 6 14 15 16 17
x1, x2, x3, x4, x5, x5 + x6 , x6, x7, x8, x9, x10,
7 8 9 10 11 12 13 2

Medians are, 18, 14 and 10. x5 + x6


Hence, median is < x6
2
18 + 14 + 10 42
Mean = = = 14
3 3 ⇒ median decreases
Statistics M-141
27. (d) Let x1, x2, x3, ..., x15 be the no. of observations 36. (b) Modal weekly consumption

x1 + x2 + x3 + x4 + ... +x8 = 384  f1 − f 0 


=l+  h = 43.6 units
 2f1 − f 0 − f 2 
x8 + x9 + x10 + ... + x15 = 424 37. (c) Mean weekly consumption
Σfi x i 3100
x1 + x2 + x3 + ... + x15 = 750 = =
Σf i 80 = 38.75 units
x1 + x2 + x3 + x4 + x5 + x6 + x7 + x8 ...+ x15 = 750 38. (a) Both assertion and reason are true, reason is the
correct explanation of the assertion.
384 – x8 + 424 = 750
39. (d) Arranging the terms in ascending order,
Hence, eighth observation is 58
0, 5, 11, 19, 21, 27, 30, 36, 42, 50, 52
 f1 − f 0  th
28. (c) Mode = l +  h = 36.82  11 + 1
 2f1 − f 0 − f 2  median value =  = 6th value = 27
 2 
29. (d) Modal class = The class with maximum frequency
1 1
40. (c) Median = (mode + 2 mean) = ( 60 + 2 × 66 ) = 64
= 35 – 45 3 3

\ Upper limit of modal class = 45 41. (A) → (s); (B) → (r); (C) → (q); (D) → (p)
Σf i x i 2830
30. (d) Mean = = = 35.4 42. (A) → (t); (B) → (r); (C) → (p); (D) → (u); (E) → (q);
Σf i 80
(F) → (s)
 f1 − f 0 
31. (a) Mode = l +  h = 41.4
 2f1 − f 0 − f 2  As we know, cumulative frequency of an interval is equal
to the sum of frequency of that interval and of previous
n 
− Cf intervals.
32. (b) Median = l +  2  × h = 40.2
 f 
  ∴ a = 12

a + b = 25 ⇒ b = 13 (∵ a = 12)
Weekly No. of Cumulative Class fixi
Now, 25 + 10 = c ⇒ c = 35
consumption families fi frequency marks
cf xi c + d = 43 ⇒ d = 8 (∵ c = 35)
1-10 16 16 5 8 43 + e = 48 ⇒ e = 5
10-20 12 28 15 180
and, f = 48 + 2 = 50
20-30 18 46 25 450
30-40 6 52 35 210 43. (A) → (t); (B) → (s); (C) → (p); (D) → (q); (E) → (r)
40-50 4 56 45 180
44. (A) → (r); (B) → (q); (C) → (s); (D) → (p)
50-60 0 0 55 0
Sfi = 56 Sfidi = 110 45. (A) → (q); (B) → (s); (C) → (t); (D) → (q)

46. (A) → (s); (B) → (u); (C) → (q); (D) → (p)


n 
− cf
33. (c) Median = l +  2  h = 20 units 47. two
 f 
 
48. zero
–) = Σfi x i = 19.64 units
34. (a) Mean (x 49. 10
Σf i

35. (c) Modal class = the class with maximum frequency 50. 25

= 20 – 30 51. 50
M-142 Mathematics
52. 35, 46 53. 27.5 63. True 64. True

54. 20 55. class-mark 65. False 66.


False

56. data 57. median 67. True 68.


False

58. mode 59. 2 69. True 70. True

60. uniform 61. Median 71. True 72. True

62. cumulative frequency of the median class. 73. True

You might also like